KAPLAN & SADOCK’S
Study Guide and Self-Examination Review in Psychiatry
Ninth Edition

KAPLAN & SADOCK’S
Study Guide and Self-Examination Review in Psychiatry
NINTH EDITION
Benjamin James Sadock, M.D.
Menas S. Gregory Professor of Psychiatry, New York University School of Medicine; Attending Psychiatrist, Tisch Hospital; Attending Psychiatrist, Bellevue Hospital Center; Honorary Medical Staff, Lenox Hill Hospital, New York, New York
Virginia Alcott Sadock, M.D.
Professor of Psychiatry, Department of Psychiatry, New York University School of Medicine; Attending Psychiatrist, Tisch Hospital; Attending Psychiatrist, Bellevue Hospital Center, New York, New York
Pedro Ruiz, M.D.
Professor and Executive Vice Chairman, Department of Psychiatry and Behavioral Sciences, University of Miami Miller School of Medicine, Miami, Florida
President, World Psychiatric Association (2011–2014)
                                                                       
Acquisitions Editor: Charley Mitchell
Product Manager: Tom Gibbons
Vendor Manager: Bridgett Dougherty
Senior Manufacturing Manager: Benjamin Rivera Marketing Manager: Brian Freiland
Design Coordinator: Steve Druding Production Service: Aptara, Inc.
© 2011 by LIPPINCOTT WILLIAMS & WILKINS, a WOLTERS KLUWER business © 2007, 2003 by LIPPINCOTT WILLIAMS & WILKINS
Two Commerce Square
2001 Market Street
Philadelphia, PA 19103 USA LWW.com
“Kaplan Sadock Psychiatry” with the pyramid logo is a trademark of Lippincott Williams & Wilkins.
All rights reserved. This book is protected by copyright. No part of this book may be reproduced in any form by any means, including photocopying, or utilized by any information storage and retrieval system without written permission from the copyright owner, except for brief quotations embodied in critical articles and reviews. Materials appearing in this book prepared by individuals as part of their official duties as U.S. government employees are not covered by the above-mentioned copyright.
Printed in China
Library of Congress Cataloging-in-Publication Data
Sadock, Benjamin J.
Kaplan & Sadock’s study guide and self-examination review in psychiatry / Benjamin James
Sadock, Virginia Alcott Sadock, Pedro Ruiz. – 9th ed. p. ; cm.
Kaplan and Sadock’s study guide and self-examination review in psychiatry
Study guide and self-examination review in psychiatry
Includes index.
Summary: “This study guide is an excellent aid in preparing for boards and other psychiatry
exams and in reinforcing your knowledge. It offers chapter overviews of Synopsis, Tenth Edition and over 1,600 multiple-choice questions with discussions of correct and incorrect answers”–Provided by publisher.
ISBN 978-1-4511-0000-6 (pbk. : alk. paper)
1. Psychiatry–Examinations–Study guides. 2. Psychiatry–Examinations, questions, etc. I. Kaplan, Harold I., 1927–1998. II. Sadock, Virginia A. III. Ruiz, Pedro, 1936–
IV. Title. V. Title: Kaplan and Sadock’s study guide and self-examination review in psychiatry. VI. Title: Study guide and self-examination review in psychiatry.
[DNLM: 1. Mental Disorders–Examination Questions. 2. Psychiatry–Examination Questions. WM 18.2]
RC454.K36 2011 616.890076–dc22
2011014402
Care has been taken to confirm the accuracy of the information presented and to describe generally accepted practices. However, the authors, editors, and publisher are not responsible for errors or omissions or for any consequences from application of the information in this book and make no warranty, expressed or implied, with respect to the currency, completeness, or accuracy of the contents of the publication. Application of the information in a particular situation remains the professional responsibility of the practitioner.
The authors, editors, and publisher have exerted every effort to ensure that drug selection and dosage set forth in this text are in accordance with current recommendations and practice at the time of publication. However, in view of ongoing research, changes in government regulations, and the constant flow of information relating to drug therapy and drug reactions, the reader is urged to check the package insert for each drug for any change in indications and dosage and for added warnings and precautions. This is particularly important when the recommended agent is a new or infrequently employed drug.
Some drugs and medical devices presented in the publication have Food and Drug Administration (FDA) clearance for limited use in restricted research settings. It is the responsibility of the health care provider to ascertain the FDA status of each drug or device planned for use in their clinical practice.
To purchase additional copies of this book, call our customer service department at (800) 638-3030 or fax orders to (301) 223-2320. International customers should call (301) 223-2300.
Visit Lippincott Williams & Wilkins on the Internet: at LWW.com. Lippincott Williams & Wilkins customer service representatives are available from 8:30 am to 6 pm, EST.
 
10 9 8 7 6 5 4 3 2 1
 
This ninth edition of Study Guide and Self-Examination Review in Psychiatry stands alone as a separate textbook. In the form of questions and answers, it can serve as a guide to the entire field of psychiatry and the behavioral sciences. Questions are designed to examine the etiology, diagnosis, and treatment of every known psychiatric disorder. A thorough discussion covers not only cor- rect answers but also wrong answers. By carefully studying both questions and answers, the reader will gain a thorough under- standing of material useful to prepare for examinations of all types.
This book was written to meet the needs of medical students, psychiatric physicians, and mental health professionals from all fields. It is designed especially to help those preparing for the United States Medical Licensing Examination (USMLE) and the American Board of Psychiatry and Neurology (ABPN); it will also prove of value to all who want to test their knowledge in psychiatry as part of their continuing medical education.
The authors have added new and different questions to each edition of Study Guide and modified and updated material from earlier editions. This Study Guide contains more than 1,500 ques- tions, more than any other book of its kind, and the format of each question is standardized to follow that used by the USMLE and ABPN. In addition, the allocation of topics is carefully weighted with attention to both clinical and theoretical issues.
The authors of the last edition of Study Guide are particularly pleased that Pedro Ruiz, M.D., a close personal and professional associate and outstanding academician, has joined them as third author. Dr. Ruiz is Professor and Executive Vice Chairman of the Department of Psychiatry and Behavioral Sciences at the Univer- sity of Miami Miller School Of Medicine. He was past-president of the American Psychiatric Association and is the current pres- ident of the World Psychiatric Association. His contributions to this edition of Study Guide are immeasurable.
COMPREHENSIVE TEACHING SYSTEM
Study Guide forms one part of a comprehensive system developed by the authors to facilitate the teaching of psychiatry and the behavioral sciences. At the head of the system is the Comprehensive Textbook of Psychiatry, which is global in depth and scope; it is designed for and used by psychiatrists, behavioral scientists, and all other workers in the men- tal health field. Kaplan & Sadock’s Synopsis of Psychiatry is a relatively brief, highly modified, original, and current version useful for medical students, psychiatric residents, practicing psychiatrists, and other mental health professionals. A special edition of Synopsis, Concise Textbook of
Preface
Clinical Psychiatry covers just the diagnosis and treatment of all psy- chiatric disorders. Other parts of the system are the pocket handbooks: Pocket Handbook of Clinical Psychiatry, Pocket Handbook of Psychiatric Drug Treatment, Pocket Handbook of Emergency Psychiatric Medicine, and Pocket Handbook of Primary Care Psychiatry. These books cover the diagnosis and the treatment of mental disorders, psychopharmacol- ogy, psychiatric emergencies, and primary care psychiatry, respectively, and are compactly designed and concisely written to be carried in the pocket by clinical clerks and practicing physicians, whatever their spe- cialty, to provide a quick reference. Finally, Comprehensive Glossary of Psychiatry and Psychology provides simply written definitions for psy- chiatrists and other physicians, psychologists, students, and other mental health professionals. Together, these books create a multiple approach to teaching, studying, and learning of psychiatry.
HOW TO USE THIS BOOK
Each chapter begins with an introduction that emphasizes areas of special significance about which the student should be aware. The authors have also prepared lists of helpful hints that present key terms and concepts essential to a basic knowledge of psy- chiatry. Students should be able to define and discuss each of the terms in depth as preparation for examinations.
The section Objective Examinations in Psychiatry provides the student with helpful hints on how to take the examinations. If the student understands how questions are constructed, his or her chances of answering correctly are greatly improved. This book defines distracters (wrong answers) as well as correct answers in each discussion.
To use this book most effectively, the student should attempt to answer all the questions in a particular chapter. By allowing about 1 minute for each answer, the student can approximate the time constraints of an actual written examination. The answers should be verified by referring to the corresponding answer sec- tion in each chapter. Pay particular attention to the discussion of the wrong answers, a feature unique to this book. If further information is needed, the reader is referred to the current edi- tions of either the Synopsis of Psychiatry or the Comprehensive Textbook of Psychiatry.
ACKNOWLEDGEMENTS
The authors wish to thank Nitza Jones-Sepulveda, Project Editor of Synopsis and Comprehensive Textbook of Psychiatry, who assisted in the preparation of the book with the help of Sara Cain.
v
vi Preface
We especially acknowledge James Sadock, M.D., and Victoria Gregg, M.D., for their help in their areas of exper- tise, emergency adult and emergency pediatric medicine, respec- tively.
We also thank Ze’ev Levin, M.D. for his contributions to the previous edition of this book. We also want to thank Rana Zahiri and Mainek Patel who served as assistants to the editors and who made valuable contributions to the text in both content and organization.
Finally, we want to express our deep thanks to Charles Marmar, M.D., Chairman of Psychiatry at New York Univer- sity School of Medicine for his leadership of the Department and for his commitment to academic excellence.
B.J.S. V.A.S. P.R.
 
Preface v
. 1 NeuralSciences 1
. 2 NeuropsychiatryandBehavioral Neurology 9
. 3 ContributionsofthePsychosocial Sciences to Human Behavior 13
. 4 ClinicalNeuropsychological
Testing 25
. 5 TheoriesofPersonalityand Psychopathology 35
. 6 The Patient–Doctor Relationship
and the Psychiatric Interview 45
. 7 ClinicalExaminationofthe
Psychiatric Patient 55
. 8 Signs and Symptoms in Psychiatry 65
. 9 ClassificationinPsychiatryand Psychiatric Rating Scales 73
Contents
10 Delirium,Dementia,andAmnestic and Other Cognitive Disorders and Mental Disorders Due to a General Medical Condition 82
11 NeuropsychiatricAspectsofHIV Infection and AIDS 97
12 Substance-Related Disorders 104 13 Schizophrenia 117 14 Other Psychotic Disorders 132 15 Mood Disorders 142 16 Anxiety Disorders 156 17 Somatoform Disorders 168 18 Factitious Disorders 177 19 Dissociative Disorders 185
20 Human Sexuality 193 vii
viii Contents
21 Gender Identity Disorders
202 34
. 22 Eating Disorders 207
. 23 NormalSleepandSleep
Psychotherapies 281 35 Biological Therapies 288
Disorders
214 36
Child Psychiatry: Assessment, Examination, and Psychological Testing 300
. 24 Impulse-ControlDisorderNot Elsewhere Classified 224
. 25 Adjustment Disorders 230
. 26 Personality Disorders 235
. 27 Psychosomatic Medicine and Consultation-Liaison Psychiatry 241
. 28 NonconventionalApproachesin Mental Health Care 252
. 29 PsychiatryandReproductive
Medicine 255
. 30 Relational Problems 260
. 31 ProblemsRelatedtoAbuse
. 37 Mental Retardation 310
. 38 Learning Disorders 317
. 39 MotorSkillsDisorder:
Developmental Coordination Disorder 323
. 40 Communication Disorders 326
. 41 PervasiveDevelopmental
Disorders 331
. 42 Attention-Deficit Disorders 337
. 43 Disruptive Behavior Disorders 342
or Neglect
. 32 AdditionalConditionsThatMay Be a Focus of Clinical Attention
. 33 Emergency Psychiatric Medicine
264 44 269 45
274 46
Feeding and Eating Disorders
of Infancy or Early Childhood 346
Tic Disorders 351 Elimination Disorders 356
. 47 Other Disorders of Infancy, Childhood, or Adolescence 361
. 48 MoodDisordersandSuicidein Children and Adolescents 364
. 49 AnxietyDisordersofInfancy, Childhood, and Adolescence 367
. 50 Early-Onset Schizophrenia 372
. 51 Adolescent Substance Abuse 376
. 52 ChildPsychiatry:Additional Conditions That May Be a Focus
of Clinical Attention 381
. 53 PsychiatricTreatmentof
Children and Adolescents 384
54
55 56
57 58
Contents ix Forensic Issues in Child and
Adolescent Psychiatry 392 Geriatric Psychiatry 395
End-of-LifeCareand
Palliative Medicine 401
Forensic Psychiatry 408
Ethics in Psychiatry 418
Appendix A: Objective
Examinations in Psychiatry 424
Index 427
 
The human brain is responsible for cognitive abilities, emotions, and behaviors. During the “decade of the brain” in the 1990s, major advances in neural sciences took place, and the brain was finally recognized as the biological substrate for all normal and abnormal mental functions. It is possible to conceive a biolog- ically based diagnostic system for psychiatric disorders. This approach will also permit and advance brain oriented investiga- tional efforts to produce better psychiatric treatments and, thus, improve the quality of care of psychiatric patients. In most fields of medicine, diagnoses are based on physical signs, symptoms, a comprehensive medical history, and laboratory, radiological, and other relevant tests and procedures. In psychiatry, however, the diagnoses are based primarily on the clinical impression of the patient’s interpretation of his or her thoughts and feelings.
If the brain is the site of focus for psychiatric disorders, one should attempt to develop a classification system on the un- derstanding of biological factors rather than primarily patient’s symptoms. Neural sciences focus primarily on brain biology. It is, therefore, essential that one start to focus more intensively on the functions of the brain from a mental illness viewpoint. Be- sides understanding the functions and dysfunctions of lobal re- gions, basal ganglia, limbic structures, hypothalamus, and other relevant areas of the brain, one should understand the ultra struc- ture of individual brain cells. Of further importance are the synap- tic connectivity and the functional organization of the brain, as well as the behavioral consequences of pathological processes that take place in the central nervous system (CNS). At the same
1
Neural Sciences
time, the role of genetics is very relevant in this regard; par- ticularly, insofar as psychiatric disorders are concerned. Thus, knowledge about gene expression, DNA replication, messenger RNA synthesis and translation into protein, as well as the out- comes of mutations at each of these stages are quite relevant in this context.
Knowledge of clinical psychopharmacology is essential, in- cluding neurotransmitters, brain location of the biogenic amine neurotransmitter nuclei, and the distribution of the axonal pro- jections. The roles of glutamate, γ -aminobutiric acid (GABA), monoamine neurotransmitters, such as serotonin, dopamine, norepinephrine, epinephrine, histamine, and acetylcholine, as well as the peptide neurotransmitters such as endorphins and enkephalins, are all crucial to the understanding of the use of psychopharmacological agents.
Also of crucial importance is the knowledge of the major neuroimaging techniques, as well as the clinical limi- tations of these neuroimaging techniques. They include mag- netic resonance imaging (MRI), computed tomography (CT), magnetic resonance spectroscopy (MRS), single photon emis- sion computed tomography (SPECT), proton emission tomog- raphy (PET), electroencephalography (EEG), and magnetoen- cephalography (MEG), as well as others.
Without question, medical students and psychiatric physi- cians need to be familiar and have knowledge about the field of neuroscience. The following questions and answers will permit them to assess their knowledge in this regard.
  
HELPFUL HINTS
The student should know the following terms, theoreticians, and concepts.
Acetylcholine Animacy
Apoptosis
Ascending pathways Broca’s area Catecholamines Caudate nucleus Chronobiology Circadian Rhythm Decoding emotions Deep brain stimulation
(DBS)
Dopamine Electroconvulsive
therapy (ECT) Endophenotypes Epigenetics Genetic factors in
cognition, temperament and personality
Genome Globus pallidus Glutamic acid
Histamine
Limbic system Locus ceruleus Metabolic syndrome Neuropeptides Neurotropic
factors Nitric Oxide
Norephinephrine and Epinephrine
Population genetics Putamen
Receptors
REM and NREM
sleep
Repetitive transcranial
magnetic stimulation
(RTMS) Seasonal affective
disorder and
circadian rhythm Self
Serotonin Substania Nigra
1
▲▲▲ ▲ ▲▲▲
▲▲▲▲▲▲ ▲▲ ▲▲
▲▲ ▲▲▲ ▲▲▲
▲▲▲▲▲▲▲▲▲▲▲
2 1. Neural Sciences Subthalamic nucleus
Syndromes of pain Transcriptome
QUESTIONS
Directions
Transporters
Vagal nerve stimulation
(VNS)
Wakefullness
γ -aminobutyric acid
(GABA)
1.6. The neuropeptides are primarily related to which of the following central nervous systems?
A. Serotonin neurotransmitter system
B. Norepinephrine and Epinephrine system C. Acetylcholine system
D. Hypothalamic regulation system
E. Catecholamines system
1.7. The strongest evidence for a role for neurotrophins in psy- chiatric diseases has come from the pathophysiology of which of the following psychiatric disorders?
A. Schizophrenia
B. Panic disorder
C. Major depressive disorder
D. Obsessive-compulsive disorder E. Antisocial personality disorder
1.8. Manipulation of one of the novel neurotransmitters, nitric oxide, is thought to have a therapeutic effect in which of the following psychiatric conditions?
A. Mood disorders
B. Addictive disorders C. Anxiety disorders
D. Personality disorders E. Dissociative disorders
1.9. The potential role of homeostatic neuronal plasticity as a therapeutic mechanism in certain psychiatric disorders is currently explored with the use of which of the following treatments?
A. Electroconvulsive therapy (ECT)
B. Vagal nerve stimulation (VNS)
C. Repetitive transcranial magnetic stimulation (RTMS) D. Deep brain stimulation (DBS)
E. All of the above
1.10. Deoxyribonucleic acid (DNA) is composed of which of the following nucleic acids (nucleotides)?
A. Adenine
B. Cystosine
C. Guanine
D. Thymine
E. All of the above
1.11. Risk factors associated with the metabolic syndrome in- clude
A. hyperglycemia B. visceral obesity C. hypertension D. hyperlipidemia E. all of the above
  
Each of the questions or incomplete statements below is followed by five suggested responses or completions. Select the one that is best in each case.
. 1.1. Thenatureofanendophenotypeisbiologicallydefinedon which of the following?
A. Neuropsychological B. Cognitive
C. Neurophysiological D. Biochemical
E. All of the above
. 1.2. Which of the following morphological regions is a part of the neuron?
A. Cell body
B. Dendrites
C. Axon
D. Axon terminals E. All of the above
. 1.3. A reduced density of interneurons in layer 2 of the pre- frontal cortex has been observed among patients with
A. Bipolar disorder
B. Major depression
C. Obsessive-compulsive disorder D. Schizophrenia
E. Panic disorder
. 1.4. The cell bodies of the serotonergic neurons are located in what region of the brain?
A. Midline raphe nuclei of the brainstem
B. Midbrain substania nigra and ventral tegmental area
C. Locusceruleusandthelateraltegmentalnoradrenergic
nuclei
D. Thetuberomamillarynucleusoftheposteriorhypotha-
lamus
E. The basal forebrain complex and the mesopontine
complex
. 1.5. The primary inhibitory neurotransmitter, γ -aminobutyric acid (GABA), in the brain is notably depleted in which of the following neuropsychiatric disorders?
A. Pick’s disease
B. Vascular dementia
C. Creutzfeldt-Jakob disease
D. Huntington’s disease
E. Normal pressure hydrocephalus
▲▲
▲▲
▲▲▲
. 1.12. The neuropsychiatric disorder that has been best charac- terized in terms of the influence of the brain on the immune system and vice versa is which one of the following?
A. Somatization disorder
B. Hypochrondriasis
C. Factitious disorder
D. Major depressive disorder E. Depersonalization disorder
. 1.13. Circadian rhythms include
A. sleep
B. temperature
C. hormone levels D. eating
E. all of the above
. 1.14. Research efforts have demonstrated that there are no elec- troencephalographic (EEG) abnormalities associated with the use of which psychopharmacological agent?
A. Olanzapine
B. Clozapine
C. Risperidone D. Pherphenazine E. Quetiapine
. 1.15. Structural MRI findings associated with schizophrenia in- clude all of the following except
A. reductions in cortical gray matter
B. reductions in cortical white matter
C. risk genes influencing MRI findings
D. progressive deviation of regional cortical volumes in
childhood onset cases
E. decreased striatum volume
. 1.16. Recent research has provided a basis for clinical indi- cations of PET and SPECT radiotracer imaging for the diagnosis and management of several neuropsychiatric disorders. Among them, we found all of the following except
A. movement disorders B. schizophrenia
C. mood disorders
D. anxiety disorders
E. antisocial personality disorder
. 1.17. Populations genetics encompasses all of the following
except
A. quantitative genetics B. genetic epidemiology C. genetic demography D. molecular genetics
E. evolutionary genetics
. 1.18. The most successful application of gene-mapping strate- gies relates to which of the following psychiatric disor- ders?
A. Alzheimer’s disease B. Bipolar disorder
C. Schizophrenia
D. Panic disorder
E. Obsessive-compulsive disorder
1.19. Animal models in psychiatric research have proven to be useful in which one of the following disorders?
A. Anxiety disorders
B. Depressive disorders
C. Substance abuse disorders D. Eating disorders
E. All of the above
1.20. Whichofthefollowingconditionscombinessymptomsof pain and psychiatric signs and symptoms?
A. Fibromyalgia
B. Migraine
C. Irritable bowel syndrome D. Chronic fatigue syndrome E. All of the above
1.21. The concept that animacy is a primary milestone of social perception can be demonstrated by which of the following observations?
A. At birth, infants preferentially track moving human faces.
B. At 3 months, infants smile and vocalize more to people than objects.
C. At 3 months, infants show preferential attention to self- propelled motion.
D. At 9 months, infants understand that animate beings, not objects, have goal-directed action.
E. All of the above
1.22. The Self as a subject (the I-Self) includes all of the fol-
lowing except
A. self-awareness B. self-continuity C. self-coherence D. me-self
E. sense of ownership
1.23. Cells that decrease their firing during NREM sleep and cease firing altogether during REM sleep are
A. nonadrenegeric cells B. cholinergic cells
C. histaminergic cells D. dopaminergic cells E. all of the above
1.24. Adequate stimuli for the identification of food and initia- tion of eating in adults include which of the following?
A. Olfactory stimuli B. Temporal stimuli
1. Neural Sciences 3
4 1. Neural Sciences
C. Cognitive stimuli D. Social stimuli
E. All of the above
1.25. Primary reinforcement by drugs of abuse engages a widespread network of the brain’s motivational pathways, which include all of the following except
A. amygdala
B. hippocampus
C. hypothalamus D. prefrontal cortex E. occipital cortex
Directions
Each group of questions consists of lettered headings followed by a list of numbered statements. For each numbered phrase or statement, select the one lettered heading that is most closely associated with it. Each lettered heading may be selected once, more than once, or not at all.
Questions 1.26–1.31
A. Computed Tomography Scan (CT)
B. Functional Magnetic Resonance Imaging (fMRI) C. Magnetic Resonance Imaging Scan (MRI)
D. Magnetic Resonance Spectroscopy (MRS)
E. Positron Emission Tomography Scan (PET)
F. Single Photon Emission Computed Scan (SPECT)
. 1.26. Used to study levels of lithium concentration in the brain
. 1.27. Cannot be used in patients with pacemakers or ferromag-
netic metals
. 1.28. Radioactive compounds are used to study dopamine and
serotonin receptors
. 1.29. Detects neuronal activity by measuring brain blood flow
using a glucose analog
. 1.30. Uses fluorine-18 to study brain structures and to detect
neuropsychiatric disorders
. 1.31. Certain tumors may be invisible because they absorb as
much radiation as the surrounding brain
ANSWERS
1.1. The answer is E (all)
The nature of an endophenotype is biologically defined on the basis of neuropsychological, cognitive, neurophysiological, neu- roanatomical, biochemical, and brain data. Endophenotype is an internal phenotype, which is a set of objective characteris- tics of an individual that are not visible to the unaided eye. A given phenotype would not be limited to a patient with a partic- ular diagnosis; for instance, schizophrenia might also be found in patients with other diagnoses, such as depression or bipolar disorder.
1.2. The answer is E (all)
The human brain contains approximately 100 billion nerve cells or neurons. In general, neurons are composed of four morpho-
logically identified regions: (1) the cell body or soma, which contains the nucleus and can be considered the metabolic center of the neuron; (2) the dendrites, which are processes that arise from the cell body, branch extensively, and serve as the major re- cipient zones of input from other neurons; (3) the axon, which is a single process that arises from a specialized portion of the cell body (the axon hillock) and conveys information to other neu- rons; and (4) the axon terminals, which are fine branches near the end of the axon and that form contacts (synapses) generally with the dendrites or the cell bodies of other neurons, release neurotransmitters, and provide a mechanism for interneuronal communication. Most neurons in the human brain are consid- ered to be multipolar in that they give rise to a single axon and several dendritic processes.
1.3. The answer is D
Research has shown that in a significant number of patients with schizophrenia, the prefrontal cortex exhibits a reduced density of interneurons in layer 2. Additionally, these patients show an upregulation of GABAA receptor binding, a potential functional compensation, as well as a relative deficiency of nitric oxide syn- thase (NOS)-expressing neurons. These observations have led to the hypothesis that schizophrenia is due to reduced GABAer- gic activity. The origin of GABA interneurons from the gan- glionic eminences and their association with specific patterning genes raises new genetic models of disease causation and possi- ble strategies for disease intervention.
1.4. The answer is A
The cell bodies of the serotonergic neurons are located in the midline raphe nuclei of the brainstem. The dopamine neurons are located in the midbrain substantia nigra and the ventral tegmental areas as well as in the periaqueductal gray, hypotha- lamus, olfactory bulb, retina, and kidney. The norepinephrine and epinephrine producing neurons are found in the pons and medulla in two major clusters: the locus ceruleus and the lateral tegmental noradrenergic nuclei. These neurons are also found in the adrenal medulla. Histaminergic cell bodies are located within the region of the posterior hypothalamus termed the tuberomam- millary nucleus.
1.5. The answer is D
This degeneration characteristically results in a depletion of GABA, the brain’s major inhibitory neurotransmitter, and acetylcholine. Huntington’s disease is an autosomal-dominant neurodegenerative disorder characterized by cognitive and phys- ical decline. Its etiology involves an abnormal expansion of a trinucleotide repeat on chromosome 4. MRI classically reveals bilateral atrophy of the caudate nucleus and putamen of the basal ganglia. CT scan further reveals a prominence of the lateral ven- tricles as a result of surrounding atrophy. Huntington’s disease has a gradual onset between the ages of 30 to 50 years. Key features of the disease include progressive subcortical dementia, chorea (rapid, involuntary, dance-like movements), depression, and psychosis. Dysfunction of GABAergic neurotransmission has further been implicated in anxiety disorders, schizophrenia, alcohol dependence, and seizure disorders. Pick’s disease is due to an accumulation of tau proteins, which lead to degeneration
of the frontal and temporal lobes. Onset of Pick’s disease is between the ages of 40 to 60 years and initial signs include per- sonality change, language impairment, and memory loss. Vas- cular or multi-infarct dementia presents acutely with cognitive decline following a cerebrovascular event and has a stepwise progression. Creutzfeldt-Jakob disease is a rare spongiform en- cephalopathy accompanied by rapidly progressive dementia and hallucinations. It is caused by an accumulation of prions leading to nerve cell death. Normal-pressure hydrocephalus clinically presents with the triad of dementia, incontinence, and gait dis- turbance. It is due to poor reabsorption of CSF and characterized by widening of the lateral ventricles.
1.6. The answer is D
Neuropeptides represent the most diverse class of signaling molecules in the CNS. They have a role in the hypothalamic regulation of pituitary hormone secretion. They also have an array of direct or neuromodulatory effects, ranging from modu- lating neurotransmitter release and neuronal firing patterns to the regulation of emotionality and complex behavior. More than 100 unique biologically active neuropeptides have been identified in the brain, a subset of which is presented in Table 1.1.
Table 1.1
Selected Neuropeptide Transmitters
Adrenocorticotropin hormone (ACTH) Angiotensin
Atrial natriuretic peptide
Bombesin
Calcitonin
Calcitonin gene-related peptide (CGRP)
Cocaine and amphetamine regulated transcript (CART) Cholecystokinin (CCK)
Corticotropin-releasing factor (CRF)
Dynorphin
β –Endorphin
Leu-enkephalin
Met-enkephalin
Galanin
Gastrin
Gonadotropin-releasing hormone (GnRH)
Growth hormone
Growth hormone-releasing hormone (GHRH; GRF) Insulin
Motilin
Neuropeptide S
Neuropeptide Y (NPY)
Neurotensin
Neuromedin N
Orphanin FQ/Nociceptin
Orexin
Oxytocin
Pancreatic polypeptide
Prolactin
Secretin
Somatostatin (SS; SRIF)
Substance K
Substance P
Thyrotropin-releasing hormone (TRH)
Urocortin (1, 2, and 3)
Vasoactive intestinal polypeptide (VIP)
Vasopressin (AVP; ADH)
1.7. The answer is C
The strongest evidence for a role for neurotrophins among psychiatric disorders has come from the pathophysiology of depression, especially depression associated with stress. For depression, it is believed that there is a fundamental dysregula- tion of synaptic plasticity and neuronal survival in regions of the brain, such as the hippocampus. In animal models, restraint stress leads to a decreased expression of the brain-derived neurotrophic factor (BDNF) in the hippocampus. In addition, chronic physical or psychosocial stress leads to atrophy and death of hippocam- pus neurons, especially in the CA3 region in rodents and pri- mates. Also, MRI studies have shown that patients with depres- sive or post-traumatic stress disorders exhibit a small decrease in hippocampal volume. It is unclear though, whether the atrophy and/or death of these neurons is directly related to the decreased availability of BDNF.
1.8. The answer is A
Recently, it was discovered that gases can function as neuro- transmitters. In this context, it was found that Nitric Oxide has a neurotransmitter property vis-a`-vis a few psychiatric disorders. Mood Disorders is one of the groups of psychiatric illnesses in which Nitric Oxide has a positive role as a neurotransmit- ter. NOS-expressing neurons are well represented in areas im- plicated in depression, including the dorsal raphe nucleus and prefrontal cortex. A role for Nitric Oxide has been suggested in antidepressant response as selective serotonin reuptake inhibitor (SSRI) antidepressants can directly inhibit NOS activity.
1.9. The answer is E (all)
In recent years, there has been increasing interest in the use of brain stimulation methods as treatment for psychiatric and neurological disorders. These methods include electroconvul- sive therapy (ECT), Vagal Nerve Stimulation (VNS), repetitive transcranial magnetic stimulation (RTMS), and deep brain stim- ulation (DBS). The development of optimal stimulation param- eters for these treatments requires knowledge about the effects of electrical stimulation on neuronal function. With respect to ECT, a major advance has been the recognition that electrical stimulation parameters play a key role in determining therapeu- tic and adverse effects. There is compelling evidence that the degree to which electrical doses exceed the seizure threshold is of substantial importance. For bilateral ECT, electrical doses just above threshold (approximately 1.5 times threshold) result in a highly effective form of treatment that minimizes cogni- tive impairment. For nondominant hemisphere (unilateral) ECT, electrical doses that are five to six times threshold are required to produce a significant benefit.
1.10. The answer is E (all)
DNA (deoxyribonucleic acid) is made of four nucleic acids, also known as nucleotides: adenine, cytosine, guanine, and thymine. A genome is defined as the total complement of DNA replicated in a living organism. A critical milestone was reached in 2001 with the completion of the first draft of the human genome. Currently, there are thousands of genomes sequenced. It is now evident that there are in total about 25,000 protein-coding genes in Homo sapiens.
1. Neural Sciences 5
  
6 1. Neural Sciences
1.11. The answer is E (all)
Metabolic syndrome is a disease characterized by a cluster of metabolic risk factors, which include hyperglycemia, vis- ceral obesity, hyperlipidemia, and hypertension. Most notably, adverse effects of atypical antipsychotics, such as olanzapine (Zyprexa) and clozapine (Clozaril), include weight gain and hyperglycemia. However, aripiprazol (Abilify) and ziprasidone (Geodon) are atypical antipsychotics that have minimal effects in this regard. Schizophrenia patients may exhibit increased cortisol and epinephrine production even when not medicated; therefore, monitoring protocols for patients on atypical antipsychotics is of utmost importance. Of note, metabolic syndrome also leads to insulin resistance and complicates glycemic control in patients with preexisting diabetes.
1.12. The answer is D
The neuropsychiatric disorder that has been best characterized in terms of the influence of the brain on the immune system and vice versa is major depressive disorder. For many years, major depressive disorder was seen as a quintessential example of how stress-related disorders may decrease immunocompe- tence. More recently, however, it has become evident that stress also activates inflammatory pathways, even while suppressing measures of acquired immunity.
1.13. The answer is E (all)
The circadian clock drives many rhythms including behavior, core body temperature, sleep, eating, drinking, and hormonal levels. One such circadian-regulated hormone is the indolamine, melatonin. Melatonin synthesis is controlled through a multisy- naptic pathway from the CNS to the pineal gland. Serum levels of melatonin become elevated at night and return to baseline during the day. Light suppresses elevated melatonin levels, im- mediately decreasing them to baseline levels. Light also shifts the phase of circadian rhythms of melatonin synthesis. Because melatonin can be assayed easily, it provides a convenient win- dow into the state of the circadian pacemaker. Any perturbation of the clock is reflected in the melatonin profile; thus, melatonin offers an output that can be used to study the regulation of the central circadian pacemaker.
1.14. The answer is E
Electroencephalographic (EEG) abnormalities have been re- ported with the use of clozapine (47 percent), olanzapine (38.5 percent), trifluoperazine and mesoridazine (about 35 per- cent), risperidone (28 percent), fluphenazine and thiothixene (just above 20 percent), pherphenazine, chlorpromazine and thioridazine (just about 10 percent), and haloperidol (just below 10 percent). There were, however, no abnormalities observed with quetiapine or loxapine. The clinical significance of EEG abnormalities associated with psychopharmacological agents, particularly in the absence of any indications of seizures or en- cephalopathic effects, remain an open research question.
1.15. The answer is E
Structural MRI finding associated with schizophrenia do not in- clude decreased striatum volume. Structural MRI has been useful
for characterizing features of the heritable risk for schizophre- nia. Family members of individuals with schizophrenia show a pattern of reductions in cortical gray and white matter vol- ume that resembles, but is milder than, that associated with schizophrenia. Among healthy individuals and patients diag- nosed with schizophrenia, risk genes appear to influence MRI findings. Structural MRI has also provided some insights into the progressive course of this disorder and the impact of an- tipsychotic treatment. The most striking findings have been in childhood onset schizophrenia, where a series of neuroimaging studies using a variety of MRI approaches have clearly shown that the development and evolution of schizophrenia are asso- ciated with a progressive deviation of regional cortical volumes from those of healthy comparison populations.
1.16. The answer is E
Over the past two decades, radio tracer imaging with PET and SPECT have gained merit as tools to image brain functioning and neurochemistry in living humans and have provided the founda- tion necessary to begin to identify the neurochemical signatures and neuropsychiatric disorders that result from abnormal brain chemistry. Radio tracer imaging can also help to assess the rela- tionship between occupancy of specific various receptors in the brain and clinical efficacies of various psychotropic drugs. For instance, imaging of dopamine Dz receptors provides critical in- formation for the differential diagnosis of movement disorders and schizophrenia, and also for the assessment of receptor oc- cupancy by neuroleptic drugs. Imaging of serotonin receptors and the serotonin transporter is useful in the diagnosis of mood and anxiety disorders, as well as the assessment of antidepressant efficacy. Imaging of nicotinic acetylcholine receptors and acetyl- cholinesterase may serve as markers of cognitive and memory impairment.
1.17. The answer is D
Population genetics, which deals with the mathematical proper- ties of genetic transmission in families and populations, can be subdivided into the partially overlapping fields of evolutionary genetics, genetic demography, quantitative genetics, and genetic epidemiology. The primary goal of evolutionary genetics is to un- derstand changes in gene frequency across generations. Genetic demography is primarily concerned with differential mortality and fertility in human populations, while genetic epidemiology deals with the distribution of disease-associated genes across human subpopulations. The goal of quantitative genetics is to partition the observed variation of phenotypes into its genetic and environmental components.
1.18. The answer is A
The most successful application of gene-mapping strategies ap- plies to Alzheimer’s disease, which is the most common form of dementia and is characterized by a progressive decline in memory, aphasia, apraxia, agnosia, and diminished excessive functioning. Abnormal deposition of beta-amyloid protein and neurofibrillary tangles are the major pathophysiological charac- teristics of Alzheimer’s disease. Associated genes include auto- somal dominant forms of the APOE4 allele on chromosomes 1,
14, 19, and a mutation of the P-App gene (amyloid precursor protein gene) on chromosome 21. Genetic alterations may be re- sponsible for up to 10 percent of cases of early onset forms of the disease, often diagnosed between the ages of 30 to 60 years. Ge- netic testing for APOE is currently available to assess whether an individual carries such a risk factor, but cannot predict definitive development of the disease.
1.19. The answer is E (all)
Animal models provide insight into biological mechanisms un- derlying psychiatric disease pathophysiology and treatment. The susceptibility to psychiatric disorders is recognized to result from highly complex interactions between genetic endowment and the cumulative effects of innumerable environmental influences. The elucidation of the human genome provides unprecedented opportunities to understand genetic determinants of behavioral traits and psychiatric disease susceptibility. However, the behav- ioral impact of genetic manipulations cannot be systematically studied in humans, nor can the effects of many types of environ- mental stimuli and stressors.
1.20. The answer is E (all)
Fibromyalgia, migraine, irritable bowel syndrome, and chronic fatigue syndrome are some of the most commonly recognized conditions in which pain is associated with psychiatric symp- toms. Fibromyalgia is a chronic nonprogressive disease in which patients will present with at least 11 musculoskeletal tender points. Depression and anxiety are frequently diagnosed in pa- tients suffering from fibromyalgia. Migraines are severe unilat- eral headaches that often present with throbbing, nausea, vom- iting, and photophobia. The prodomal phase of a migraine may include depression, hypersomnia, irritability, and fatigue. Irrita- ble bowel syndrome is an idiopathic gastrointestinal tract disor- der in which symptoms of alternating diarrhea and constipation are commonly preceded by anxiety and depression. Chronic fa- tigue syndrome is a condition in which patients present with 6 months or more of fatigue not alleviated by rest, compromise of short-term memory and concentration, insomnia, depression, and musculoskeletal pain.
1.21. The answer is E (all)
Animacy is the inability of the brain to quickly and efficiently differentiate animate from inanimate objects. The first thing a brain must do in any healthy social interaction is detect animacy. Psychophysical research has shown that our attentional and per- ceptual systems are uniquely tuned for detecting animate things versus other object types. Time-consuming mental calculations would be wasted attempting to predict the thoughts, feelings, and actions of objects that could not think, feel, or act.
1.22. The answer is D
William James made a distinction between two separable but intimately interrelated aspects of the “self.” The “self” as a subject is known as the I-Self and “self” as object is known as the Me-Self. He also identified subcomponents within them. Components of the I-Self include Self-agency (sense of owner- ship), self-awareness, self-continuity, and self-coherence. The
1. Neural Sciences 7 Me-Self included “material me,” “social me,” and “spiritual
me.”
1.23. The answer is A
Noradrenergic cells from the locus coeruleus project directly throughout the forebrain and cortex and show their highest dis- charge rates during wakefulness. These cells decrease their fir- ing during NREM sleep and cease firing altogether during REM sleep.
1.24. The answer is E (all)
Adequate stimuli for the identification of food and initiation of eating in adults comprise a bewildering farrago of olfac- tory, visual, auditory, temporal, circadian, metabolic, cogni- tive, and social stimuli. Most of them are conditioned stimuli whose potency depends on individual experience from infancy onward.
1.25. The answer is E
While the mesolimbic dopamine pathway is critical, it must be noted that primary reinforcement of drugs of abuse engages a widespread network of the brain’s motivational pathways, in- cluding cortical regions and limbic structures such as the pre- frontal cortex, amygdala, hippocampus, and hypothalamus.
Answers 1.26–1.31 1.26. The answer is D 1.27. The answer is C 1.28. The answer is F 1.29. The answer is B 1.30. The answer is E
1.31. The answer is A
Computed tomography (CT) scanners are currently the most widely available and convenient imaging tools available in clin- ical practice. They effectively take a serious of head X-ray pic- tures from all vantage points, 360 degrees around a patient’s head. CT scanners allow assessment of structural brain lesions such as tumors or strokes. However, since CT images are deter- mined only by the degree to which tissues absorb X-irradiation, certain tumors may be invisible on CT because they absorb as much radiation as the surrounding brain.
Magnetic resonance imaging (MRI) scans, however, do not rely on the absorption of X-rays. MRI is based on nuclear mag- netic resonance (NMR). MRI scanners collect the emissions of individual, realigning nuclei and use computer analysis to generate a series of two-dimensional images that represent the brain. MRI scans cannot be used for patients with pacemakers or implants of ferromagnetic metals due to the strong magnetic fields created during the MRI. The resolution of brain tissue of even the lowest power MRI scan exceeds that of CT scanning (Figure 1.1).
8 1. Neural Sciences

ABC
FIGURE 1.1
Comparison of CT and MRI. A. Computed tomography (CT) scan in the axial plane at the level of the third ventricle. The cerebrospinal fluid (CSF) within the ventricle appears black, the brain tissue appears gray, and the skull appears white. There is very poor discrimination between the gray and white matter of the brain. The arrow indicates a small calcified lesion in a tumor of the pineal gland. Detection of calcification is one role in which CT is superior to magnetic resonance imaging (MRI). B. T2-weighted image of the same patient at roughly the same level. With T2, the CSF appears white, the gray matter appears gray, the white matter is clearly distinguished from the gray matter; the skull and indicated calcification appear black. Much more detail of the brain is visible than with the CT. C. T1-weighted image of the same patient at roughly the same level. With T1, the CSF appears dark, the brain appears more uniformly gray, and the skull and indicated calcification appear black. T1 MRI images are the most similar to CT images. (Reprinted with permission from Grossman CB. Magnetic Resonance Imaging and Computed Tomography of the Head and Spine. 2nd ed. Baltimore: Williams & Wilkins; 1996:101.)
Whereas routine MRI detects hydrogen nuclei to determine brain structure, magnetic resonance spectroscopy (MRS) can de- tect several odd-number nuclei. The ability of MRS to detect a wide range of biologically important nuclei permits the use of the technique to study many metabolic processes. MRS can be used to measure concentrations for psychotherapeutic drugs in the brain. One study used MRS to measure lithium concentra- tions in the brains of patients with bipolar disorder and found that lithium concentrations in the brain were half those in the plasma during depressed and euthymic periods but exceeded those in the plasma during manic episodes.
Functional magnetic resonance imaging (fMRI) detects neu- ronal activity by measuring brain blood flow using a glucose analog. Neuronal activity within the brain causes a local increase in blood flow, which in turn increases the local hemoglobin con- centration. Functional MRI is useful to localize neuronal ac- tivity to a particular lobe or subcortical nucleus and has even been able to localize activity to a single gyrus. Functional MRI is widely used to study brain abnormality related to cognitive dysfunction.
Single photon emission computed tomography (SPECT)
scans also provide information on the cerebral blood flow, but un- like fMRIs, they do not measure neuronal metabolism directly. In SPECT, manufactured radioactive compounds are used to study
regional difference in cerebral blood flow within the brain. It records the pattern of photon emission from the bloodstream ac- cording to the level of perfusion in different regions of the brain. SPECT uses compounds labeled with single photon-emitting iso- topes: iodine-123, technetium-99m, and xenon-133. In addition to these compounds used for measuring blood flow, iodine-123- labeled ligands for the muscarinic, dopaminergic, and seroton- ergic receptor can be used to study these receptors by SPECT.
The isotopes used in positron emission tomography (PET) decay by emitting positrons, antimatter particles that bind with and annihilate electrons, thereby giving off photons that travel in 180-degree opposite directions. Because detectors have twice as much signal from which to generate an image as SPECT scan- ners have, the resolution of the PET image is higher. A wide range of compounds can be used in PET studies, and the res- olution of PET continues to be refined closer to its theoretical minimum of 3 mm, which is the distance positrons move be- fore colliding with an electron. The most commonly used iso- topes in PET are fluorine-18, nitrogen-13, and oxygen-15. These isotopes are usually linked to another molecule, except in the case of oxygen-15. PET has been used increasingly to study normal brain development. Relatively few PET scanners are available because they require an on-site cyclotron to make the isotopes.

2
Neuropsychiatry and Behavioral Neurology

Psychiatry discontinued the term “organic” from the official nomenclature a couple of decades ago, but its significance is still relevant today because the care of patients with identifi- able, acquired brain disease (such as epilepsy, movement disor- ders, and traumatic brain injury) requires the physician to have expertise, knowledge base, and a familiarity with assessment and treatment methods not usually required for patients with primary psychiatric disorders. Patients with organic psychiatric syndromes are common in clinical practice and are often difficult to manage for most general psychiatrists, even with consultation from other specialists who may themselves not be experts in the mental and emotional phenomena, which often accompany brain disease.
Neuropsychiatry is the psychiatric subspecialty that deals with the psychological and behavioral manifestations of brain disorders. Neuropsychiatry is, therefore, closely allied with cognitive and behavioral neurology–that is, the neurological specialty that focuses on psychological phenomena present in patients with brain disorders. In many ways, neuropsychiatry can offer a distinctive perspective on idiopathic psychiatric dis- orders; its limitations in this respect must also be noted. The limitations include consideration of the multifactorial nature of many psychiatric diseases–namely, interaction between environ- mental and genetic factors. To date, no psychiatric disorder has been completely mapped. The unraveling of a biological basis for many psychiatric diseases is a novel aspect of the field, which is in its inception phase.
HELPFUL HINTS
The student should know the following terms, theoreticians, and concepts.
Broca’s area
Brain Circuits Demyelinating disorders
Fibromyalgia Hemispheric
lateralization
HIV-assisted dementia Human prion diseases Limbic system
Movement disorders Neurometabolic
syndromes
QUESTIONS
Directions
Each of the questions or incomplete statements below is followed by five suggested responses. Select the one that is best in each question.
2.1. Disinhibition is associated with which of the following brain areas or localizations?
A. Left frontal lobe B. Third frontal gyrus C. Right frontal lobe D. Left hemisphere E. Left limbic area
2.2. The most common mental disorder associated with cere- brovascular disease is
A. Mania
B. Anxiety
C. Psychosis D. Alcoholism E. Depression
2.3. The most common site of brain tumors is
A. Frontal lobes
B. Occiput lobes
C. Diencephalic regions D. Pituitary gland
E. Parietal lobes
9
▲▲
▲▲▲
▲▲
▲▲▲
10 2. Neuropsychiatry and Behavioral Neurology
. 2.4. Epilepsy patients are prone to have which of the following
psychiatric conditions?
A. Psychosis
B. Depression
C. Personality Disorders D. Hyposexuality
E. All of the above
. 2.5. Which of the following is the single greatest risk factor for traumatic brain injury (TBI)?
A. Male gender
B. Alcohol or drug abuse
C. African American ethnicity D. Low socioeconomic status E. None of the above
. 2.6. Which of the following movement disorders is present in patients with tic disorders?
A. Bradykinesia B. Hyperkinesia C. Hypokinesia D. Akinesia
E. Ataxia
. 2.7. Which of the following is the most common demyelinating disorder?
A. Tay-Sachs disease
B. Hurler’s syndrome
C. Acquired immunodeficiency syndrome (AIDS) D. Multiple sclerosis
E. Adrenoleukodystrophy
2.11. Psychopharmacological agents that appear to help patients with fibromyalgia include
A. Pregabalin B. Duloxetine C. SSRIs
D. SNRIs
E. All of the above
2.12. Prematurity is associated with which of the following psy- chiatric conditions?
A. Anxiety
B. Aggression
C. Schizophrenia D. Conduct disorder E. All of the above
2.13. A 32-year-old man presents to the emergency department after suffering a seizure earlier in the day. His history reveals 3 days of mild headache and fever, but is oth- erwise unremarkable. The patient’s cerebral spinal fluid reveals 91 percent lymphocytes and elevated protein. CT of his head is unremarkable. What is the most likely diagnosis?
A. Meningioma
B. Herpes simplex encephalitis C. Subarachnoid hemorrhage D. Intracerebral hemorrhage
E. Cryptococcal meningitis
ANSWERS
2.1. The answer is C
Although left lateralization of language and right lateralization of visuospatial function are widely recognized, lateral special- ization in the prefrontal region is less obvious but still of clinical significance. Frontal lobe degeneration involving the right more than the left frontal lobe is particularly associated with disinhibi- tion. Women tend to show less lateralization of language, so left hemisphere lesions are less likely to produce severe impairment.
2.2. The answer is E
Depressive disorders are probably the most common emotional disorder associated with cerebrovascular disease. The prevalence depends upon whether community based samples or hospitalized patients are examined or whether patients with acute stroke or those with chronic stroke are evaluated. Mania and generalized anxiety disorder (GAD) occur much less frequently than depres- sion following stroke.
2.3. The answer is A
The most common sites of brain tumors are the frontal and tem- poral lobes. The least common sites are the occipital lobes, dien- cephalic regions, pituitary gland, and parietal and infratentorial areas. Brain tumors are slightly more common in men than in women.
. 2.8. Which of the following infectious agents have been as- sociated with the onset of obsessive-compulsive disorder (OCD)?
A. Streptococcal infection B. Borrelia Burgdorferi C. Retrovirus
D. Influenza virus
E. Varicella zoster virus
. 2.9. Which of the following are human prion diseases?
A. Sporadic Creutzfeld-Jakob disease (sCJD)
B. Fatal familial insomnia (FFI)
C. Gerstmann-Straussler-Scheinker (GSS) disease D. Kuru
E. All of the above
. 2.10. The diagnosis of a primary headache disorder requires the exclusion of which condition?
A. Vascular malformation B. Bacterial meningitis C. Diabetes mellitus
D. Pseudotumor cerebri E. All of the above
2.4. The answer is E (all)
Epidemiological studies from communities, psychiatric hospi- tals, and epilepsy clinics report a 20 to 60 percent prevalence of psychiatric problems among epilepsy patients. Epilepsy patients are prone to psychosis, depression, personality disorders, hypo- sexuality, and other behavioral disorders. These problems are approximately equally divided between those that occur ictally or peri-ictally and those that occur interictally or are variably related to the ictus. The percentage of epilepsy patients in psy- chiatric hospitals was also higher than the general prevalence of epilepsy and ranged from 4.7 percent of all inpatients in a British psychiatric hospital to 9.7 percent in a U.S. Veterans Affairs psy- chiatric facility.
2.5. The answer is B
The single greatest risk for traumatic brain injury (TBI) is alco- hol and drug abuse. Epidemiological studies have reported that close to one-third of brain injury patients had identifiable alco- holism before trauma. Of note, however, males are two to three times more likely to suffer traumatic brain injuries than females. African Americans also have higher rates of traumatic brain in- juries, a finding that may be explained by increased firearms ex- posure. Lower socioeconomic status, especially unemployment, and lower levels of education also correlate with a higher inci- dence of TBI. The risk of experiencing a second brain injury is three times greater after a single previous brain injury.
2.6. The answer is B
Tics are repeated, uncontrolled, involuntary muscular contrac- tions or vocalizations. They frequently occur in children and are more common in boys due to genetic predisposition. Motor tics result in stereotyped actions that can be voluntarily suppressed for brief periods of time. They are characterized by hyperki- nesia or excessive motility. Bradykinesia is defined as slowed movement. Hypokinesia is a decrease in movement, while aki- nesia refers to an inability to initiate movement. Bradykinesia, hypokinesia, and akinesia are all features of Parkinson’s dis- ease. Ataxia refers to a lack in coordination during voluntary movement, a common feature of cerebellar and posterior column lesions. Of note, Tourette’s syndrome is a disorder that includes both motor and vocal tics with an onset before the age of 18 years. Tics can also occur independently, as motor tic dis- order or vocal tic disorder.
2.7. The answer is D
The etiology of the demyelinating disorders is diverse and largely conforms to the traditional, binary classification of etiology– congenital or acquired. Under the congenital disorders are in- cluded presumed genetic and chromosomal abnormalities. The chromosomal abnormalities include the ganglioside disorders, such as Tay-Sachs disease, which usually present in infancy or early childhood. Another group of congenital disorders includes the leukodystrophies, of which there are numerous subtypes, such as metachromatic, globoid cell, adrenoleukodystrophy, and Hurler’s syndrome. The acquired demyelinating disorders in- clude infectious disease, particularly acquired immunodeficiency syndrome (AIDS); trauma, including open and, especially, closed brain trauma; vascular disorders, including the vascular demen-
tias; toxins, including alcohol and other solvents; and autoim- mune disorders, the most prominent of which is multiple scle- rosis. This list is certainly not inclusive, and many disease types may cause a demyelinating encephalopathy. By far, the most common of these disorders is multiple sclerosis.
2.8. The answer is A
Evidence links streptococcal infections with the onset of obsessive-compulsive disorder (OCD) as well as tic disorders in children. Borrelia burgdorferi is the spirochete that causes Lyme disease and may lead to psychosis and cognitive impair- ment in the late stage of infection. Retroviruses such as human immunodeficiency virus (HIV) may lead to a host of neuropsy- chiatric problems, most notably AIDS dementia, which begins with memory impairment and may progress to aphasia in the later stages. Varicella zoster virus may rarely progress to encephalitis, especially in immunocompromised patients. Studies have linked Influenza A and Influenza B to the onset of encephalitis (acute or postinfectious) with a basis in immune modulation.
2.9. The answer is E (all)
Transmissible spongiform encephalopathies (TSEs) are an un- usual and uncommon group of infectious neurodegenerative disorders that are caused by conformational changes, mis- processing, and malfunction of the prion protein (PRP). The human prion diseases are sporadic Creutzfeldt-Jakob disease (sCJD), the variant Creutzfeldt-Jakob disease (vCJD), the intro- genic Creutzfeldt-Jakob disease (iCJD), fatal familial insomnia (FFI), Gerstmann-Straussler-Scheinker (GSS) disease, and Kuru (Table 2.1).
2.10. The answer is E (all)
The diagnosis of a primary headache disorder requires the exclu- sion of other conditions, which include structural lesions, vas- cular malformations, viral or bacterial meningitis, encephali- tis, cerebral contusion, metabolic disorders, vasculitis, brain tu- mors, sinusitis, and pseudotumor cerebri (a condition of the brain that mimics a tumor but is caused by swelling or inflammation). Young, obese females are especially prone to pseudotumor cere- bri, also known as idiopathic intracranial hypertension. One of the most important findings of the past decades is evidence point- ing to an increased risk of ischemic stroke among young women with migraines. In children, arteriovenous malformations are a known cause of subarachnoid hemorrhage, which often present as “the worst headache” of one’s life.
2.11. The answer is E (all)
There is a significant overlap and comorbidity between patients with symptoms of fibromyalgia and other psychiatric conditions, such as depression, panic and anxiety, and posttraumatic stress syndromes. The significance of this comorbidity is not under- stood in terms of understanding how the symptomatology arises, but these secondary psychiatric syndromes can provide thera- peutic targets for psychopharmacology. There is also significant comorbidity between patients with fibromyalgia and rheumato- logic arthritis, systemic lupus, and others. Despite this comorbid- ity, the acute symptomatology of fibromyalgia does not correlate well with disease activity of associated medical diseases. When
2. Neuropsychiatry and Behavioral Neurology 11
12 2. Neuropsychiatry and Behavioral Neurology Table 2.1

Human and Animal Prion Diseases
Sporadic Creutzfeldt-Jakob disease (sCJD) Variant Creutzfeldt-Jakob disease (vCJD)
Iatrogenic Creutzfeldt-Jakob disease (iCJD)
Fatal familial insomnia (FFI) Gerstmann-Stra ̈ussler-Scheinker disease (GSS) Kuru
Scrapie
BSE (mad cow disease)
Chronic wasting disease (CWD) Transmissible mink encephalopathy

Disease
Cause
Unknown
Exposure to bovine spongiform
encephalopathy (BSE) Genetic
Familial
Genetic
Ritual cannibalism
Unknown
Animal feed with animal body parts,
initially from sheep Caged elk and deer Farm-raised
Distribution/Incidence
Global
As of 2006, more than 150 cases of vCJD
have been recorded and all were associated with methionine homozygotic status at codon 129 of PRNP gene
Accidental transmission of CJD to human hosts through various medical/surgical procedures, such as tissue transplantation
Rare
Extremely rare
Papua
Europe, Iceland, United States, Canada Europe, United States
United States, Canada United States
 
From Dormont D. Prion disease: pathogenesis and public health concerns. FEBS Lett. 2002;529:17.
such diseases are present, however, varieties of psychotropic drugs are commonly prescribed for fibromyalgia, especially an- tidepressants. The antiepileptic agent pregabalin has recently been approved by the U.S. Food and Drug Administration (FDA) for the treatment of pain associated with fibromyalgia. A wide spectrum of analgesics is also prescribed for such patients. The selective serotonin reuptake inhibitor (SSRI) and serotonin nore- pinephrine reuptake inhibitor (SNRI) antidepressant, duloxetine, has been reported to be effective in treating patients with this disorder. However, experience suggests that benefits from such therapies are neither long lasting nor associated with return to employment.
2.12. The answer is E (all)
A large prospective cohort study of 2,032 adolescents found that those born premature with low birth weight were 11-fold more likely to develop a depressive disorder. Prematurity was also associated with elevated risk of anxiety, social isolation, conduct disorder, aggression, thought disorders, and schizophrenia.
2.13. The answer is B
Herpes Simplex virus type 1 (HSV-1) is the most common acute cause of fatal encephalitis in the United States. Lesions are seen in the temporal lobe area, as revealed by an MRI. EEG will demonstrate slow waves with high amplitudes. Polymerase chain reaction (PCR) analysis of the viral DNA is considered the defini- tive diagnostic tool and has replaced brain biopsies. Presenta- tion of HSV-1 encephalitis can include seizures (as in the above case), altered mental status, hypomania, and amnesia. Kluver- Bucy syndrome, a constellation of hyperphagia, hypersexuality, hyperdocility, and hyperorality, may also result as a consequence of herpes encephalitis with bilateral amygdala lesions. Crypto- coccal meningitis is more often seen in immunocompromised patients. Meningiomas will not result in an increase in lym- phocytes and generally do not present acutely. Subarachnoid and intracranial hemorrhages are cerebrovascular lesions that do not match the CSF finding of lymphocytosis and elevated protein.

3
Contributions of the Psychosocial Sciences to Human Behavior

The psychosocial sciences integrate social sciences and psychol- ogy. Psychology is concerned with mental processes and behav- ior and can be broken down into several areas. Clinical psychol- ogy is concerned with the application of psychological principles to the prevention, treatment, and understanding of psychopathol- ogy. Educational psychology is concerned with the application of psychological theories to teaching. Sociology is the study of col- lective human behavior, including the developmental structure and interactions of their social institutions. Anthropology is the study of humans in relation to distribution, origin, classification, and relationship of races, physical characteristics, environmen- tal relations, social relations, and culture. Ethology is the study of animal behavior and is often applied to human behavior. Epi- demiology is the study of the incidence, prevalence, control, and distribution of disease within a particular population.
One of the major contributions to psychosocial sciences is the work of Jean Piaget (1896–1980), a widely renowned child (or developmental) psychologist. He is best known for his work in the understanding of the way children think and acquire knowl- edge. Piaget is also recognized for his famous theory of the four stages of cognitive development.
Over time, many other professionals have made contributions to our understanding of human behavior. Two of the major the- ories that have stemmed from such contributions are learning theory and attachment theory. Learning theory developed from the work of such behavioral researchers as Ivan Petrovich Pavlov (1849–1936), John B. Watson (1878–1958), and B.F. Skinner (1904–1990). Three different types of learning emerged: clas- sic conditioning, operant conditioning, and social learning the- ory. Attachment theory originated in the work of John Bowlby (1907–1990) who pointed out that the mother–child attachment was an essential medium of human interaction that had impor- tant consequences for development and personality functioning. Rene ́ Spitz (1887–1974) described anaclitic depression, or hosp- tialism, in which normal children who were separated for long periods from adequate caregiving failed to thrive and, therefore, became depressed and nonresponsive. Ethologists, such as Kon- rad Lorenz (1903–1989) and Harry Harlow (1905–1981), studied bonding and attachment behaviors in animals, and showed how studying animal behavior could help illuminate human behavior.
The questions and answers below will help students test their knowledge of the subjects highlighted.
HELPFUL HINTS
The student should know the following terms, theoreticians, and concepts.
abstract thinking acculturation
altruism
Aplysia
assimilation attachment phases attribution theory aversive stimuli behaviorism biofeedback biostatistics
cognitive dissonance cognitive organization cognitive triad
concrete operations cross-cultural studies
and syndromes double-blind method drift hypothesis epigenesis
escape and avoidance
conditioning ethology
experimental neurosis extinction
flooding and implosion frustration–aggression
hypothesis
imprinting
inductive reasoning inhibition
learning theory object permanence operant and classical
conditioning positive and negative
reinforcement preattachment stage preoperational stage reciprocal
determinism reciprocal inhibition
respondent behavior segregation hypothesis sensory deprivation separation anxiety social learning stranger anxiety syllogistic reasoning systematic
desensitization tension-reduction
theory
token economy trial-and-error learning vulnerability theory
13
▲▲▲▲▲▲▲▲ ▲ ▲▲▲
▲▲▲▲▲▲ ▲ ▲▲▲ ▲
▲▲ ▲▲▲▲ ▲▲▲▲▲
▲▲▲▲▲▲▲▲▲▲▲▲▲▲
14 3. Contributions of the Psychosocial Sciences to Human Behavior
QUESTIONS
Directions
Each question or incomplete statement below is followed by five suggested responses or completions. Select the one that is best in each case.
. 3.1. The drift hypothesis states that
A. one’sexperienceasamemberofaparticularclassleads to the development of individual differences in coping capacity
B. womenaredisadvantagedrelativetomenbecausetheir adult roles expose them to chronic stress
C. a mental illness results in a decline in one’s social clan
D. differential exposure to stress explains group differ-
ences in mental illness
E. ethnic minorities and immigrants appear to have more
effective coping skills over nonminorities
. 3.2. A young woman presents with a history of agoraphobia since adolescence. Her agoraphobia progressed over the subsequent decade until she was essentially housebound. She could only leave home accompanied by her mother or husband and, even in that circumstance, with consid- erable anxiety. Leaving home alone had, in the past, of- ten precipitated a panic attack. She had been treated with various medications and with psychotherapy without sig- nificant improvement. She was admitted to a clinical re- search center as part of a study examining the use of so- cial reinforcement in various phobic conditions. A ther- apist with whom she had developed a good relationship delivered reinforcement in the form of praise contingent on progress. In the baseline period, the patient was en- couraged to walk as far away from the clinical research unit as she could. Reinforcement for staying outside the unit resulted in only a small increase in distance walked away from the unit. In the next phase, praise for progress was given on a shaping schedule. For example, if the patient had been reinforced at a criterion of 100-yards distance on one trial and walked 150 yards on the next trial, the criterion would become 125 yards. She would be praised on the next trial only if she walked 125 yards or more. In this phase, the distance walked began to in- crease. When reinforcement was stopped, distance walked increased dramatically and then decreased. Finally, when reinforcement was reintroduced, the patient was able to walk long distances away from the unit. This was then generalized to the patient’s home environment. (Cour- tesy of W. Stewart Agras, M.D., and G. Terence Wilson, Ph.D.)
The therapy used in the above case is an example of
A. Classic Conditioning B. Operant Conditioning C. Social Learning
D. Cognitive Learning E. None of the above
3.3. American psychologist Edward L. Thorndike described
A. drive reduction theory B. behaviorism
C. trial-and-error learning D. respondent behavior E. operant behavior
3.4. Premack’s principle states that
A. people will attribute other people’s behavior to stabi- lize their own personality traits
B. high-frequencybehaviorscanbeusedtoreinforcelow- frequency behavior
C. a person can learn by imitating the behavior of another person
D. the more people feel capable of controlling a threaten- ing event, the less anxious they will be
E. people will attribute their own behavior to situational causes
3.5. The catharsis hypothesis is the belief that
A. participation in activities such as kickboxing can re- duce aggressive behavior
B. aggression may be due to a lack of basic social skills C. punishment can be an effective deterrent to overt ag-
gression
D. exposure to signs of pain or discomfort on the victim’s
part inhibits further aggression
E. humorous materials can often reduce anger
3.6. Attribution theory states that
A. behavior results from the interplay between cognitive and environmental factors
B. persons are likely to attribute their own behavior to situational causes
C. abehaviorengagedinwithhighfrequencycanbeused to reinforce a low-frequency behavior
D. an organism changes its behavior to avoid a painful stimulus
E. an animal learns a response to get out of a place where it does not want to be
3.7. Which of the following chromosomal abnormalities has been implicated as having an influence on aggressive be- havior?
A. 45-XO
B. 48-XXXY C. 47-XXY D. 47-XYY E. 47-XXX
3.8. Which of the following statistical procedures is used to evaluate the frequency of events in a population?
A. Analysis of Variance (ANOVA) B. T-test
C. Chi-squared test
D. Discriminant analysis E. Z-score
. 3.9. Attachment theory states that
A. infants are generally polytropic in their attachments
B. attachment disorders may lead to a failure to thrive
C. attachment occurs instantaneously between the mother
and the child
D. attachment is synonymous with bonding
E. separation anxiety is most common when an infant is
5 months old
. 3.10. Which of the following statements regarding crossover
studies is true?
A. It eliminates selection bias.
B. They are a variation of the double-blind study.
C. They contain a treatment group and a control group. D. They are a type of prospective study.
E. All of the above
. 3.11. Prevalence is the
A. risk of acquiring a condition at some point in time
B. ratio of people who acquire a disorder during a year’s
time
C. proportion of a population that has a condition at any
given moment in time
D. standard deviation
E. rate of first admissions to a hospital for a disorder
. 3.12. In which of the following age groups is the stage of pre- operational thought present?
A. Birth to 2 years
B. 2 to 7 years
C. 7 to 11 years
D. 11 through the end of adolescence E. None of the above
. 3.13. Asian patients seem to achieve a clinical response com- parable to those of non-Asian patients, even though they require a significantly lower dose of
A. lithium
B. antipsychotics C. tricyclics
D. benzodiazepines E. All of the above
. 3.14. The “choo-choo” phenomenon is associated with which of the following types of social deprivation in monkeys?
A. Total-isolation–reared monkeys B. Mother-only–reared monkeys
C. Peer-only–reared monkeys
D. Partial-isolation–reared monkeys E. Separation-reared monkeys
3. Contributions of the Psychosocial Sciences to Human Behavior 15 Directions
Each set of lettered headings below is followed by a list of num- bered phrases. For each numbered phrase, select
A. if the item is associated with A only
B. if the item is associated with B only
C. if the item is associated with both A and B D. if the item is associated with neither A nor B
Questions 3.15–3.19
A. Behavioral Model of learning
B. Psychoanalytic Model of learning
. 3.15. Childhood experiences are the focus of the analysis
. 3.16. Theory is based on experimentation
. 3.17. Subjective methods of interpretation
. 3.18. Testable hypotheses that can be evaluated through exper-
imentation
. 3.19. Theory is predominantly based on case histories
Questions 3.20–3.25
A. Classical conditioning B. Operant conditioning
. 3.20. Ivan Petrovich Pavlov
. 3.21. Instrumental conditioning
. 3.22. B. F. Skinner
. 3.23. Repeatedpairingofaneutralstimuluswithonethatevokes
a response
. 3.24. Learning occurs as the consequence of action
. 3.25. Learning takes place as a result of the co-occurrence of
environmental events
Directions
Each group of questions below consists of lettered headings fol- lowed by a list of numbered words or phrases. For each num- bered word or phrase, select the one lettered heading that is most closely associated with it. Each lettered heading may be selected once, more than once, or not at all.
Questions 3.26–3.31
A. Sigmund Freud’s theory on aggression B. Konrad Lorenz’s theory on aggression C. Albert Bandura’s theory on aggression D. John Dollard’s theory on aggression
3.26. The root of human aggression is neither innate violence nor aggressive drive aroused by frustration.
3.27. Aggression stems from the redirection of Thanatos away from the self and towards others.
3.28. Aggression springs from a fighting instinct that humans share with other organisms.
3.29. Aggression always stems from frustration.
3.30. Aggression is a learned behavior that varies depending on
culture and experience.
16 3. Contributions of the Psychosocial Sciences to Human Behavior
3.31. Aggression is a reaction to the blocking or thwarting of libido.
Questions 3.32–3.36
A. Positive reinforcement B. Negative reinforcement C. Punishment
D. Classical conditioning
. 3.32. Anorexicwomanbeginseatingandgainingweightinorder to get out of the hospital
. 3.33. Dog begins salivating to the sound of a bell after learning food is coming soon after
. 3.34. Amanbeginsleavinghomeearlierinthemorningtoavoid rush hour traffic
. 3.35. A woman gives her dog a treat every time he sits when told
. 3.36. Child has his favorite toy taken away every time he wets his bed
Questions 3.37–3.40
A. Ivan Petrovich Pavlov B. Eric Kandel
C. Konrad Lorenz
D. Harry Harlow
. 3.37. Imprinting
. 3.38. Surrogate mother
. 3.39. Experimental neurosis
. 3.40. Aplysia
Questions 3.41–3.44
A. John Bowlby
B. Harry Harlow C. Mary Ainsworth D. Rene ́ Spitz
. 3.41. “Secure base” effect
. 3.42. Protest, despair, detachment
. 3.43. First described anaclitic depression
. 3.44. Primarily associated with ethological studies
Questions 3.45–3.49
A. Fixed-ratio schedule
B. Variable-ratio schedule
C. Fixed-interval schedule D. Variable-interval schedule
. 3.45. Leads to the most rapid rate of response
. 3.46. Seen with the use of slot machines
. 3.47. Associated with scalloping
. 3.48. Generates an oscillating rate of response
. 3.49. Seen with the three-strike rule of baseball
ANSWERS
3.1. The answer is C
The drift hypothesis states that a mental illness results in a de- cline in one’s social clan. Most of the evidence for the drift hypothesis comes from studies of major mental illnesses, pri- marily schizophrenia. Those studies show that the early onset of a disorder can reduce one’s chances of socioeconomic achieve- ment, a fact that seems true for people who become ill before establishing a career.
A large part of sociological research on psychopathology has focused on structural correlates of psychiatric illness such as so- cial class, race, ethnicity, sex, and age. The associations between these variables and the prevalence of psychiatric disorders are substantial. The most obvious hypothesis to test in examining such associations is that differential exposure to stress explains group differences in mental illness. It is now clear that this hy- pothesis can be rejected. Although it is true that people in com- paratively disadvantaged positions in society are exposed to more stress than their advantaged counterparts, differential exposure cannot totally explain their higher rates of anxiety, depression, and nonspecific distress in general population samples.
Studies have shown that a class-linked vulnerability to stress accounts for the major part of the association between social class and depression and between social class and nonspecific distress. Another explanation is that one’s experience as a member of a particular class leads to the development of more effective coping skills.
A related area of research concerns racial and ethnical dif- ferences in mental illness. One intriguing and still only partially understood pattern is that ethnic minority immigrants appear to have better coping skills over nonminorities that disappear among second-generation and later-generation minorities.
Another area of sociological interest concerns gender differ- ences in anxiety and mood disorders. There are several lines of research to pursue gender differences in nonspecific distress and affective disorders. The dominant perspective in sociology since the 1980s holds that women are disadvantaged relative to men because their adult roles expose them to more chronic stress.
3.2. The answer is B
The therapy used in the above case is an example of operant conditioning. Operant conditioning, developed by B. F. Skin- ner, is a form of learning in which behavior frequency is altered through the application of positive and negative consequences. The therapist uses positive reinforcement (praise) to help the patient overcome her agoraphobia. Positive reinforcement is the process by which certain consequences of a response increase the probability that the response will recur. Classic conditioning is a form of learning in which a neutral stimulus becomes a condi- tioned stimulus when presented with an unconditioned stimulus. Unlike operant conditioning, where learning occurs as a conse- quence of action, in classic conditioning, the examiner reinforces behavior. Ivan Petrovich Pavlov developed classic conditioning through his famous experiment conditioning a dog to salivate in response to a ringing bell.
Social learning theory relies on role modeling, identification, and human interactions. A person can learn by imitating behavior
of another person, but personal factors are involved. When a per- son dislikes the role model, imitative behavior is unlikely. Albert Bandura is a major proponent of the social learning school. Ac- cording to Bandura, behavior results from the interplay between cognitive and environmental factors, a concept known as recipro- cal determinism. Cognitive learning theories focus on the role of understanding: Cognition implies understanding the connection between cause and effect, between action and the consequences of action.
3.3. The answer is C
Operant conditioning is related to trial-and-error learning, as described by the American psychologist Edward L. Thorndike (1874–1949). In trial-and-error learning, a person or animal at- tempts to solve a problem by trying different actions until one proves successful. A freely moving organism behaves in a way that is instrumental in producing a reward. For example, a cat in the Thorndike puzzle box must learn to lift a latch to escape from the box. For this reason, operant conditioning is sometimes called instrumental conditioning. Thorndike’s law of effect states that certain responses are reinforced by reward, and the organism learns from these experiences.
One of the first theorists to explore the neuropsychological as- pect of learning was Clark L. Hull (1884–1952), who developed a drive reduction theory of learning. Hull postulated that neu- rophysiological connections established in the central nervous system reduce the level of a drive (e.g., obtaining food reduces hunger). An external stimulus stimulates an efferent system and elicits a motor impulse. The critical connection is between the stimulus and the motor response, which is a neurophysiological reaction that leads to what Hull called a habit.
American psychologist John B. Watson (1878–1958), the fa- ther of behaviorism, used Ivan Petrovich Pavlov’s theory of clas- sical conditioning to explain certain aspects of human behavior. In 1920, Watson described producing a phobia in an 11-month- old boy called Little Albert. At the same time that the boy was shown a white rat that he initially did not fear, he was exposed to a loud, frightening noise. After such pairings, Albert became fearful of the white rat, even when he heard no loud noise. Many theorists believe that this process accounts for the development of childhood phobias, which are considered learned responses based on classic conditioning.
B.F. Skinner described two types of behavior: respondent behavior, which results from known stimuli (e.g., the knee jerk reflex to patellar stimulation or the papillary constriction to light), and operant behavior, which is independent of a stimulus (e.g., the random movements of an infant).
3.4. The answer is B
A concept developed by David Premack states that a behavior engaged in with high frequency can be used to reinforce a low- frequency behavior. In one experiment, Premack observed that children spent more time playing with a pinball machine than eating candy when both were freely available. When he made playing with the pinball machine contingent on eating a certain amount of candy, the children increased the amount of candy they ate. In a therapeutic application of this principle, patients with schizophrenia were observed to spend more time in a rehabili- tation center sitting down doing nothing than they did working
at a simple task. When being able to sit down for 5 minutes was made contingent on a certain amount of work, the work output was considerably increased, as was the skill acquisition. This principle is also known as Grandma’s rule (“If you eat your spinach, you can have dessert”).
The social learning theory relies on role modeling, identifica- tion, and human interactions. This theory states that a person can learn by imitating the behavior of another person, but personal factors are involved.
The attribution theory is a cognitive approach concerned with how a person perceives the causes of behavior. According to the attribution theory, people are likely to attribute their own behavior to situational causes but are likely to attribute other people’s behavior to their own stable internal personality traits.
The self-efficacy theory predicts that the more people feel capable of predicting and controlling threatening events, the less vulnerable they are to anxiety and stress disorders in response to traumatic experiences.
3.5. The answer is A
The catharsis hypothesis is the belief that participation in activ- ities, such as running or kickboxing, allows people to vent their anger and hostility and, therefore, reduces aggressive behavior. Some people, however, may become more aggressive as a re- sult of the expressive behaviors. Catharsis, therefore, may not be effective for long-term reduction of aggression.
Punishment is sometimes an effective deterrent to overt ag- gression. Research findings indicate that the frequency or inten- sity of aggressive behavior can be reduced by even mild forms of punishment, such as social disapproval. However, punishment does not always, or even usually, produce such effects. The recip- ients of punishment often interpret it as an attack against them. To that extent, aggressors may respond even more aggressively. Strong punishment is more likely to provoke desires for revenge or retribution than to instill lasting restraints against violence. For these reasons, certain punishments may backfire and actu- ally encourage, rather than inhibit, the dangerous actions they are designed to prevent.
A major reason why many people become involved in re- peated aggressive encounters is their lack of basic social skills. These people do not know how to communicate effectively and thus adopt an abrasive style of self-expression. Their social deficits seem to ensure that they experience repeated frustration and frequently anger those with whom they have direct contact. A technique for reducing the frequency of such behavior involves providing these people with the social skills that they lack. The results are encouraging and indicate that training in appropriate social skills can offer a promising approach to the reduction of human violence.
When aggressors attack other people in face-to-face con- frontations, the aggressors may block out, ignore, or deny signs of pain and suffering on the part of their victims. In several exper- iments, exposure to signs of pain and discomfort on the victim’s part has inhibited further aggression. If aggressors are exposed to such feedback, they may feel empathy and subsequently re- duce further aggression.
Informal observation indicates that anger can often be re- duced through exposure to humorous material. Several types of
3. Contributions of the Psychosocial Sciences to Human Behavior 17
18 3. Contributions of the Psychosocial Sciences to Human Behavior
humor, presented in various formats, can induce reactions or emotions incompatible with aggression among the persons who observe the humor.
3.6. The answer is B
According to attribution theory, persons are likely to attribute their own behavior to situational causes but are likely to at- tribute others’ behaviors to stable internal personality traits. In psychiatry, attribution theory may help explain why some per- sons attribute a change in behavior to an external event (situation) or to a change in internal state (disposition or ability).
Albert Bandura is a major proponent of the social learning school. According to Bandura, behavior results from the inter- play between cognitive and environmental factors, a concept known as reciprocal determinism. People learn by observing others, intentionally or accidentally. This process is described as modeling or learning through imitation. A person’s choice of model is influenced by a variety of factors such as age, sex, status, and similarity.
The Premack’s principle is a concept developed by David Premack that states that a behavior engaged in with high fre- quency can be used to reinforce a low-frequency behavior. In one experiment, Premack observed that children spent more time playing with a pinball machine than eating candy when both were freely available. When he made playing with the pinball machine contingent on eating a certain amount of candy, the children in- creased the amount of candy they ate.
In adverse control or conditioning, an organism changes its behavior to avoid a painful, noxious, or aversive stimulus. Elec- tric shocks are common aversive stimuli used in laboratory ex- periments. Any behavior that avoids an aversive stimulus is re- inforced as a result.
Negative reinforcement is related to two types of learning, escape learning and avoidance learning. In escape learning, an animal learns a response to get out of a place where it does not want to be (e.g., an animal jumps off an electric grid whenever the grid is charged). Avoidance learning requires an additional response. The rat on the grid learns to avoid a shock if it quickly pushes a lever when a light signal goes on.
3.7. The answer is D
Behavior research involving the influence of chromosomes on aggressive behavior has concentrated primarily on abnormalities in X and Y chromosomes, particularly the 47-XYY syndrome. Early studies indicate that people with the syndrome could be characterized as tall, with below-average intelligence, and likely to be apprehended and in prison for engaging in criminal be- havior. Subsequent studies indicated, however, that, at most, the XYY syndrome contributes to aggressive behavior in only a small percentage of cases. Studies of the androgen and go- nadotropin characteristics of persons with XYY syndrome have been inconclusive. However, none of the other listed chromoso- mal abnormalities have been associated with increases in ag- gressive behavior.
3.8. The answer is C
The chi-square test is used to evaluate the relative frequency or proportion of events in a population that falls into well-defined
categories. Research on whether parents who were abused as children are more likely to abuse their children could be tested using the chi-square test of association. A t-test is a statistical procedure designed to compare the means of two sets of obser- vations. Analysis of variance, or ANOVA, is a set of statistical procedures designed to compare the means of two or more groups of observations. Discriminant analysis is a multivariate method for finding the relation between a single discrete outcome and a linear combination of two or more predictors. The z-score is the deviation of a score from its group mean expressed in standard deviations.
3.9. The answer is B
Attachment disorders are characterized by biopsychosocial pathology that results from maternal deprivation, a lack of care by, and interaction with, the infant’s mother or caretaker. Psy- chosocial dwarfism, separation anxiety disorder, avoidant per- sonality disorder, depressive disorders, delinquency, learning disorders, borderline intelligence, and failure to thrive have been traced to negative attachment experiences. Failure to thrive re- sults in the infant being unable to maintain viability outside a hospital setting. When maternal care is deficient because the mother is mentally ill, because the child is institutionalized for a long time, or because the primary object of attachment dies, the child suffers emotional damage.
John Bowlby formulated a theory that states normal attach- ment is crucial to healthy development. According to Bowlby, attachment occurs when the infant has a warm, intimate, and continuous relationship with its mother, and both mother and infant find satisfaction and enjoyment. Infants are generally monotropic, not polytropic, in their attachments, but multiple attachments may also occur (i.e., attachment may be directed toward the father or a surrogate). Attachment does not occur instantaneously between the mother and the child; it is a grad- ually developing phenomenon. Attachment results in one per- son’s wanting to be with a preferred person who is perceived as stronger, wiser, and able to reduce anxiety or distress. Attach- ment produces a feeling of security in the infant. It is a process that is facilitated by interaction between the mother and the in- fant. The amount of time together is less important than the quality of activity between the two.
Attachment is not synonymous with bonding; they are dif- ferent phenomena. Bonding concerns the mother’s feelings for her infant. It differs from attachment in that a mother does not normally rely on her infant as a source of security, a requirement of attachment behavior. A great deal of research on the bonding of a mother to her infant reveals that it occurs when they have skin-to-skin contact or other types of contact, such as voice and eye contact.
Separation from the attachment may or may not produce intense anxiety, depending on the child’s developmental level and the current phase of attachment. Separation anxiety is ex- pressed as tearfulness or irritability in a child who is isolated or separated from its mother or caretaker. Separation anxiety is most common when an infant is 10 to 18 months of age (not 5 months), and it disappears generally by the end of the third year. Table 3.1 delineates aspects of normal attachment at different ages.
Table 3.1
Normal Attachment
Birth to 30 days Reflexes at birth
Rooting
Head turning
Sucking
Swallowing
Hand–mouth
Grasp
Digital extension
Crying—signal for particular kind of distress Responsiveness and orientation to mother’s face, eyes, and
voice
4 days—anticipatory approach behavior at feeding
3 to 4 weeks—infant smiles preferentially to mother’s voice
Age 30 days through 3 months
Vocalization and gaze reciprocity further elaborated from 1 to
3 months; babbling at 2 months, more with the mother than
with a stranger
Social smile
In strange situations, increased clinging response to mother
Age 4 through 6 months
Briefly soothed and comforted by sound of mother’s voice Spontaneous, voluntary reaching for mother
Anticipatory posturing to be picked up
Differential preference for mother intensifies
Subtle integration of responses to mother
Age 7 through 9 months
Attachment behaviors further differentiated and focused
specifically on mother
Separation distress, stranger distress, strange-place distress
Age 10 through 15 months
Crawls or walks toward mother
Subtle facial expressions (coyness, attentiveness)
Responsive dialogue with mother clearly established
Early imitation of mother (vocal inflections, facial expression) More fully developed separation distress and mother preference Pointing gesture
Walking to and from mother
Affectively positive reunion responses to mother after separation
or, paradoxically, short-lived, active avoidance or delayed
protest
Age 16 months through 2 years
Involvement in imitative jargon with mother (12 to 14 months) Head-shaking “no” (15 to 16 months)
Transitional object used during the absence of mother Separation anxiety diminishes
Mastery of strange situations and persons when mother is near Evidence of delayed imitation
Object permanence
Microcosmic symbolic play
Age 25 months through 3 years
Able to tolerate separations from mother without distress when
familiar with surroundings and given reassurances about
mother’s return
Two- and three-word speech
Stranger anxiety much reduced
Object consistency achieved—maintains composure and
psychosocial functioning without regression in absence of
mother
Microcosmic play and social play; cooperation with others begins
Based on material by Justin Call, M.D.
3.10. The answer is E (all)
A crossover study is a variation of the double-blind study. The treatment group and the control or placebo group change places at some point so that the placebo group gets the treatment and the treatment group now receives the placebo. That procedure eliminates selection bias. If the treatment group improves in both instances and the placebo group does not, one can conclude that the makeup of the two groups was truly random. Each group serves as the control for the other.
3.11. The answer is C
Prevalence is the proportion of a population that has a condition at any given time. The ratio of people who acquire a disorder during a year’s time (new cases) is called the annual incidence. In a stable situation, the prevalence is approximately equal to the annual incidence times the average duration, measured in years, of the condition. The risk of acquiring a condition at some time in the future is the accumulation of age-specific annual incidence rates over a period of time.
Standard deviation (SD) is a statistical measure of variability within a set of values. For a normal distribution, about 68 percent of the values fall within one SD of the mean, and about 95 percent lie within two SDs of the mean. It is sometimes presented by , the Greek letter sigma.
The rate of first admissions to a hospital for a disorder is the number of all first admissions to any hospital during a particular time.
3.12. The answer is B
The state of preoperational thought is present in children 2 to 7 years old. During this stage, children use symbols and language more extensively than in the sensorimotor stage. Thinking is intuitive; children learn without the use of reasoning. Preopera- tional thought is midway between socialized adult thought and the completely autistic Freudian unconscious.
Jean Piaget (1896–1980) used the term sensorimotor to de- scribe the first stage, present from birth to 2 years of age. Infants begin to learn through sensory observation, and they gain control of their motor functions through activity, exploration, and ma- nipulation of the environment. The stage of concrete operations is present in children from 7 to 11 years of age. The stage of concrete operations is so named because in this period, children operate and act on the real and perceivable world of objects and events. Egocentric thought is replaced by operational thought, which involves dealing with a wide array of information in the world outside of the child. Therefore, children can now see things from someone else’s perspective.
The formal operations stage is present from 11 years of age through the end of adolescence. This stage is named so because a young person’s thoughts operate in a formal, highly logical, systematic, and symbolic manner. This stage is characterized by the ability to think abstractly, to reason deductively, and to define concepts. It is also characterized by the emergence of skills for dealing with permutations and combinations; young people can grasp the concept of probabilities.
3. Contributions of the Psychosocial Sciences to Human Behavior 19
  
Table 3.2 lists Piaget’s stages of intellectual development.
20 3. Contributions of the Psychosocial Sciences to Human Behavior
Table 3.2
Stages of Intellectual Development Postulated by Piaget
 
Age (Yrs)
0–1.5 (to 2)
2–7
7–11
3.13. The answer is E (all)
Asian patients require lower dosages of dopamine receptor an- tagonist (typical antipsychotic) medications than comparable non-Asian patients to achieve a desirable clinical outcome. Also, when treated on a fixed-dosage schedule, Asians seem to develop significantly greater extrapyramidal adverse effects. One study found a 52 percent higher plasma concentration of haloperidol (Haldol) in Chinese schizophrenics living in China than in non- Asian schizophrenic patients residing in the United States when both groups received treatment on a fixed-dosage schedule. An- other study demonstrated that Chinese schizophrenics residing in Taiwan and Taipei achieved haloperidol plasma concentrations comparable to those of white, African American, and Hispanic patients hospitalized in San Antonio while using significantly lower daily dosages of haloperidol.
As is the case with neuroleptics, studies with tricyclic drugs have shown that average dosages prescribed for Asians are sig- nificantly lower (up to 50 percent lower) than dosages prescribed in the United States for non-Asians. The reasons for this respon- siveness have not been clearly established, although preliminary evidence suggests differential responsiveness of relevant recep- tors, differences in resulting plasma concentrations, or both.
Studies of prescription patterns as well as those compar- ing the pharmacokinetics and pharmacodynamics of benzodi- azepines across ethnic groups have established the enhanced sensitivity of Asians to the effects of benzodiazepines. Typically prescribed doses are one-half to two-thirds those of similar non- minority populations. The ethnic differences in benzodiazepine metabolism are most often linked to polymorphisms in the (S)- mephenytoin phenotype, yielding a higher percentage of poor metabolizers in the Chinese ethnic group.
As with tricyclic drugs, antipsychotics, and benzodiazepines,
Asians seem to achieve clinical responses comparable to those of non-Asian patients using significantly lower dosages of lithium, 0.5 to 0.7 mEq/L versus the 0.8 to 1.2 mEq/L generally required by white populations.
3.14. The answer is C
An area of animal research that has relevance to human behav- ior and psychopathology is the longitudinal study of nonhuman

11 through the end of adolescence
Period
Sensorimotor
Preoperations subperioda Concrete operations Formal operations
Cognitive Developmental Characteristics
Divided into six stages, characterized by: 1. Inborn motor and sensory reflexes
2. Primary circular reaction
3. Secondary circular reaction
4. Use of familiar means to obtain ends
5. Tertiary circular reaction and discovery through active experimentation
6. Insight and object permanence
Deferred imitation, symbolic play, graphic imagery (drawing), mental imagery,
and language
Conservation of quantity, weight, volume, length, and time based on reversibility
by inversion or reciprocity; operations; class inclusion and seriation Combinatorial system, whereby variables are isolated and all possible
combinations are examined; hypothetical-deductive thinking a This subperiod is considered by some authors to be a separate developmental period. Printed with permission.

primates. Monkeys have been observed from birth to maturity, not only in their natural habitats and laboratory facsimiles but also in laboratory settings that involve various degrees of social deprivation early in life. Socially isolated monkeys are raised in varying degrees of isolation and are not permitted to develop normal attachment bonds. Social isolation techniques illustrate the effects of an infant’s early social environment on subsequent development and separation techniques, which illustrate the ef- fects of loss of a significant attachment figure. The choo-choo phenomenon is observed in peer-only–reared infant rhesus mon- keys. It is the actual physical alignment these monkeys have been observed to form (that of a “choo-choo train”), in addition to ex- hibiting behavior such as becoming easily frightened, clinging to each other, being reluctant to explore, and engaging in minimal play (Figure 3.1).
Answers 3.15–3.19 3.15. The answer is B
3.16. The answer is A
FIGURE 3.1
Choo-choo phenomenon in peer-only-reared infant rhesus mon- keys.

3. Contributions of the Psychosocial Sciences to Human Behavior 21
Table 3.3
Behavioral and Psychoanalytic Models
Behavioral Model
Behavior is determined by current contingencies, reinforcement history, and genetic endowment
Problem behavior is the focus of study and treatment
Contemporary variables, such as contingencies of reinforcement, are the focus of the analysis
Treatment entails the application of the principles of operant or classical conditioning
Objective observation, measurement, and experimentation are the methods used; the focus is on observable behavior and environmental events (antecedents and consequences)
Theory is based on experimentation
Tenets can be formulated into testable hypotheses and evaluated
through experimentation
Reprinted with permission from Dorsett PC. Behavioral and social learning psychology. In: Stoudemire A, ed. Human Behavior: An Introduction for Medical
 
Psychoanalytic Model
Behavior is determined by intrapsychic processes
Behavior is but a symbol of intrapsychic processes and a symptom of unconscious conflict; the underlying conflict is the focus of treatment
Historical variables, such as childhood experiences, are the focus of the analysis
Treatment consists of bringing unconscious conflicts into consciousness
Subjective methods of interpretation of behavior and inference regarding unobservable events (e.g., intrapsychic processes) are used
Theory is predominantly based on case histories
Many tenets cannot be formulated into testable hypotheses to

be evaluated through experimentation

Students. Philadelphia: JB Lippincott; 1990:105. 3.17. The answer is B
3.18. The answer is A
3.19. The answer is B
Attempts have been made to reconcile behavioral theory and Freudian psychodynamics by stressing the commonalities be- tween the two. In the tension-reduction theory of behavior, be- havior is motivated by an organism’s attempt to reduce tension produced by unsatisfied or unconscious drives. Similarly, Sig- mund Freud’s pleasure principle is a tension-reducing force and, consequently, a strong motivator. When a drive is repressed, anx- iety occurs and acts as an acquired drive; a person’s behavior may be motivated by an attempt to reduce that anxiety. Adults may avoid situations that are likely to stimulate anxiety, but they may be completely unaware of their avoidance patterns. Ther- apy, in part, is an unlearning process. The patient learns that certain behaviors can reduce anxiety, and avoidance patterns are replaced by approach patterns. Table 3.3 gives a comparison of the behavioral and psychoanalytic models.
Answers 3.20–3.25 3.20. The answer is A 3.21. The answer is B 3.22. The answer is B 3.23. The answer is A 3.24. The answer is B
3.25. The answer is A
Among the building blocks of learning theory are classical con- ditioning and operant conditioning. In classical conditioning, learning is thought to take place as a result of the contiguity
of environmental events. When events occur closely together in time, people will come to associate the two. In the case of operant conditioning, learning is thought to occur as a result of the consequences of a person’s actions and the resultant effect on the environment. As B. F. Skinner (1904–1990) stated, “A person does not act upon the world, the world acts upon him.”
Classical or respondent conditioning results from the re- peated pairing of a neutral (conditioned) stimulus with one that naturally evokes a response (unconditioned stimulus), such that the neutral stimulus eventually comes to evoke the response. The time relation between the presentation of the conditioned and un- conditioned stimuli is important and varies for optimal learning from a fraction of a second to several seconds.
The Russian physiologist and Nobel prize winner, Ivan Petro- vich Pavlov (1849–1936), observed in his work on gastric secre- tion that a dog salivated not only when food was placed in its mouth but also at the sound of the footsteps of the person com- ing to feed the dog, even though the dog could not see or smell the food. Pavlov analyzed these events and called the saliva flow that occurred with the sound of footsteps a conditioned response (CR)—a response elicited under certain conditions by a partic- ular stimulus.
In a typical Pavlovian experiment, a stimulus (S) that had no capacity to evoke a particular response before training did so after consistent association with another stimulus. For example, under normal circumstances, a dog does not salivate at the sound of a bell, but when the bell sound is always followed by the presentation of food, the dog ultimately pairs the bell and the food. Eventually, the bell sound alone elicits salivation (CR).
Skinner’s theory of learning and behavior is known as operant or instrumental conditioning. Whereas in classical conditioning an animal is passive or restrained and behavior is reinforced by the experimenter, in operant conditioning the animal is active and behaves in a way that produces a reward; thus, learning occurs as a consequence of action. For example, a rat receives a reinforcing stimulus (food) only when it responds correctly by pressing a lever. Food, approval, praise, good grades, or any
22 3. Contributions of the Psychosocial Sciences to Human Behavior
Table 3.4
Reinforcement Schedules in Operant Conditioning
 
Reinforcement Schedule
Fixed-ratio (FR) schedule
Variable-ratio (VR) schedule
Fixed-interval (FI) schedule
Variable-interval (VI) schedule
Example
Reinforcement occurs after every 10 responses (10:1 ratio); 10 bar presses release a food pellet; workers are paid for every 10 items they make.
Variable reinforcement occurs (e.g., after the third, sixth, then second response, and so on).
Reinforcement occurs at regular intervals (e.g., every 10 minutes or every third hour).
Reinforcement occurs after variable intervals (e.g., every 3, 6, and then 2 hours), similar to VR schedule.
Behavioral Effect
Rapid rate of response to obtain the greatest number of rewards. Animal knows that the next reinforcement depends on a certain number of responses being made.
Generates a fairly constant rate of response because the probability of reinforcement at any given time remains relatively stable.
Animal keeps track of time. Rate of responding drops to near 0 after reinforcement and then increases at about the expected time of reward.
Response rate does not change between reinforcements. Animal responds at a steady rate to get the reward when it is available; common in trout fishing, use of slot machines, checking mailbox.
 
other response that satisfies a need in an animal or a person can serve as a reward.
Operant conditioning is related to trial-and-error learning, as described by the American psychologist Edward L. Thorndike (1874–1949). In trial-and-error learning, a person or an animal attempts to solve a problem by trying different actions until one proves successful. In other words, a freely moving organism behaves in a way that is instrumental in producing a reward. For example, a cat in a Thorndike puzzle box must learn to lift a latch to escape from the box. For this reason, operant conditioning is sometimes called instrumental conditioning. Thorndike’s law of effect states that certain responses are reinforced by reward and the organism learns from these experiences.
Four kinds of instrumental or operant conditioning are de- scribed in Table 3.4.
Answers 3.26–3.31 3.26. The answer is C 3.27. The answer is A 3.28. The answer is B 3.29. The answer is D 3.30. The answer is C
3.31. The answer is A
In his early writings, Sigmund Freud held that all human behavior stems either directly or indirectly from Eros—the life instinct— whose energy, or libido, is directed toward the enhancement or reproduction of life. In this framework, aggression was viewed simply as a reaction to the blocking or thwarting of libido and was neither an automatic nor an inevitable part of life. After the tragic events of World War I, Freud gradually came to adopt a gloomier position about the nature of human aggression. He proposed the existence of a second major instinct—Thanatos, the death force—whose energy is directed toward the destruction or termination of life. Because the death instinct, if unrestrained,
soon results in self-destruction, Freud hypothesized that through mechanisms, such as displacement, the energy of Thanatos is redirected outward and serves as the basis for aggression against others. Thus, in Freud’s latter view, aggression stems primarily from the redirection of the self-destructive death instinct (i.e., Thanatos) away from the self and towards others.
According to Konrad Lorenz, aggression that causes physical harm to others springs from a fighting instinct that humans share with other organisms. The energy associated with this instinct is produced spontaneously in organisms at a more or less constant rate. The probability of aggression increases as a function of the amount of stored energy and the presence and strength of aggression-releasing stimuli. Aggression is inevitable, and, at times, spontaneous eruptions occur.
According to Albert Bandura, neither innate urges towards violence nor aggressive drives aroused by frustration are the roots of human aggression. Rather, persons acquire aggression, much like other forms of social behavior, through either personal experience or by observation of others. These learned behaviors vary between cultures depending on experience. At the same time, people also learn through experience which persons or groups, behaviors, and situations warrant aggression.
John Dollard’s frustration–aggression hypothesis, in its orig- inal form, indicated that frustration always leads to a form of aggression and that aggression always stems from frustration. Frustrated persons, however, do not always respond with aggres- sive thoughts, words, or deeds. They may show a wide variety of reactions, ranging from resignation, depression, and despair to attempts to overcome the sources of their frustration. And not all aggression results from frustration. People (e.g., boxers and football players) act aggressively for many reasons and in response to many stimuli.
Answers 3.32–3.36 3.32. The answer is B 3.33. The answer is D 3.34. The answer is B
3.35. The answer is A
3.36. The answer is C
In operant conditioning, learning is thought to occur as a result of the consequences of one’s actions and the resultant effect on the environment. In classical conditioning, in contrast, learning is thought to take place as the result of the contiguity of envi- ronmental events; when events occur closely together in time, persons will probably come to associate the two. An example is Pavlov’s salivating dogs experiment.
In operant conditioning, positive reinforcement is the process by which certain consequences of a response increase the prob- ability that the response will occur again. Food, water, praise, and money, as well as substances such as opium, cocaine, and nicotine, all may serve as positive reinforcers.
Negative reinforcement is the process by which a response that leads to the removal of an aversive event increases that re- sponse. Any behavior that enables one to avoid or escape a pun- ishing consequence is strengthened. Therefore, a patient with anorexia nervosa eating and gaining weight in order to get out of the hospital (presuming she prefers going home to a prolonged hospitalization), as well as getting up early to avoid traffic are examples of negative reinforcement.
Negative reinforcement is not punishment. Punishment is an aversive stimulus (for example, a slap on the face for bad behav- ior, or the removal of a desired object). It is presented explicitly to weaken or suppress an undesired response.
Answers 3.37–3.40 3.37. The answer is C 3.38. The answer is D 3.39. The answer is A
3.40. The answer is B
Imprinting has been described as the process by which certain stimuli become capable of eliciting certain innate behavior pat- terns during a critical period of an animal’s behavioral develop- ment. The phenomenon is associated with Konrad Lorenz, who in 1935 demonstrated that the first moving object (in that case, Lorenz himself) a duckling sees during a critical period shortly after hatching is regarded and reacted to thereafter as the mother duck.
Harry Harlow is associated with the concept of the surrogate mother from his experiments in the 1950s with Rhesus monkeys. Harlow designed a series of experiments in which infant mon- keys were separated from their mothers during the earliest weeks of life. He found that the infant monkeys, if given the choice be- tween a wire surrogate mother and a cloth-covered surrogate mother, chose the cloth-covered surrogates, even if the wire sur- rogates provided food.
Ivan Petrovich Pavlov coined the terms “experimental neu- rosis” to describe disorganized behavior that appears in the ex- perimental subject (in Pavlov’s case, dogs) in response to an inability to master the experimental situation. Pavlov described extremely agitated behavior in his dogs when they were unable
to discriminate between sounds of similar pitch or test objects of similar shapes.
Eric Kandel contributed to the knowledge of the neurophys- iology of learning. He demonstrated in the study of the sea slug species Aplysia that synaptic connections are altered as a result of learning. His work earned him the Nobel Prize in Medicine in 2001.
Answers 3.41–3.44 3.41. The answer is C 3.42. The answer is A 3.43. The answer is D
3.44. The answer is B
Attachment can be defined as the emotional tone between children and their caregivers and is evidenced by an infant’s seeking and clinging to the caregiving person, usually the mother. By their first month, infants usually begin to show such behavior, which is designed to promote proximity to the desired person.
John Bowlby, a British psychoanalyst (1907–1990), for- mulated the theory that normal attachment in infancy is cru- cial to healthy development. Bowlby described a predictable set and sequence of behavior patterns in children who are separated from their mothers for long periods (more than 3 months): protest, in which the child protests against the sepa- rations by crying, calling out, and searching for the lost per- son; despair, in which the child appears to lose hope that the mother will return; and detachment, in which the child emo- tionally separates himself or herself from the mother. Bowlby believed that this sequence involves ambivalent feelings toward the mother; the child both wants her and is angry with her for her desertion.
Mary Ainsworth built on Bowlby’s observations and found that the interaction between the mother and her baby during the attachment period influences the baby’s current and future behav- ior significantly. Many observers believe that patterns of infant attachment affect future adult emotional relationships. Patterns of attachment vary among babies; for example, some babies sig- nal or cry less than others. Sensitive responsiveness to infant signals, such as cuddling the baby when it cries, causes infants to cry less in later months. Close bodily contact with the mother when the baby signals for her is also associated with the growth of self-reliance, rather than with a clinging dependence, as the baby grows older. Unresponsive mothers produce anxious ba- bies; these mothers often have lower intelligence quotients (IQs) and are emotionally more immature and tend to be younger than responsive mothers are.
Ainsworth also confirmed that attachment serves the purpose of reducing anxiety. What she called the secure base effect en- ables a child to move away from the attachment figures and to explore the environment. Inanimate objects, such as a teddy bear or a blanket (called the transitional object by Donald Winnicott), also serve as a secure base, one that often accompanies children as they navigate the world.
3.
Contributions of the Psychosocial Sciences to Human Behavior 23
24 3. Contributions of the Psychosocial Sciences to Human Behavior
Harry Harlow’s ethological studies with monkeys are relevant to attachment theory. Harlow demonstrated the emotional and behavioral effects of isolating monkeys from birth and keeping them from forming attachments. The isolates were withdrawn, unable to relate to peers, unable to mate, and incapable of caring for their offspring.
Anaclitic depression, also known as hospitalism, was first described by Rene ́ Spitz in infants who had made normal at- tachments but were then separated suddenly from their mothers for varying times and placed in institutions or hospitals. The children became depressed, withdrawn, nonresponsive, and vul- nerable to physical illness. However, they recovered when their mothers returned or when surrogate mothering became available.
Answers 3.45–3.49 3.45. The answer is A 3.46. The answer is D
3.47. The answer is C
3.48. The answer is C
3.49. The answer is A
Often given in the form of attention and praise contingent on certain behaviors, reinforcement is a basic ingredient of most therapists’ repertoire. Much is known about various schedules of reinforcement, defined as the pattern or frequency with which a reward is delivered as a consequence of behavior. The most frequently used schedules are listed in Table 3.4. The fixed-ratio schedule leads to the most rapid rate of response and is seen in baseball rules when a player is called out after three strikes. The use of slot machines demonstrates a variable-interval schedule of reinforcement.
The fixed-interval schedule has an oscillating rate of response that increases near the expected time of reward, a concept known as scalloping.

4
Clinical Neuropsychological Testing

The neuropsychological examination has a valuable place in di- agnosing and treating behavioral syndromes that are associated with medical, psychological, and psychiatric conditions. Clinical neuropsychology is a specialty within psychology that examines the relationship between behavior and brain functioning in the realms of cognitive, motor, sensory, and emotional functioning. Neuropsychologists integrate the medical and psychosocial his- tory with the chief complaint and pattern of performance on neu- ropsychological procedures. Such analysis determines whether results are consistent with a particular area of brain damage or a particular diagnosis.
The neuropsychological examination systematically assesses functioning in the realm of attention and concentration, mem- ory, language, spatial skills, sensory and motor abilities, as well as executive functioning and emotional status. Assessment in- struments are standardized against normal control subjects. This ensures that test administration and scoring are invariant across
time and examiners. Resulting data show whether the test is valid and reliable. The aim of neuropsychological tests is to achieve quantifiable and reproducible results that can be compared with the test scores of normal people of comparable age and demo- graphic background.
There are two types of tests: objective and projective. Objective tests are typically pencil-and-paper tests based on specific items and questions. They yield numerical scores and profiles subject to mathematical and statistical analy- sis. Projective tests present stimuli whose meaning is not immediately obvious. Projective tests presumably have no right or wrong answers. Those being tested impute mean- ings to the stimulus, based on psychological and emotional factors.
The student should be familiar with the types of neuropsy- chological assessment tests that are available, how they are administered, and their indications for use.
HELPFUL HINTS
The student should know the following terms, theoreticians, and concepts.
abstract reasoning accurate profile battery tests behavioral flexibility bell-shaped curve Bender Visual Motor
Gestalt Test catastrophic reaction clang association classification of
intelligence coping phase
dressing apraxia dysgraphia
dyslexia
EEG abnormalities Eysenck personality
inventory fluency
Gestalt psychology Halstead-Reitan House-Tree-Person Test individual and group
tests
intelligence quotient
(IQ)
learning disability left versus right
hemisphere disease Luria-Nebraska
Neuropsychological
Battery (LNNB) manual dexterity maturational levels memory: immediate,
recent, recent past,
remote mental age
mental status cognitive tasks
MMPI
motivational aspects of
behavior neuropsychiatric
tests
objective tests organic dysfunction performance subtests perseveration personality testing primary assets and
weaknesses projective tests psychodynamic
formulations Raven’s Progressive
Matrices
reaction times recall phase response sets Rorschach Test scatter pattern Shipley Abstraction
Test Stanford-Binet stimulus words
TAT
temporal orientation verbal subtests visual-object agnosia WAIS
WISC word-association
technique
25
▲▲▲▲▲▲ ▲▲▲▲▲▲▲▲▲
▲▲▲▲▲▲▲▲▲▲▲▲▲
▲▲▲▲▲▲▲▲ ▲▲▲ ▲
▲▲▲▲▲▲ ▲▲▲ ▲▲▲▲▲▲ ▲
26 4. Clinical Neuropsychological Testing
QUESTIONS
Directions
Each question or incomplete statement below is followed by five suggested responses or completions. Select the one that is best in each case.
. 4.1. For the general population, an intelligence quotient (IQ) of 100 corresponds to intellectual ability in the
A. 20th percentile B. 25th percentile C. 40th percentile D. 50th percentile E. 65th percentile
. 4.2. Neuropsychological deficits associated with left hemi- spheric damage include all of the following except
A. limb apraxia
B. visuospatial deficits
C. finger agnosia
D. aphasia
E. right–left disorientation
. 4.3. Neuropsychological referrals are made for
A. establishing a baseline of performance B. diagnostic purposes
C. ascertaining brain impairment
D. planning for rehabilitation
E. all of the above
. 4.4. True statements about projective personality tests include
A. They often focus on latent or unconscious aspects of personality.
B. The variety of responses is limited.
C. They tend to be more direct and structural than objec-
tive personality tests.
D. Instructions are usually specific.
E. None of the above
D. visual and motor coordination E. all of the above
4.7. Which is not true of the Wisconsin Card Sorting Test?
A. It assesses abstract reasoning
B. It assesses parietal lobe dysfunction
C. The patient is told during testing whether their re-
sponses are correct or incorrect
D. The examiner records the number of trials required to
achieve ten consecutive correct responses
E. The examiner changes the principle of sorting when
the task is mastered
4.8. In the Wechsler Adult Intelligence Scale (WAIS),
A. digit span is a subtest of the verbal component of the test
B. its latest revision is designed for persons aged 16 to 60
C. mentalretardationcorrespondstothelowest1%ofthe
population
D. the average range of IQ is 100 to 120
E. the verbal scale is more sensitive to normal aging
. 4.5. After taking the Wechsler Adult Intelligence Scale (WAIS), a patient was found to have poor concentration and attention. Select the WAIS subtest that most likely screened the patient for these symptoms.
A. block design
B. comprehension
C. arithmetic
D. digit symbol
E. picture completion
. 4.6. The Bender Visual Motor Gestalt Test is administered to assess
A. maturation levels in children B. organic dysfunction
C. loss of function
4.9. An
his mathematics instructor after failing a series of exami- nations since the beginning of the school year. The boy’s mother states that he complains about not being able to read as well as the other students. He also complains of difficulty hearing the teacher.
Which of the following tests of personality, intelli- gence, and achievement is most appropriate in assessing the boy’s specific learning challenges?
A. Rorschach Test
B. Wechsler Preschool and Primary Scale of Intelligence C. Wechsler Intelligence Scale for Children
D. Wide-Range Achievement Test
E. Minnesota Multiphasic Personality Inventory
Directions
Each set of lettered headings below is followed by a list of num- bered phrases. For each numbered phrase, select
A. If the item is associated with A only
B. If the item is associated with B only
C. If the item is associated with both A and B D. If the item is associated with neither A nor B
Questions 4.10–4.13
A. Rorschach Test
B. Thematic Apperception Test (TAT)
4.10. Assesses patient’s personality as a whole
4.11. Henry Murray
4.12. Is often included in a battery of tests for personality as-
sessment
4.13. John E. Exner’s Comprehensive System
8-year-old boy is referred for psychological testing by
Questions 4.14–4.15
A. Brief Psychiatric Rating Scale (BPRS)
B. Schedule for Affective Disorders and Schizophrenia
(SADS)
. 4.14. Ratings are made on the basis of mental status interview and do not require that the examiner ask any specific ques- tions
. 4.15. Highly structured interview
Directions
Each group of questions below consists of lettered headings fol- lowed by a list of numbered phrases or statements. For each numbered phrase or statement, select the one lettered heading that is most associated with it. Each lettered heading may be selected once, more than once, or not at all.
Questions 4.16–4.19
A. Battery Approach
B. Hypothesis Testing Approach C. Screening Approach
D. Mental Status Examination
. 4.16. Determines if a diagnosis can be made with less informa- tion
. 4.17. May help identify problems that were not mentioned by the patient
. 4.18. Is not considered part of neuropsychological testing
. 4.19. May overlook unexpected areas of deficits
Questions 4.20–4.24
A. Intellectual functioning B. Language
C. Executive functions
D. Memory
E. Visuospatial-constructional
. 4.20. California Verbal Learning Test II
. 4.21. Shipley Scale
. 4.22. Token Test
. 4.23. Wisconsin Card Sorting Test
. 4.24. Rey-Osterreith Complex Figure Test
Questions 4.25–4.29
A. Rorschach test
B. Luria-Nebraska Neuropsychological Battery
C. Halstead-Reitan Battery of Neuropsychological Tests D. Stanford-Binet Intelligence Scale
E. None of the above
. 4.25. Consists of ten tests, including the trail-making test and the critical flicker frequency test
. 4.26. Consists of 120 items, plus several alternative tests, appli- cable to ages 2 years to adulthood
4.27. A test of diffuse cerebral dysfunction to which normal children by the age of 7 years respond negatively
4.28. Furnishes a description of the dynamic forces of person- ality through an analysis of the person’s responses
4.29. Isextremelysensitiveinidentifyingdiscreteformsofbrain
damage, such as dyslexia
Questions 4.30–4.34
A. Raven’s Progressive Matrices
B. Sentence Completion Test
C. Shipley Abstraction Test
D. Thematic Apperception Test (TAT)
E. Wechsler Adult Intelligence Scale (WAIS)
. 4.30. A broad set of complex verbal and visuospatial tasks that are normatively summarized by three scales
. 4.31. Impaired performance is associated with posterior lesions of either cerebral hemisphere
. 4.32. Series of 20 black-and-white pictures depicting individ- uals of different ages and sexes involved in a variety of settings
. 4.33. More direct than most projective tests in soliciting re- sponses from the patient
. 4.34. Requires the patient to complete logical sequences
Questions 4.35–4.39
A. Frontal lobes
B. Dominant temporal lobe C. Nondominant parietal lobe D. Dominant parietal lobe
E. Occipital lobe
. 4.35. Patient not able to name common objects
. 4.36. Patient not able to name a camouflaged object but able to
name it when it is not camouflaged
. 4.37. Anyimproperlettersequenceinspelling“earth”backward
. 4.38. The loss of gestalt, loss of symmetry, and distortion of
figures
. 4.39. Two or more errors or two or more 7-second delays in
carrying out tasks of right–left orientation
Questions 4.40–4.43
A. Short-term memory loss
B. Signs of organic dysfunction
C. Korsakoff’s syndrome
D. Posterior right hemisphere lesion E. Damage to frontal lobes or caudate
. 4.40. Bender Visual Motor Gestalt
. 4.41. Benton Visual Retention Test
. 4.42. Wechsler Memory Scale
. 4.43. Wisconsin Card Sorting Test
Questions 4.44–4.48
A. Executive functions
B. Attention and concentration
4. Clinical Neuropsychological Testing 27
28 4. Clinical Neuropsychological Testing
C. Visuospatial–constructional D. Motor
E. None of the above
. 4.44. Clock drawing and facial recognition
. 4.45. Finger tapping
. 4.46. Trail-making test
. 4.47. Digit span
. 4.48. Wisconsin Card Sorting Test
Questions 4.49–4.53
A. Immediate memory B. Episodic memory C. Semantic memory D. Recent past memory E. Implicit memory
. 4.49. Memoryforautomaticskills,likespeakinggrammatically or driving a car
. 4.50. Recall of perceived material within 30 seconds of presen- tation
. 4.51. Memory for specific events, like a phone message
. 4.52. Retention of information over the past few months, like
current events
. 4.53. Memory for knowledge and facts, like the first president
of the United States
ANSWERS
4.1. The answer is D
An intelligence quotient (IQ) of 100 corresponds to the 50th per- centile in intellectual ability for the general population. Modern psychological testing began in the first decade of the 20th cen- tury when Alfred Binet (1857–1911), a French psychologist, developed the first intelligence scale to identify mentally chal- lenged youngsters. The Stanford-Binet Scale is a standardized intelligence test assessing both cognitive ability and intelligence from age 2 onward. It is useful for determining the presence of a learning disability, developmental delay, or brain damage in people with neurological insults. Its components include gen- eral intelligence, visuospatial processing, quantitative reasoning, fluid reasoning, knowledge, and memory.
4.2. The answer is B
Many functions are mediated by both the right and left cere- bral hemispheres. However, important qualitative differences be- tween the two hemispheres can be demonstrated by the presence of lateralized brain injury. Various cognitive skills that have been linked to the left or right hemisphere in right-handed people are listed in Table 4.1. Although language is the most obvious area that is largely controlled by the left hemisphere (with injuries leading to aphasias), the left hemisphere is also generally con- sidered to be dominant for limb praxis (i.e., performing complex movements, such as brushing teeth, commanding, or imitation). The left hemisphere has been associated with a cluster of deficits identified as Gerstmann syndrome (i.e., finger agnosia, dyscal- culia, dysgraphia, and right–left disorientation). In contrast, the
Table 4.1
Selected Neuropsychological Deficits Associated with Left or Right Hemisphere Damage
 
Left Hemisphere
Aphasia
Right–left disorientation
Finger agnosia
Dysgraphia (aphasic) Dyscalculia (number alexia) Constructional apraxia (details) Limb apraxia
Right Hemisphere
Visuospatial deficits
Impaired visual perception Neglect
Dysgraphia (spatial, neglect) Dyscalculia (spatial) Constructional apraxia (Gestalt) Dressing apraxia
Anosognosia
 
right hemisphere is thought to play a more important role in con- trolling visuospatial abilities and hemispatial attention, which are associated with the clinical presentations of constructional apraxia and neglect, respectively.
Although lateralized deficits such as these are typically char- acterized in terms of damage to the right or left hemisphere, it is important to keep in mind that the patient’s performance can also be characterized in terms of preserved brain functions. In other words, it is the intact brain tissue that drives many behavioral responses following injury to the brain.
4.3. The answer is E (all)
Most neuropsychological referrals are made for diagnostic pur- poses, to ascertain if brain impairment is present, or to dif- ferentiate among neurological or psychiatric disorders. Other important uses of testing include establishing a baseline of performance for assessing future change and planning for re- habilitation and management of behaviors affected by brain im- pairment. The specific methods of neuropsychological assess- ment reflect the individual’s unique presentation of symptoms and complaints, history and development, the perspective of the neuropsychologist, and the reason for referral.
A common referral issue involves documentation of level of functioning for a variety of purposes, including assessment of change or competence, especially in the presence of diagnoses such as dementia, stroke, and head injury.
4.4. The answer is A
Projective personality tests, in contrast to objective personality instruments, are more indirect and unstructured. Unlike objec- tive tests, in which the patient may simply mark “true” or “false” to given questions, the variety of responses to projective person- ality tests is almost unlimited. Instructions are usually general, allowing the patient’s fantasies to be expressed. Patients gener- ally do not know how their responses will be scored or analyzed, making it difficult to obtain a desired result. Projective tests typ- ically do not measure one particular personality characteristic such as type A personality (i.e., narrow-band measurement) but instead are designed to assess a personality as a whole (i.e., broad-band measurement).
Projective tests often focus on latent or unconscious aspects of personality. Obviously, psychologists and others differ in the degree to which they rely on unconscious information. In many projective techniques, patients are simply shown a picture of
something and asked to tell what the picture reminds them of. Projective techniques assume that when presented with an am- biguous stimulus such as an inkblot, for which there are an almost unlimited number of responses, the patients’ responses will re- flect fundamental aspects of their personalities. The ambiguous stimulus is a sort of screen on which individuals project their own needs, thoughts, or conflicts. In particular, a schizophrenic patient’s responses will often reflect a rather bizarre, idiosyn- cratic view of the world. Hundreds of different projective tech- niques have been developed—most of which are not used widely today.
4.5. The answer is C
The arithmetic subtest uses simple arithmetic tasks to assess at- tention and concentration. The block design subtest requires a subject to arrange a series of pictures to tell a story. This process tests performance and cognitive styles. The digit symbol subtest requires a subject to match digits and symbols in as little time as possible, as a test of performance. The comprehension sub- test reveals a subject’s ability to adhere to social consequences and to understand social judgments when the subject answers questions about how people should behave. On the picture com- pletion subtest, a subject must complete a picture with a missing part. Visuospatial defects appear when errors are made on this picture completion procedure.
4.6. The answer is E (all)
The Bender Visual Motor Gestalt Test, devised by the American neuropsychiatrist Lauretta Bender in 1938, is a technique that consists of nine figures that are copied by the subject (Fig. 4.1). It is administered as a means of evaluating maturation levels in children and organic dysfunction. Its chief applications are to determine retardation, loss of function, and organic brain defects in children and adults. The designs are presented one at a time to the subject, who is asked to copy them onto a sheet of paper. The subject is then asked to copy the designs from memory (Figs. 4.2 and 4.3); thus, the Bender designs can be used as a test of both visual and motor coordination and immediate visual memory.
4.7. The answer is B
Persons with damage to the frontal lobes or to the caudate nu- cleus, and some patients with schizophrenia, give abnormal re- sponses to the Wisconsin Card Sorting Test (WCST). It does not address parietal lobe damage. This test assesses abstract reasoning and flexibility in problem solving. Stimulus cards are presented to patients to sort into groups according to a principle established by the examiner but unknown to the patient. As the patient sorts the cards, he or she is told whether the responses are correct or incorrect. The number of trials required to achieve ten consecutive correct responses is recorded. When or if the pa- tient has mastered the task, the examiner changes the principle of sorting, and the number of trials required to achieve correct sorting is again recorded.
4.8. The answer is A
The Wechsler Adult Intelligence Scale (WAIS) comprises 11 subtests made up of six verbal (information, comprehension,
FIGURE 4.1
Test figures from the Bender Visual Motor Gestalt Test, adapted from Max Wertheimer. (Reprinted with permission from Bender L. A Vi- sual Motor Gestalt Test and Its Clinical Use. New York: American Orthopsychiatric Association; 1938:33.)
FIGURE 4.2
Bender Visual Motor Gesalt Test drawings of a 57-year-old brain- damaged woman.
4. Clinical Neuropsychological Testing 29
 
30 4.
Clinical Neuropsychological Testing

FIGURE 4.3
Bender Visual Motor Gesalt Test recall by the 57-year-old brain- damaged patient who drew Figure 4.2.
arithmetic, similarities, digit span, vocabulary), and five perfor- mance (picture completion, block design, picture arrangement, object assembly, digit symbol) subtests, which yield a verbal IQ, a performance IQ, and a combined full-scale IQ. The latest edi- tion, the WAIS-III, is designed for persons 16 to 89 years of age. There are also scales for children ages 4 to 6.5 and 5 through 15. The average or normal range of IQ is 90 to 110; IQ scores of at least 120 are considered superior. Mental retardation, de- fined as IQ below 70, corresponds to the lowest 2.2 percent of the population; 2 of every 100 people have IQ scores consistent with mental deficiency. The performance scale is more sensitive to normal aging than the verbal scale, which is more sensitive to education.
4.9. The answer is D
The Wide-Range Achievement Test evaluates content-specific foundation in mathematics, spelling, reading, and sciences. It may test individuals ranging from ages 5 to 94 and is standard- ized to account for stratifications in gender, ethnicity, geographic region, and socioeconomic status. The Rorschach Test is a pro- jective assessment test that requires interpretation of ink blots. It may be used to interpret defense mechanisms and thought disorders. The Wechsler Preschool and Primary Scale of Intel- ligence tests the ability to organize, process, and reason by way of picture completion, verbal comprehension, and block design. It may be completed without reading or writing and is designed for children between the ages of 4 and 6. The Wechsler Intelli- gence Scale for Children uses an identical approach for children and adolescents between the ages of 6 and 16. The Minnesota Multiphasic Personality Inventory is the most common objective personality test aimed at determining personality types, includ- ing pathologies and behavioral patterns.
Answers 4.10–4.13 4.10. The answer is C 4.11. The answer is B 4.12. The answer is C
4.13. The answer is A
Projective personality tests are indirect and unstructured. The variety of responses to projective personality tests are almost unlimited. Instructions are usually very general in nature, allow- ing the patient’s fantasies to be expressed. The patient does not
know how his or her responses will be scored or analyzed. Con- sequently, trying to feign the test becomes difficult. Projective tests typically do not measure one particular personality char- acteristic but are instead designed to assess one’s personality as a whole. Two of the most widely used projective personal- ity tests are the Rorschach Test and the Thematic Apperception Test (TAT). Many clinicians will include both of these tests in a battery of tests for personality assessment.
The Rorschach test was created by Herman Rorschach, a Swiss psychologist, around 1910. The Rorschach test is the most frequently used projective personality instrument. The test consists of ten ambiguous symmetrical inkblots. Using John E. Exner’s Comprehensive System, the examiner gives the patient the following instructions: “You’ll be given a series of inkblots. Look at each inkblot and tell me what you see.” The examiner gives the patient each card and asks, “What might this be?” The examiner writes down verbatim what the patient says, along with the initial reaction times and total time spent on each card. After completion of what is called the free-association phase, the ex- aminer conducts an inquiry phase to determine important aspects of each response that are crucial to scoring.
The TAT was designed by Henry Murray and Christina Morgan as part of a normal personality study conducted in the Harvard Psychological Clinic in 1943. The TAT consists of a series of 30 picture cards and a blank card. Typically, a patient is shown ten TAT cards and asked to make up stories about them. The patient is asked to tell what is going on in the picture, what was going on before the picture was taken, what the individu- als in the picture are thinking and feeling, and what is likely to happen in the future.
Answers 4.14–4.15
4.14. The answer is A
4.15. The answer is B
One of the more commonly used interview rating scales is the Brief Psychiatric Rating Scale (BPRS). To use the BPRS, the psychologist or psychiatrist completes a mental status interview with the patient and then rates that patient on a series of 18 psychiatric symptoms such as motor retardation, blunted affect, conceptual disorganization, anxiety, and guilt. Expanded defini- tions of each of these terms are provided to the examiner. The interviewer rates each domain on a seven-point scale from “not present,” the lowest rating, to “extremely severe,” the highest rat- ing. An experienced interviewer can complete the ratings in 2 or 3 minutes. The BPRS has been used extensively in drug outcome and other studies. The advantages of the BPRS are the reasonably high interrater reliability, the ease and speed of rating, and the well-defined symptom description. The BPRS ratings are made on the basis of a mental status interview and do not require that the examiner ask any specific questions of the patient.
The SADS is a highly structured interview instrument. The interviewer is required to ask each patient a series of prescribed questions to ensure that all relevant areas are addressed. For example, a patient is asked a question similar to the following: “Have you ever heard voices or other things that weren’t there or
that other people couldn’t hear or see?” Based on the patient’s response, the examiner asks other detailed, prescribed follow- up questions concerning hallucinations, or if the response was negative, the interviewer moves on to the next question in a different area. This approach ensures that all areas are covered in a comprehensive fashion. The SADS is especially helpful for establishing a reliable diagnosis. The SADS can also be used as an index of behavioral severity. Behavioral changes can be determined by repeated administration.
The advantage of the SADS is that it is comprehensive and has reasonably good reliability. One disadvantage is its length; another is that in the structured interview the interviewer must read questions from a lengthy booklet, which makes it difficult to establish eye contact and rapport with the patient. The SADS has been used for both research and clinical purposes, probably more frequently for the former.
Answers 4.16–4.19 4.16. The answer is C 4.17. The answer is A 4.18. The answer is D
4.19. The answer is B
The battery approach to neuropsychological assessment typi- cally includes a large variety of tests that measure most cogni- tive domains as well as sensory and motor skills. The battery approach has the advantage of identifying problems that the pa- tient might not have mentioned and that the medical history may not necessarily predict. However, it has the disadvantage of being very time-consuming (i.e., 6 to 8 hour examination).
In the screening approach, the neuropsychologist utilizes a core set of screening procedures as a first step in determin- ing whether a diagnosis can be made with less information or whether additional testing is necessary. Standardized screening approaches have the advantage of uniform normative reference groups, although many clinicians routinely use a set of tests drawn from many sources.
In some cases, usually involving an acute or severe cogni- tive impairment, it is simply not feasible to administer extensive cognitive examination procedures. The neuropsychologist might appropriately rely on bedside mental status examinations or very brief cognitive screening procedures to address the referral issues.
The qualitative hypothesis testing approach is characterized by detailed evaluation of areas of functioning that are related to the patient’s complaints and predicted areas of impairment, with relatively less emphasis on aspects of functioning that are less likely to be impaired. The hypothesis testing approach has been particularly helpful in illuminating the different roles of the two brain hemispheres. This approach has the advantage of efficiently honing in on areas of impairment and producing a detailed description of the deficits from a cognitive standpoint, but it has the shortcoming of potentially overlooking unexpected areas of deficits.
4. Clinical Neuropsychological Testing 31 Answers 4.20–4.24
4.20. The answer is D 4.21. The answer is A 4.22. The answer is B 4.23. The answer is C
4.24. The answer is E
A list of examples of common neuropsychological tests and tech- niques is provided in Table 4.2.
Answers 4.25–4.29 4.25. The answer is C 4.26. The answer is D 4.27. The answer is E 4.28. The answer is A
4.29. The answer is B
Various neuropsychiatric tests, including the Halstead-Reitan and the Luria-Nebraska batteries, are sometimes useful in bring- ing to light subtle organic dysfunctions that are undetected in standard psychiatric, psychological, and even neurological as- sessments. The Halstead-Reitan Battery of Neuropsychological Tests consists of ten tests, including the trail-making test and the critical flicker frequency test. It was developed in an attempt to improve the reliability of the criteria used to diagnose brain damage. Assessment data were gathered on a group of patients with left hemisphere injury, right hemisphere injury, and global involvement. The trail-making test is a test of visuomotor percep- tion and motor speed, and the critical flicker frequency test (not- ing when a flicker light becomes steady) tests visual perception.
The Luria-Nebraska Neuropsychological Battery (LNNB) is extremely sensitive in identifying discrete forms of brain damage, such as dyslexia (an impairment in the ability to read) and dyscalculia (an inability to perform arithmetical operations), rather than global forms of damage.
The Stanford-Binet Intelligence Scale is one of the tests most frequently used in the individual examination of children. It consists of 120 items, plus several alternative tests, applicable to the ages between 2 years and adulthood. The tests have a variety of graded difficulties, both verbal and performance, designed to assess such functions as memory, free association, orientation, language comprehension, knowledge of common objects, abstract thinking, and the use of judgment and reasoning.
The Rorschach Test is a psychological test consisting of ten inkblots that the person is asked to look at and interpret. It furnishes a description of the dynamic forces of personality through an analysis of the person’s responses.
The face-hand test, devised by Lauretta Bender, is a test of diffuse cerebral dysfunction to which normal children by the age of 7 years respond negatively. The person, whose eyes are
32 4. Clinical Neuropsychological Testing
Table 4.2
Selected Tests of Neuropsychological Functioning
 
Area of Function
INTELLECTUAL FUNCTIONING
Wechsler Intelligence Scales
Shipley Scale
ATTENTION AND CONCENTRATION
Digit Span
Visual Memory Span
MEMORY
Wechsler Memory Scale III
California Verbal Learning Test II
LANGUAGE
Boston Diagnostic Aphasia
Examination
Boston Naming Test—Revised Token Test
VISUOSPATIAL-CONSTRUCTIONAL
Clock Drawing Rey-Osterreith Complex
Figure Test
MOTOR
Finger Tapping Grooved Pegboard
EXECUTIVE FUNCTIONS
Wisconsin Card Sorting Test Trail-Making Test
PSYCHOLOGICAL FACTORS
Beck Depression Inventory
Minnesota Multiphasic Personality Inventory 2
Comment
Age-stratified normative references; appropriate for adults up to age 89, adolescents, and young children
Scale Brief (20 minute) paper and pencil measure of multiple choice vocabulary and open-ended verbal abstraction
Auditory–verbal measure of simple span of attention (Digits Forward) and cognitive manipulation of increasingly longer strings of digits (Digits Backward)
Visual-spatial measure of ability to reproduce a spatial sequence in forward and reverse order
Comprehensive set of subtests measuring attention and encoding, retrieval, and recognition of various types of verbal and visual material with both immediate recall and delayed retention; excellent age-stratified normative comparisons for adults up to age 89 with intellectual data for direct comparison
Documents encoding, recognition, and both immediate and 30-minute recall; affords examination of possible learning strategies as well as susceptibility to semantic interference with alternate and short forms available
Comprehensive assessment of expressive and receptive language functions
Documents word finding difficulty in a visual confrontation format
Systematically assesses comprehension of complex commands using standard token stimuli that vary
in size, shape, and color
Useful screening technique is sensitive to organization and planning as well as constructional ability Ability to draw and later recall a complex geometric configuration; sensitive visual memory as well as
executive deficits in development of strategies and planning
Standard measure of simple motor speed; particularly useful for documenting lateralized motor impairment
Ability to rapidly place notched pegs in slotted holes; measures fine finger dexterity as well as eye–hand coordination
Measure of problem-solving efficiency is particularly sensitive to executive deficits of perseveration and impaired ability to flexibly generate alternative strategies in response to feedback
Requires rapid and efficient integration of attention, visual scanning, and cognitive sequencing
Brief (5 to 10 minutes) self-report measure that is sensitive to symptoms of depression; best for screening depression in adults up to late middle-age, who can be expected to frankly report symptoms; available in standard (21 four-choice items) or short (13-item) form
This psychometrically developed self-report instrument remains highly useful for documenting quantitative levels of self-reported symptoms that can be objectively compared to known populations; drawbacks include administration time (567 true–false questions, requires about 1 to 1.5 hours or more) for frail individuals, and the emphasis on pathological features for persons who are generally psychologically healthy; advantages include well-developed validity scales and availability of many symptom-specific subscales that have been identified over the years
 
closed, is touched simultaneously on the cheek and the hand; retesting is done with the person’s eyes open. The results are considered positive if the person fails consistently to identify both stimuli within ten trials.
Answers 4.30–4.34 4.30. The answer is E 4.31. The answer is A
4.32. The answer is D
4.33. The answer is B
4.34. The answer is C
Assessment of intellectual functioning serves as the cornerstone of the neuropsychological examination. The Wechsler Intelli- gence Scales, based on carefully developed normative standards, represent the traditional gold standard in intellectual assessment. The latest revision of this instrument is the Wechsler Memory
Scale III. In general, the Wechsler intelligence scales use a broad set of complex verbal and visuospatial tasks that are normatively summarized as a Verbal IQ, Performance IQ, and Full Scale IQ. In the context of a neuropsychological examination, the patient’s performance across the procedures provides useful information regarding longstanding abilities as well as current functioning. Most neuropsychologists recognize that the summary IQ values provide only a ballpark range for characterizing an individual’s general level of functioning. Therefore, it is usually more appro- priate and meaningful to characterize an individual’s intellectual functioning in terms of range of functioning (e.g., borderline, low average, average, high average, superior) represented by the IQ value rather than the specific value.
Raven’s Progressive Matrices require the patient to complete a design by selecting the stimulus that completes a design in which a part is omitted from a multiple-choice pictorial display. Impaired performance is associated with poor visuoconstructive ability and with posterior lesions of either cerebral hemisphere, but receptive language deficit may contribute to poor perfor- mance in patients with dominant hemisphere damage.
The TAT consists of a series of 20 black-and-white pictures that depict individuals of both sexes and of different age groups involved in a variety of different settings. For example, on Card 1, a young boy is shown sitting at a table looking at a violin. Card 2 depicts a farm scene in which a young woman in the foreground is carrying books in her hands; a man is working in the fields nearby, and an older woman is seen in the background. Typically, a patient is shown ten TAT cards and asked to make up stories about them. The patient is asked to tell what is going on in the picture, what was going on before the picture was taken, what the individuals in the picture are thinking and feeling, and what is likely to happen in the future. The stories the patient make up concerning the pictures, according to the projective hypothesis, reflect the patient’s own needs, thoughts, feelings, stresses, wishes, desires, and view of the future. An example of a TAT card is presented in Figure 4.4.
Although a projective instrument, the Sentence Completion Test is much more direct in soliciting responses from the patient. The patient is simply presented with a series of incomplete sen- tences and asked to complete each sentence stem with the first response that comes to mind. Examples of possible incomplete sentences are as follows:
My father seldom . . .
Most people don’t know that I’m afraid of . . . When I was a child, I . . .
When encountering frustration, I usually . . .
The purpose of the test is to elicit, somewhat indirectly, information about the patient that cannot be elicited by other measures. Since the patient responds in writing, the examiner’s time is limited. The length of time it takes to complete this test varies greatly, depending on the number of incomplete sentences.
The Shipley Abstraction Test requires the patient to complete logical sequences; it assesses the patient’s capacity to think ab- stractly. Because performance on a test of this type is related to educational background, an accompanying vocabulary test is also given to the patient, and a comparison is made between
FIGURE 4.4
Card 12F of the Thermatic Apperception Test. (Reprinted with per- mission from Murray HA. Thematic Apperception Test, Cambridge, MA: Harvard University Press, Copyright ⃝c 1943 President and Fel- lows of Harvard College, ⃝c 1971 Henry A. Murray.)
the patient’s performances on the two tests. A low abstraction score in relation to vocabulary level is interpreted as reflecting an impairment in conceptual thinking.
Answers 4.35–4.39 4.35. The answer is B 4.36. The answer is E 4.37. The answer is A 4.38. The answer is C
4.39. The answer is D
Numerous mental status cognitive tasks are available to test and localize various brain dysfunctions. Construction apraxia—the loss of gestalt, loss of symmetry, and distortion of figures—seen in the task of copying the outline of simple objects, is localized to the nondominant parietal lobe. Dysfunction of the occipital lobes is suggested when a patient cannot name a camouflaged object but can name it when it is not camouflaged. Two or more errors or two or more 7-second delays in carrying out tasks of right–left orientation (e.g., place left hand to right ear, right elbow to right knee) are localized to dysfunction of the domi- nant parietal lobe. A dysfunction in concentration is thought to be localized to the frontal lobes and can be tested by eliciting any improper letter sequence in spelling “earth” backward. In anomia, the patient cannot name common objects (e.g., watch and key); the impairment is localized to the dominant temporal lobe.
4. Clinical Neuropsychological Testing 33

34 4. Clinical Neuropsychological Testing
Answers 4.40–4.43 4.40. The answer is B 4.41. The answer is A 4.42. The answer is C
4.43. The answer is E
The Wechsler Memory Scale screens for verbal and visual mem- ory and yields a memory quotient. The results can reveal whether a subject has amnestic Korsakoff’s syndrome. The Wisconsin Card Sorting Test assesses a person’s abstract reasoning ability and flexibility in problem solving. The results can reveal whether a person has damage to the frontal lobes or to the caudate.
The Benton Visual Retention Test screens for short-term mem- ory loss.
Signs of organic dysfunction may be screened for by the Ben- der Visual Motor Gestalt Test. Posterior right hemisphere lesions can be revealed through a facial recognition test.
Answers 4.44–4.48 4.44. The answer is C 4.45. The answer is D 4.46. The answer is A 4.47. The answer is B
4.48. The answer is A
Digit Span is the auditory–verbal measure of simple span of at- tention (Digits Forward) and cognitive manipulation of increas- ingly longer strings of digits (Digits Backward). Facial Recog- nition assesses matching and discrimination of unfamiliar faces. Clock Drawing is a useful screening technique sensitive to or- ganization and planning as well as constructional ability. Finger Tapping is the standard measure of simple motor speed, particu- larly useful for documenting lateralized motor impairment. The Wisconsin Card Sorting Test is a measure of problem-solving efficiency. It is particularly sensitive to executive deficits of per- severation and impaired ability to generate alternative strategies in response to feedback. The Trail-Making Test requires rapid and efficient integration of attention, visual scanning, and cog- nitive sequencing.
Answers 4.49–4.53 4.49. The answer is E 4.50. The answer is A 4.51. The answer is B 4.52. The answer is D
4.53. The answer is C
Immediate (or short-term) memory may be defined as the re- production, recognition, or recall of perceived material within a period up to 30 seconds after presentation. It is most often assessed by digit repetition, reversal (auditory), and memory- for-designs (visual) tests. Both an auditory–verbal task, such as digit span or memory for words or sentences and a nonverbal visual task, such as memory for designs or for objects or faces, should be given to assess a patient’s immediate memory. Pa- tients can also be asked to listen to a standardized story and then repeat it as accurately as possible. Patients with lesions of the right hemisphere are likely to show more severe defects on vi- sual nonverbal tasks than on auditory–verbal tasks. Conversely, patients with left hemisphere disease, including those who are not aphasic, are likely to show severe deficits on the auditory– verbal tests, with variable performance on the visual nonverbal tasks.
Recent past memory concerns the retention of information over the past few months. Patients can be asked questions about current events.
Remote memory is the ability to remember events in the dis- tant past. It is commonly believed that remote memory is well preserved in patients who show pronounced defects in recent memory, but the remote memory of senile and amnesic patients is usually significantly inferior to that of normal persons of com- parable age and education. Even patients who appear to be able to recount their past fairly accurately show gaps and inconsis- tencies in their recitals on close examination.
Memory theorists have described three other types of mem- ories: episodic, for specific events (e.g., a telephone message); semantic, for knowledge and facts (e.g., the first president of the United States); and implicit, for automatic skills (e.g., speaking grammatically or driving a car). Semantic and implicit memory do not decline with age, and persons continue to accumulate in- formation over a lifetime. A minimal decline in episodic memory with aging may relate to impaired frontal lobe functioning.

5
Theories of Personality and Psychopathology

Psychoanalysis is one of the fundamental principles within psy- chiatry. It was developed by Sigmund Freud (1856–1939), and although it has advanced far beyond Freud, it can be said that his influence is still strong and pervasive. Psychoanalysis is the bedrock of psychodynamic understanding and forms the fundamental point of reference for a variety of types of ther- apeutic intervention. It embraces not only psychoanalysis it- self but also various forms of psychoanalytically oriented psy- chotherapies and related therapies involving psychodynamic concepts.
All major psychological theories of personality involve the basic premise that a person’s early psychosocial development shapes what comes later—that the impact of childhood events, beliefs, experiences, and fantasies continue, consciously or un- consciously, throughout life and account for adult behavior.
Freud’s revolutionary contributions to the understanding of the human mind and psyche continue to stimulate, provoke, and challenge students of personality and psychopathology today.
His basic tenets of the unconscious mind, psychosexual devel- opment, and psychodynamics remain the bedrock of psychoan- alytic theory even though many have disagreed with, modified, or expanded on his ideas. No matter how a particular theorist may feel about Freud’s ideas, each must begin with a thorough knowledge of his contributions.
There have been many other psychoanalytic personality the- orists with widely varying views on development. These in- clude Alfred Adler (1870–1937), Erik Erikson (1902–1994), Karen Horney (1885–1952), Carl Gustav Jung (1875–1961), Melanie Klein (1882–1960), Harry Stack Sullivan (1892–1949), and Heinz Kohut (1913–1981). Schools of thought, encompass- ing a variety of different theories, include ego psychology, object relations, self-psychology, and interpersonal psychology, among others. Each approach has its own perspective on personality de- velopment and the development of psychopathology.
Students should study the questions and answers below to test their knowledge in this area.
HELPFUL HINTS
Students should know the various theorists, their schools of thought, and their theories.
Karl Abraham abreaction
Alfred Adler
Franz Alexander Gordon Allport analytical process attention cathexis birth trauma
Joseph Breuer cathexis
conscious
defense mechanisms displacement
dream work
The Ego and the Id
ego functions
ego psychology Erik Erikson
Eros and Thanatos Anna Freud
free association
Erich Fromm fundamental rule Karen Horney hypnosis
hysterical phenomena infantile sexuality instinctual drives
The Interpretation of Dreams
Carl Gustav Jung latent and manifest
dreams
libido and instinct
theories Abraham Maslow Adolph Meyer multiple self-
organizations
nocturnal sensory stimuli
object constancy object relations parapraxes preconscious system pregenital
primary and secondary gains
primary and secondary processes
primary autonomous functions
psychic determinism psychoanalytic theory psychodynamic thinking psychoneurosis psychosexual
development Otto Rank
reality principle reality testing regression
Wilhelm Reich repetition compulsion repression
resistance
secondary revision signal anxiety
Studies on Hysteria Harry Stack Sullivan symbolic representation synthetic functions of
the ego
topographic theory transference unconscious motivation wish fulfillment
35
▲▲▲▲▲▲▲▲▲▲▲▲▲ ▲▲▲▲
▲▲▲▲▲▲▲▲▲▲▲▲▲▲▲
▲▲▲▲▲▲▲▲▲ ▲▲ ▲ ▲▲▲
▲▲▲▲▲▲▲▲▲▲▲▲▲▲▲▲▲▲▲▲
36 5. Theories of Personality and Psychopathology
QUESTIONS
Directions
Each question or incomplete statement below is followed by five suggested responses or completions. Select the one that is best in each case.
. 5.1. Mr. A was a 26-year-old white man who had a history of bipolar I disorder. He was brought in for treatment after not completing the last required course for his advanced degree and being arrested for disturbing the peace. He had consistently lied to his family about where he stood with his coursework and about having skipped an examination that would have qualified him to use his professional de- gree. He had also not told them that he had been using marijuana almost daily for a number of years and occa- sionally used hallucinogens. His arrest for disorderly con- duct was for swimming naked in an apartment complex in the middle of the night while under the influence of hallu- cinogens. Mr. A spent most of his time reading and trying to write but compared himself unfavorably with other fa- mous writers, feeling himself to be inferior. (Adapted from Paul C. Mohl, MD, and Adam M. Brenner, MD.)
How would Alfred Adler view Mr. A?
A. Mr. A’s problems are a failure of adaptation to his men- tal illness and adult life.
B. Mr. A’s rebellion against conformity is a defense against his fear of being away from his mother’s pro- tection and domination.
C. Mr. A uses drugs and psychosis as escapes to maintain some degree of self-esteem.
D. Mr. A has been attempting to move from inferiority to mastery in fantasy rather than in realistic achievement.
E. Mr. A’s process of self-realization has been blocked.
. 5.2. Adolf Meyer would treat Mr. A by
A. assistingMr.Ainintegratingalienpartsofhimselfinto his ego complex
B. focusing on the adequate treatment of Mr. A’s mental disorder
C. challenging Mr. A to develop attachments outside of the family
D. encouraging Mr. A to join a support and educational group to better understand and accept his mental ill- ness
E. none of the above
. 5.3. A young woman presents to you complaining of lack of energy, trouble sleeping, depression, and hopelessness that has been present for the past year. You diagnose her with major depressive disorder. Which of the following would have been Freud’s explanation of this disorder?
A. Her depression is actually internally directed anger.
B. Her internal good objects have been destroyed by ag-
gression and greed.
C. She never mastered the trust versus mistrust stage of
ego development.
D. She is being persecuted by a tormenting internal object. E. She feels despair that her self-object needs will not be
met by others.
5.4. Lacanian theory
A. places heavy emphasis on linguistics
B. has little place for biology or drives
C. postulates that an individual is embedded in political
and societal structure
D. views the analytical process as an effort to recognize
alienation from one’s true self
E. all of the above
5.5. The Oedipus complex as described by Freud involves all of the following except
A. adult sexuality
B. rivalries
C. anal phase
D. intense love relationships E. both mother and father
5.6. The work of Anna Freud, daughter of Sigmund Freud, included all of the following except
A. contributions to child psychoanalysis
B. development of modern ego psychology
C. studies on the function of the ego in personality devel-
opment
D. contradictions to her father’s claims about psychosex-
ual development
E. expansion on individual defense mechanisms
5.7. According to Freud, the Oedipus complex is resolved through
A. the castration complex
B. the acting out of symbolic rivalries
C. moving on to the genital stage of development D. the realization of one’s gender identity
E. identification with the opposite-sex parent
5.8. EriksondiffersfromFreudbyhisplacinggreateremphasis on
A. interpersonal relationships
B. cultural factors in development C. instinctual drives
D. psychosexual development
E. object relations
5.9. You have a 72-year-old patient who has been very con- cerned with her appearance ever since you met her. She has had three facelifts, never leaves the house without makeup, and refuses to allow her grandchildren to call her “grand- mother.” Which of the following of Erikson’s stages is this woman having difficulty mastering?
A. Narcissistic
B. Generativity versus stagnation
C. Egocentric
D. Identity versus role confusion E. Integrity versus despair
. 5.10. Which of the following about Melanie Klein is false? A. She denounced Freud’s “death instinct.”
B. She described the “depressive position.”
C. She coined the term “persecutory anxiety.” D. She stressed the role of intrapsychic fantasy. E. She was a child analyst.
. 5.11. WhichofthefollowingstatementsregardingFreud’sview of hypnosis is not true?
A. Freud believed hypnosis encouraged the patient to please the hypnotist.
B. Freudbelievednopatientwascompletelyrefractoryto hypnosis.
C. Freud eventually abandoned the use of hypnosis for the use of free association.
D. Freudbelievedhypnosisconcealedaspectsoftransfer- ence.
E. None of the above
. 5.12. According to Otto Rank, death fear is
A. the fear of dying usually associated with a phobia
B. the fear of losing all ties in the process of becoming
separate
C. the fear of dying “before one’s time”
D. the fear of losing one’s identity by fusing with another
person
E. none of the above
. 5.13. A patient of yours reports having recurrent dreams of snakes shedding their skins. According to Carl Gustav Jung, this image is an example of which of the follow- ing?
A. Illusions
B. Phallic symbol C. Archetypes
D. Manifest content E. Primary process
. 5.14. Whichofthefollowingtheoristsisconsideredthefounder of the attachment theory?
A. John Bowlby B. Otto Kernberg C. Melanie Klein D. Heinz Kohut E. Adolf Meyer
. 5.15. Erik Erikson’s epigenetic principle states
A. the genetic component of personality must be explored to fully understand the ego
B. development spans the entire life cycle, from infancy through old age and senescence
5.16.
5.17.
5.18.
5.19.
5.20.
C. each developmental stage must be completed but in no particular order
D. each sequential stage must be satisfactorily resolved for development to proceed smoothly
E. none of the above
According to Erich Fromm, which of the following char- acter types are typical of modern capitalist society?
A. Exploitative
B. Hoarders
C. Marketers
D. Receptive
E. All of the above
Separation and individuation
A. have no associated anxiety
B. involve attaining a sense of object permanence C. involve a “practicing” subphase
D. begins at approximately 8 or 9 months of age E. is based on the work of Dan Stern, MD
Whichofthefollowingisnotconsideredamaturedefense mechanism?
A. Suppression B. Somatization C. Asceticism D. Anticipation E. Altruism
According to Carl Gustav Jung, archetypes are
A. instinctual patterns
B. expressed in mythological images
C. expressed in representational images
D. organizational units of the personality
E. all of the above
All of the following statements concerning the concept of the preconscious are true except
A. It is those mental events brought to consciousness by focusing attention.
B. It acts as a censor to unacceptable wishes and desires.
C. It is characterized by primary process thinking.
D. It interfaces with both the unconscious and the con-
scious.
E. It is part of the topographical model of the mind for-
mulated by Freud.
Freud declared the road to understanding the unconscious lies in which of the following?
A. Instinct control
B. Repression of preconscious desires
C. Mastering the phases of ego development D. Understanding infant sexuality
E. Interpretation of dreams
5.21.
5. Theories of Personality and Psychopathology 37
38 5. Theories of Personality and Psychopathology
5.22. A 64-year-old woman works as a seamstress in the garment factory of a clothing company. The factory is poorly lit, poorly ventilated, and overcrowded with employees. She works 16 hours per day, is allowed only one restroom break per day, and may not eat lunch. However, the seamstress continues to work at the factory because she believes her work to be meaningful and that it will someday make her wealthy. Her outlook pertains to which of the following theorists’ beliefs?
A. Gordon Allport B. Abraham Maslow C. George Kelly
D. Carl Rogers
E. Viktor Frankl
Directions
Each group of questions below consists of lettered headings fol- lowed by a list of numbered phrases or statements. For each numbered phrase or statement, select the one lettered heading that is most associated with it. Each lettered heading may be selected once, more than once, or not at all.
Questions 5.23–5.28
A. Autistic phase B. Symbiotic phase C. Hatching
D. Practicing
E. Rapprochement
F. Object constancy
. 5.23. The child gradually differentiates out of the symbiotic matrix
. 5.24. The infant functions as though he and his mother were an omnipotent system
. 5.25. The infant’s ability to move physically away from the mother by locomotion
. 5.26. A stage of absolute primary narcissism
. 5.27. The crisis is separation anxiety
. 5.28. The ability to maintain a relationship with one object
whether or not needs are being satisfied
Questions 5.29–5.33
A. Initiative versus guilt
B. Intimacy versus isolation
C. Trust versus mistrust
D. Industry versus inferiority
E. Generativity versus stagnation
. 5.29. Competence
. 5.30. Care
. 5.31. Love
. 5.32. Hope
. 5.33. Purpose
Questions 5.34–5.38
A. Basic trust versus mistrust B. Integrity versus despair
C. Initiative versus guilt
D. Intimacy versus isolation
E. Identity versus role diffusion
5.34. Late adulthood
5.35. Early adulthood
5.36. Puberty and adolescence 5.37. Early childhood
5.38. Infancy
Questions 5.39–5.43
A. Erik Erikson
B. Anna Freud
C. Sigmund Freud
D. Melanie Klein
E. Harry Stack Sullivan
. 5.39. Childhood and Society
. 5.40. Civilization and its Discontents
. 5.41. Envy and Gratitude
. 5.42. The Ego and the Mechanisms of Defense
. 5.43. Schizophrenia as a Human Process
Questions 5.44–5.47
A. Manifest dream content B. Latent dream content C. Dream work
D. Secondary revision
. 5.44. Unconscious thoughts and wishes
. 5.45. What is recalled by the dreamer
. 5.46. Mental operation of making the unconscious known
. 5.47. Work of organizing aspects of a dream into less bizarre
form
Questions 5.48–5.51
A. Urethral stage B. Genital stage C. Latency stage D. Anal stage
E. Phallic stage
. 5.48. Further integration of oedipal identifications
. 5.49. Striving for separation from dependence and control by
parents
. 5.50. Lying the foundation for gender identity
. 5.51. The ultimate separation from dependence on and attach-
ment to the parents
Questions 5.52–5.55
A. Displacement B. Projection
C. Reaction formation D. Repression
E. Sublimation
. 5.52. A young woman gets into an argument with her boyfriend. Although very upset, she remains silent as he tells her she is worthless. When she gets home, the young woman picks a fight with her younger sister over nothing and begins yelling at her.
. 5.53. A young man is very envious of his best friend. Although it is difficult to admit, he believes his friend is more success- ful and better looking and is always the life of the party. To the contrary, this young man tells his family that his best friend is envious of him despite there being no evidence that this is so.
. 5.54. An 18-year-old young man lives alone with his mother and despises her. He is embarrassed that he has these feelings and compensates by hovering over her, attending to her every need.
. 5.55. A middle-aged man has had an unconscious desire to con- trol others for as long as he can remember. To fulfill this need, he became a prison guard.
ANSWERS
5.1. The answer is D
Alfred Adler (1870–1937) posited striving for self-esteem through overcoming a sense of inferiority, which he saw as an inevitable force in the human condition as a result of its extended childhood. As seen from the Alderian point of view, Mr. A has been attempting to make the normal step of moving from inferi- ority to mastery in fantasy instead of through realistic achieve- ment. He maintains himself in fantasy as a writer while fail- ing at the accomplishments that would enable him to become a writer.
Sandor Rado (1890–1972) theorized that cultural factors of- ten cause excessive hedonic control by interfering with the or- ganism’s ability for self-regulation. Rado would have framed Mr. A’s difficulties as failures of adaptation to his mental illness and adult life and as a regression to the hedonic level of adaptation in which pleasure is sought and pain avoided.
Wilhelm Reich (1897–1957) made major contributions to psychoanalysis in the area of character formation and character types. The term character armor refers to the personality’s de- fenses that serve as resistance to self-understanding and change. Reich might see Mr. A’s rebellion against conformity as a defense against his fear of being away from his mother’s protection and domination. The more Mr. A rebels, the more tightly he binds himself to his mother.
Harry Stack Sullivan (1892–1949) would probably see Mr. A as arrested in childhood. His fear of displeasing his mother led him to give up healthy self-esteem strivings for independence, favoring a distant yet dependent position. Mr. A uses drugs and psychosis as escapes to maintain some degree of self-esteem.
Karen Horney (1885–1952) proposed three separate concepts of the self: the actual self (the sum total of a person’s experience), the real self (the harmonious, healthy person), and the idealized
self (the neurotic expectation or glorified image that a person believes he or she should be). From the standpoint of Horney, Mr. A’s process of self-realization has been blocked in all three directions. He has not developed the ability to love and trust; he expresses opposition in an unhealthy way, and he has made self-defeating moves toward independence.
5.2. The answer is B
Adolf Meyer (1866–1952) would focus first on the adequate treatment of Mr. A’s mental disorder. Meyer emphasized the in- terrelationship of symptoms and individual psychological and biological functioning. Controlling biological forces at work to disrupt Mr. A’s life is always a primary goal. Because Mr. A is still dependent on his parents, they are included in the treatment plan and might be seen separately by a social worker. Both Mr. A and his parents are told that failure to control the symptoms of his mental illness could cost him his life and any satisfaction that he might derive from it. Mr. A’s mood swings are stabilized on an appropriate medication. Later, Mr. A would begin the dis- tributive analysis phase of his treatment and would be asked to reflect on the impact of his bipolar I disorder and his avoidance of responsibility in his life.
5.3. The answer is A
Freud originally understood depression as internally directed anger. In his view, the self-reproaches and the loss of self-esteem commonly experienced by depressed patients are directed not at the self but rather at an ambivalently experienced introject. He noted that in some cases, the only way the ego can give up an object is to introject it, so the anger directed at the ambivalently held object takes on the clinical manifestation of the depression.
From the self-psychological point of view associated with Heinz Kohut, depression is related to a sense of despair about getting one’s self object needs met by people in the environment. Melanie Klein suggested that depression is linked to a reactiva- tion of the depressive position; depressed patients are convinced that they have destroyed their internal good objects because of their own aggression and greed. As a result, they feel persecuted by internal bad objects while longing for the lost love objects.
Erikson hypothesized that the depressed patient’s experience of being empty and of being no good is an outgrowth of a devel- opmental derailment. Depression therefore results from failing to develop a basic sense of trust or the virtue of hope. From an object relations perspective, many depressed patients uncon- sciously experience themselves to be at the mercy of a torment- ing internal object that is unrelenting in its persecution of them. In cases of psychosis, that primitive forerunner of the superego may actually be hallucinated as a voice that is unrelentingly critical.
5.4. The answer is E (all)
The French psychoanalyst Jacques Lacan (1901–1981) made a lasting impression on French psychoanalysis as well as on literary and film criticism in academic departments throughout the world. Lacan’s reading of Freud relies heavily on linguistics. The notion that human beings are constituted by language is one of three basic principles endorsed by Lacan. The unconscious is
5. Theories of Personality and Psychopathology 39
40 5. Theories of Personality and Psychopathology
structured like a language that consists only of signifiers; biology and drives have no place in his theory. Second, the ego does not exist as an autonomous structure. A third principle is that an individual is inevitably embedded in political and societal structures that cannot be transcended.
Lacan thrived on being unorthodox. He denied the signifi- cance of diagnoses, rules, or established schools of thought. He saw the analytical process as an effort to recognize the alien- ation from one’s true self. Analysis was also designed to bring out underlying structures and contexts in the unconscious.
5.5. The answer is C
The phallic period (not anal phase), during which the Oedi- pus complex emerges, is a critical phase of development for the budding formation of the child’s own sense of gender identity— as decisively male or female—based on the child’s discovery and realization of the significance of anatomical sexual differ- ences. Adult sexual adjustment is said to rely on the attachment to one parent and the identification with the other. The events associated with the phallic phase also set the stage for the devel- opmental predisposition to later psychoneuroses. Freud used the term Oedipus complex to refer to the intense love relationships, together with their associated rivalries, hostilities, and emerging identifications, formed during this period between the child and parents.
5.6. The answer is D
Anna Freud (1895–1982) was not known to have contradicted her father’s claims on psychosexual development. Anna Freud, the daughter of Sigmund Freud, ultimately made her own set of unique contributions to psychoanalysis. Whereas her father fo- cused primarily on repression as the central defense mechanism, Anna Freud greatly elaborated on individual defense mecha- nisms, including reaction formation, undoing, introjection, iden- tification, projection, turning against the self, reversal, and subli- mation. She was also a key figure in the development of modern ego psychology in that she emphasized that there was “depth in the surface.” In other words, the defenses marshaled by the ego to avoid unacceptable wishes from the id were in and of them- selves complex and worthy of attention. Up to that point, the primary focus had been on uncovering unconscious sexual and aggressive wishes. She also made seminal contributions to the field of child psychoanalysis and studied the function of the ego in personality development.
5.7. The answer is A
There is some differentiation between the sexes in the pattern of development. Freud explained the nature of this discrepancy in terms of genital differences. Under normal circumstances, he believed that, for boys, the Oedipal complex was resolved by the castration complex. Specifically, the boy had to give up his strivings for his mother because of the threat of castration— castration anxiety. In contrast, the Oedipus complex in girls was also evoked by reason of the castration complex. Unlike in boys, little girls are already castrated, and as a result, they turn to their fathers as bearers of the penis out of a sense of disappointment over their own lack of penises. Little girls are thus more threat- ened by a loss of love than by actual castration fears.
5.8. The answer is B
Erikson’s work concentrated on the effects of social, cultural, and psychological factors in development. Although Erikson acknowledged the important role of sexuality, it was less cen- tral to his theory. The concepts of instinctual drives and psy- chosexual development are essential parts of Freud’s theories, not Erikson’s. Object relations, which refer not to interpersonal relationships but to the interactions of internalized constructs of external relationships, is the central idea in object relation psychology.
5.9. The answer is E
Erikson’s theory of psychosocial development centers around eight stages of ego development. Integrity versus despair is the last of the stages and takes place between age 65 years and death. If this stage is successfully mastered, the individual arrives at a peaceful acceptance of his or her own mortality without losing interest in life. The patient in this case, however, is clearly having difficulties with this stage as she attempts to deny the passage of time and refuses to prepare for this endpoint of the life cycle. A person in a stage of despair is unconsciously fearful of death and lives in basic self-contempt.
Generativity versus stagnation is another of Erikson’s stages that takes place from ages 40 to 65 years and is focused on raising children with nurturance and love and not living in isolation. The stage of identity versus role confusion occurs between age 11 years and the end of adolescence, during which the adolescent must begin to establish a future role in adult society.
5.10. The answer is A
Melanie Klein thought that Freud’s concept of the “death instinct” was central to understanding aggression, hatred, sadism, and other forms of badness, which she viewed as deriva- tives of the death instinct. Her perspective grew largely from her psychoanalytic work with children. She became impressed with the role of unconscious intrapsychic fantasy and proposed that infants project derivatives of the death instinct onto the mother and then fear attack from the “bad mother,” a phenomenon re- ferred to as persecutory anxiety. This anxiety is associated with Klein’s paranoid-schizoid position. Through the integration of good and bad internal object relations, infants develop concern for the mother toward whom they have developed harmful fan- tasies. This concern results in repression and entering of the depressive position.
5.11. The answer is B
Freud’s concept of transference was to become a cornerstone of psychoanalytic theory and technique, a discovery that eventually contributed to his abandonment of hypnosis as a tool. Freud first observed that many patients were simply refractory to hypnosis. When a patient could be hypnotized, he noted, after the discovery of transference, that hypnosis concealed aspects of the transfer- ence, so these aspects could not be investigated as part of the process. He also believed that hypnosis encouraged the patient to please the hypnotist instead of learning about the origins and the meanings of symptoms. By the late 1890s, Freud abandoned hypnosis. Instead, he had the patient lie on the couch and say whatever came to mind without censorship, which is the method
of free association that remains a central part of psychoanalytic technique today.
5.12. The answer is D
Death fear is the fear of losing one’s identity by fusing with another person. The weaker one’s personal identity, the stronger the death fear.
Life fear, by contrast, is fear of losing all ties in the process of becoming separate. Every person experiences the cycle of movement from union to separation and back again as part of the life process. This movement takes place at various levels, including family, societal, artistic, and spiritual. At each level, there is one or more movement toward union and rebirth. Each person, for example, usually yields to a love experience in which personal differences are set aside to experience unity with an- other, to experience self-worth and to be relieved of the sense of difference. The yielding to another ends when the will asserts its separateness, and a new affirmation of individuality occurs.
Fear of dying associated with a phobia or “before one’s time” are not ideas associated with Otto Rank (1884–1939).
5.13. The answer is C
Carl Gustav (1875–1961) Jung believed archetypes to be in- stinctual patterns. Archetypes represent the basic motivations and drives that become organizational units of the personality. Snakes shedding their skins are usually interpreted as a symbol for change and renewal. Primary processes, manifest content of dreams, and phallic representations are Freudian concepts.
5.14. The answer is A
John Bowlby (1907–1990) is generally considered the founder of attachment theory. He formed his ideas about attachment in the 1950s while he was consulting with the World Health Organi- zation (WHO) on the problems of homelessness in children. He stressed that the essence of attachment is proximity (i.e., the tendency of a child to stay close to the mother or caregiver). A basic sense of security and safety is derived from a continuous and close relationship with the caregiver, according to Bowlby. He believed that without this early proximity to the mother or caregiver, the child does not develop a secure base, which he considered a launching pad for independence. In the absence of a secure base, the child feels frightened or threatened, and development is severely compromised.
Otto Kernberg works as one of the most influential object relations theorists in the United States. Melanie Klein evolved a theory of internal object relations intimately likened to moti- vational drives. Heinz Kohut is best known for his writings on narcissism and the development of self-psychology. Adolf Meyer introduced the concept of common sense psychiatry, which fo- cused on the ways a patient’s current life situation could be re- alistically improved.
5.15. The answer is D
Erikson’s formulations were based on the concept of epigene- sis, a term borrowed from embryology. His epigenetic princi- ple holds that development occurs in sequential, clearly defined stages and that each stage must be satisfactorily resolved for development to proceed smoothly. According to the epigenetic
model, if successful resolution of a particular stage does not occur, all subsequent stages reflect the failure in the form of physical, cognitive, social, or emotional maladjustment.
5.16. The answer is E (all)
As a social philosopher and critic, Fromm did not really develop a systematic theory of psychopathology. Rather, he identified three major mechanisms of retreat from individuation. Some individ- uals, he said, may seek an authoritarian solution, trying to live through someone or something external to themselves, relying on that for their sense of adequacy. Others may become destructive, attacking anything that confronts them with their separateness and loneliness. Most individuals develop a conformist attitude, warding off the anxiety of experiencing their own intentionality by accepting socially offered thoughts, roles, and attitudes.
These mechanisms result in four different unproductive ori- entations or characters typical of modern capitalist society: re- ceptive, exploitative, hoarding, and marketing. The receptive character often appears to be cooperative and open; however, the primary agenda is to establish a passive relationship with a leader who solves problems magically. Exploitative charac- ters are likewise interested in filling themselves up from the outside; however, they aggressively manipulate and usurp what- ever reduces their terror. Hoarders collect, store, and close in on themselves, often being cold and aloof in their efforts to feel secure. Marketers treat themselves as a plastic commodity to be manipulated as needed to achieve externally validated success.
5.17. The answer is C
Margaret Mahler (1897–1985) has conceptualized the process of development in terms of phases of separation and individ- uation. The “practicing” subphase of this process follows the first or “hatching” subphase, which arises at approximately 4 or 5 months of age. The third subphase, “rapprochement” involves separation anxiety. The child’s wishes to be separate from the mother are tempered by an increasing awareness of the need for and dependence on the mother. The fourth, and final phase, the subphase of “object constancy,” involves consolidation of indi- viduality and mature psychological involvement with others. Ob- ject permanence, the knowledge that objects in the external world have an existence independent of the child’s actions on them or interactions with them, is a major accomplishment of Piaget’s sensorimotor period of intellectual development. Dan Stern, a psychiatrist and psychoanalyst, who focused his work on infant observation, brought into clearer focus the intense affective and interactional matrix between mother and child and directed at- tention more specifically to the emergence of a sense of self.
5.18. The answer is B
Somatization is an immature defense mechanism. In it, psychic derivatives are converted into bodily symptoms, and patients re- act with somatic manifestations rather than psychic manifesta- tions. All of the others are mature defenses. Anticipation is goal directed and involves realistic anticipation or planning for fu- ture inner discomfort. Suppression involves the conscious post- ponement of attention to a conscious impulse or conflict. Al- truism uses constructive and instinctually satisfying service to others to undergo a vicarious experience. Asceticism involves the
5. Theories of Personality and Psychopathology 41
42 5. Theories of Personality and Psychopathology assignment of value to specific pleasure and is directed against
all base pleasures.
5.19. The answer is E (all)
Carl Gustav Jung believed archetypes to be instinctual patterns. He believed that all psychic energy is transmitted in forms of ex- perience, behavior, and emotion, which are expressed in repre- sentational or mythological images. Thus, the archetypes repre- sent the basic motivations and drives that become organizational units of the personality.
5.20. The answer is C
The unconscious system (not the preconscious) is characterized by primary process thinking. The publication of The Interpreta- tion of Dreams in 1900 heralded the arrival of Freud’s topograph- ical model of the mind, in which he divided the mind into three regions: the conscious system, the preconscious system, and the unconscious system. The preconscious system comprises mental events, processes, and contents that can be brought to conscious awareness by the act of focusing attention. Although most per- sons are not consciously aware of the appearance of their first- grade teacher, they ordinarily can bring this image to mind by deliberately focusing attention on the memory. Conceptually, the preconscious interfaces with both the unconscious and conscious regions of the mind. To reach conscious awareness, contents of the unconscious must become linked with words and thus be- come preconscious. The preconscious also serves to maintain the repressive barrier and to censor unacceptable wishes and desires.
5.21. The answer is E
Freud became aware of the significance of dreams when he noted that patients frequently reported their dreams in the process of free association. Through their further associations to dream con- tent, he learned that dreams were definitely meaningful even though meanings were often hidden or disguised. Most of all, Freud was struck by the intimate connection between dream content and unconscious memories or fantasies that were long repressed. This observation led Freud to declare that the inter- pretation of dreams was the royal road to understanding the unconscious.
5.22. The answer is E
Viktor Frankl (1905–1997), an Australian neurologist and philosopher, had a distinctive view of human nature. Having spent time in Nazi concentration camps, Frankl came to the con- clusion that the most appalling circumstances could be tolerated if one found a way of making them meaningful. He also believed that those who fail to find meaning face alienation, despair, and existential neuroses. Carl Rogers (1902–1987) articulated a for- mal theory of personality holding that all organisms tend toward their own actualization. As such, he believed that mental health and personal growth are the natural conditions of human kind. Therefore, psychopathology would be a distortion of the actual- ization process. Gordon Allport (1897–1967) believed that man’s behavior is proactive, reflecting internal, self-initiating charac- teristics. He believed personality to be rational and organized and influenced by goals, plans, and philosophies. Abraham Maslow
(1908–1970) interpreted personality in motivational terms. The individual’s whole life, including perceptions, values, and goals, is focused on the satisfaction of a set of needs. Maslow believed needs were organized in a hierarchy. George Kelly (1905–1967) argued that human beings should be seen as scientists trying to make sense of their world. Kelly held that the basic unit for un- derstanding personality is the personal construct, a schema for classifying and interpreting experiences.
Answers 5.23–5.28 5.23. The answer is C 5.24. The answer is B 5.25. The answer is D 5.26. The answer is A 5.27. The answer is E
5.28. The answer is F
The lettered choices above represent the phases of Margaret Mahler’s separation-individuation process. The theory empha- sizes the process of separation from the maternal orbit and the establishment of personal autonomy.
The first phase of Mahler’s theory of development describes the autistic phase: “During the first few weeks of extrauterine life, a stage of absolute primary narcissism, marked by the in- fant’s lack of awareness of a mothering agent prevails. . . . It is followed by a stage of dim awareness that need satisfaction cannot be provided by oneself, but comes from somewhere out- side the self.” The theory also articulates the origin of the initial differentiation of self and object, in which infants can be said to experience something outside themselves, to which they can relate, as satisfying their inner needs.
The second phase is the symbiotic phase “in which the infant behaves and functions as though he and his mother were an omnipotent system—a dual unity within one common boundary.” Boundaries become temporarily differentiated only in the state of “affect hunger” but disappear again when the need is gratified. Consequently, the object is recognized as separate from the self only at moments of need, so that when the need is satisfied, the object ceases to exist—from the infant’s (subjective) point of view—until a need again arises.
During the hatching period, the child gradually differentiates out of the symbiotic matrix. The first behavioral signs of such differentiation seem to arise at about 4 to 5 months of age, at the high point of the symbiotic period. As “hatching” and separation from the mother gradually increase, there is a move to a second or practicing subphase of separation-individuation.
The crisis in the rapprochement phase is particularly that of separation anxiety. The child’s wishes and desires to be sepa- rate, autonomous, and omnipotent are tempered by an increasing awareness of the need for and dependence on the mother. Thus, the mother’s availability and the reassurance of her continuing love and support become all the more important. As the conflicts and crisis of rapprochement are gradually resolved, the child
Table 5.1
Erikson’s Psychosocial Stages
Psychosocial Stage
Trust vs. mistrust (birth–) Autonomy vs. shame and doubt
(∼18 months–)
Initiative vs. guilt (∼3 years–)
Industry vs. inferiority (∼5 years–) Identity vs. role confusion (∼13 years–) Intimacy vs. isolation (∼20s–) Generativity vs. stagnation (∼40s–) Integrity vs. despair (∼60s–)
Associated Virtue
Hope Will
Purpose Competence Fidelity Love
Care Wisdom
gles to resolve the oedipal struggle, guilt may grow because of aggressive thoughts or wishes. Initiative arises as the child be- gins to desire to mimic the adult world and as the child finds enjoyment in productive activity.
The stage of puberty and adolescence (age 11 years through the end of adolescence) is characterized by identity versus role diffusion, during which the adolescent must begin to establish a future role in adult society. During this psychosocial crisis, the adolescent is peculiarly vulnerable to social and cultural influ- ences.
Early adulthood (ages 21 to 40 years) is characterized by in- timacy versus isolation. The crisis is characterized by the need to establish the capacity to relate intimately and meaningfully with others in mutually satisfying and productive interactions. The failure to achieve a successful resolution of that crisis results in a sense of personal isolation. Late adulthood (age 65 years and older) is characterized by integrity versus despair. The crisis implies and depends on the successful resolution of all the pre- ceding crises of psychosocial growth. It entails the acceptance of oneself and of all of the aspects of life and the integration of their elements into a stable pattern of living.
Answers 5.39–5.43 5.39. The answer is A 5.40. The answer is C 5.41. The answer is D 5.42. The answer is B
5.43. The answer is E
Anna Freud is probably best known for her book The Ego and the Mechanisms of Defense. In the book, she gives a partic- ularly clear description of how the defenses work, including some special attention to adolescents’ use of defenses. Melanie Klein’s Envy and Gratitude introduces her theory of primary envy. In Civilization and Its Discontents, Sigmund Freud pro- poses that the conflict between sexual needs and societal mores is the source of mankind’s propensity for dissatisfaction, aggres- sion, hostility, and ultimately violence. Erik Erikson, in Child- hood and Society, writes about the influence of society on culture and child development. He correlated personality growth with parental and societal values. Harry Stack Sullivan, in Schizophre- nia as a Human Process, expounds the idea that schizophre- nia developed as a result of the cultural forces that bore on the individual.
Answers 5.44–5.47 5.44. The answer is B 5.45. The answer is A 5.46. The answer is C 5.47. The answer is D
5. Theories of Personality and Psychopathology 43
   
Adapted from Erikson E. Insight and Responsibility. New York: WW Norton; 1964; Erikson E. Identity: Youth and Crisis. New York: WW Norton: 1968.
enters the final phase of separation and individuation—namely, the phase of consolidation of individuality and the beginnings of emotional object constancy. Object constancy implies a capac- ity to differentiate among objects and to maintain a meaningful relationship with one specific object, whether needs are being satisfied or not.
Answers 5.29–5.33
5.29. The answer is D
5.30. The answer is E
5.31. The answer is B
5.32. The answer is C
5.33. The answer is A
See Table 5.1.
Answers 5.34–5.38 5.34. The answer is B 5.35. The answer is D 5.36. The answer is E 5.37. The answer is C
5.38. The answer is A
The first of Erik Erikson’s developmental stages, infancy (birth to 1 year), is characterized by the first psychosocial crisis the infant must face, that of basic trust versus basic mistrust. The crisis takes place in the context of the intimate relationship between the infant and his or her mother. The infant’s primary orientation to reality is erotic and centers on the mouth. The successful resolution of the stage includes a disposition to trust others, a basic trust in oneself, a capacity to entrust oneself, and a sense of self-confidence.
During early childhood (ages 3 to 5 years), the crisis ad- dressed by the child is initiative versus guilt. As the child strug-
44 5. Theories of Personality and Psychopathology
Freud distinguished between two layers of dream content. The manifest content refers to what is recalled by the dreamer; the latent content involves the unconscious thoughts and wishes that threaten to awaken the dreamer. Freud described the un- conscious mental operations by which latent dream content is transformed into manifest dream content as the dream work. Re- pressed wishes and impulses must attach themselves to innocent or neutral images to pass the scrutiny of the dream censor. This process involves selection of apparently meaningless or trivial images from the dreamer’s current experience, images that are dynamically associated with the latent images that they resem- ble in some respect. Secondary revision is the process by which primitive aspects of dreams are made into more coherent and less bizarre form.
Answers 5.48–5.51
5.48. The answer is C
5.49. The answer is D
5.50. The answer is E
5.51. The answer is B
The awareness of the external world of objects develops grad- ually in infants. Soon after birth, they are primarily aware of physical sensations, such as hunger, cold, and pain, which give rise to tension, and caregivers are regarded primarily as persons who relieve their tension or remove painful stimuli. Recent in- fant research, however, suggests that awareness of others begins much sooner than Freud originally thought. Table 5.2 provides a summary of the objectives for the stages of psychosexual devel- opment and the object relationships associated with each stage.
Answers 5.52–5.55 5.52. The answer is A 5.53. The answer is B 5.54. The answer is C
5.55. The answer is E
In Freudian psychoanalytic theory, defense mechanisms repre- sent the ego’s attempts to mediate between the pressure of the instinctual drives emerging from the id and the restrictions im- posed by the societal rules from the superego. Defense mecha- nisms alleviate anxiety by maintaining conflict out of awareness.
Displacement is the shifting of unacceptable emotions or im- pulses to a more appropriate object. Although the woman is angry at her boyfriend, she waits until she gets home and displaces this anger on her younger sibling, a less dangerous outlet. Projection
Table 5.2
Stages of Psychosexual Development and Their Objectives
 
Stage
Oral
Anal
Urethral Phallic
Latency
Genital
Objective
To establish a trusting dependence on nursing and sustaining objects, to establish comfortable expression and gratification of oral libidinal needs without excessive conflict or ambivalence from oral sadistic wishes.
The anal period is essentially a period of striving for independence and separation from the dependence on and control by the parent. The objectives of sphincter control without overcontrol (fecal retention) or loss of control (messing) are matched by the child’s attempts to achieve autonomy and independence without excessive shame or self-doubt from loss of control.
Issues of control and urethral performance and loss of control. It is not clear whether or to what extent the objectives of urethral functioning differ from those of the anal period.
The objective of this phase is to focus erotic interest in the genital area and genital functions. This focusing lays the foundation for gender identity and serves to integrate the residues of previous stages of psychosexual development into a predominantly genital–sexual orientation.
The primary objective in this period is the further integration of oedipal identifications and a consolidation of sex-role identity and sex roles. The relative quiescence and control of instinctual impulses allow for the development of ego apparatuses and mastery skills.
The primary objectives of this period are on the ultimate separation from dependence on and attachment to the parents and the establishment of mature, nonincestuous object relations. Related to this are the achievement of a mature sense of personal identity and acceptance and the integration of a set of adult roles and functions that permit new adaptive integrations with social expectations and cultural values.
 
is a defense mechanism in which the patient reacts to an inner unacceptable impulse as if it were outside the self. In this case, the man projects his envy as coming from his friend when it is actually an inner unacceptable impulse. In reaction formation, an unacceptable unconscious impulse is transformed into its op- posite. The teenage boy, rather than accepting his feelings of hatred toward his mother, treats her with love and constant at- tention. Sublimation is satisfaction of an objectionable impulse obtained by using socially acceptable means. The man in this case is therefore channeling his urges to control others into his job as a prison guard rather than preventing them. Repression is the unconscious act of preventing a thought or feeling from entering consciousness.

6
The Patient–Doctor Relationship and the Psychiatric Interview

The quality of the patient–doctor relationship is crucial to the practice of medicine and psychiatry. The relationship between any one patient and physician varies depending on each of their personalities and past experiences as well as the setting and pur- pose of the encounter. However, there are general principles that, when followed, help to ensure that the relationship established is helpful. One of the essential qualities of a clinician is interest in humanity because the secret to the care of the patient is in car- ing for the patient. A good physician knows his or her patients through and through. Time, sympathy, and understanding must be dispensed, and the personal bond with the patient is one of the greatest satisfactions of the practice of medicine.
An effective relationship is characterized by good rapport. Rapport is the spontaneous, conscious feeling of harmony that promotes the development of a therapeutic alliance. It implies an understanding and trust between the doctor and the patient. The patient comes to a doctor seeking help. A desire for help motivates the patient to share information and feelings that are distressing, personal, and private with a stranger. From the first encounter, the patient’s willingness to share depends on the ver- bal and nonverbal interventions of the physician and staff. As the physician’s behaviors demonstrate respect and considera-
tion, rapport begins to develop when the patient feels safe and comfortable.
Obtaining a good patient–doctor relationship can at times be difficult. Almost all physicians at some point treat patients who are difficult, not because of their medical illness but because they engage in power struggles or are demanding or uncooperative. The issue is especially pertinent for psychiatrists because the un- derlying pathology of their patients may manifest as behavioral interactions that themselves provoke negative responses. Diffi- cult patients need acknowledgement, understanding, and special skills.
Various types of patients fall under the rubric of special pa- tient populations. They include patients with urgent issues, pa- tients who are severely mentally ill, patients from different cul- tural backgrounds who are unassimilated, patients who cannot communicate well because of difficulties with the English lan- guage, and patients whose personality problems make them diffi- cult. Inherent in the management of all such cases is the doctor’s understanding of the emotions, fears, and conflicts that the pa- tient’s behavior represents.
The student should test their knowledge by addressing the following questions and answers.
The key terms listed below should be understood by the student.
active versus passive patients
aggression and counteraggression
agitative patients belligerent patients biopsychosocial model burnout
coercion
closed-ended questions compliance versus
noncompliance confrontation
content versus process
countertransference cross-cultural issues defensive attitudes demanding patients dependant patients emotionally charged
statements empathy
George Engel “good patients” grievance collector illness behavior insight interpretation
isolated patients malingering misperception misrepresentation mutual participation narcissistic patients need–fear dilemma obsessive patients open-ended questions overcompensatory anger passive suicidal patients patient–doctor models rapport
reflection
secondary gain seductive patients self-monitoring
sick role
somatizing patients sublimation therapeutic limitations thought disorder transference unconscious guilt uncooperative patients unresolved conflicts
HELPFUL HINTS
45
▲▲▲▲▲▲▲▲▲▲▲▲
▲▲▲▲▲▲▲▲▲▲▲▲▲▲
▲▲▲▲▲▲ ▲▲▲▲▲▲▲
▲ ▲ ▲▲▲▲▲▲▲ ▲▲
46 6. The Patient–Doctor Relationship and the Psychiatric Interview
QUESTIONS
Directions
Each question or incomplete statement below is followed by five suggested responses or completions. Select the one that is best in each case.
. 6.1. In response to the question, “Why did you come to the clinic?” a patient said: “When I got up this morning, I showered and dressed. I was angry at my landlord for not fixing the faucet in my bathroom. I tried to get him on the phone. He wouldn’t talk to me. I’ll call my lawyer. You see, my rent is supposed to be paid by the Department of Welfare, but they’re so nasty. [But why did you come to the clinic?] I’m coming to that, Doctor. You see, they don’t care about an upright citizen. I did so much for my community. No one can say I wasn’t a hard worker,” and so on. After repeated questioning, the patient finally stated that she was worried about being constipated.
The above patient is an example of
A. a patient with a thought disorder B. a delusional patient
C. a somatizing patient
D. a demanding patient
E. an agitated patient
. 6.2. The psychiatric interview serves all of the following func-
tions except
A. to establish a therapeutic relationship B. to implement a treatment plan
C. to assess the nature of the problem
D. to demonstrate the physician’s expertise E. none of the above
. 6.3. During an interview, a patient sarcastically asks a physi- cian, “Did you really go to medical school?” Which is the best way for the physician to respond?
A. Promptly end the interview
B. Answer the question directly
C. Address the issue that provoked the comment D. Do not answer
E. None of the above
. 6.4. Rapport is
A. based on a doctor projecting feelings onto the patient
B. based on a patient projecting feelings from past rela-
tionships to the doctor
C. a feeling of harmony that promotes a therapeutic rela-
tionship
D. of little significance in obtaining the history
E. none of the above
. 6.5. Mr. M, a 60-year-old man, 10 months after the death of his wife of 40 years, reluctantly told his daughter that he wished he were dead but would never act on these wishes.
Alarmed, she took him to a psychiatrist for an evaluation. Which of the following is true?
A. Feelingthiswayisanormalgriefreaction,sonoaction is required.
B. His daughter has overreacted in light of the absence of described intent.
C. Detailed questions about his suicidality are essential for prevention.
D. Euphemistic inquiries about his suicide risk would fos- ter rapport.
E. Asking this man about suicide may increase his risk.
6.6. Transference feelings
A. are based on a patient projecting feelings from past relationships toward the doctor
B. are a main reason for lawsuits filed by mistreated pa- tients
C. arebasedonadoctorprojectingfeelingsontoapatient
D. do not occur with highly experienced physicians
E. none of the above
6.7. At
discuss her perception of why she felt ill, but the physi- cian wants to know the chronology of her symptoms. The physician should
A. inform her that time is of the essence
B. inform her that an extra charge will be made if more
time is needed for the appointment
C. allow the patient to complete her thoughts
D. immediately discuss how compliance will be affected
by her perceptions and responses
E. politely interrupt the patient and continue with closed-
ended questions
6.8. Which of the following statements on the person-centered
approach is false?
A. Theindividualityofthepatient’sexperienceisacentral theme.
B. The focus is on illness and deficits.
C. The focus is on strengths and assets.
D. The treatment plan should be based on the patient’s
goals.
E. None of the above
6.9. Illness behavior refers to
A. the influence of culture on illness
B. the way the condition presents itself
C. the role society ascribes to the sick person D. being excused from responsibilities
E. all of the above
6.10. Which of the following minimizes agitation and the risk of harm by potentially violent patients?
A. Asking the patient if he is carrying a weapon B. Terminating the interview if necessary
the beginning of an appointment, a patient wants to
C. Evaluating the patient in a nonstimulating environment D. Heeding one’s subjective sense of fear
E. All of the above
. 6.11. Which of the following statements about transference is true?
A. Transference is a conscious process.
B. Transference reactions may be strongest with psychi-
atrists.
C. Transference is exclusively positive because patients
know doctors are trying to help them.
D. Transference occurs only in patient interactions with
psychiatrists, not with clinicians from other disci-
plines.
E. Transference implies that the way a clinician interacts
with the patient has no direct bearing on the emotional reactions of the patient.
. 6.12. Antisocial patients
A. rarely malinger
B. rarely present as socially adept or intelligent
C. seldom cause physicians to feel threatened
D. should never be confronted directly about inappropri-
ate behavior
E. must be approached with a heightened sense of vigi-
lance
. 6.13. Which of the following is considered important in estab- lishing rapport with a patient?
A. Showing expertise
B. Evaluating a patient’s insight C. Putting the patient at ease
D. Expressing compassion
E. All of the above
. 6.14. Choose the answer that best fits the example given in the below case study.
A soldier in basic training presents at the Mental Hygiene Clinic with a long list of symptoms. It is later learned that he has visited the base library and read about psychiatric illnesses. The symptoms reported are increasingly more dramatic as the response of the psychiatrist is not what the soldier expected. Finally, after describing some elaborate visual hallucinations that do not impress the psychiatrist, the soldier blurts out, “What’s the matter, you never heard of paranoid schizophrenia?”
A. A seductive patient
B. A depressed patient
C. A noncooperative patient D. A lying patient
E. None of the above
. 6.15. Which of the following is not considered a necessary part of a psychiatric patient–doctor relationship?
B. Clarification of the doctor’s availability between scheduled appointments
C. Discussion of payment
D. The understanding that confidentiality may be broken
in some situations
E. Awareness of the consequences for missed appoint-
ments
6.16. With paranoid patients, a physician
A. should not allow a patient to remain evasive
B. must react defensively to a patient’s suspicions
C. should be as relaxed and friendly as possible
D. should be prepared to explain in detail every decision E. should not take seriously a patient’s hostile or conspir-
atorial misperception of a neutral event
6.17. In which instance is an autocratic patient–doctor relation- ship most appropriate?
A. A 54-year-old woman with hypertension wishes to monitor her own blood pressure at home.
B. A 22-year-old man is brought into the emergency de- partment with a gunshot wound to the chest.
C. A young woman confides in her doctor about wanting to have an abortion.
D. A patient has a life-threatening illness with various treatment options.
E. Awomanwhoisacarrierofthegeneforcysticfibrosis consults her doctor about whether she and her husband should conceive.
6.18. Which type of patient would say the following: “I have a friend who is in the business and is great friends with some very famous celebrities. I could introduce you to them. . .”
A. A seductive patient
B. A noncooperative patient C. A lying patient
D. A somatizing patient
E. None of the above
6.19. Confronting a patient with the topic of suicide in a psy- chiatric interview
A. should not be done in a straightforward manner
B. hinders the doctor’s rapport
C. is necessary with all depressed patients
D. increases the chances of depressed patients attempting
or committing suicide E. none of the above
6.20. Which of the following doctor factors is associated with treatment compliance?
A. Older doctors with experience B. Increased frequency of visits C. Positive physician attitude
D. Short waiting room time
E. All of the above
A. The patient’s familiarity with the doctor’s personal life
6. The Patient–Doctor Relationship and the Psychiatric Interview 47
48 6. The Patient–Doctor Relationship and the Psychiatric Interview
6.21. A 29-year-old woman with a history of schizophrenia presents to your office for a refill of her prescriptions. She is convinced her mother hid her bottle of olanzapine (Zyprexa). The patient states that she feels betrayed and is having auditory hallucinations commanding her to kill her mother. However, when questioned further, the pa- tient denies having any specific plans to harm her mother. Which of the following is the most important next step in management?
A. Refill the patient’s prescription with a more potent dosage of olanzapine.
B. Immediately contact the patient’s mother.
C. Admit the patient.
D. Simply refill the same dosage of olanzapine.
E. Refill the prescription and then contact the patient’s
mother.
Directions
Each set of lettered headings below is followed by a list of num- bered phrases. For each numbered phrase, select
A. if the item is associated with A only
B. if the item is associated with B only
C. if the item is associated with both A and B D. if the item is associated with neither A nor B
Questions 6.22–6.26
A. Psychiatric interview
B. Nonpsychiatric interview
6.32. Are essential in obtaining necessary data for diagnosis and treatment
6.33. The interviewer selects topics
6.34. Are ideal at the beginning of the interview
6.35. Have low time efficiency
6.36. Are effective in assessing the duration of symptoms
Directions
Each group of questions below consists of lettered headings fol- lowed by a list of numbered phrases or statements. For each numbered phrase or statement, select the one lettered heading that is most associated with it. Each lettered heading may be selected once, more than once, or not at all.
Questions 6.37–6.42
A. Paternalistic model B. Informative model C. Interpretive model D. Deliberative model
. 6.37. Also called the autocratic model
. 6.38. A doctor is trying to get a patient to stop smoking
. 6.39. Patient may think the doctor is cold and uncaring
. 6.40. Risks a clash in values
. 6.41. Shared decision-making between doctor and patient
. 6.42. Appropriate for some one-time consultations
ANSWERS
6.1. The answer is A
The patient described is an example of a patient with a thought disorder. Disorders of thought can seriously impair effective communication. The evaluating psychiatrist should note formal thought disorders while minimizing an adverse impact on the interview. When derailment is evident, as in this case, the psy- chiatrist typically proceeds with questions that call for short an- swers.
Somatizing patients pose a number of difficulties for consulting and treating psychiatrists because they may be reluctant to engage in self-reflection and psychological exploration. Many somatizing patients live with the fear that their symptoms are not being taken seriously and the parallel fear that something medically serious may be overlooked. Psychiatrists’ main task in dealing with these patients is to acknowledge the suffering conveyed by the symp- toms without necessarily accepting the patient’s explanation for the symptoms.
Delusions are fixed, false beliefs not shared by members of one’s culture. Delusional patients often come for a psychiatric evaluation after having had their beliefs dismissed or belittled by friends and family. They are on guard for similar reactions from the examiner. It is possible to ask questions about delusions with- out revealing belief or disbelief (e.g., “Does it seem that people are intent on hurting you?” rather than “Do you believe there is a plot to hurt you?”). Many psychiatrists have found that patients can speak more freely when asked to talk about the accompanying emotions rather than the belief itself (“It must be frightening to
. 6.22. Assess the nature of the problem
. 6.23. Symptom classification
. 6.24. Develop and maintain a therapeutic relationship
. 6.25. Recognitionofthepsychologicaldeterminantsofbehavior
. 6.26. Communicate information
Questions 6.27–6.31
A. Countertransference B. Transference
. 6.27. In a conversation about his patient, a physician sponta- neously comments, “She’s a very nice lady; she reminds me a lot of my mother.”
. 6.28. May be encouraged as integral to some intensive psychi- atric treatment.
. 6.29. A patient sees her doctor as overly critical because her mother had always criticized her life choices.
. 6.30. Can be upsetting and interfere with good medical care.
. 6.31. A patient, when asked if she shared her concerns about her sexual relationship with her husband to her physician, says, “Well, I tried to bring it up a couple of times, but he looked pretty uncomfortable, and he changed the subject.
I didn’t bring it up again; I didn’t want to bother him.”
Questions 6.32–6.36
A. Open-ended questions B. Close-ended questions
think there are people you do not know who are plotting against you”).
Demanding patients have a difficult time delaying gratifica- tion and demand that their discomfort be eliminated immediately. They are easily frustrated and can become petulant or even angry and hostile if they do not get what they want when they want it. Beneath their surface behavior, they may fear that they will never get what they need from others and thus must act in that inap- propriately aggressive way. The doctor must be firm with these patients from the outset and must clearly define acceptable and un- acceptable behavior. These patients must be treated with respect and care, but they must also be confronted with their behavior.
An agitated patient is emotionally restless and excited. He or she may make physical threats to the physician. When interviewing agitated and potentially violent patients, the tasks are to conduct an assessment, contain behavior, and limit the potential for harm.
6.2. The answer is D
Three functions of medical interviews are to assess the nature of the problem, to develop and maintain a therapeutic relationship, and to communicate information and implement a treatment plan. See Table 6.1. Although showing expertise may be a good way to establish rapport, it is not a function of psychiatric interview.
6.3. The answer is C
At times, patients will ask questions about the psychiatrist. A good rule of thumb is that questions about the physician’s qual- ifications and position should generally be answered directly (e.g., board certification, hospital privileges). On occasion, such a question might actually be a sarcastic comment (“Did you re- ally go to medical school?”). In this case, it would be better to address the issue that provoked the comment rather than respond concretely. A major reason for not answering personal questions
Table 6.1
Three Functions of the Medical Interview
directly is that the interview may become psychiatrist centered rather than patient centered.
6.4. The answer is C
Rapport is the spontaneous, conscious feeling of harmony that promotes the development of a therapeutic alliance. It implies an understanding and trust between the doctor and patient. Fre- quently, the doctor is the only person to whom a patient can talk about things that he or she cannot tell anyone else. Most patients trust their doctors to keep secrets, and this confidence must not be disobeyed.
Transference describes the process of patients unconsciously projecting feelings from their past relationships to the doctor. When doctors unconsciously project their feelings to the patient, the process is called countertransference; these feelings are not directly related to rapport. For example, a patient may remind the doctor of his father’s narcissism.
6.5. The answer is C
A thorough assessment of suicide potential addresses intent, plans, means, and perceived consequences, as well as a his- tory of attempts, and family history of suicide. The examiner must ask clear, straightforward, noneuphemistic questions. Ask- ing about suicide does not increase risk; the psychiatrist is not raising a topic that the patient has not already contemplated. Some patients may report a wish that they were dead but would never intentionally do anything to take their own lives. Others express greater degrees of determination. Either way, any ex- pression of a wish to die should be considered and evaluated seriously. It should never be considered lightly or as a part of a “normal” grief reaction. Normal grief may include sadness and guilt; however, this resolves within 1 year. There are some
Skills
1. Knowledge base of diseases, disorders, problems, and clinical hypotheses from multiple conceptual domains: biomedical, sociocultural, psychodynamic, and behavioral
2. Ability to elicit data for the above conceptual domains (encouraging the patient to tell his or her story; organizing the flow of the interview, the form of questions, the characterization of symptoms, the mental status examination).
1. Defining the nature of the relationship
2. Relief of physical and psychological distress 3. Hearing, bearing, and tolerating the patient’s
expression of painful feelings
4. Appropriate and genuine interest, empathy,
support, and cognitive understanding
1. Determining the nature of the problem (function I) 2. Developing a therapeutic relationship (function II) 3. Establishing the differences in perspective
between physician and patient
6. The Patient–Doctor Relationship and the Psychiatric Interview 49
 
Functions
I. Determining the nature of the problem
II. Developing and maintaining a therapeutic relationship
III. Communicating information and implementing a treatment plan
Objectives
1. To enable the clinician to establish a diagnosis or recommend further diagnostic procedures, suggest a course of treatment, and predict the nature of the illness
1. The patient’s willingness to provide diagnostic information
2. Allowing the patient to tell his or her story 3. Willingness to accept a treatment plan or a
process of negotiation
1. Patient’s understanding of the illness
2. Patient’s understanding of the suggested
diagnostic procedures
3. Patient’s understanding of the treatment
possibilities
 
Adapted from Lazare A, Bird J, Lipkin M Jr, Putnam S. Three functions of the medical interview: An integrative conceptual framework. In: Lipkin Jr M, Putnam S, Lazare A, eds. The Medical Interview. New York: Springer; 1989:103.
50 6. The Patient–Doctor Relationship and the Psychiatric Interview
patients who tell no one of their suicide plans and proceed in a deliberate, systematic manner. The daughter did not overreact because her father experienced a loss, and suicidal statements need to be taken seriously.
6.6. The answer is A
Transference describes the process of patients unconsciously projecting feelings from their past relationships to the doctor. A patient may come to see the doctor as cold, harsh, critical, threatening, seductive, caring, or nurturing, not because of any- thing the physician says or does but because that has been part of the patient’s past. The residue of such experience leads the patient to unwittingly “transfer” the feeling from past relation- ships to the doctor. The transference can be positive or negative, and it can swing back and forth—sometimes abruptly—between the two. Many a physician has become unsettled when a pleas- ant, cooperative, and admiring patient suddenly and for no dis- cernible reason becomes enraged and breaks off the relationship or threatens a lawsuit. Physicians are not immune to distorted perceptions of the patient–doctor relationship. When doctors un- consciously project their feelings to the patient, the process is called countertransference.
Although physicians can be sued for anything, transference feelings are not one of the main reasons for lawsuits filed by mistreated patients.
A doctor’s level of expertise does not have any effect on whether transference feelings will occur or not.
6.7. The answer is C
The early part of the interview is generally the most open ended in that the physician allows patients to speak as much as pos- sible in their own words by asking open-ended questions and permits them to finish. An open-ended question is one that can- not be answered by a simple “yes” or “no” (e.g., “Can you tell me more about that?”). This type of questioning is important to establish rapport, which is the first step in an interview. In one survey of 700 patients, the patients substantially agreed that physicians do not have the time or the inclination to listen and to consider the patient’s feelings. They also stated that physicians do not have enough knowledge of the emotional problems and socioeconomic background of the patient’s family and that physi- cians increase patients’ fears by giving explanations in technical language. Psychosocial and economic factors exert a profound influence on human relationships, so the physician should have as much understanding as possible of the patient’s environment and subculture.
As for time and charges, physicians should inform patients about their fee policies but should not interrupt patients to do so. Instead, those areas of business should be dealt with before the initial visit so that an ongoing relationship with a patient can be established. The matter of fees must be discussed openly from the outset; the discussion should include the physician’s charges, whether the physician is willing to accept insurance payments directly (known as assignments), the policy concerning payment for missed appointments, and whether the physician is part of a managed care plan. Discussing these questions and any others about fees at the beginning of the relationship can minimize misunderstandings later.
A discussion of compliance with a medical plan is important but is premature early in the interview. Furthermore, compli- ance, which is the degree to which the patient carries out clinical recommendations by the treating physician, is a two-way street. Studies have shown that noncompliance is associated with physi- cians perceived as rejecting and unfriendly. Noncompliance is also associated with asking a patient for information without giving any feedback and with failing to explain a diagnosis. A physician who is aware of the patient’s belief system, feelings, and habits establishes a treatment regimen that increases com- pliant behavior.
6.8. The answer is B
Traditionally, medicine has focused on illness and deficits rather than strengths and assets. A person-centered approach focuses on strength and assets as well as deficits. During the assessment, it is often helpful to ask the patient, “Tell me about some of the things you do best” or “What do you consider your greatest asset?” A more open-ended question such as, “Tell me about yourself” may elicit information that focuses more on either strengths or deficits, depending on a number of factors, including the patient’s mood and self-image.
A psychiatric interview should be person (patient) centered. That is, the focus should be on understanding the patient and enabling the patient to tell his or her story. The individuality of the patient’s experience is a central theme, and the patient’s life history is elicited. It is subject to the constraints of time, the patient’s willingness to share some of this material, and the skill of the interviewer. It is especially important that the re- sulting treatment plan be based on the patient’s goals, not the psychiatrist’s. Numerous studies have demonstrated that often the patient’s goals for treatment (e.g., safe housing) are not the same as the psychiatrist’s (e.g., decrease in hallucinations).
6.9. The answer is E (all)
The term illness behavior describes patients’ reactions to the experience of being sick. Aspects of illness behavior have some- times been termed the sick role, the role that society ascribes to people when they are ill. The sick role can include being excused from responsibilities and the expectation of wanting to obtain help to get well. Illness behavior and the sick role are affected by people’s previous experiences with illness and by their cultural beliefs about disease. For some disorders, this varies little among cultures, but for others, the way a person deals with the disorder may strongly shape the way the condition presents itself. The re- lationship of illness to family processes, class status, and ethnic identity is also important. The attitudes of peoples and cultures about dependency and helplessness greatly influence whether and how a person asks for help, as do such psychological factors as personality type.
6.10. The answer is E (all)
Most unpremeditated violence is preceded by a prodrome of ac- celerating psychomotor agitation. Several steps can be taken to minimize the agitation and potential risk. The interview should be conducted in a quiet, nonstimulating environment. The psy- chiatrist should avoid any behavior that could be misconstrued as menacing, such as standing over the patient, staring, or
touching. The psychiatrist should ask whether the patient is car- rying any weapons and ask the patient to leave the weapon with a guard or in a holding area. If the agitation continues to in- crease, the psychiatrist may need to terminate the interview. The physician should always heed his or her own sense of comfort or fear.
6.11. The answer is B
Transference reactions may be strongest with psychiatrists, es- pecially when the therapeutic modality used requires the psy- chiatrist to be more neutral. The more neutral the psychiatrist is, the more transferential fantasies and concerns are mobilized in the patient and transferred onto the doctor. Transference de- scribes the process of patients unconsciously attributing to their doctors aspects of important past relationships, especially those with their parents. Transference is ubiquitous and plays a role in the interaction of all patients with all clinicians. The transfer- ence can be positive or negative, and it can swing back and forth between the two. The words and deeds of doctors have power- ful effects on their patients because of the unique authority the doctor has and the patients’ dependence on them. How a partic- ular physician behaves and interacts has a direct bearing on the emotional, and even the physical, reactions of the patient.
6.12. The answer is E
Doctors must treat antisocial patients with respect but also with a heightened sense of vigilance. These patients can inspire fear in others, often legitimately so, because many have vio- lent histories. Doctors who feel threatened by patients should unashamedly seek assistance and not feel compelled to see the patients alone. Firm limits must be set on behavior (e.g., no drugs in the hospital and no sexual activity with other patients), and the consequences of transgressing must be stated firmly and adhered to (e.g., discharge from the hospital if the patient is medically stable, isolation if not). If inappropriate behavior is discovered, patients must be confronted directly and nonangrily, and they must be held responsible for their actions. On the surface, these patients may appear charming, socially adept, and intelligent be- cause over many years, they have perfected the behaviors they know to be appropriate, and they perform almost as actors. An- tisocial patients often malinger, the term for willfully feigning
Table 6.2
Checklist for Clinicians
illness for a clear secondary gain (e.g., to obtain drugs, get a bed for the night, or hide from people pursuing them). They obvi- ously get sick like any other patient. When they are sick, they need to be cared for in the same ways as others.
6.13. The answer is E (all)
Establishing rapport is the first step of a psychiatric interview. It encompasses six strategies, as defined by Ekkhard and Sieglinde Othmer: putting patients at ease; finding patients’ pain and ex- pressing compassion; evaluating patients’ pain and expressing compassion; evaluating patients’ insight and becoming an ally; showing expertise; and establishing authority and balancing the roles of empathic listener, expert, and authority. As part of a strategy for increasing rapport, Othmer and Othmer developed a checklist that enables interviewers to recognize problems and refine their skills in establishing rapport. The first 10 questions in the checklist are shown in Table 6.2.
6.14. The answer is D
Patients lie or deceive their psychiatrists for many different rea- sons. Some are motivated by secondary gain (e.g., for financial resources, absence from work, or a supply of medication). Some may deceive, not for an external advantage, but for the psycho- logical benefits of assuming the sick role or factitious illness. Psychiatrists are dependent on the patient’s self-report. It may be useful, especially when there is a question about the patient’s reliability, to gather collateral information regarding the patient. This allows the psychiatrist to have a more broad understanding of the patient outside of the interview setting. Discrepancies in symptom severity between self-report and collateral information may suggest deception. Although it is helpful and necessary to maintain some skepticism, a clinician who becomes jaded by suspiciousness risks making empathetic treatment impossible.
6.15. The answer is A
Although a clinician may choose to reveal aspects of his or her personal life when appropriate, it is not necessary to the patient– doctor relationship that the patient be familiar with the doctor’s personal life. Limited, discreet self-disclosure, or self-revelation, by physicians may be useful in certain situations, and physi- cians should feel at ease and should communicate a sense of

1. I put the patient at ease.
2. I recognized the patient’s state of mind.
3. I addressed the patient’s distress.
4. I helped the patient warm up.
5. I helped the patient overcome suspiciousness. 6. I curbed the patient’s intrusiveness.
7. I stimulated the patient’s verbal production. 8. I curbed the patient’s rambling.
9. I understood the patient’s suffering.
Yes No N/A
6. The Patient–Doctor Relationship and the Psychiatric Interview 51

The following checklist allows clinicians to rate their skills in establishing and maintaining rapport. It helps them detect and eliminate weaknesses in interviews that failed in some significant way. Each item is rated “yes,” “no,” or “not applicable.”
                          
10. I expressed empathy for the patient’s suffering.
   
52 6. The Patient–Doctor Relationship and the Psychiatric Interview
self-comfort. Conveying this sense may involve answering a pa- tient’s questions about whether a physician is married and where he or she comes from. A doctor who practices self-revelation ex- cessively, however, is using a patient to gratify unfulfilled needs in his or her own life and is abusing the role of the physician. If a doctor believes that a piece of information will help a patient be more comfortable, the doctor can decide in each case whether to be self-revealing. The decision depends on whether the infor- mation will further a patient’s care or if it will provide nothing useful.
Before psychiatric clinicians can establish an ongoing rela- tionship with patients, it is necessary to address certain issues. For example, they must openly discuss payment of fees. Dis- cussing these issues and any other questions about fees from the beginning of the relationship can minimize misunderstanding later. Psychiatrists should also discuss the extent and limitations of confidentiality with patients so that patients are clear about what can and cannot remain confidential. As much as physicians must legally and ethically respect patients’ confidentiality, it may be wholly or partially broken in some specific situations. For ex- ample, if a patient makes clear that he or she intends to harm someone, the doctor has a responsibility to notify the intended victim.
Patients need to be informed about a doctor’s policies for missed appointments. Some doctors ask patients to give 24 hours’ notice to avoid being billed for a missed session. Others bill for missed sessions regardless of advance notification. Patients must know in advance to make an informed decision about whether to accept the doctor’s policy or to choose another doctor.
6.16. The answer is D
Paranoid patients fear that people want to hurt them and intend to do them harm. Doctors should be able to explain in detail every decision and planned procedure to paranoid patients and should react nondefensively to patients’ suspicions. Patients may misperceive cues in their environment to the degree that they see conspiracies in neutral events. They are critical, evasive, and suspicious. They are often called grievance seekers because they tend to blame others for everything bad that happens in their lives. They are extremely mistrustful and may question everything that doctors advise doing. Physicians must remain somewhat formal, albeit always respectful and courteous, with these patients because they often view expressions of warmth and empathy with suspicion (“What does he want from me?”).
6.17. The answer is B
In a paternalistic, or autocratic, patient–doctor relationship, it is assumed that the doctor knows best. He or she will prescribe treatment, and the patient is expected to comply without ques- tioning. Moreover, the doctor may decide to withhold informa- tion when it is believed to be in the patient’s best interests. In this model, the physician asks most of the questions and generally dominates the interview.
There are circumstances in which an autocratic approach is desirable. In emergency situations, the doctor needs to take con- trol and make potentially life-saving decisions without long de- liberation. In addition, some patients feel overwhelmed by their illness and are comforted by a doctor who can take charge. In
general, however, the paternalistic autocratic approach risks a clash of values, especially in situations in which the patient must play a role, such as a life-threatening illness or issues concern- ing high-risk conception. In most instances, illnesses that can be monitored and controlled by the combined efforts of the doctor and patient, such as hypertension, should not be treated autocrat- ically.
6.18. The answer is A
Sex is not the only enticement with which psychiatrists can be seduced. Patients may offer insider information for profitable trading in the stock market, may promise an introduction to a movie star friend, or may suggest that they will dedicate their next novel to the psychiatrist. Although it is easy to understand that some offers by patients, such as the possibility of a sex- ual involvement, cannot be accepted without considerable harm to the patient, others may seem more innocuous. However, be- cause they nearly always introduce a different agenda into the therapy than that originally contracted for and because they cre- ate additional, more ambiguous levels of obligation between the therapist and patient, any psychiatric work is inevitably contam- inated. The ability to help the patient is compromised. Conse- quently, gaining any material or social benefit from the patient other than the agreed-on fee is unethical.
6.19. The answer is C
All patients must be asked about suicidal thoughts; however, depressed patients may need to be questioned more fully. A thorough assessment of suicide potential addresses intent, plans, means, and perceived consequences, as well as history of at- tempts and family history of suicide. Many patients mention their thoughts of suicide spontaneously. If not, the examiner can begin with a somewhat general question, such as: “Do you ever have thoughts of hurting yourself?” or “Does it ever seem that life isn’t worth living?” These questions can then be followed up with more specific questions. The examiner must feel com- fortable enough to ask simple, straightforward, noneuphemistic questions. Asking about suicide does not increase the risk. The psychiatrist is not raising a topic that the patient has not already contemplated. Specific, detailed questions are essential for pre- vention. Confronting a patient with the topic of suicide does not hinder the psychiatrist’s rapport.
6.20. The answer is E (all)
Compliance increases when physicians have a positive attitude and are enthusiastic and nonpunitive. Older doctors with experi- ence, the amount of time spent talking to patients, a short waiting room time, and increased frequency of visits are also associated with high compliance rates. The patient–doctor relationship, or match, is one of the most important factors in compliance is- sues. When a doctor and patient have different priorities and beliefs and different styles of communication (including a dif- ferent understanding of medical advice and different medical expectations), compliance decreases.
6.21. The answer is C
It is the psychiatrist’s responsibility to admit the patient (even if it must be on an involuntary basis) for stabilization and treatment
as the next best step in management. The patient is exhibiting paranoid delusions and is subject to auditory command halluci- nations. However, the focus of the interview should rest on the pa- tient’s homicidal ideation. The patient’s mother is in danger and must be immediately protected. Despite the patient’s denial of a specific plan to harm her mother, the patient’s mother should still be warned. Because the hallucinations likely occurred in the ab- sence of medical management (because the pills were “hidden”), it is unlikely that the dosage needs adjustment as the immediate next step in management. Suicidal and homicidal ideations, sub- stance intoxication or withdrawal, a threatening demeanor, and grave disorganization or inability to care for oneself are among the indications for admission.
Answers 6.22–6.26 6.22. The answer is C 6.23. The answer is A 6.24. The answer is C 6.25. The answer is A
6.26. The answer is C
Mack Lipkin, Jr., described three functions of medical inter- views: to assess the nature of the problem, to develop and main- tain a therapeutic relationship, and to communicate information and implement a treatment plan. These functions are exactly the same in psychiatric interviews. Also universal are the predomi- nant coping mechanisms used in illness, both adaptive and mal- adaptive. These mechanisms include such reactions as anxiety, depression, regression, denial, anger, and dependency. Physi- cians must anticipate, recognize, and address such reactions if treatment and intervention are to be effective. Psychiatric in- terviews have two major technical goals: (1) recognition of the psychological determinants of behavior and (2) symptoms clas- sification.
Answers 6.27–6.31 6.27. The answer is A 6.28. The answer is B 6.29. The answer is B 6.30. The answer is C
6.31. The answer is A
Transference and countertransference are very significant ex- pressions of unconscious processes in an interview. They are purely hypothetical constructs, but they have proven extremely useful as organizing principles for explaining certain develop- ments of the patient–doctor relationship that can be upsetting and that can interfere with good medical care.
Transference is the process of the patient’s unconsciously and inappropriately displacing onto the doctor aspects of important
past relationships, especially those with parents. A patient may come to see the doctor as cold, harsh, critical, threatening, seduc- tive, caring, or nurturing, not because of anything the physician says or does but because that has been the patient’s experience in the past. Transference can be positive or negative, and it can swing back and forth—sometimes abruptly—between the two.
Transference reactions may be strongest with psychiatrists for a number of reasons. For example, as part of intensive, insight- oriented psychotherapy, the encouragement of transference feel- ings is an integral part of treatment.
Countertransference is the process whereby the physician unconsciously displaces onto the patient patterns of behavior or emotional reactions as if he or she was a significant figure from earlier in the physician’s life. Countertransference may take the form of negative, disruptive feelings, but it may also encompass disproportionally positive, idealizing, or even eroticized reac- tions. Psychiatrists should be alert to signs of countertransfer- ence issues (missed appointment by the psychiatrist, boredom, discomfort, or sleepiness in a session). Supervision or consul- tations can be helpful as can personal therapy in helping the psychiatrist recognize and deal with these issues.
Answers 6.32–6.36 6.32. The answer is C 6.33. The answer is B 6.34. The answer is A 6.35. The answer is A
6.36. The answer is B
Interviewing any patient involves a fine balance between allow- ing the patient’s story to unfold at will and obtaining the neces- sary data for diagnosis and treatment. Most experts agree that an ideal interview begins with broad, open-ended questioning; continues by becoming specific; and closes with detailed direct questioning.
Open-ended questions (e.g., “Tell me about your pain”) iden- tify an area but provide minimal structure as to how to respond. This is in contrast to close-ended questions (e.g., “Is your pain sharp”), which provide much structure and narrow the field from which a response may be chosen. The ultimate close-ended ques- tion leads to a “yes” or “no” response. Close-ended questions can be effective in generating specific and quick responses about a clearly delineated topic. They have also been found to be effective in assessing such factors as the presence or absence, frequency, severity, and duration of symptoms. Table 6.3 summarizes some of the pros and cons of open- and closed-ended questions.
Answers 6.37–6.42 6.37. The answer is A 6.38. The answer is D 6.39. The answer is B
6. The Patient–Doctor Relationship and the Psychiatric Interview 53
54 6. The Patient–Doctor Relationship and the Psychiatric Interview Table 6.3

Pros and Cons of Open- and Closed-Ended Questions Broad, Open-Ended Questions

Aspect
Genuineness
Reliability
Precision
Time efficiency
Completeness of diagnostic coverage Acceptance by patient
High
They produce spontaneous formulations. Low
They may lead to nonreproducible answers. Low
Intent of question is vague.
Low
Circumstantial elaborations.
Low
Patient selects topic.
Varies
Most patients prefer expressing themselves
Narrow, Closed-Ended Questions
Low
They lead the patient.
High
Narrow focus, but they may suggest answers. High
Intent of question is clear.
High
May invite yes or no answers.
High
Interviewer selects topic.
Varies
Some patients enjoy clear-cut checks; others

freely; others feel guarded and insecure.
Reprinted with permission from Othmer E, Othmer SC. The Clinical Interview Using DSM-IV. Washington, DC: American Psychiatric Press; 1994.
hate to be pressed into a yes or no format.

6.40. The answer is A
6.41. The answer is C
6.42. The answer is B
The interactions between a doctor and patient—the questions a patient asks, the way in which news is conveyed and treatment recommendations are made—can take different shapes. In the paternalistic model, it is assumed that the doctor knows best. He or she prescribes treatment, and the patient is expected to com- ply without questioning. In this model, also called the autocratic model, the physician asks most of the questions and generally dominates the interview. Circumstances arise in which the pa- ternalistic approach is desirable (e.g., emergency situations). In general, however, the paternalistic approach risks a clash of val- ues. A paternalistic obstetrician, for example, might insist on spinal anesthesia for delivery when the patient wants to experi- ence natural childbirth.
In the informative model, the doctor dispenses information. All available data are freely given, but the choice is left wholly up to the patient. This model may be appropriate for certain one-
time consultations in which no established relationship exists and the patient will be returning to the regular care of a known physician. At other times, the informative model places the pa- tient in an unrealistically autonomous role and leaves him or her believing the doctor is cold and uncaring.
In the interpretive model, the doctor has come to know his or her patients better and understand something of the circum- stances of their lives, their families, their values, and their hopes and aspirations. The doctor is then better able to make recom- mendations that take into account the unique characteristics of an individual patient. A sense of shared decision making is es- tablished as the doctor presents and discusses alternatives, with the patient’s participation, to find the one that is best for that particular person.
In the deliberative model, the physician acts as friend or coun- selor to the patient, not just by presenting information but also in actively advocating a particular course of action. The deliber- ative approach is commonly used by doctors hoping to modify injurious behavior, for example, in trying to get a patient to stop smoking or lose weight.

7
Clinical Examination of the Psychiatric Patient

The psychiatric examination consists of two parts. The first is the psychiatric history, and the second is the mental status. The psychiatric history is the record of the patient’s life; it allows the psychiatrist to understand who the patient is, where the patient has come from, and where the patient is likely to go in the fu- ture. The history is the patient’s life story told in the patient’s own words from his or her own point of view. Information may be obtained from other sources, such as the patient’s parents or spouse. A thorough psychiatric history is essential to making a correct diagnosis and formulating a specific and effective treat- ment plan.
A patient’s history remains stable, whereas the mental sta- tus can change daily or hourly. The mental status examination (MSE) is a description of the patient’s appearance, speech, ac- tions, and thoughts during the interview. It is a systematic for- mat for recording findings about thinking, feeling, and behav- ior. Only phenomena observed at the time of the interview are recorded in the mental status. Other data are recorded in the history.
In this day and age of increased monitoring of medical care by third parties, the astute clinician must be aware of good doc- umentation of care and attend to the medical record. Reviews of cases are often conducted by persons with little or no back- ground in psychiatry who do not recognize the complexities of psychiatric diagnosis and treatment.
Similarly, psychiatrists must have a knowledge and under- standing of physical signs and symptoms. They must often de- cide whether a patient needs a medical examination and what that should include. There are numerous medical conditions that can manifest as psychiatric symptoms. Each of these conditions argues for a different set of laboratory and diagnostic tests. Ad- vances in biological psychiatry have made laboratory tests more and more useful. Laboratory tests are used to monitor dosing, treatment adherence, and toxic effects of various psychotropic medications.
The student should address the following questions and study the answers to gain knowledge of the clinical examination of the psychiatric patient.
HELPFUL HINTS
Students should familiarize themselves with these terms, especially the acronyms and names of laboratory tests.
anamnesis appearance, behavior,
attitude, and speech catecholamines
chief complaint
clang associations concentration, memory,
and intelligence confabulation consciousness and
orientation CSF
CT
delusional beliefs
EEG
family history
history of present illness;
medical history initial interview and
greeting interviewing
variations judgment and insight lithium
marital history mental status
examination military history
mood, feelings, and affect
neologisms occupational and
educational history paraphasia
perception
PET polysomnography prognosis psychiatric history psychiatric report psychodynamic
formulation
psychosexual history punning
reliability
sensorium and cognition sexuality
stress interview thought process treatment plan TRH
TSH
tricyclic antidepressants uncovering feelings VDRL
word salad
55
▲▲▲▲▲▲▲▲▲▲▲▲▲▲
▲ ▲▲ ▲▲▲▲▲▲▲▲
▲▲▲▲▲▲▲▲▲▲
▲▲ ▲▲▲▲ ▲▲ ▲▲▲
56 7. Clinical Examination of the Psychiatric Patient
QUESTIONS
Directions
Each of the questions or incomplete statements below is followed by five suggested responses or completions. Select the one that is best in each case.
. 7.1. Systematic errors are
A. the physician’s fault
B. caused by flaws in the hospital system
C. attributed to a specific member of the treatment
team
D. sending an email with patient information
E. most common in solo fee-for-service practices
. 7.2. Which of the following substances has been implicated in mood disorders with a seasonal pattern?
A. Estrogen
B. Gonadotropin-releasing hormone (GnRH) C. Luteotropic hormone (LTH)
D. Melatonin
E. Testosterone
. 7.3. If a patient receiving clozapine shows a white blood count (WBC) of 2,000 per cc, the clinician should
A. stop the administration of clozapine at once.
B. increase the dosage of clozapine at once.
C. monitor the patient’s WBC every 10 days.
D. terminate any antibiotic therapy.
E. institute weekly complete blood count (CBC) tests with differential.
. 7.4. Somatizing patients can be difficult to treat because
A. they have difficulty speaking spontaneously
B. they are violent
C. they may initially idealize the doctor
D. they may be reluctant to engage in self-reflection and
psychological exploration
E. they complain of pain
. 7.5. Common pretreatment lithium tests include
A. serum electrolytes B. serum BUN
C. ECG
D. pregnancy test
E. all of the above
. 7.6. The medical record
A. cannot be used in malpractice litigation B. is accessible to patients
C. is used only by the treating team
D. cannot be used by regulatory agencies E. is absolutely confidential
7.7. In a psychiatric interview
A. delusions should be challenged directly
B. the psychiatrist must not ask depressed patients if they
have suicidal thoughts
C. the psychiatrist should have a seat higher than the pa-
tient’s seat
D. the psychiatrist may have to medicate a violent patient
before taking a history
E. a violent patient should be interviewed alone to estab-
lish a patient–doctor relationship
7.8. True statements about diagnostic tests in psychiatric dis- orders include
A. increased serum calcium has been associated with de- pression
B. serum bicarbonate may be elevated in patients with bulimia nervosa
C. serum amylase may be increased in patients with bu- limia nervosa
D. serum bicarbonate may be decreased in patients with panic disorder
E. all of the above
7.9. Polysomnography (sleep EEG) abnormalities include
A. an increase in REM sleep in dementia
B. an increased sleep latency in schizophrenia
C. a decrease in the amount of REM sleep in major de-
pressive disorder
D. a lengthened REM latency in major depressive
disorder
E. none of the above
7.10. The following thyroid function test changes are true in patients with hypothyroidism except
A. Serum protein-bound iodine (PBI) is decreased. B. Serum-free thyroxine is decreased.
C. Serum T3 uptake is decreased.
D. Serum T3-to-T4 ratio is decreased.
E. Serum T3 concentration is decreased.
7.11. The psychiatric history
A. has no formal structure
B. does not address medical issues
C. attends to the patient’s anamnesis
D. focuses exclusively on information obtained from the
patient
E. focuses primarily on symptoms
7.12. Tests of concentration include all of the following except
A. calculations
B. proverb interpretation
C. spelling “world” backward
D. repeating a series of random numbers
E. repeating three or four unrelated objects after 5 to
10 minutes
. 7.13. A good test for recent memory is to ask patients
A. to subtract 7 from 100
B. their date of birth
C. how many siblings they have
D. what they had to eat for their last meal E. who is the president of the United States
. 7.14. Each of the following statements is true except
A. Panic attacks triggered by sodium lactate are inhibited by alprazolam.
B. Panic attacks triggered by sodium lactate are not in- hibited by propranolol.
C. Sodium lactate provokes panic attacks in a majority of patients with panic disorder.
D. Sodium lactate can trigger flashbacks in patients with posttraumatic stress disorder.
E. Hyperventilation is as sensitive as lactate provocation in inducing panic attacks.
. 7.15. True statements about the lengths of time drugs of abuse can be detected in urine include
A. alcohol for 7 to 12 hours
B. benzodiazepine for 2 to 3 weeks C. cocaine for 1 to 2 weeks
D. marijuana for 24 to 48 hours
E. morphine for 8 days
. 7.16. Psychiatrist: Did you have a happy childhood?
Patient: (Pause) Yes, I guess so.
Psychiatrist: Did you have friends?
Patient: Well, I guess it depends on what you mean by a friend.
Psychiatrist: Kids you did things with. How many? This is an example of
A. facilitating intervention B. expanding intervention C. obstructive intervention D. reinforcement
E. none of the above
. 7.17. A 33-year-old woman is fired from her job as a hairdresser. She had been working at the salon for 12 years and is in complete disbelief. As soon as she arrives home, her hus- band immediately notes that she is in an unpleasant mood. When asked what’s bothering her, the woman begins to verbally assault her husband for not giving their 5-year- old daughter a bath before dinner. Which of the follow- ing defense mechanisms are being demonstrated by this woman?
A. Rationalization B. Acting out
C. Displacement D. Dissociation
E. Conversion
7. Clinical Examination of the Psychiatric Patient 57 Directions
Each group of questions below consists of lettered headings fol- lowed by a list of numbered phrases or statements. For each numbered phrase or statement, select the one lettered heading that is most associated with it. Each lettered heading may be selected once, more than once, or not at all.
Questions 7.18–7.22
A. Omnipotence B. Altruism
C. Sublimation D. Projection
E. Reaction formation
7.18. Patient falsely attributes his or her own unacceptable feel- ings, impulses, or thoughts onto another.
7.19. Patient channels potentially maladaptive feelings or im- pulses into socially acceptable behavior.
7.20. Patient substitutes behavior, thoughts, or feelings that are diametrically opposed to his or her own unacceptable thoughts or feelings.
7.21. Patientfeelsoractsasifheorshepossessesspecialpowers or abilities and is superior to others.
7.22. Patient dedicates him- or herself to meeting the needs of others.
Questions: 7.23–7.27
A. Elevated level of 5-HIAA B. Decreased level of 5-HIAA
. 7.23. Aggressive behavior
. 7.24. Carcinoid tumors
. 7.25. High banana intake
. 7.26. Phenothiazine medications
. 7.27. Suicidal patients
Questions 7.28–7.32
A. Bromide intoxication
B. Bulimia nervosa
C. Creutzfeldt-Jakob disease
D. Epstein-Barr virus (EBV)
E. Mean corpuscular volume (MCV)
7.28. Elevated in alcoholism and vitamin B12 and folate defi- ciency
7.29. Causative agent for infectious mononucleosis; associated with depression, fatigue, and personality change
7.30. Decreased serum chloride
7.31. Psychosis, hallucinations, delirium 7.32. Generalized sharp waves on EEG
Questions 7.33–7.37
A. Circumstantiality B. Derailment
C. Perseveration
58 7. Clinical Examination of the Psychiatric Patient D. Flight of ideas
E. Thought blocking
. 7.33. Multipleassociationssothatthoughtsmoveabruptlyfrom idea to idea
. 7.34. A sudden break in the flow of ideas
. 7.35. Overinclusion of trivial details that impedes getting to the
point
. 7.36. A breakdown in the logical connection between ideas and
overall goal directedness
. 7.37. Repetition of words, phrases, or ideas that are out of con-
text
Questions 7.38–7.42
A. Broca’s aphasia
B. Wernicke’s aphasia C. Conduction aphasia D. Global aphasia
. 7.38. Significantly impaired ability to repeat words and phrases
. 7.39. Also called expressive aphasia
. 7.40. Patients are unaware of their communication problems
. 7.41. Impairment in all three dimensions of fluency, compre-
hension, and repetition
. 7.42. Attributed to damage to the arcuate fasciculus
ANSWERS
7.1. The answer is B
Systematic errors are caused by flaws in the hospital system or in the transfer of information. The failure is in the health care delivery system as a whole rather than in the individual doctor. An example is an inability to retrieve a patient’s medical records. Individual errors are attributed to a particular physician or to a specific member of the treatment team. The physician bears the responsibility even if the error is made by a person under his or her supervision. Examples include prescribing the wrong med- ication or operating on the wrong leg. It is appropriate to use email that contains patient information providing all releases have been obtained. Systematic errors are more common in in- stitutions where many people have to interact. Solo practices have few people, so there is less chance for errors to occur.
7.2. The answer is D
Melatonin is the substance that has been implicated in mood dis- orders with a seasonal pattern. Melatonin’s exact mechanism of action is unknown, but its production is stimulated in the dark, and it may affect the sleep–wake cycle. Melatonin is synthesized from serotonin, an active neurotransmitter. Decreased noctur- nal secretion of melatonin has been associated with depression. A number of other substances also affect behavior, and some known endocrine diseases (e.g., Cushing’s disease) have associ- ated psychiatric signs such as psychosis. Symptoms of anxiety or depression may also be explained in some patients by changes in endocrine function.
Luteotropic hormone (LTH) is an anterior pituitary hor- mone whose action maintains the function of the corpus lu- teum. Gonadotropin-releasing hormone (GnRH), produced by
the hypothalamus, increases the pituitary secretion of LTH and follicle-stimulating hormone (FSH).
Testosterone is the hormone responsible for secondary sex characteristics in men. A decreased testosterone level has been associated with erectile dysfunction and depression. Testos- terone is also formed in small amounts by the ovaries and the adrenal cortex.
Estrogen is the hormone responsible for pubertal changes in girls. Exogenous estrogen replacement therapy has been associ- ated with depression.
7.3. The answer is A
A patient who shows a WBC of 2,000 while taking clozapine (Clozaril) is at high risk for agranulocytosis. If agranulocytosis develops (i.e., if the WBC is less than 1,000) and there is evidence of severe infection (e.g., skin ulcerations), the patient should be placed in protective isolation on a medical unit. The clinician should stop the administration of clozapine at once, not increase the dosage of clozapine. The patient may or may not have clinical symptoms, such as fever and sore throat. If the patient does have such symptoms, antibiotic therapy may be necessary. Depending on the severity of the condition, the physician should monitor the patient’s WBC every 2 days, not 10 days, or institute daily, not weekly, CBC tests with differential.
7.4. The answer is D
Somatizing patients pose a number of difficulties for the consult- ing and the treating psychiatrist because they may be reluctant to engage in self-reflection and psychological exploration. Many somatizing patients live with the fear that their symptoms are not being taken seriously and the parallel fear that something med- ically serious may be overlooked. The psychiatrist’s main task is to acknowledge the suffering conveyed by the patient without necessarily accepting the patient’s explanation for symptoms. Clinicians should be curious about both the nature of the psy- chiatric complaint and the impact of those complaints on the patient’s life. While pain may be a complaint, it is not a major impediment to treatment.
Severely depressed patients may have difficulty concentrat- ing, thinking clearly, and speaking spontaneously. The psychia- trist evaluating a depressed patient may need to be more forceful and directive than usual. Although depressed patients should not be badgered, long silences are seldom useful, and the examiner may need to repeat questions more than once.
Narcissistic patients act as though they are superior to ev- eryone around them, including the doctor. They may initially idealize the doctor out of a need to have their doctor be as per- fect as they are, but idealization can quickly turn to disdain when they realize that the doctor is only human. Underneath their sur- face arrogance, narcissistic patients feel desperately inadequate and fear that others will see through them.
7.5. The answer is E (all)
The common lithium pretreatment tests include serum elec- trolytes, blood urea nitrogen (BUN), serum creatinine, urinal- ysis, thyroid function tests (TFTs) (e.g., thyroid-stimulating hormone, thyroxine [T4 ], T3 resin uptake [T3 RU]), and an elec- trocardiogram (ECG). In patients with a history suggestive of
possible kidney problems, a 24-hour urine test for creatinine and protein clearance is recommended.
Lithium has effects on a number of organ systems of which the clinician should be aware. Lithium therapy is associated with a benign elevation of the white blood cell count (WBC), which may reach 15,000 cells per mm3 . This WBC elevation can some- times be mistaken for signs of infection or wrongly attributed to lithium in the context of other signs of infection. Furthermore, lithium can have adverse effects on electrolyte balance (espe- cially in patients taking thiazide diuretics), thyroid function, the kidney, and the heart. Its levels may also be altered by nons- teroidal antiinflammatory drugs and aspirin.
Lithium may also lead to nephrogenic diabetes insipidus. Its levels can increase with dehydration. It has been argued that antithyroid antibody testing is helpful in assessing the possibil- ity of lithium-induced hypothyroidism. Because of the potential cardiac teratogenicity of lithium, a pregnancy test in women of childbearing age should be ordered. Periodic follow-up of serum electrolytes, BUN, creatinine, glomerular filtration rate, TFTs, ECG, and 24-hour urine for creatinine and protein clearance are recommended. The frequency and exact makeup of the follow- up testing battery should be dictated by the patient’s medical condition.
7.6. The answer is B
Patients have a legal right to access their medical records. This right derives from the belief that medical care is a collaborative process between doctor and patient. The medical record is a nar- rative that documents all events that occur during the course of treatment. It is used not only by the treating team but also by reg- ulatory agencies and managed care companies. It is also crucial in malpractice litigation. Although in theory it is accessible to authorized persons only and is safeguarded for confidentiality, absolute confidentiality cannot be guaranteed.
7.7. The answer is D
Psychiatrists often encounter violent patients in a hospital set- ting. Frequently, the police bring a patient into the emergency department in some type of physical restraint (e.g., handcuffs). The psychiatrist must establish whether effective verbal contact can be made with the patient or whether the patient’s sense of reality is so impaired that productive interviewing is impossible. If impaired reality testing is an issue, the psychiatrist may have to medicate a violent patient before taking a history.
With or without restraints, a violent patient should not be interviewed alone to establish a patient–doctor relationship. At least one other person should always be present; in some situ- ations, that other person should be a security guard or a police officer. Other precautions include leaving the interview room’s door open and sitting between the patient and the door so the interviewer has unrestricted access to an exit if it becomes nec- essary. The psychiatrist must make it clear, in a firm but calm manner, that the patient may say or feel anything but is not free to act in a violent way.
Delusions should never be directly challenged. Delusions are fixed false ideas that may be thought of as a patient’s defen- sive and self-protective, albeit maladaptive, strategy against over-
whelming anxiety, low self-esteem, and confusion. Challenging a delusion by insisting that it is not true or possible only increases the patient’s anxiety and often leads the patient to defend the belief desperately. However, clinicians should not pretend that they believe patients’ delusions. Often, the best approach is for clinicians to indicate they understand that the patient believes the delusion to be true but he or she does not hold the same belief.
Being mindful of the possibility of suicide is imperative when interviewing any depressed patient, even if a suicidal risk is not apparent. The psychiatrist must ask depressed patients if they have suicidal thoughts. Doing so does not make patients feel worse. Instead, many patients are relieved to talk about their suicidal ideas. The psychiatrist should ask specifically, “Are you suicidal now?” or “Do you have plans to take your own life?” A suicide note, a family history of suicide, or previous suicidal behavior by the patient increases the risk for suicide. Evidence of impulsivity or of pervasive pessimism about the future also places patients at risk. If the psychiatrist decides that the patient is in imminent risk for suicidal behavior, the patient must be hospitalized or otherwise protected.
The way chairs are arranged in the psychiatrist’s office affects the interview. The psychiatrist should not have a seat higher than the patient’s seat. Both chairs should be about the same height so that neither person looks down on the other.
7.8. The answer is E (all)
The patient’s history and physical examination typically dictate which tests are ordered. Laboratory abnormalities are typically useful when they optimize outcomes, that is, if the test results will contribute to the detection of a previously unrecognized medical condition or otherwise influence treatment. Diagnostic testing can also serve a therapeutic function by reassuring the patient or family that other serious medical problems do not appear to be present.
Serum amylase may be increased in patients with bulimia ner- vosa. Serum bicarbonate may be decreased in patients with panic disorder and may be elevated in patients with bulimia nervosa. Serum calcium may be increased in patients with depression in addition to hyperparathyroidism and bone metastases.
7.9. The answer is B
Electroencephalography (EEG) obtained during sleep is a po- tentially powerful biological marker of psychiatric illness. In schizophrenia (not major depressive disorder), increased sleep latency has been reported, especially during relapse. Sleep EEG abnormalities described in major depressive disorder include an increase (not a decrease) in the overall amount of rapid-eye movement (REM) sleep and a shortened (not lengthened) REM latency. Medical conditions giving rise to pseudodepressions are typically associated with decreased REM sleep. Patients with dementia usually have increased amounts of non-REM sleep.
7.10. The answer is D
Several thyroid function tests, including tests for thyroxine (T4), are used to rule out hypothyroidism, which can appear with symptoms of depression. Table 7.1 lists the thyroid function test changes associated with hypothyroidism. Common signs and
7. Clinical Examination of the Psychiatric Patient 59
60 7. Clinical Examination of the Psychiatric Patient
Table 7.1
Thyroid Function Test Changes in Patients with Hypothyroidism
1. Serum T4 concentration is decreased. 2. Serum-free thyroxine is decreased.
3. Serum T3 concentration is decreased. 4. Serum T3 uptake is decreased.
5. Serum PBI is decreased.
6. Serum TBG is normal.
7. Serum T3-to-T4 ratio is increased. 8. Serum TSH is increased
PBI, protein-bound iodine; T3, triiodothyronine; T4, thyroxine; TBG, thyroxine-binding globulin; TSH, thyroid-stimulating hormone.
Reprinted from MacKinnon RA, Yudofsky SC. Principles of the Psychiatric Evaluation. Philadelphia: JB Lippincott; 1991:97, with permission.
symptoms associated with both depression and hypothyroidism include fatigue, weakness, stiffness, poor appetite, constipation, menstrual irregularities, slowed speech, apathy, impaired mem- ory, and even hallucinations and delusions. Neonatal hypothy- roidism results in mental retardation and is preventable if the diagnosis is made before birth.
7.11. The answer is C
The patient’s personal and developmental history, or anamne- sis, is an essential focus of the psychiatric history (Table 7.2). The history, which includes psychiatric, medical, and family in- formation, comes from the patient but is often supplemented by collateral information from family members, social referral agencies, previous treating physicians, and old hospital records. It allows the psychiatrist to understand where the patient is, where
Table 7.2
Outline of Psychiatric History
I. Identifying data
II. Chief complaint
III. History of present illness A. Onset
B. Precipitating factors
IV. Past illnesses
A. Psychiatric
B. Medical
C. Alcohol and other substance history
V. Family history
VI. Personal history (anamnesis)
A. Prenatal and perinatal
B. Early childhood (through age 3 years) C. Middle childhood (ages 3–11 years)
D. Late childhood (puberty through adolescence) E. Adulthood
1. Occupational history
2. Marital and relationship history 3. Military history
4. Educational history
5. Religion
6. Social activity
7. Current living situation
8. Legal history
F. Sexual history
G. Fantasies and dreams H. Values
the patient has come from, and where the patient is likely to go in the future. It thereby focuses on much more than a symptom checklist. It has a structure, which is not a rigid plan for inter- viewing a patient but is a guide for organizing the psychiatrist’s thoughts and questions.
7.12. The answer is B
Concentration is the ability to sustain attention over time. Patients who forget the examiner’s question, are distracted by extraneous stimuli, or lose track of what they are saying have impaired concentration.
Memory, which involves concentration, must be evaluated across the spectrum of immediate to remote. One test of immediate re- call is to say (without inflection or verbal spacing) a series of random numbers and have the patient repeat the series. A pro- gressively longer sequence of numbers is presented, and both forward and backward recalls are tested. Most adults can eas- ily recall five or six numbers forward and three or four in re- verse. Recent memory is for events several minutes to hours old and may be evaluated by giving patients the names of three or four unrelated objects and asking them to repeat them after 5 to 10 minutes. Remote memory describes events 2 or more years old. It is usually revealed in the course of obtaining patients’ histories, although it may be necessary to confirm facts through collateral sources.
Calculations describe the ability to manipulate numbers men- tally. Simple addition, subtraction, or multiplication questions may be used. Problems of money and change are often helpful with patients who have a limited educational background. For ex- ample, if a magazine costs $3.50 and you pay with a $10 bill, how much change should you be given? Other tests of concentration include counting backward by 3s, reciting the alphabet backward, spelling “world ” backward, and naming the months of the year backward.
Abstract reasoning describes the ability to mentally shift back and forth between general concepts and specific examples. A frequently used way to test abstract reasoning is asking proverb interpretation. For example, a clinician might ask the patient, “What does it mean when someone says, ‘People who live in glass houses shouldn’t throw stones?’ ” A conventional response, one that is able to generalize from the specifics of the proverb to the generalization, might be, “Don’t criticize others of what you are guilty yourself.” A nonabstract response would address the concrete particulars without grasping the larger meaning, for example, “You would break the glass.” (Some answers will be id- iosyncratic and difficult to classify as either abstract or concrete: “The police would see you and would come to arrest you.”)
7.13. The answer is D
Recent memory is the ability to remember what has been expe- rienced within the past few hours, days, or weeks. It is assessed by asking patients to describe how they spent the past 24 hours, such as what they had to eat for their last meal.
Remote, or long-term, memory is the ability to remember events in the distant past. Memory for the remote past can be evaluated by inquiring about important dates in patients’ lives, such as their date of birth or how many siblings they have. The answers must be verifiable. Subtracting 7 from 100 is more a test
     
of concentration. Asking who the president of the United States is tests the general fund of information.
7.14. The answer is E
Table 7.4
Facilitating, Expanding, and Obstructive Interventions

Even though hyperventilation can trigger panic attacks in pre- disposed persons, hyperventilation is not as sensitive as lactate provocation in inducing panic attacks. Sodium lactate provokes panic attacks in a majority (up to 72 percent) of patients with panic disorder. Therefore, lactate provocation is used to confirm a diagnosis of panic disorder. Sodium lactate can also trigger flashbacks in patients with posttraumatic stress disorder. Car- bon dioxide (CO2) inhalation also precipitates panic attacks in those so predisposed. Panic attacks triggered by sodium lactate are not inhibited by peripherally acting β-blockers, such as pro- pranolol (Inderal), but are inhibited by alprazolam (Xanax) and tricyclic drugs.
7.15. The answer is A
The laboratory is useful for detecting substances of abuse and for evaluating the impact the substance use is having on the patient’s body. Often the laboratory detection of abused substances and certain diagnostic test abnormalities related to substance abuse (e.g., abnormal liver function tests in alcohol-abusing patients) are used therapeutically to confront the denial of a patient with a substance abuse disorder.
The most commonly used specimen for the detection of drugs of abuse is urine, although toxicological analyses can also be per- formed on blood specimens. The period of time that the clinician can detect drugs in blood specimens is typically shorter than the length of time drugs can be detected in urine specimens. How- ever, the length of time that a particular drug of abuse can be detected in the urine is somewhat variable, depending on the specific drug, the duration and amounts of the substance used, and concomitant medical problems (e.g., liver or kidney disease). Table 7.3 provides a list of some common drugs of abuse that can be detected in urine specimens along with a typical length of time after recent use in which the substance can be detected. Other specimens that have been studied to detect substance abuse include saliva and hair samples.
Facilitating
Reinforcement Reflection Summarizing Education Reassurance Encouragement Acknowledging
emotion Humor
Nonverbal communication
Silence
Expanding
Clarifying Associations Leading Probing Transitions Redirecting
Obstructive
Closed-ended questions Compound questions Why questions Judgmental questions Minimizing patient’s
concerns
Table 7.3
Drugs of Abuse That Can Be Tested in Urine
Adapted from McIntyre KM, Norton JR, McIntyre JS. Psychiatric interview, history, and mental status examination. In: Sadock BJ, Sadock VA, Ruiz P, eds. Kaplan & Sadock’s Comprehensive Textbook of Psychiatry. 9th Ed. Baltimore: Lippincott Williams & Wilkins; 2009:886.
7.16. The answer is C
Obstructive interventions are interviewing techniques that are not helpful for gathering information and the development of a positive patient–doctor relationship. These activities are unclear, unconnected, poorly timed, and not responsive to the patient’s issues or concerns (Table 7.4). The interview described above illustrates how a series of close-ended questions can retard the natural flow of the patient’s history. It reinforces the patient giv- ing a one-word or brief answer with little or no elaboration. Patients can be a partner in the interview unless blocked by the psychiatrist as with the interview above. Many patients, some of whom have previous experiences with therapy, come prepared to talk about even painful matters.
Facilitating interventions are effective in enabling the patient to continue sharing his or her story and are also helpful in promoting a positive patient–doctor relationship (Table 7.4). An example of a facilitating intervention is reinforcement. Brief phrases such as, “I see,” “Go on,” “Yes,” and “Tell me more” all convey the inter- viewer’s interest in the patient’s continuing. Although seemingly simplistic, such interventions are very important in the patient’s sharing material about him- or herself and other important indi- viduals and events in his or her life. Without these reinforcements, the interview will often become less productive.
Expanding interventions are used to expand the focus of the in- terview. Such interventions include clarifying, associations, lead- ing, probing, transitions, and redirecting (Table 7.4). These tech- niques are helpful when the line of discussion has been sufficiently mined, at least for the time being, and the interviewer wants to en- courage the patient to talk about other issues. These interventions are most successful when a degree of trust has been established in the interview and the patient believes that the psychiatrist is nonjudgemental about what is being shared.
7.17. The answer is C
In this vignette, the woman uses displacement as a neurotic de- fense mechanism, shifting emotions from an undesirable situa- tion to one that is more personally acceptable to her. Acting out is giving in to unconscious impulses by physical means. It is
7. Clinical Examination of the Psychiatric Patient 61
    
Drug
Alcohol Amphetamine Barbiturate
Benzodiazepine Cocaine
Codeine
Heroin
Marijuana Methadone Methaqualone Morphine Phencyclidine (PCP) Propoxyphene
Length of Time Detected in Urine
7–12 hours
48 hours
24 hours (short acting)
3 weeks (long acting)
3 days
6–8 hours (metabolites for 2–4 days) 48 hours
36–72 hours
3 days to 4 weeks (depending on use) 3 days
7 days
48–72 hours
8 days
6–48 hours
 
62 7. Clinical Examination of the Psychiatric Patient
considered an immature defense mechanism. Rationalization is also a neurotic defense mechanism. It makes negative outcomes more acceptable by way of justification (e.g., “I was fired be- cause my boss was in a bad mood”). Conversion defense mech- anisms are when physical symptoms become the manifestation of an emotional disturbance (e.g., if the woman had become temporarily unable to speak after learning that she was fired). Dissociation is another neurotic defense mechanism that con- sists of blocking out unpleasant feelings or thoughts to avoid emotional consequences.
Answers 7.18–7.22 7.18. The answer is D 7.19. The answer is C 7.20. The answer is E 7.21. The answer is A
7.22. The answer is B
All of the lettered responses are examples of specific defense mechanisms, that is, the unconscious processes used by the pa- tient to deal with emotional conflict or internal or external stres- sors. Omnipotence is when the patient, in order to cope with emotions or stressors, feels or acts as if he or she possesses
special powers or abilities and is superior to others. Altruism is when the patient gets pleasure from giving to others what the patient would have liked to receive. For example, a former al- coholic serves as an Alcoholics Anonymous (AA) sponsor to a new member, thereby achieving a transformative process that may be lifesaving to both the giver and receiver. Many acts of altruism involve free will, but some involuntarily sooth unmet needs. Sublimation is when the patient channels potentially mal- adaptive feelings or impulses into socially acceptable behavior. By analogy, sublimation permits the oyster to transform an irri- tating grain of sand into a pearl. In writing his Ninth Symphony, the deaf, angry, and lonely Beethoven transferred his pain into triumph by putting Schiller’s “Ode to Joy” to music. Projection is when the patient falsely attributes his or her own unaccept- able feelings, impulses, or thoughts onto another. For example, a cheating husband accuses his wife of infidelity. Reaction forma- tion is when the patient substitutes behavior, thoughts, or feelings that are diametrically opposed to his or her own unacceptable thoughts or feelings. For example, a former alcoholic works to ban the sale of alcohol in his town and annoys his social drinking friends.
Answers 7.23–7.27 7.23. The answer is B 7.24. The answer is A
Comments
Elevated in alcoholism and vitamin B12 and folate deficiency
Part of herpes virus group
EBV is causative agent for infectious mononucleosis,
which can present with depression, fatigue, and
personality change
CMV can produce anxiety, confusion, mood disorders EBV may be associated with chronic mononucleosis-like
syndrome associated with chronic depression and
fatigue
Decreased in patients with bulimia and psychogenic
vomiting
Mild elevation in hyperventilation syndrome, panic
disorder
Bromide intoxication can cause psychosis, hallucinations,
delirium
Part of dementia workup, especially when serum chloride
is elevated
Seizures, brain death, lesions; shortened REM latency in
depression
High-voltage activity in stupor; low-voltage fast activity in
excitement, functional nonorganic cases (e.g., dissociative states), alpha activity present in the background, which responds to auditory and visual stimuli
Generalized sharp waves on EEG seen in dementia of Creutzfeldt-Jakob disease
Table 7.5
Some Laboratory Findings in Mental Disorders
 
Test
MCV (average volume of a red blood cell)
EBV, CMV
Chloride), serum Bromide, serum EEG
Major Psychiatric Indication
Alcohol abuse
Cognitive and medical workup
Anxiety
Mood disorders
Eating disorders
Panic disorder
Dementia
Psychosis
Cognitive and medical workup
 
CMV, cytomegalovirus; EBV, Epstein-Barr virus; EEG, electroencephalography; MCV, mean corpuscular volume; RBC, red blood cell; REM, rapid eye movement.
7.25. The answer is A
Answers 7.33–7.37
7.33. The answer is D
7.34. The answer is E
7.35. The answer is A
7.36. The answer is B
7.37. The answer is C
See Table 7.6.
Answers 7.38–7.42 7.38. The answer is C 7.39. The answer is A 7.40. The answer is B 7.41. The answer is D
FIGURE 7.1
Brodmann’s areas of the human cortex, showing convex surface (top) and medial surface (bottom). (From Elliot HC. Textbook of Neuroanatomy. Philadelphia: Lippincott; 1969, with permission.)
7.26. The answer is A
7.27. The answer is B
The serotonin metabolite 5-hydroxyindoleacetic acid (5-HIAA) is elevated in the urine of patients with carcinoid tumors; at times in patients who take phenothiazine medications; and in persons who eat foods high in l-tryptophan, the chemical precursor of serotonin (e.g., walnuts, bananas, and avocados). The amount of 5-HIAA in cerebrospinal fluid is decreased in some persons who display aggressive behavior and in some suicidal patients who have committed suicide in particularly violent ways.
Answers 7.28–7.32 7.28. The answer is E 7.29. The answer is D 7.30. The answer is B 7.31. The answer is A
7.32. The answer is C
There are physiological changes that occur in some psychiatric disorders. Current laboratory tests do not allow all of them to be demonstrated. However, in some cases, as listed in Table 7.5 (see p. 62), those changes are very much in evidence and are useful in confirming diagnoses.
Table 7.6
Formal Thought Disorders
Circumstantiality: Overinclusion of trivial or irrelevant details that impede the sense of getting to the point.
Clang associations: Thoughts are associated by the sound of words rather than by their meaning (e.g., through rhyming or assonance).
Derailment (synonymous with loose associations): A breakdown in both the logical connection between ideas and the overall sense of goal directedness. The words make sentences, but the sentences do not make sense.
Flight of ideas: A succession of multiple associations so that thoughts seem to move abruptly from idea to idea; often (but not invariably) expressed through rapid, pressured speech.
Neologism: The invention of new words or phrases or the use of conventional words in idiosyncratic ways.
Perseveration: Out-of-context repetition of words, phrases, or ideas.
Tangentiality: In response to a question, the patient gives a reply that is appropriate to the general topic without actually answering the question. Example:
Doctor: “Have you had any trouble sleeping lately?”
Patient: “I usually sleep in my bed, but now I’m sleeping on the
sofa.”
Thought blocking: A sudden disruption of thought or a break in
the flow of ideas.
7. Clinical Examination of the Psychiatric Patient 63
   
64 7. Clinical Examination of the Psychiatric Patient
7.42. The answer is C
Broca’s aphasia (also called nonfluent or expressive aphasia) has traditionally been characterized by nonfluent speech but in- tact auditory comprehension and somewhat impaired repetition. It has long been thought to be associated with damage to the Broca’s area of the brain (i.e., left inferior frontal convolution) or Brodman’s area 44 (see Figure 7.1 on p. 63). However, more recent neuroimaging data in stroke patients have shown that the full syndrome of Broca’s aphasia, including agrammatism (tele- graphic speech), is found only in the presence of more extensive damage.
Wernicke’s aphasia (also called fluent or receptive aphasia) is characterized by fluent but nonsensical speech, impaired com- prehension, and somewhat impaired repetition. It has been as- sociated with damage to Wernicke’s area in the region of the superior temporal gyrus. The impaired ability to comprehend language directly affects the individual’s ability to self-monitor language output and may be related to a breakdown of the
syntactic structure of language. Unlike patients with Broca’s aphasia, who are usually painfully and obviously aware of their communication difficulty, patients with Wernicke’s aphasia are typically not aware of their communication problems. This is because Wernicke’s area is critical for comprehending their own speech as well as the language of others.
Patients with conduction aphasia demonstrate relatively in- tact auditory comprehension and spontaneous speech because of the preservation of Wernicke’s and Broca’s areas. However, the ability to repeat words and phrases is significantly impaired and has traditionally been attributed to damage to the arcuate fasciculus, which interconnects Wernicke’s and Broca’s areas. This type of aphasia is much more subtle and tends to have less negative impact on daily functioning.
Global aphasia is characterized by impairment in all three dimensions of fluency, comprehension, and repetition caused by damage to core language areas on the lateral surface of the left hemisphere.

8
Signs and Symptoms in Psychiatry

Signs are objective (observed by the clinician), and symptoms are subjective (perceived by the patient). The clinician observes the patient’s agitation; the patient’s complains of feeling de- pressed. In psychiatry, signs and symptoms are not as clearly demarcated as in other fields of medicine; they often overlap. Because of this, disorders in psychiatry are often described as syndromes—a group of signs and symptoms that together make up a recognizable condition. There are hundreds of terms used to describe the signs and symptoms of psychiatric illness, and students of psychiatry are encouraged to familiarize themselves with as many as possible. The language of psychiatry is pre- cise, and this allows clinicians to articulate their observations reliably.
In psychiatry, the presentation of signs and symptoms is not always straightforward. A patient may insist that nothing is wrong (that there are no symptoms) when it is obvious to most observers that certain behaviors or ways of thinking are bizarre, damaging, or disruptive. These might be defined as ego-syntonic
symptoms. Ego-dystonic symptoms are those of which the pa- tient is aware and that are experienced as uncomfortable or un- acceptable. Another complicating factor is that a clinician may not be able to literally observe or to hear a described symptom (e.g., an auditory hallucination) and may have to depend on in- direct evidence (e.g., a patient’s preoccupation or distraction) to diagnose it.
Many psychiatric signs and symptoms can be understood as various points of a spectrum or behavior ranging from normal to pathological. It is extremely rare to have a pathognomonic sign or symptom in psychiatry, although in some cases, the dis- turbance is specific to a neurological deficit. As John Nemiah wrote: “Psychiatry is a science of inexhaustible complexity. It is as infinite as the range of human emotions and behavior. One cannot possibly learn it all.” Because of that, psychiatry still remains as much art as science.
Student should study the questions and answers below for a useful review of these topics.
HELPFUL HINTS
The student should be able to define and categorize the signs and symptoms and other terms listed below.
affect and mood agnosias
anxiety
aphasic disturbances belle indifference cerea flexibilitas coma
de ́ja` entendu de ́ ja` pense ́ de ́ja` vu delirium
delusion
dementia depersonalization disorientation disturbances of conation disturbances of
consciousness and
attention disturbances of intelligence
disturbances of memory
disturbances of perception
dysmegalopsia dysmetria
fausse reconnaissance folie a` deux
hypnosis
illusions
insight and judgment jamais vu
noesis
panic
phobias pseudodementia psychoneurosis somatopagnosia speech form stereotypy synesthesia
thought content trailing phenomenon
QUESTIONS
Directions
Each of the questions or incomplete statements below is followed by five suggested responses or completions. Select the one that is best in each case.
8.1. Psychoneurosis includes all of the following disorders except
A. anxiety disorder
B. obsessive-compulsive disorder (OCD) C. sexual dysfunction
D. dysthymia
E. delusional disorder
8.2. Stereotypy is
A. ingrained, habitual involuntary movement
B. repetitive fixed pattern of physical action or speech
65
▲▲▲▲▲▲▲▲▲▲
▲ ▲▲▲▲▲▲▲▲▲
▲▲▲▲▲▲ ▲ ▲
▲▲▲▲▲▲▲▲▲▲▲
66
8. Signs and Symptoms in Psychiatry
C. pathological imitation of movements of one person by another
D. subjectivefeelingofmusculartensionandrestlessness secondary to antipsychotic or other medication
E. temporary loss of muscle tone and weakness precipi- tated by a variety of emotional states
. 8.3. Alexithymia is
A. an unpleasant mood
B. a state in which a person is easily annoyed and pro-
voked to anger
C. a loss of interest in and withdrawal from pleasurable
activities
D. an inability to describe or to be aware of emotions or
mood
E. a normal range of mood, implying absence of de-
pressed or elevated emotional state
. 8.4. Sundowning
A. is a result of overmedication
B. is associated with akathisia
C. is associated with stupor
D. occurs usually as a function of mania E. usually occurs in young individuals
. 8.5. A psychiatric patient who, although coherent, never gets to the point has a disturbance in the form of thought called
A. blocking
B. tangentiality
C. verbigeration
D. circumstantiality E. word salad
. 8.6. In DSM-IV-TR, delirium tremens is called
A. opiate withdrawal delirium
B. alcohol withdrawal delirium
C. benzodiazepine withdrawal delirium D. alcohol intoxication delirium
E. amphetamine intoxication delirium
. 8.7. Asking a patient to interpret a proverb is used as a way of assessing
A. abstract thinking B. impulse control C. insight
D. intelligence
E. judgment
. 8.8. Which of the following is a paramnesia?
A. Eidetic images B. Jamais vu
C. Lethologica D. Repression
E. Screen memories
8.9. The “Alice in Wonderland” effect is another name for
A. dysmegalopsia
B. dysphasia
C. trailing phenomenon D. dysmetria
E. fausse reconnaissance
8.10. Broca’s aphasia is another term used for
A. fluent aphasia B. jargon aphasia C. global aphasia D. paraphasia
E. expressive aphasia
8.11. Compared with primary process thinking, secondary pro- cess thinking is
A. illogical
B. related to the id
C. influenced by the environment D. immediate gratification seeking E. magical
8.12. A 41-year-old man presents to his psychiatrist upon his wife’s request. Four weeks earlier, the patient was hospi- talized for a mild injury to his head after crashing his car into a tree. He has a 3-year history of alcoholism. The pa- tient recounts that while shopping in his local grocery store 2 weeks ago, he had the realization that the store clerk is simply an imposter replacing his wife. The patient’s wife insists that her husband is completely delusional and is on the verge of divorcing him if he continues to neglect her as his actual, lawful wife of 15 years. Which of the following types of delusions is the patient expressing?
A. Capgras syndrome
B. Clerambault syndrome C. Delusional jealousy
D. Delusion of doubles E. Fregoli phenomenon
Directions
Each group of questions below consists of lettered headings fol- lowed by a list of numbered phrases or statements. For each numbered phrase or statement, select the one lettered heading that is most associated with it. Each lettered heading may be selected once, more than once, or not at all.
Questions 8.13–8.17
A. Belle Indifference B. Condensation
C. Eidetic Image
D. Glossolalia
E. Unio mystica
8.13. Feeling of mystic unity with an infinite power
8.14. A person showing disinterest in his or her physical com-
plaint
. 8.15. Unusually vivid or exact mental image of objects previ- ously seen or imagined
. 8.16. Unintelligible jargon that has meaning to the speaker but not the listener
. 8.17. One symbol stands for a number of components
Questions 8.18–8.21
A. Confabulation B. De ́ja` entendu C. De ́ja` pense ́ D. De ́ja` vu
E. Jamais vu
. 8.18. Illusion of auditory recognition
. 8.19. Regarding a new thought as a repetition of a previous
thought
. 8.20. Feeling of unfamiliarity with a familiar situation
. 8.21. Regarding a new situation as a repetition of a previous
experience
Questions 8.22–8.26
A. Amnestic aphasia B. Broca’s aphasia
C. Coprophasia
D. Syntactical aphasia E. Wernicke’s aphasia
. 8.22. Difficulty finding the correct name for an object
. 8.23. Spontaneous but incoherent speech
. 8.24. Inability to arrange words in proper sequence
. 8.25. Understanding remains but speech is grossly impaired
. 8.26. Involuntary obscene language
Questions 8.27–8.31
A. Simultagnosia
B. Astereognosis
C. Apraxia
D. Anosognosia
E. Adiadochokinesia
. 8.27. Inability to carry out specific tasks
. 8.28. Inability to recognize objects by touch
. 8.29. Inability to perform rapid alternating movements
. 8.30. Inability to recognize one’s own neurological deficit
. 8.31. Inability to comprehend more than one element of a visual
scene at a time
Questions 8.32–8.36
A. Apiphobia
B. Cynophobia
C. Musophobia D. Ophidiophobia E. Spheksophobia
. 8.32. Fear of mice
. 8.33. Fear of bees
8.34. Fear of wasps 8.35. Fear of snakes 8.36. Fear of dogs
Questions 8.37–8.39
A. Recall
B. Registration C. Retention
8.37. Capacity to hold memories in storage
8.38. Capacity to add new material to memory
8.39. Capacity to return previously stored memories to con-
sciousness
Questions 8.40–8.44
A. Cataplexy
B. Klein-Levin syndrome C. Narcolepsy
D. Nocturnal myoclonus E. Restless legs syndrome
8.40. Peculiar feelings during sleep, causing an irresistible need to move around
8.41. Periods of sleepiness, alternating with confusion, hunger, and sexual activity
8.42. Sudden attacks of generalized muscle weakness, leading to physical collapse while alert
8.43. Sudden attacks of irresistible sleepiness that may include hallucinations and cataplexy
8.44. Repetitive jerking of the legs during sleep, waking patients as well as their partners
Questions 8.45–8.49
A. Blocking
B. Clang association
C. Flight of ideas
D. Loosening of associations E. Neologism
8.45. “I was gigglifying, not just tempifying; you know what I mean.”
8.46. “I was grocery training; but when I ride the grocery, I drive the food everywhere on top of lollipops.”
8.47. “Cain and Abel—they were cannibals. You see, brothers kill brothers—that is laudable. If you ask me, though, never name your son Huxtibal. OK.”
8.48. Patient: “I never wanted.” Physician: “Go on. What were you saying?” Patient: “I don’t know.”
8.49. “Tired, mired, schmired, wired.” Questions 8.50–8.54
A. Haptic hallucinations
B. Olfactory hallucinations C. Ictal hallucinations
D. Autoscopic hallucinations E. Migrainous hallucinations
8. Signs and Symptoms in Psychiatry 67
68 8. Signs and Symptoms in Psychiatry
. 8.50. Most are simple visual hallucinations of geometric pat- terns, but phenomena such as micropsia and macropsia may occur
. 8.51. Hallucinations of one’s own physical self
. 8.52. Involve the sense of smell and are most often associated
with organic brain disease or psychotic depression
. 8.53. Occur as part of seizure activity and are typically brief and
stereotyped
. 8.54. Formication
Questions 8.55–8.58
A. Synesthesia
B. Paramnesia
C. Hypermnesia D. Eidetic images E. Lethologica
. 8.55. Exaggerated degree of retention and recall
. 8.56. Temporary inability to remember a name
. 8.57. Confusion of facts and fantasies
. 8.58. Sensationsthataccompanysensationsofanothermodality
Questions 8.59–8.63
A. Nihilistic delusion B. Somatopagnosia C. Simultanagnosia D. Twilight state
E. Verbigeration
. 8.59. Also called ignorance of the body and autotopagnosia
. 8.60. Disturbed consciousness with hallucinations
. 8.61. Depressive delusion that the world and everything related
to it have ceased to exist
. 8.62. Inability to recognize a part of one’s body as one’s own
. 8.63. Impairmentintheperceptionorintegrationofvisualstim-
uli appearing simultaneously
ANSWERS
8.1. The answer is E
A neurosis is a chronic or recurrent nonpsychotic disorder char- acterized mainly by anxiety (not psychosis), which is experi- enced directly or is altered through a defense mechanism. It re- mains a useful term and can be sufficient to impair the person’s functioning in a number of areas. It appears as a symptom, such as an obsession, a compulsion, a phobia, or a sexual dysfunction. In DSM-III, a neurotic disorder was defined as follows:
A mental disorder in which the predominant disturbance is a symptom or group of symptoms that is distressing to the individual and is recognized by him or her as unacceptable (ego-dystonic); reality testing is grossly intact. Behavior does not actively violate gross social norms (though it may be quite disabling). The distur- bance is relatively enduring or recurrent without treatment and is not limited to a transitory reaction to stressor.
Using the above definition, all of the above disorders are consid- ered neuroses except delusional disorder, which is a psychotic disorder.
8.2. The answer is B
Motor behavior is that aspect of the psyche that includes im- pulses, motivations, wishes, drives, instincts, and cravings as expressed by a person’s behavior or motor activity.
Stereotypy is a repetitive fixed pattern of physical action or speech. Echopraxia is a pathological imitation of movements of one person by another. Cataplexy is a temporary loss of mus- cle tone and weakness precipitated by a variety of emotional states. Mannerism is an ingrained, habitual involuntary move- ment. Akathisia is a subjective feeling of muscular tension sec- ondary to antipsychotic or other medication, which can cause restlessness, pacing, and repeated sitting and standing. It can be mistaken for psychotic agitation.
8.3. The answer is D
Mood is a pervasive and sustained emotion that is subjectively experienced and reported by a patient and observed by others; examples include depression, elation, and anger.
Euthymic mood is a normal range of mood, implying absence of depressed or elevated mood. Alexithymia is a person’s inability to describe or difficulty in describing or being aware of emotions or mood. Irritable mood is a state in which a person is easily annoyed and provoked to anger. Anhedonia is a loss of interest in and withdrawal from all regular and pleasurable activities. It is often associated with depression.
8.4. The answer is A
Sundowning, or sundowner’s syndrome, is the result of being overly sedated with medications. It is also seen with Alzheimer’s disease. It occurs in older (not younger) people and usually oc- curs at night. It is characterized by drowsiness, confusion, ataxia, and falling. It has no association with mania. Stupor is a distur- bance of consciousness that is defined by a lack of reaction to and unawareness of one’s surroundings; this is not true of sun- downing.
8.5. The answer is B
Tangentiality is the inability to have a goal-directed association of thoughts. The patient never gets from the desired point to the desired goal. Word salad is an incoherent mixture of words and phrases. Circumstantiality is indirect speech that is delayed in reaching the point but eventually gets there. Circumstantiality is characterized by an overinclusion of details. Verbigeration is a meaningless repetition of specific words or phrases. Blocking is an abrupt interruption in the train of thinking before a thought or idea is finished. After a brief pause, the person indicates no recall of what was being said or what was going to be said. It is also known as thought deprivation.
8.6. The answer is B
In the DSM-IV-TR, delirium tremens is called alcohol with- drawal delirium. Delirium tremens is an acute and sometimes fatal reaction to withdrawal from alcohol, usually occurring 72 to 96 hours after the cessation of heavy drinking. Distinctive characteristics are marked autonomic hyperactivity (tachycar- dia, fever, hyperhidrosis, and dilated pupils), usually accompa- nied by tremulousness, hallucinations, illusions, and delusions. Patients admitted to the hospital, especially for surgery, may
present several days into admission with alcohol withdrawal and may be at risk for delirium tremens. Benzodiazepine withdrawal delirium may appear similar to alcohol withdrawal and may also be accompanied by seizures. The onset of symptoms depends on the half-life of the particular benzodiazepine. Delirium may also occur with opioid withdrawal and has been reported with switching from transdermal fentanyl (Duragesic) to morphine (Duramorph). The presentation may occur because of unknown prior use of opiates in the context of abuse or prescription or methadone (Dolophine) maintenance.
8.7. The answer is A
Asking a patient to interpret a proverb is generally used as a way of assessing whether the person has the capacity for abstract thought. Abstract thinking, as opposed to concrete thinking, is characterized primarily by the ability to shift voluntarily from one aspect of a situation to another and to think symbolically. Concrete thinking is characterized by an inability to conceptu- alize beyond immediate experience or beyond actual things and events. Psychopathologically, it is most characteristic of persons with schizophrenia or organic brain disorders. For example, if the psychiatrist says, “People in glass houses shouldn’t throw stones,” a patient with schizophrenia may think “because the house might break.”
Judgment is the patient’s ability to comprehend the meaning of events and to appreciate the consequences of actions. It is of- ten tested by asking how the patient would act in certain standard circumstances (e.g., if the patient smelled smoke in a crowded movie theater). Impulse control is the ability to control acting on a wish to discharge energy that is, at the moment, felt to be dan- gerous, inappropriate, or otherwise ill advised. Insight is a con- scious understanding of forces that have led to a particular feel- ing, action, or situation. Intelligence is the capacity for learning, recalling, integrating, and applying knowledge and experience.
8.8. The answer is B
Paramnesias are the falsification of memory by distortion of re- call. Jamais vu is the false feeling of unfamiliarity with a real situation that the person has experienced and, similar to confab- ulation, de ́ja` vu, and false memory, is an example of a param- nesia. Eidetic images are visual memories of almost hallucina- tory vividness. Repression is an unconscious defense mechanism characterized by unconscious forgetting of unacceptable ideas. A screen memory is a consciously tolerable memory that cov- ers a painful memory. Lethologica is the temporary inability to remember a name or proper noun.
8.9. The answer is A
Dysmegalopsia is sometimes called the “Alice in Wonderland” effect. Dysmegalopsia is a distortion in which the size and shape of objects is misperceived. Objects may appear larger (macrop- sia) or smaller (micropia) than they actually are. It can result from a retinal disease and more commonly from temporal and parietal lobe lesions. In rare instances, it can be associated with schizophrenia. Dysphasia is a difficulty in comprehending oral language (reception dysphasia) or in trying to express verbal lan- guage (expressive dysphasia). It is caused by an acquired lesion of the brain. Trailing phenomenon is a perceptual abnormality as-
sociated with hallucinogenic drugs in which moving objects are seen as a series of discrete and discontinuous images. Dysmetria is an impaired ability to gauge distance relative to movements. It is commonly seen in neurological deficits. Fausse reconnais- sance is a false recognition and a feature of paramnesia. It can occur in delusional disorders.
8.10. The answer is E
Broca’s aphasia is another term used for expressive aphasia. Aphasia is any disturbance in the comprehension or expression of language caused by a brain lesion. There are different types of aphasias. Expressive aphasia is a disturbance of speech in which understanding remains intact but the ability to speak is grossly impaired. For these patients, speech is difficult to initi- ate, labored, nonfluent, and halting. Writing is also difficult in that sentence structure is poor. Expressive aphasia is also known as Broca’s aphasia, nonfluent aphasia, or motor aphasia. Fluent aphasia is characterized by the inability to understand the spo- ken word. Fluent but incoherent speech is present. Speech is well articulate and grammatically correct, but it lacks in content. It is also known as Wernicke’s aphasia, sensory aphasia, and recep- tive aphasia. Jargon aphasia is an aphasia in which the words that are produced are neological or nonsensical. Speech is fluent and effortless with proper grammar, but the patient has difficulty with noun selection. The patient will either replace words that sound or look like the original word or will replace the original word with sounds. The jargon makes sense to the patient but may not make sense to others. Global aphasia is a combination of grossly nonfluent aphasia and severe fluent aphasia. All aspects of spo- ken and written language are impaired in both expression and comprehension. Other cognitive skills remain intact. Paraphasia is a form of abnormal speech in which one word is substituted for another, the irrelevant word generally resembling the required one in morphology, meaning, or phonetic composition. The in- appropriate word may be a legitimate one used incorrectly (i.e., clover instead of hand) or a bizarre nonsense expression (i.e., treen instead of train). Paraphasic speech may be seen in or- ganic aphasias and in mental disorders such as schizophrenia.
8.11. The answer is C
Compared with primary process thinking, secondary process thinking is influenced by the demands of the environment. Secondary process thinking is defined as a form of thinking in psychoanalysis that is logical (not illogical), organized, reality oriented (not magical), and influenced by the demands of the en- vironment. It characterizes the mental activity of the ego (not the id). Primary process thinking, on the other hand, is directly re- lated to the functions of the id and is characteristic of unconscious mental processes. It is marked by primitive, prelogical thinking and the tendency to seek immediate gratification of instinctual demands. Primary process thinking includes thinking that is il- logical and magical. According to Sigmund Freud’s The Project for a Scientific Psychology, secondary process thinking corrects and regulates the primary process in accord with principles of logic, rationality, and reality. It is aimed at avoiding unpleasure, at delaying instinctual discharge, and at binding mental energy in accordance with the demands of external reality and the subject’s moral principles or values.
8. Signs and Symptoms in Psychiatry 69
70 8. Signs and Symptoms in Psychiatry
8.12. The answer is A
Delusions are fixed, false beliefs, strongly held and immutable in the face of refuting evidence, that are not consonant with the person’s education, social, and cultural background.
Delusions of misidentification are prominently reported be- cause of their inherently intriguing nature. In Capgras’s syn- drome, the patient believes that someone close to him or her has been replaced by an exact double, as in the above case. It is com- mon in patients with schizophrenia, brain injuries, and demen- tia. In Fre ́goli’s phenomenon, strangers are identified as familiar persons in the patient’s life. In the delusion of doubles, patients believe that another person has been physically transformed into themselves. In Cle ́rambault’s syndrome, patients believe that a person is erotically attached to them when in fact they are not. Delusional jealousy is a false belief about a spouse’s infidelity.
Answers 8.13–8.17 8.13. The answer is E 8.14. The answer is A 8.15. The answer is C 8.16. The answer is D
8.17. The answer is B
Belle indifference is when a person shows disinterest in his or her physical complain and may occur in conversion disorder. Condensation is a mental process in which one symbol stands for a number of components. An eidetic image is an unusually vivid or exact mental image of objects previously seen or imag- ined. Glossolalia is unintelligible jargon that has meaning to the speaker but not to the listener. Unio mystica is a feeling of mystic unity with an infinite power.
Answers 8.18–8.21 8.18. The answer is B 8.19. The answer is C 8.20. The answer is E
8.21. The answer is D
De ́ja` vu is regarding a new situation as a repetition of a previous experience. De ́ja` entendu is an illusion of auditory recognition. De ́ja` pense ́ is regarding a new thought as a repetition of a pre- vious thought. Jamais vu is a feeling of unfamiliarity with a familiar situation. Confabulation is the unconscious filling in of memory by imagining experiences that have no basis in fact.
Answers 8.22–8.26 8.22. The answer is A 8.23. The answer is E 8.24. The answer is D 8.25. The answer is B
8.26. The answer is C
Broca’s, nonfluent, and expressive aphasia are motor aphasias in which there is a disturbance of speech caused by a cognitive dis- order in which understanding remains but the ability to speak is grossly impaired, with halting, laborious, and inaccurate speech. In a syntactical aphasia, there is an inability to arrange words in proper sequence. In coprophasia, there is the involuntary use of vulgar language, seen in people with Tourette’s disorder and some people with schizophrenia. Nominal aphasias, also termed amnestic aphasias or anomies, result in difficulty finding the cor- rect name for an object. Wernicke’s aphasia is a sensory, fluent, or receptive aphasia in which there is an organic loss of abil- ity to comprehend the meaning of words; speech is fluid and spontaneous but incoherent and nonsensical.
Answers 8.27–8.31 8.27. The answer is C 8.28. The answer is B 8.29. The answer is A 8.30. The answer is D
8.31. The answer is A
Simultagnosia is the inability to comprehend more than one el- ement of a visual scene at a time or to integrate the parts into a whole. Anosognosia, or the ignorance of illness, is a person’s in- ability to recognize a neurological deficit as occurring to him- or herself. Apraxias refer to the inability to carry our specific tasks. Astereognosis is the inability to recognize objects by touch. Adi- adochokinesia is the inability to perform rapid alternating move- ments.
Answers 8.32–8.36 8.32. The answer is C 8.33. The answer is A 8.34. The answer is E 8.35. The answer is D
8.36. The answer is B
Phobias are irrational fears. In an effort to reduce the intense anxiety attached to phobic objects and situations, patients do their best to avoid the feared stimuli. Thus, phobias consist both of the fears and the avoidance components. The fear itself may include all the symptoms of extreme anxiety, up to and including panic. In specific phobias, persistent, irrational fears are provoked by specific stimuli. Table 8.1 lists some specific phobias. Common specific phobias include fear of dust, excreta, snakes, spiders, heights, and blood.
Answers 8.37–8.39 8.37. The answer is C 8.38. The answer is B
Table 8.1 Specific Phobias
Acrophobia Agoraphobia Amathophobia Apiphobia Astrapophobia Blennophobia Claustrophobia Cynophobia Decidophobia Electrophobia Eremophobia Gamophobia Gatophobia Gephyrophobia Gynophobia Hydrophobia Kakorrhaphiophobia Katagelophobia Keraunophobia Musophobia Nyctophobia Ochlophobia Odynophobia Ophidiophobia Pnigerophobia Pyrophobia Scholionophobia Sciophobia Spheksophobia Technophobia Thalassophobia Triskaidekaphobia Tropophobia
8.39. The answer is A
Fear of heights
Fear of open spaces
Fear of dust
Fear of bees
Fear of lightning
Fear of slime
Fear of enclosed spaces
Fear of dogs
Fear of making decisions
Fear of electricity
Fear of being alone
Fear of marriage
Fear of cats
Fear of crossing bridges
Fear of women
Fear of water
Fear of failure
Fear of ridicule
Fear of thunder
Fear of mice
Fear of night
Fear of crowds
Fear of pain
Fear of snakes
Fear of smothering
Fear of fire
Fear of school
Fear of shadows
Fear of wasps
Fear of technology
Fear of the ocean
Fear of number 13
Fear of moving or making changes
ories are not passively retrieved but are actively reconstructed. Each act of recollection requires an act of putting the memory together, not simply lifting it ready made from a file. At times, failure to recall may signify that the memory traces themselves have disappeared and are no longer retrievable. However, diffi- culties in recall can occur separately, as in the everyday event of forgetting the name of a person or object, only to spontaneously remember it hours or days later.
Answers 8.40–8.44 8.40. The answer is E 8.41. The answer is B 8.42. The answer is A 8.43. The answer is C
8.44. The answer is D
In narcolepsy, the patient has sudden attacks of irresistible sleepiness, a symptom that may be part of a broader syndrome that includes cataplexy (sudden attacks of generalized muscle weakness leading to physical collapse in the presence of alert consciousness).
Periodic hypersomnia occurs in the Klein-Levin syndrome, a condition that typically affects young men, in which periods of sleepiness alternate with confusional states, ravenous hunger, and protracted sexual activity.
Sensory symptoms during sleep, typically described by pa- tients as peculiar feelings in their legs, causing an irresistible need to move around, are characteristic of restless legs syndrome. The motor abnormality of repetitive myoclonic jerking of the legs, awakening both patients and their partners, is known as nocturnal myoclonus.
Answers 8.45–8.49 8.45. The answer is E 8.46. The answer is D 8.47. The answer is C 8.48. The answer is A
8.49. The answer is B
All of the lettered responses are examples of specific distur- bances in form of thought. Neologisms are new words created by the patient, often by combining syllables of other words, for idiosyncratic psychological reasons. Loosening of associations is a flow of thoughts in which ideas shift from one subject to another in completely unrelated ways. When the condition is se- vere, the patient’s speech may be incoherent. Flight of ideas is a rapid, continuous verbalization or play on words that produces a constant shifting from one idea to another; the ideas tend to be connected, and when the condition is not severe, a listener may be able to follow them; the thought disorder is most characteristic of someone in a manic state. Blocking is an abrupt interruption in a train of thinking before a thought or idea is finished; after
8. Signs and Symptoms in Psychiatry 71
  
Memory functions have been divided into three stages: registra- tion, retention, and recall. Registration or acquisition refers to the capacity to add new material to memory. The material may be sensory, perceptual, or conceptual and may come from the environment or from within the person. For new material to be acquired, the person must attend to the information presented; it must then be processed or cortically organized. Registration and short-term memory retention are usually impaired in disorders that affect vigilance and attention, such as head trauma, delir- ium, intoxication, psychosis, spontaneous or induced seizures, anxiety, depression, and fatigue. Retention is the ability to hold memories in storage. Recall is the capacity to return previously stored memories to consciousness.
Benzodiazepine use has been associated with working memory difficulties, especially in elderly patients. Some short-acting high- potency benzodiazepines used as sleeping pills may be particu- larly troublesome in this regard.
The retention of memories is impaired in posttraumatic amne- sia as well as in a number of cognitive disorders, such as demen- tia of the Alzheimer’s type and Wernicke-Korsakoff syndrome. The latter, which ordinarily results from the chronic thiamine deficiency seen with alcoholism, is associated with pathological alterations in the mammillary bodies and thalamus.
Disturbances in recall can occur even when memories have been registered and are in storage. Research has shown that mem-
72 8. Signs and Symptoms in Psychiatry
a brief pause, the person indicates no recall of what was being said or what was going to be said. The condition is also known as thought deprivation. A person who is using clang association uses an association of words similar in sound but not in mean- ing; the words used have no logical connections and may include examples of rhyming and punning.
Answers 8.50–8.54 8.50. The answer is E 8.51. The answer is D 8.52. The answer is B 8.53. The answer is C
8.54. The answer is A
Hallucinations are perceptions that occur in the absence of corre- sponding sensory stimuli. Hallucinations are ordinarily subjec- tively indistinguishable from normal perceptions. Hallucinations are often experienced as being private, so others are not able to see or hear the same perceptions. The patient’s explanation for this is typically delusional. Hallucinations can affect any sensory system and sometimes occur in several concurrently.
Autoscopic hallucinations are hallucinations of one’s own physical self. Such hallucinations may stimulate the delusion that one has a double (doppelganger). Reports of near-death out-of- body experiences in which individuals see themselves rising to the ceiling and looking down at themselves in a hospital bed may be autoscopic hallucinations.
Haptic hallucinations involve touch. Simple haptic halluci- nations, such as the feeling that bugs are crawling over one’s skin (formication) are common in alcohol withdrawal syndromes and in cocaine intoxication. When unkempt and physically neglect- ful patients complain of these sensations, they may be caused by the presence of real physical stimuli such as lice. Some tac- tile hallucinations (e.g., having sexual intercourse with God) are highly suggestive of schizophrenia but may also occur in pa- tients with tertiary syphilis and other conditions and may in fact be stimulated by local genital irritation. Olfactory and gusta- tory hallucinations, involving smell and taste, respectively, have most often been associated with organic brain disease, particu- larly with the uncinate fits of complex partial seizures. Olfactory hallucinations may also be seen in patients with psychotic de- pression, typically as odors of decay, rotting, or death.
Ictal hallucinations, occurring as part of seizure activity, are typically brief, lasting only seconds to minutes, and stereotyped. They may be simple images, such as flashes of light, or elabo- rate ones, such as visual recollections of past experiences. Dur- ing the hallucinations, the patient ordinarily experiences altered consciousness or a twilight sleep.
Migrainous hallucinations are reported by about 50 percent of patients with migraines. Most are simple visual hallucinations of geometric patterns, but fully formed visual hallucinations, sometimes with micropsia and macropsia, may also occur. This complex has been called the Alice in Wonderland syndrome after Lewis Carroll’s descriptions of the world in Through the Looking Glass, mirroring some of his own migrainous experiences. In turn, these phenomena closely resemble visual hallucinations induced by psychedelic drugs such as mescaline.
Answers 8.55–8.58 8.55. The answer is C 8.56. The answer is E 8.57. The answer is B
8.58. The answer is A
In synesthesia, the patient experiences sensations that accom- pany sensations of another modality (e.g., an auditory sensation is accompanied by or triggers a visual sensation or a sound is experienced as being seen or accompanied by a visual experi- ence). Paramnesia is a confusion of facts and fantasies; it leads to a falsification of memory with the distortion of real events by fantasies. Hypermnesia is an exaggerated degree of retention and recall or an ability to remember material that ordinarily is not retrievable. Eidetic images, also known as primary memory images, are visual memories of almost hallucinatory vividness. Lethologica is the temporary inability to remember a name or a proper noun.
Answers 8.59–8.63 8.59. The answer is B 8.60. The answer is D 8.61. The answer is A 8.62. The answer is B
8.63. The answer is C
A nihilistic delusion is a depressive delusion that the world and everything related to it have ceased to exist. Somatopagnosia (also called ignorance of the body and autotopagnosia) is the inability to recognize a part of one’s body as one’s own. Si- multanagonisa is impairment in the perception or integration of visual stimuli appearing simultaneously. A twilight state is dis- turbed consciousness with hallucinations. Verbigeration is the meaningless and stereotyped repetition of words or phrases as seen in people with schizophrenia.

9
Classification in Psychiatry and Psychiatric Rating Scales

Classification is the systematic arrangement of items in groups or categories according to established criteria. The classification of mental disorders consists of specific mental disorders that are grouped into various classes on the basis of some shared char- acteristics. Systems of classification for psychiatric diagnoses have several purposes: (1) to distinguish one psychiatric diagno- sis from another, so that the clinicians can offer the most effective treatment; (2) to provide a common language among health care professionals; and (3) to explore the still unknown causes of many mental disorders.
The two most important psychiatric classifications are the Diagnostic and Statistical Manual of Mental Disorders (DSM) developed by the American Psychiatric Association (APA) in collaboration with other groups of mental health professionals and the International Classification of Diseases (ICD) developed by the World Health Organization (WHO). The ICD is the offi- cial classification system used in Europe and many other parts of the world, and the DSM is the official coding system used in
the United States. The DSM describes manifestations of men- tal disorders in terms of its associated features; age, culture, and gender-related features; prevalence, incidence, and risks; course; complications; predisposing factors; familial pattern; and differ- ential diagnosis. It is a multiaxial system that evaluates patients along several variables and contains five axes.
Psychiatric rating scales are a variety of questionnaires, inter- views, checklists, outcome assessments, and other instruments that are available to inform psychiatric practice, research, and administration. They aid clinicians by helping them confirm their diagnoses and clarify their thinking in ambiguous situa- tions. They can also provide a baseline for follow-up of the progress of an illness over time or in response to specific in- terventions. Without these scales, psychiatrists are left with only their clinical impressions, which does not allow for reliable comparison.
Students should study the questions and answers below for a useful review of these topics.
HELPFUL HINTS
The student should be able to define the terms below, especially the diagnostic categories.
atheoretical classification
clinical syndromes competence
descriptive approach diagnostic criteria differential diagnosis disability determination DSM-IV-TR
ego dystonic ego syntonic
familial pattern Global Assessment
of Functioning
Scale
gross social norms highest level of
functioning ICD-10
impairment Emil Kraepelin Likert scale
Mini-Mental State Examination
multiaxial system
partial and full remission predictive validity predisposing factors prevalence
psychosocial and
environmental
stressors reality testing
residual type severity-of-stress rating sex ratio
Social and Occupational
Functioning
Assessment Scale Structured Clinical
Interview for DSM validity and reliability
73
▲▲▲▲▲▲
▲ ▲▲▲▲▲▲ ▲
▲▲ ▲▲ ▲▲▲▲
▲▲▲▲▲▲▲▲▲▲▲
74 9. Classification in Psychiatry and Psychiatric Rating Scales
CLASSIFICATION IN PSYCHIATRY QUESTIONS
Directions
Each of the questions or incomplete statements below is followed by five suggested responses or completions. Select the one that is best in each case.
. 9.1. AxisIIoftheDSM-IV-TRincludeswhichofthefollowing disorders?
A. Occupational problem B. Mental retardation
C. Neoplasms
D. Depressive disorder E. Adjustment disorders
. 9.2. Which of the following about the multiaxial diagnostic classification is true?
A. It is required for proper use of DSM-IV-TR.
B. Axis I precludes disorders diagnosed in infancy, child-
hood, or adolescence.
C. Axis II consists of personality disorders only.
D. Axis III lists only the physical disorders that are
causative of the patient’s mental disorder.
E. AxisIVstressorsareevaluatedbasedontheclinician’s assessment of the stress that an average person with similar sociocultural values and circumstances would
experience from the psychosocial stressors.
. 9.3. A16-year-oldmalehighschoolstudentpresentstoaclinic because of episodes of diaphoresis and palpitations. These episodes typically last 5 minutes and occur without warn- ing. The patient becomes quite scared during the episodes and begins to tremble. He is very embarrassed about his “mental disorder.” The DSM-IV-TR definition of a mental disorder includes all of the following except
A. deviant behaviors that are primarily between the indi- vidual and society
B. not merely expectable responses to particular events
C. significantly increased morbidity
D. significant disability
E. significant distress
. 9.4. The differentiation of in full remission from recovered requires consideration of which factor?
A. The characteristic course of the disorder
B. The length of time since the last period of disturbance C. The total duration of the disturbance
D. All of the above
E. None of the above
. 9.5. Dementia in Alzheimer’s disease may be characterized by each of the following terms except
A. mixed type B. atypical type
C. acute onset
D. with late onset E. with early onset
. 9.6. “Not otherwise specified (NOS)” categories of DSM-IV- TR may be used when
A. the cause is uncertain
B. there is an atypical presentation
C. the symptoms are below the threshold for a specific
disorder
D. thesymptompatterncausessignificantdistressbuthas
not been included in the DSM-IV-TR classification
E. all of the above
. 9.7. Which of the following is an appendix diagnosis in DSM- IV-TR?
A. Minor depressive disorder
B. Caffeine withdrawal
C. Premenstrual dysphoric disorder D. Factitious disorder by proxy
E. All of the above
. 9.8. In DSM-IV-TR, the severity of all the following disorders are captured by the diagnostic code except
A. major depression B. mental retardation C. conduct disorder D. mania
E. mixed manic-depressive episodes
. 9.9. True statements about DSM-IV-TR include
A. Axis I and Axis II comprise the entire classification of mental disorders.
B. ManypatientshaveoneormoredisordersonbothAxis I and Axis II.
C. The habitual use of a particular defense mechanism can be indicated on Axis II.
D. On Axis III, the identified physical condition may be causative, interactive, and effect or unrelated to the mental state.
E. All of the above
. 9.10. Which of the following factors are used to designate a principal disorder in a patient with multiple psychiatric disorders?
A. Age of onset
B. Relative degree of impairment C. Reason for seeking treatment D. All of the above
E. None of the above
Directions
Each group of questions below consists of lettered headings fol- lowed by a list of numbered phrases or statements. For each numbered phrase or statement, select the one lettered heading
that is most associated with it. Each lettered heading may be selected once, more than once, or not at all.
Questions 9.11–9.15
A. Axis I B. Axis II C. Axis III D. Axis IV E. Axis V
. 9.11. Repeated theft of items that are not needed for personal use or monetary gain
. 9.12. GAF = 45 (on admission), GAF = 65 (at discharge)
. 9.13. Alcohol gastritis secondary to alcohol dependence
. 9.14. The adult manifestation of childhood conduct disorder
. 9.15. Inadequate health insurance
ANSWERS
9.1. The answer is B
Mental retardation is defined as deficits in cognitive abilities as well as in behaviors required for social and personal suffi- ciency, known as adaptive functioning. Recently, the field has replaced the term mental retardation with the more precise term of intellectual instability. This disorder is found in Axis II of the DSM-IV-TR. This axis also includes personality disorders, which are defined as enduring subjective experiences and behav- iors that deviate from cultural standards, are rigidly pervasive, are stable through time and lead to unhappiness and impairment.
Occupational problem is included in Axis IV, which consists of psychosocial and environmental problems. Occupational prob- lems often arise during stressful changes in work, namely, at ini- tial entry into the workforce or when making job changes within the same organization to a higher position. Distress occurs partic- ularly if these changes are not sought and no preparatory training has taken place, as well as during layoffs and at retirement. This is especially true if the retirement is mandatory and the person is not prepared for this event. Neoplasms are abnormal growths and dysplastic changes in tissue in animals and plants. Neoplasms are included in Axis III, which consists of physical disorders and general medical conditions that are present in addition to a mental disorder. The physical condition may be causative, the result of a mental disorder, or unrelated to the mental disorder. Major depres- sive disorder and adjustment disorders are both included in Axis I, which consists of clinical disorders and other conditions that may be the focus of clinical attention. Major depressive disorder is characterized by an all-encompassing low mood accompanied by low self-esteem and loss of interest or pleasure in normally en- joyable activities. It occurs without a history of a manic, mixed, or hypomanic episode and must last at least 2 weeks. Adjustment disorders are characterized by an emotional response to a stressful event. Typically, the stressor involves financial issues, a medical illness, or a relationship problem.
9.2. The answer is E
Axis IV codes the psychological and environmental problems
that contribute significantly to the development or exacerbation of the current disorder. The evaluation of the stressors is based on the clinician’s assessment of the stress that an average person
with similar values and circumstances would experience. This judgment is based on the amount of change that the stressor causes in the person’s life, the degree to which the event is desired and under the person’s control, and the number of stressors.
The DSM-IV-TR is a multiaxial system of evaluation in which different domains of information are described on five different axes. Proper use of the DSM-IV-TR does not require the use of the multiaxial format. The DSM-IV-TR also allows clinicians who do not wish to use the multiaxial format to list the diagnoses serially, with the principal diagnosis listed first. The purpose of multiaxial evaluation is to promote a comprehensive, biopsy- chosocial approach toward clinical assessment. Axis I consists of clinical disorders and other conditions that may be a focus of clinical attention. It includes disorders first diagnosed in in- fancy, childhood, or adolescence, excluding mental retardation and personality disorders, which are coded on Axis II. Axis III lists any physical disorder or general medical condition that is present in addition to the mental disorder. The physical condi- tion may be causative of, the result of, or unrelated to the mental disorder.
9.3. The answer is A
Mental disorders must be considered a manifestation of a behav- ioral, psychological, or biological dysfunction in the individual. Neither deviant behavior nor conflicts that are primarily be- tween the individual and society are mental disorders unless the deviance or conflict is a symptom of a dysfunction in the individ- ual. They are conceptualized as clinically significant behavioral or psychological syndromes or patterns that occur in an individ- ual and that are associated with present distress or disability or with a significantly increased risk of suffering death, pain, dis- ability, or an important loss of freedom. The syndromes must not be an expectable and culturally sanctioned response to a particular event.
9.4. The answer is D (all)
In full remission means that there are no longer any symptoms or signs of the disorder present, but it is still clinically relevant to note the disorder. Recovered means that the disorder would no longer be noted. The differentiation of in full remission and recovered requires consideration of many factors, including the characteristic course of the disorder, the length of time since the last period of disturbance, and the total duration of the distur- bance. However, the differentiation is not absolute. According to DSM-IV-TR, even when a patient is considered “recovered,” a history of the disorder may be noted if the physician believes the information is valuable to the case.
9.5. The answer is C
Acute onset refers to vascular dementia, not dementia in Alzheimer’s disease, which has a progressive onset. The other four types—mixed, atypical, with late onset, and with early onset—all refer to dementia in Alzheimer’s disease.
9.6. The answer is E (all)
Each diagnosis has a “not otherwise specified” (NOS) cate- gory. According to DSM-IV-TR, an NOS diagnosis may be appropriate either when the symptoms are below the diagnos- tic threshold for one of the specific disorders or when there is an
9.
Classification in Psychiatry and Psychiatric Rating Scales 75
76 9. Classification in Psychiatry and Psychiatric Rating Scales
atypical or mixed presentation. It is similarly applicable if the symptom pattern has not been included in the DSM-IV-TR classi- fication but it causes clinically significant distress or impairment of functioning or the cause is uncertain.
9.7. The answer is E (all)
The DSM-IV-TR contains proposed criteria for 20 specific disor- ders that were not included in the official classification. They are instead included in an appendix so research can be conducted on their reliability, validity, and potential clinical utility (Table 9.1). Many of these disorders are currently captured by the classifica- tion under the “not otherwise specified” (NOS) designation. For example, depressive disorder NOS can be use for minor depres- sive disorder or premenstrual dysphoric disorder.
9.8. The answer is C
When the full criteria for a disorder are met, its severity can be specified as mild, moderate, or severe. Severity ratings are based on the number and intensity of the symptoms of the dis- order and the impairment in occupational and social functioning caused by symptoms. In the DSM-IV-TR, the severity specifier can be applied to all disorders, although specific guidelines for making this rating are provided for only a few disorders. Ex- amples are mental retardation, conduct disorder, mania, mixed manic-depressive episodes, and major depression. For each of
Table 9.2
Axes of DSM-IV-TR
Axis I: Clinical Disorders and Other Disorders That May Be a Focus of Clinical Attention
Disorders usually first diagnosed in infancy, childhood, or adolescence (excluding mental retardation)
Delirium, dementia, and amnestic and other cognitive disorders Mental disorders caused by general medical condition not
elsewhere classified
Substance-related disorder
Schizophrenia and other psychotic disorders
Mood disorders
Anxiety disorders
Somatoform disorders
Factitious disorders
Dissociative disorders
Sexual and gender identity disorder
Eating disorders
Sleep disorders
Impulse-control disorders not elsewhere classified Adjustment disorders
Other conditions that may be a focus of clinical attention
Axis II: Personality Disorder and Mental Retardation
Paranoid personality disorder
Schizoid personality disorder
Schizotypal personality disorder
Antisocial personality disorder
Borderline personality disorder
Histrionic personality disorder
Narcissistic personality disorder
Avoidant personality disorder
Dependent personality disorder Obsessive-compulsive personality disorder Personality disorder not otherwise classified Mental retardation
Table 9.1
Appendix Diagnoses in DSM-IV-TR
Postconcussional disorder
Mild neurocognitive disorder
Caffeine withdrawal
Postpsychotic depressive disorder of schizophrenia
Simple deteriorative disorder
Minor depressive disorder
Recurrent brief depressive disorder
Premenstrual dysphoric disorder
Mixed anxiety depressive disorder
Factitious disorder by proxy
Dissociative trance disorder
Binge-eating disorder
Depressive personality disorder
Passive-aggressive personality disorder (negativistic personality
disorder)
these disorders, except conduct disorder, the disorder’s severity is captured by the diagnostic code.
9.9. The answer is E (all)
The DSM-IV-TR is a multiaxial system that comprises five axes and evaluates the patient along each. Axis I and Axis II comprise the entire classification of mental disorders, including 17 major groupings, more than 300 specific disorders, and almost
Axis III: General Medical Conditions
Infectious and parasitic diseases
Neoplasms
Endocrine, nutritional, and metabolic diseases and immunity
disorders
Diseases of blood and blood-forming organs
Diseases of the nervous system and sense organs Diseases of the circulatory system
Diseases of the respiratory system
Diseases of the digestive system
Diseases of the genitourinary system
Complications of pregnancy, childbirth, and puerperium Diseases of the skin and subcutaneous
tissues
Diseases of the musculoskeletal system and connective tissue Congenital anomalies
Certain conditions originating in the perinatal period Symptoms, signs, and ill-defined conditions
Injury and poisoning
Axis IV: Psychosocial and Environmental Problems
Problems with primary support group
Problems related to the social environment
Educational problems
Occupational problems
Housing problems
Economic problems
Problems with access to health care services
Problems related to interaction with crime or the legal system Other psychosocial and environmental problems
           
400 categories. In many instances, the patient has one or more disorders on both Axes I and II. For example, a patient may have major depressive disorder noted on Axis I and borderline and narcissistic personality disorders on Axis II. In general, multiple diagnoses on each axis are encouraged. Axis II consists of per- sonality disorders and mental retardation. The habitual use of a particular defense mechanism can be indicated on Axis II.
Axis III lists any physical disorder or general medical condi- tion that is present in addition to the mental disorder. The iden- tified physical condition may be causative (e.g., hepatic failure causing delirium), interactive (e.g., gastritis secondary to alco- hol dependence), and effect (e.g., dementia and human immun- odeficiency virus [HIV]-related pneumonia), or unrelated to the mental disorder. When a medical condition is causally related to a mental disorder, a mental disorder due to a general medical condition is listed in Axis I and the general medical condition is listed on both Axis I and III.
9.10. The answer is D (all)
Many patients have more than one psychiatric disorder. When more than one psychiatric disorder is present in a patient, the
Table 9.3
Global Assessment of Functioning (GAF) Scale
clinician denotes one as the principal diagnosis. Several meth- ods have been used by researchers in identifying principal dis- orders in patients with multiple psychiatric disorders. Some of the factors used to designate a principal disorder include age of onset (i.e., the principal diagnosis being the one that came first), relative degree of impairment (i.e., the principal diagnosis being the one responsible for the greatest degree of psychiatric impairment), and reason for seeking treatment (i.e., the principal diagnosis being the one that is chiefly responsible for the seek- ing of treatment). In the DSM-IV-TR, the determination of the principal diagnosis is based on the reason for clinical service.
Answers 9.11–9.15 9.11. The answer is A 9.12. The answer is E 9.13. The answer is C 9.14. The answer is B
9.
Classification in Psychiatry and Psychiatric Rating Scales 77
 
Consider psychological, social, and occupational functioning on a hypothetical continuum of mental health–illness. Do not include impairment in functioning due to physical (or environmental) limitations.
Code
100
91 90
81 80
71 70
61 60
51 50
41
(Note: Use intermediate codes when appropriate, e.g., 45, 68, 72.)
Superior functioning in a wide range of activities, life’s 40 problems never seem to get out of hand, is sought out by
others because of his or her many positive qualities. No
symptoms.
Absent or minimal symptoms (e.g., mild anxiety before an
exam), good functioning in all areas, interested and
involved in a wide range of activities, socially effective,
generally satisfied with life, no more than everyday
problems or concerns (e.g., an occasional argument with
family members). 30
Some impairment in reality testing or communication (e.g., speech is at times illogical, obscure, or irrelevant) OR major impairment in several areas, such as work or school, family relations, judgment, thinking, or mood (e.g., depressed man avoids friends, neglects family, and is unable to work; child frequently beats up younger children, is defiant at home, and is failing
   
If symptoms are present, they are transient and expectable reactions to psychosocial stressors (e.g., difficulty concentrating after family argument): no more than slight impairment in social, occupational, or school functioning (e.g., temporarily falling behind in schoolwork).
Some mild symptoms (e.g., depressed mood and mild insomnia) OR some difficulty in social, occupational, or school functioning (e.g., occasional truancy, or theft within the household), but generally functioning pretty well, has some meaningful interpersonal relationships.
Moderate symptoms (e.g., flat affect and circumstantial speech, occasional panic attacks) OR moderate difficulty in social, occupational, or school functioning (e.g., few friends, conflicts with peers or coworkers).
Serious symptoms (e.g., suicidal ideation, severe obsessional rituals, frequent shoplifting) OR any serious impairment in social, occupational, or school functioning (e.g., no friends, unable to keep a job).
31 at school).
Behavior is considerably influenced by delusions or hallucinations OR serious impairment in communication or judgment (e.g., sometimes incoherent, acts grossly inappropriately, suicidal preoccupation) OR inability to function in almost all areas (e.g., stays in bed all day; no job, home,

21 or friends).
20 Some danger of hurting self or others (e.g., suicide attempts without clear expectation of death, frequently violent, manic excitement) OR occasionally fails to maintain minimal personal hygiene (e.g., smears feces) OR gross impairment in communication (e.g., largely incoherent or
11 mute).
10 Persistent danger of severely hurting self or others (e.g., recurrent violence) OR persistent inability to maintain minimal personal hygiene OR serious
1 suicidal act with clear expectation of death. 0 Inadequate information.
     
The GAF Scale is a revision of the GAS (Endicott J, Spitzer RL, Fleiss JL, Cohen I. The Global Assessment Scale: a procedure for measuring overall severity of psychiatric disturbance. Arch Gen Psychiatry. 1976;33:766) and CGAS (Shaffer D, Gould MS, Brasio J, et al. Children’s Global Assessment Scale (CGAS). Arch Gen Psychiatry. 1983;40:1228). They are revisions of the Global Scale of the Health-Sickness Rating Scale (Luborsky I. Clinicians’ judgments of mental health. Arch Gen Psychiatry. 1962;7:407).
(From American Psychiatric Association. Diagnostic and Statistical Manual of Mental Disorders, 4th ed. Text rev. Washington, DC: American Psychiatric Association; copyright 2000, with permission.)
78 9. Classification in Psychiatry and Psychiatric Rating Scales
9.15. The answer is D
The DSM-IV-TR is a multiaxial system that consists of five axes. Each axis covers a different aspect of functioning (Ta- ble 9.2). Each axis should be covered for each diagnosis. Axis I consists of clinical disorders and conditions that may be a fo- cus of clinical attention. Examples include mood disorders, anx- iety disorders, and impulse disorders not otherwise specified (i.e., kleptomania). Axis II consists of personality disorders (i.e., antisocial personality disorder) and mental retardation. The habitual use of a particular defense mechanism can be indicated on Axis II.
Axis III lists any physical disorder or general medical con- dition that is present in addition to the mental disorder. The physical condition may be causative (e.g., kidney failure causing delirium), the result of a mental disorder (e.g., alcoholic gastri- tis secondary to alcohol dependence), or unrelated to the mental disorder. Axis IV is used to code psychosocial and environmen- tal problems that contribute significantly to the development or exacerbation of the current disorder. Examples include divorce, threat of job loss, and inadequate health insurance. Axis V is a global assessment in which the clinician evaluates the highest level of functioning by the patient in the past year. The Global Assessment of Functioning (GAF) Scale (see Table 9.3 on p. 77) is a 100-point scale, with 100 representing the highest level of functioning in all areas.
PSYCHIATRIC RATING SCALES QUESTIONS
Directions
Each of the questions or incomplete statements below is followed by five suggested responses or completions. Select the one that is best in each case.
. 9.16. A Likert scale is
A. a formal interview that specifies the exact wording of questions to be asked.
B. a formal interview that provides only some specific wording.
C. an ordinal scale of grading with 3 to 7 points that mea- sures severity, intensity, frequency, or other attributes.
D. a construct that describes the presence or absence of a
given attribute.
E. a construct that provides a quantitative assessment
along a continuum of intensity, frequency, or severity.
. 9.17. Which of the following scales is not used to rate a mood disorder?
A. Raskin Depression Rating Scale B.Montgomery-A ̊sbergScale
C. Hamilton Rating Scale
D. Defensive Functioning Scale
E. Beck Depression Inventory
. 9.18. The global assessment of function
A. is a composite of social, occupational, and psycholog- ical functioning
B. is assessed by a 50-point global assessment of func- tioning scale
C. included impairment attributable to physical limita- tions
D. is recorded on Axis IV of a multiaxial evaluation
E. bears no relation to prognosis
. 9.19. The Hamilton Anxiety Rating Scale
A. is a 10-item scale
B. includes an item on mood C. addresses suicidality
D. is exclusively history based E. excludes somatic symptoms
. 9.20. The Social and Occupational Functioning Assessment Scale (SOFAS)
A. is included on Axis III
B. may not be used to rate functioning of a past period
C. may not be used to rate functioning at the time of the
evaluation
D. is scored independently from the person’s psycholog-
ical symptoms
E. does not include impairment in functioning that is
caused by a general medical condition
. 9.21. Which of the following is true of the Defensive Function- ing Scale?
A. Apathetic withdrawal is measured on the “action level.”
B. Idealizationismeasuredinthe“majorimage-distorting level.”
C. Splitting is measured in the “minor image-distorting level.”
D. Sublimation is measured on the “mental inhibitions level.”
E. Suppression is measured on the “disavowal level.”
. 9.22. In the Scale for the Assessment of Negative Symptoms (SANS), which of the following is assessed?
A. Vocal inflection
B. Sexual activity
C. Impersistence at work D. Social inattentiveness E. All of the above
. 9.23. An example of a rating scale that measures severity and tracking change in specific symptoms is
A. the Structured Clinical Interview for the DSM-IV-TR (SCID)
B. the Short Form 36 (SF-36)
C. the Behavior and Symptom Identification Scale
(BASIS)
D. the Mini-Mental State Examination (MMSE)
E. the CAGE
Directions
Each group of questions below consists of lettered headings fol- lowed by a list of numbered phrases or statements. For each
numbered phrase or statement, select the one lettered heading that is most associated with it. Each lettered heading may be selected once, more than once, or not at all.
Questions 9.24–9.28
A. Social and Occupational Functioning Assessment Scale (SOFAS)
B. Positive and Negative Syndrome Scale (PANSS)
C. Global Assessment of Functioning (GAF)
D. Brief Psychiatric Rating Scale (BPRS)
E. Defensive Functioning Scale (DFS)
. 9.24. Coversthedefensemechanismsusedbythepatienttocope with stressors
. 9.25. Used in Axis V of DSM-IV-TR
. 9.26. Measures the negative and positive symptoms of
schizophrenia and other psychotic disorders
. 9.27. Used to track a patient’s progress in social and occupa-
tional areas
. 9.28. Used to measure the severity of psychiatric symptoma-
tology
Questions 9.29–9.31
A. Hamilton Rating Scale
B. Social and Occupational Functioning Assessment
Scale
C. Yale-Brown Scale
D. None of the above
. 9.29. Abnormal involuntary movements
. 9.30. Depression and anxiety
. 9.31. Obsessive-compulsive symptoms
Questions 9.32–9.36
are most often partially or fully anchored, assigning a meaning to each numeric level. The same anchors can apply to all of the items, or the instrument may provide specific anchors for each. Examples of rating scales scored on a Likert scale are the Brief Psychiatric Rating Scale (BPRS), which measures the severity of psychiatric symptomatology, and the Montgomery- A ̊sbergDepressionRatingScale(MADRS),whichisusedin clinical trials and is aimed at maximizing sensitivity to change in established depression.
Assessment of
B. CompositeInternationalDiagnosticInstrument(CIDI)
Scale
The Defensive Functioning Scale (DFS) covers the defensive mechanisms used by the patient to cope with stressors. All of the other scales listed are rating scales used for mood disorders.
9.18. The answer is A
The Global Assessment of Functioning (GAF) Scale is consid- ered a composite of three major areas: social functioning, occu- pational functioning, and psychological functioning. It is docu- mented on Axis V and is a 100-point scale based on a continuum of mental health and illness. It is specifically not designed to include impairment in functioning attributable to physical or environmental limitations. People with a high level of function- ing before an episode of illness generally have a better prognosis than those who had a low level of functioning.
9.19. The answer is B
The Hamilton Anxiety Rating Scale addresses depressed mood, focusing on loss of interest, lack of pleasure in hobbies, depres- sion, early wakening, and diurnal swing. It is a 14-item scale that includes assessment of sensory, cardiovascular, respiratory, gastrointestinal, genitourinary, and autonomic symptoms. It in- cludes an assessment of behavior at the time of interview, focus- ing on fidgeting, restlessness, tremor, a furrowed brow, a strained face, sighing, facial pallor, swallowing, belching, brisk tendon jerks, dilated pupils, and exophthalmos. There is no item dedi- cated to assessing suicidal ideation or intent.
A. Systematic Events (SAFTEE)
Treatment-Emergent
C. Montgomery-A ̊sberg (MADRS)
Depression
E. Abnormal Involuntary Movement Scale (AIMS)
D. Conners Rating Scales
. 9.32. A family of instruments designed to measure a range of childhood and adolescent psychopathology
. 9.33. Aimed at maximizing sensitivity to change in established depression
. 9.34. A systematic tool used to assess side effects in clinical trials
. 9.35. Used to measure dyskinetic symptoms in patients taking antipsychotic drugs
. 9.36. A fully structured diagnostic interview designed for lay administration
ANSWERS
9.16. The answer is C
A Likert scale is an ordinal scale of grading with 3 to 7 points that measure severity, intensity, frequency, or other attributes. They
Rating
9.
Classification in Psychiatry and Psychiatric Rating Scales 79
9.17.
Rating scales are available in a variety of formats. Some are simply checklists or guides to observation that help the clinician achieve a standardized rating. Others are self-administered questionnaires or tests. Still others are formal interview that may be fully struc- tured or partly structured. A fully structured formal interview specifies the exact wording of questions to be asked. A partly struc- tured formal interview provides only some specific wording along with suggestions for additional questions or probes. Whether fully structured or not, instruments may be written such that all ques- tions are always included, or they may have formal skip-out sec- tions to limit administration time. Psychiatric practitioners and in- vestigators assess a broad range of areas, referred to as constructs, to underscore the fact that they are not simple, direct observations of nature. Some constructs are viewed as categorical, or classify- ing, and others are seen as continuous or measuring. A categorical construct describes the presence or absence of a given attribute (e.g., competency to stand trial) or the category best suited to a given individual among a finite set of options (e.g., assigning a diagnosis). A continuous construct provides a quantitative as- sessment along a continuum of intensity, frequency, or severity. In addition to symptom severity and functional status, multidi- mensional personality traits, cognitive status, social support, and many other attributes are generally measured continuously.
The answer is D
80 9. Classification in Psychiatry and Psychiatric Rating Scales 9.20. The answer is D
The Social and Occupational Functioning Assessment Scale (SOFAS) is a new scale included in a DSM-IV-TR appendix. The scale differs from the Global Assessment of Functioning (GAF) Scale in that it focuses only on the person’s level of so- cial and occupational functioning. It is scored independently of the severity of the person’s psychological symptoms. And unlike the GAF scale, the SOFAS may include impairment in function- ing that is caused by a general medical condition. The SOFAS may be used to rate functioning at the time of the evaluation, or it may be used to rate functioning of a past period. The SOFAS is included on Axis V, not Axis III.
9.21. The answer is A
The “action level” is characterized by defensive functioning that deals with internal or external stressors by action or withdrawal, such as apathetic withdrawal. Suppression and sublimation are rated on the “high adaptive level,” the level of defensive func- tioning that result in the optimal adaptation in the handling of stressors. Idealization is rated on the “minor image-distorting” level, the level characterized by distortion in the image of self, body, or others that may be used to regulate self-esteem. Splitting is rated on the “major image-distorting level,” the level charac- terized by gross distortion or misattribution of the image of self or others.
9.22. The answer is E (all)
The Scale for the Assessment of Negative Symptoms (SANS) has five main categories: affective blunting, alogia, avolition-apathy, anhedonia-asociality, and attention. In the affective blunting category, vocal inflection, eye contact, facial expression, sponta- neous movements, expressive gestures, and affective responsiv- ity are measured. The avolition-apathy category measures im- persistence at work or school, grooming, and physical anergia. The anhedonia-asocialty category looks at sexual activity, recre- ational interests, ability to feel intimacy, and relationships with peers and friends. The attention category assesses social inatten- tiveness and inattentiveness during the mental status examina- tion. Another scale, used extensively for both positive and neg- ative symptoms is called the Positive and Negative Syndrome Scale (PANSS).
9.23. The answer is D
Most psychiatric rating scales in common use fall into one or more of the following categories: making a diagnosis, measur- ing severity and tracking change in specific symptoms, in general functioning, or in overall outcome and screening for conditions that may or may not be present. The Mini-Mental State Exami- nation (MMSE) is an example of a rating scale that measures the severity and tracking change in specific symptoms. It is used to provide a bedside assessment of a broad array of cognitive func- tion, including orientation, attention, memory, construction, and language. The MMSE is used commonly to screen for dementia and in following the progression of dementia over time in the clinic or in clinical trials. A patient with mild dementia tends to score from 20 to 24, moderate from 11 to 19, and severe from 0 to 10.
The Structured Clinical Interview for the revised fourth edition of the Diagnostic and Statistical Manual of Mental Disorders (DSM-IV-TR) (SCID) is a rating scale used to make a diagnosis. It was developed to provide standard DSM-III-R Axis I diag- nosis (SCID-I) based on an efficient but thorough evaluation. It has since been updated for the DSM-IV-TR. SCID-II is a similar measure for Axis II disorders. The Short Form 36 (SF-36) is a rating scale used in general functioning. It is a general measure of health status from the patient’s point of view that would be inde- pendent to specific diseases. SF-36 is focused on the individual’s functional status as it relates to physical problems, pain, and emo- tional difficulties over 4 weeks. An acute version, focused on the past week, is also available; however, the 4-week version is most widely used. The Behavior and Symptom Identification Scale (BA- SIS) is a rating scale used in overall outcome. It was developed to provide a broad but brief overview of psychiatric symptoms and functional status from the patient’s point of view for use in assessing the outcome of psychiatric treatment. The CAGE is used to screen for significant alcohol problems in a variety of settings, which could then be followed up by clinical inquiry. CAGE is an acronym for the four questions that comprise the instrument: (1) Have you ever felt you should cut down on your drinking? (2) Have people annoyed you by criticizing your drinking? (3) Have you ever felt bad or guilty about your drinking? (4) Have you ever had a drink first thing in the morning to steady your nerves or to get rid of a hangover (eye-opener)? Each “yes” answer is a score of 1. A score of 1 or more warrant a follow-up, and a score of 2 or more strongly suggests significant alcohol problems.
Answers 9.24–9.28 9.24. The answer is E 9.25. The answer is C 9.26. The answer is B 9.27. The answer is A
9.28. The answer is D
The Social and Occupational Functioning Assessment Scale (SOFAS) is used to track a patient’s progress in social and oc- cupational areas. It is independent of the psychiatric diagnosis and the severity of the patient’s psychological symptoms. It is clinician rated on a 100-point scale based on all available infor- mation, with clear descriptions of each 10-point interval. It is not widely used. The Positive and Negative Syndrome Scale (PANSS) measures negative and positive symptoms of schizophrenia and other psychotic disorders. It includes 30 items on three subscales: seven items covering positive symptoms (e.g., hallucinations and delusions), seven covering negative symptoms (e.g., blunt affect), and 16 covering general psychopathology (e.g., guilt or uncooperativeness). Each item is scored on a seven-point, item- specific Likert scale ranging from 1 to 7. The PANSS has become the standard tool for assessing clinical outcome in treatment studies of schizophrenia and other psychotic disorders. The Global Assessment of Functioning (GAF) is use in Axis V of the DSM-IV-TR to report a clinician’s judgment of a patient’s overall level of functioning. The information is used to decide on a treatment plan and later to measure the plan’s effect. Its ratings
9.
are often required for billing purposes. Similar to the SOFAS, it is clinician-rated on a 100-point scale. The Brief Psychiatric Rating Scale (BPRS) is a short scale used to measure the severity of psychiatric symptomatology. It is most useful for patients with fairly significant impairment. Its 18 items are rated on a 7-point item-specific Likert scale from 0 to 6, with the total score ranging from 0 to 108. Reliability is good to excellent when raters are experienced. The Defensive Functioning Scale (DFS) covers the defense mechanisms used by the patient to cope with stressors. Humor, suppression, anticipation, and sublimation are among the healthiest defense mechanisms. Denial, acting out, projection, and projective identification are some of the most pathological.
Answers 9.29–9.31 9.29. The answer is D 9.30. The answer is A
9.31. The answer is C
Psychiatric rating scales can be specific or comprehensive and can measure both internally experienced variables (e.g., mood) and externally observable variables (e.g., behavior). Specific scales measure discrete thoughts, moods, or behaviors, such as obsessive thoughts and temper tantrums; comprehensive scales measure broad abstractions, such as depression and anxiety. Well-known rating scales include the Hamilton Rating Scale for depression and anxiety and the Yale-Brown Scale for obsessive- compulsive symptoms.
Social adjustments (e.g., occupational success and quality of relationships with the Social and Occupational Functioning Assessment Scale [SOFAS]) and psychoanalytic concepts (e.g., ego strength and defense mechanisms) are also measured by some rating scales, although the reliability and the validity of such scales are lower.
Answers 9.32–9.36 9.32. The answer is D
Classification in Psychiatry and Psychiatric Rating Scales 81 9.33. The answer is C
9.34. The answer is A
9.35. The answer is E
9.36. The answer is B
The Systematic Assessment of Treatment-Emergent Events (SAF- TEE) is a systematic tool used to assess side effects in clinical trials. It has two versions, a general inquiry form (SAFTEE- GI) and a much more detail specific inquiry form (SAFTEE-SI) that incorporates a formal review of symptoms. The SAFTEE- GI is rated based on three open-ended questions regarding new symptoms experienced in the past week. To avoid bias, the ques- tions are focused on novelty of the symptoms rather than on any relationship to the drug. The Composite International Diagnos- tic Instrument (CIDI) is a fully structured diagnostic interview designed for lay administration. It was developed from the Diag- nostic Interview Schedule (DIS) for international use and covers both ICD and DSM criteria of 11 diagnostic modules; it does not cover antisocial personality or childhood onset disorders. The Montgomery-A ̊sberg Depression Rating Scale (MADRS) is aimed at maximizing sensitivity to change in established depres- sion. It can be administered by a psychiatrist, other physician, or psychiatric nurse, and a structured interview guide and self- report version are also available. It is frequently used in clinical trials of antidepressants and may also have a role in assessing the impact of treatment in clinical practice. The Conners Rating Scales are a family of instruments designed to measure a range of childhood and adolescent psychopathology. They are most com- monly used in the assessment of attention-deficit/hyperactivity disorder (ADHD). The Abnormal Involuntary Movement Scale (AIMS) is used to measure dyskinetic symptoms in patients taking antipsychotic drugs. It has 12 items, each of which is rated on an item-specific 5-point severity scale ranging from 0 to 4. Total scores are not generally reported. Instead, changes in global severity and individual areas can be monitored over time.

10
Delirium, Dementia, and Amnestic and Other Cognitive Disorders and Mental Disorders Due to a General Medical Condition

Cognition includes language, memory, judgment, orienta- tion, conducting interpersonal relationships, performing actions (praxis), and problem solving. Cognitive disorders reflect dis- ruption in one or more of these domains, and are frequently complicated by behavioral symptoms. These disorders include delirium, dementia, and amnestic disorders. Although other psy- chiatric disorders can exhibit some cognitive impairment as a symptom, with the aforementioned disorders, cognitive impair- ment is the cardinal symptom.
Delirium is marked by short-term confusion and changes in cognition. It is a life-threatening yet potentially reversible disor- der of the central nervous system that often involves perceptual disturbances, abnormal psychomotor activity, sleep cycle im- pairment, and a waxing and waning level of consciousness. It is not a disease but rather a syndrome and has many causes, all of
which result in similar patterns of signs and symptoms relating to the patient’s level of consciousness and cognitive impairment. Dementia is marked by severe impairment of memory, judg- ment, orientation, and cognition. Unlike delirium, deficits in de- mentia occur in the context of a clear sensorium. The critical clinical points of dementia are the identification of the syndrome
and the clinical workup of its cause.
Amnestic disorder is marked by memory impairment and for-
getfulness. Memory impairments are associated with significant deficits in social or occupational functioning. Amnestic disor- ders are causally related to general medical conditions, such as head trauma. This characteristic distinguishes them from the dis- sociative disorders involving memory impairments.
The questions and answers below can test knowledge of the subject.
  
82
HELPFUL HINTS
Students should be able to define the signs, symptoms, and syndromes listed below.
Addison’s disease
AIP
ALS
amnestic disorders anxiety disorder due to a
general medical
condition auditory, olfactory,
and visual
hallucinations beclouded dementia beriberi
black patch catastrophic reaction cognitive disorders confabulation cretinism
Creutzfeldt-Jakob disease
Cushing’s syndrome delirium
delusional disorder dementia
dementia of the Alzheimer’s
type
diabetic ketoacidosis Down syndrome dysarthria
epilepsy
general paresis granulovacuolar
degeneration Huntington’s disease
hypnagogic and hypnopompic hallucinations
hypoglycemic, hepatic, and uremic
encephalopathy interictal
intoxication and withdrawal
intracranial neoplasms Korsakoff’s syndrome kuru
Lilliputian
hallucinations memory
mild cognitive impairment
mood disorder due to a general medical condition
multiple sclerosis myxedema neurofibrillary tangles normal aging normal-pressure
hydrocephalus parkinsonism
partial versus generalized seizures
pellagra
pernicious anemia personality change due
to a general medical condition
▲ ▲▲▲▲▲ ▲▲ ▲▲▲
▲ ▲ ▲▲▲▲▲▲▲▲
▲ ▲▲▲▲▲ ▲▲▲▲▲▲ ▲
▲▲▲▲▲ ▲ ▲▲▲▲▲▲▲
10.
Delirium, Dementia, and Amnestic and Other Cognitive Disorders and Mental Disorders 83
 
Pick’s disease prion disease pseudobulbar
palsy pseudodementia
QUESTIONS
Directions
retrograde versus anterograde amnesia
senile plaques short-term versus long-
term memory loss
sundowner syndrome systemic lupus
erythematosus tactile or haptic
transient global amnesia transient ischemic attack vascular dementia vertebrobasilar disease
Each of the questions or incomplete statements below is followed by five suggested responses or completions. Select the one that is best in each case.
. 10.1. Which statement below about the interrelationship between delirium and dementia is true?
A. Delirium is a risk factor for the development of dementia.
B. Fully two-thirds of cases of dementia occur in patients with delirium.
C. Thevulnerabilityofthebraininpatientswithdemen- tia may predispose the patient to delirium.
D. Dementia contributes to a loss of independence among patients with delirium.
E. Dementia can alter the course of an underlying delirium.
. 10.2. Alzheimer’s dementia is
A. associated with hypoactive levels of acetylcholine
B. associated with pathognomonic neurofibrillary tan-
gles
C. more common in men
D. linked to chromosome 7
E. a clinical diagnosis
. 10.3. Creutzfeldt-Jakob disease is characterized by
A. rapid deterioration
B. myoclonus
C. diffuse, symmetric, rhythmic slow waves and sharp
spikes in EEG
D. postmortem definitive diagnosis
E. all of the above
. 10.4. The core features of dementia with Lewy bodies include
A. disturbance of consciousness
B. recurrent detailed visual hallucinations
C. cognitive decline caused by cerebrovascular disease D. progressive language dysfunction
E. gradual changes in personality
. 10.5. Transient global amnesia
A. is more common in women
B. is more common in young people
C. is associated with loss of self-identity
10.6.
D. has a characteristically abnormal EEG pattern E. has been linked to vascular instability
Which of the following is a true statement about Parkin- son’s disease?
A. It is the result of the degeneration of the substantia nigra, globus pallidus, putamen, and caudate.
B. It cannot be distinguished from parkinsonian syn- dromes that arise from a variety of causes.
C. Dementia is more common in early-onset disease.
D. It is a prototype of a cortical degenerative disease.
E. The only cells affected are those containing
dopamine.
Clinical characteristics of vascular dementia
A. are the same regardless of the area of infarction
B. are the same regardless of the number of infarc-
tions
C. are the same regardless of the type of vasculature
involved
D. are the same regardless of whether or not deficits ac-
cumulate or resolve quickly after small strokes
E. none of the above
Huntington’s disease
A. is linked to the long arm of chromosome 4
B. is associated with “boxcar” ventricles on brain scan-
ning
C. is not usually associated with emotional symptoms
D. shows striatal hypermetabolism on positron emission
tomography (PET)
E. affects men only
The criteria for mild cognitive impairment includes all of the following except
A. memory complaints
B. objective memory impairment
C. preserved general cognitive function D. intact daily living activities
E. dementia
Which of the following is a prion disease?
A. Creutzfeldt-Jakob disease
B. Variant Creutzfeldt-Jakob disease C. Kuru
D. Fatal familial insomnia
E. All of the above
10.7.
10.8.
10.9.
10.10.
hallucinations

▲▲▲▲
▲▲▲
▲▲▲
▲▲▲ ▲
84 10.
Delirium, Dementia, and Amnestic and Other Cognitive Disorders and Mental Disorders

FIGURE 10.1
Pick’s disease gross pathology. This demonstrates the marked frontal and temporal atrophy seen in frontotemporal dementias, such as Pick’s disease. (Courtesy of Dashyant Purohit, MD, Associate Professor, Department of Neuropathology, Mount Sinai School of Medicine, New York, NY.)
. 10.11. ThebraindepictedinFigure10.1showsfrontalandtem- poral atrophy associated with frontotemporal dementia. Which of the following statements is true?
A. Pick bodies are found in all the frontotemporal de- mentias.
B. Pick’s disease has its own separate diagnostic criteria in the DSM-IV-TR.
C. Frontotemporaldementiaismorelikelytoaffectolder populations.
D. Progressive nonfluent aphasia is a frontotemporal dementia.
E. Genetic linkage to chromosome 9 has been found in frontotemporal dementia.
. 10.12. True statements about Alzheimer’s disease include all of the following except
A. The age at onset is earlier in patients with a family history of the disease.
B. Brain imaging studies are used to exclude other iden- tifiable causes.
C. The early-onset type may have a more rapidly pro- gressive course.
D. There is clear phenomenological separation between early-onset and late-onset cases.
E. No features of the physical examination or laboratory evaluation are pathognomonic.
. 10.13. Factors that predispose to delirium include all of the following except
A. vision impairment
B. use of bladder catheterization C. smoking history
D. age older than 60 years
E. abnormal glucose level
10.14. Whichofthefollowingstatementsaboutdeliriumistrue?
A. It is independent of age.
B. It has no bearing on overall patient prognosis.
C. Male gender is an independent risk factor.
D. Postcardiotomy patients rarely develop delirium.
E. Few hospitalized medically ill patients develop
delirium.
10.15. The most common cause of delirium within 3 days post- operatively in a 40-year-old man with a history of alcohol dependence is
A. delirium tremens B. infection
C. pain medication D. postoperative pain E. stress of surgery
10.16. Whichclinicalfeaturesmaybeassociatedwithdelirium?
A. Disorganized thought processes B. Hallucinations
C. Illusions
D. Mood alterations
E. All of the above
10.17. Which of the following drugs is best used to treat acute delirium?
A. Amobarbital (Amytal)
B. Chlorpromazine (Thorazine)
C. Diazepam (Valium)
D. Haloperidol (Haldol)
E. Physostigmine salicylate (Antilirium)
10.18. In delirium,
A. epinephrine is hypothesized to be the major neuro- transmitter involved.
B. the major pathway implicated is the dorsal tegmental pathway.
C. the electroencephalogram (EEG) usually shows dif- fuse background quickening.
D. there is hyperactivity in the nucleus accumbens.
E. the level of consciousness is preserved.
10.19. Mr. E is 68 years old and married with two children. His wife reports changes in his memory and behavior over the past 9 years. She reports that he frequently forgets his keys, he goes into the house to get something and then forgets what he wants, and he has changed from an out- going pleasant person to one who avoids conversation. She says that he seems hostile at times for no apparent reason. Mr. E is in good general health, taking no medi- cations, and his alcohol consumption is limited to two to three beers a day.
What may you observe on examination of Mr. E?
A. A grasp reflex
B. Paranoid delusions
10. Delirium, Dementia, and Amnestic and Other Cognitive Disorders and Mental Disorders 85
C. Pathologicalcrying D. Poor hygiene
E. All of the above
. 10.20. In the case described above, what is the most likely di- agnosis other than dementia for Mr. E?
A. Chronic paranoid schizophrenia B. Delirium
C. Factitious disorder
D. Major depression
E. Normal aging
. 10.21. Whichofthefollowingwouldbeconsistentwiththepro- gression of Mr. E’s illness?
A. Frequent nighttime pacing
B. Increased agitation
C. Physical threats toward his wife D. Urinary incontinence
E. All of the above
. 10.22. The EEG shown in Figure 10.2 is an example of
A. partial seizure
B. grand mal epilepsy
C. petit mal epilepsy or absence seizure D. psychomotor epilepsy
E. none of the above
. 10.23. True statements about the epidemiology of dementia include all of the following except
A. The risk for vascular dementia is six times greater than that for Alzheimer’s disease among people older than 75 years.
B. Thereappearstobeahigherrateofvasculardementia in men, and a higher rate of Alzheimer’s disease in women.
C. In geriatric psychiatric populations, Alzheimer’s dis- ease is much more common than vascular dementia. D. The estimated prevalence in a population older than age 65 years is consistently reported to be about
5 percent.
E. Dementia of the Alzheimer’s type is the most com-
mon dementing disorder in North America, Scandi- navia, and Europe.
10.24. Delirium
A. has an insidious onset.
B. rarely has associated neurological symptoms.
C. generallyhasanunderlyingcauseresidinginthecen-
tral nervous system.
D. may be successfully treated with lithium.
E. generally causes a diffuse slowing of brain activity.
10.25. Of the following cognitive functions, the one most likely to be difficult to evaluate and interpret on formal testing is
A. abstraction
B. calculations
C. memory
D. reading and writing
E. visuospatial and constructional ability
10.26. Amnestic disorders
A. are invariably persistent, lasting at least 1 month.
B. are defined by a better memory for remote events than
recent ones.

FIGURE 10.2
Electroencephalogram (EEG).
86
10. Delirium, Dementia, and Amnestic and Other Cognitive Disorders and Mental Disorders
C. do not typically impair the ability to immediately repeat a sequential string of information (e.g., digit span).
D. typically have a gradual onset.
E. none of the above
. 10.27. Amnestic disorders
A. are secondary syndromes caused by primary etiolo- gies.
B. are most often caused by nutritional deficiencies related to chronic alcohol dependence.
C. may be diagnosed in the context of delirium.
D. may be diagnosed in the context of dementia.
E. none of the above
. 10.28. True statements about vascular causes of dementia include
A. Approximately15percentofcerebrovasculardisease is caused by cerebral hemorrhage related to hyperten- sion.
B. Itisbelievedthattissuedamageininfarctionunderlies vascular dementia.
C. The most common cause of cerebral infarction is thromboembolism from a large vessel plaque.
D. It comprises the second most common cause of de- mentia.
E. All of the above
. 10.29. Frontal lobe degeneration is associated with
A. apathy
B. disinhibition
C. lack of insight
D. social misconduct E. all of the above
. 10.30. Risk factors for the development of delirium include
A. increased severity of physical illness B. older age
C. preexisting dementia
D. use of anticholinergics
E. all of the above
. 10.31. Bovinespongiformencephalopathyisassociatedwithall
of the following except
A. amyloid plaques
B. astrocyte proliferation in the cerebral cortex C. “bulls eye” rash on the thigh
D. neuronal loss
E. spongiform vacuolization
. 10.32. Whichofthefollowingstatementsaboutpsychoticsymp- toms in dementia is false?
A. Psychotic symptoms usually occur in the early stages of illness.
B. Psychotic symptoms often co-occur with behavioral disturbances.
C. A common early psychotic symptom is paranoia. D. Patients may see deceased relatives.
E. Psychotic symptoms can lead to aggression.
10.33. A 77-year-old widower presents for a routine health check-up accompanied by his daughters. His medical his- tory is significant for diabetes, hypertension, and periph- eral vascular disease. Eight months ago, he had a stroke and still experiences left leg weakness. His daughter re- ports that her father sometimes does not remember to call her back and forgets to take his diabetes medications at least once a week. Once in a while, the patient is noted to “word hunt” during conversations. The patient insists he is “fine” and that he drives places without getting lost, does his own grocery shopping, and is never in a bad mood. The patient visits his best friend once a month. He admits that occasionally he has difficulty falling asleep. Which of the following is the most likely diagnosis?
A. Depression
B. Alzheimer’s dementia
C. Frontotemporal dementia D. Vascular dementia
E. Normal aging changes
Directions
Each set of lettered headings below is followed by a list of num- bered phrases. For each numbered phrase, select
A. if the item is associated with A only.
B. if the item is associated with B only.
C. if the item is associated with both A and B. D. if the item is associated with neither A nor B.
Questions 10.34–10.39
A. Delirium B. Dementia
. 10.34. Catastrophic reaction
. 10.35. Decreased acetylcholine activity
. 10.36. Hallucinations
. 10.37. High mortality rate
. 10.38. Insight present
. 10.39. Sundowning
Questions 10.40–10.45
A. Cortical dementia
B. Subcortical dementia
10.40. Huntington’s chorea
10.41. Alzheimer’s disease
10.42. Early decline in calculation, naming, and copying skills 10.43. Language is relatively spared
10.44. Presenting symptoms more likely to be a personality
change or mood disturbance
10.45. Presenting symptoms more often reflect cognitive im-
pairment
10. Delirium, Dementia, and Amnestic and Other Cognitive Disorders and Mental Disorders 87
Directions
Each group of questions below consists of lettered headings fol- lowed by a list of numbered phrases or statements. For each numbered phrase or statement, select the one lettered heading that is most associated with it. Each lettered heading may be selected once, more than once, or not at all.
Questions 10.46–10.50
A. Dementia caused by Huntington’s disease B. Dementia caused by HIV disease
C. Dementia caused by Parkinson’s disease D. Dementia caused by head trauma
E. Dementia caused by Creutzfeldt-Jakob disease
. 10.46. This is the third most common dementia in individuals younger than age 50 years.
. 10.47. The incidence has been reduced because of highly active antiretroviral therapy.
. 10.48. The disease is transmitted by a mutation on chromo- some 4.
. 10.49. This is a subcortical dementia with slowed processing speed.
. 10.50. The only treatment is supportive care.
Questions 10.51–10.55
A. Creutzfeldt-Jakob disease B. Huntington’s disease
C. Multiple sclerosis
D. Neurosyphilis
E. Normal-pressure hydrocephalus
. 10.51. Death occurring 15 to 20 years after the onset of the disease, with suicide being common
. 10.52. Death occurs within 2 years of the diagnosis
. 10.53. Manic syndrome with neurological signs in up to 20 per-
cent of cases
. 10.54. Treatment of choice is a shunt
. 10.55. More prevalent in cold and temperate climates than in
the tropics and subtropics
ANSWERS
10.1. The answer is C
Delirium and dementia are highly interrelated. Dementia is the leading risk factor for delirium (not vice versa), and fully two- thirds of cases of delirium occur in patients with dementia (not vice versa). Thus, the underlying vulnerability of the brain in patients with dementia may predispose them to the development of delirium as a result of insults related to acute medical illnesses, medications, or environmental perturbations.
Delirium is defined by the acute onset of fluctuating cog- nitive impairment and a disturbance of consciousness. The hallmark symptom is an impairment of consciousness, usually occurring in association with global impairments of cognitive functions. Dementia is defined as progressive impairment of
cognitive functions occurring in clear consciousness (i.e., in the absence of delirium). Global impairment of intellect is the essential feature, manifested as difficulty with memory, atten- tion, thinking, and comprehension. Although it is not likely that delirium itself causes the pathological change in dementia, there is no question that delirium (not dementia) contributes to worsening functional status, loss of independence, and poorer outcome among patients with dementia. Delirium can alter the course of an underlying dementia, with dramatic worsen- ing of the trajectory of cognitive decline, resulting in more rapid progression of functional losses and worse long-term outcomes.
10.2. The answer is A
The neurotransmitters that are most often implicated in the patho- physiological condition of Alzheimer’s disease are acetylcholine and norepinephrine, both of which are hypoactive in the disease. The disease has shown linkage to chromosomes 1, 14, and 21, not chromosome 7. Although it is commonly diagnosed in the clini- cal setting after other causes of dementia have been excluded, the final diagnosis of Alzheimer’s disease requires a brain biopsy. The classic gross neuroanatomical observation of the brain from patients with this disease is diffuse atrophy. The classic micro- scopic findings are senile plaques, neuronal loss, synaptic loss, and granulovascular degeneration of the neurons. Neurofibril- lary tangles are not unique to Alzheimer’s disease but also occur in Down syndrome, dementia pugilistica, Parkinson-dementia complex of Guam, and the brains of normal people as they age. The risk factors for developing Alzheimer’s dementia include be- ing female, having a first-degree relative with the disorder, and having a history of head injury.
10.3. The answer is E (all)
Creutzfeldt-Jakob disease is an infection that causes a rapidly progressive cortical-pattern dementia. The infectious agent, a prion, is a subviral replicative protein that is now known to cause a variety of so-called spongiform diseases in animals and humans. The age at onset of Creutzfeldt-Jakob disease is usu- ally in the sixth or seventh decade of life, although onset can occur at any age. The incidence is one in 1,000,000. The clin- ical symptoms vary with progression of the illness and depend on the regions of the brain that become involved. Patients may present initially with nonspecific symptoms, including lethargy, depression, and fatigue. Within weeks, however, more fulmi- nant symptoms develop, including progressive cortical-pattern dementia, myoclonus, and pyramidal and extrapyramidal signs. Although blood, cerebrospinal fluid, and imaging studies are un- remarkable, the electroencephalogram (EEG) may demonstrate a characteristic pattern of diffuse, symmetric, rhythmic slow waves and sharp spikes. A presentation with rapid deterioration, myoclonus, and the characteristic EEG pattern should raise sus- picion of Creutzfeldt-Jakob disease. The definitive diagnosis is made by postmortem microscopic examination, which demon- strates spongiform neural degeneration and gliosis throughout the cortical and subcortical gray matter; white matter tracts are usually spared. Prion disease can incubate for decades be- fore the emergence of clinical symptoms and subsequent rapid progression. Reported routes of transmission include invasive
88 10. Delirium, Dementia, and Amnestic and Other Cognitive Disorders and Mental Disorders
  
ABC
FIGURE 10.3
Photomicrographs of Lewy body pathology. (A) Abnormal accumulation of α-synuclein aggregates demonstrated by immunocytochemistry in the amygdala of a subject with dementia. Lewy bodies appear as dense intracellular inclusions (arrows), but staining of neuronal processes can be seen throughout the neuropil (arrowheads). In individuals in whom Lewy body pathology occurs concurrently with Alzheimer’s disease, the amygdala is often the only region affected. (B) Classic appearance of a Lewy body (arrow) in a large pigmented neuron of the substantia nigra. (C) Lewy body pathology in the neocortex. Both Lewy bodies (arrows) and substantial labeling of neuronal processes in the neuropil (arrowheads) are evident. (Magnification for [A] and [B] 200×; for [C], 400×. All images provided courtesy of Dr. Ronald L. Hamilton, Department of Pathology, Division of Neuropathology, University of Pittsburgh School of Medicine.)
body contacts, such as direct tissue transplantation (e.g., corneal transplants) or hormonal extracts (e.g., human growth hormone before synthetic supplies were developed). Familial patterns have also been reported, which suggests that there may be ge- netic susceptibility to infection or vertical transmission of the disease agent. No antiviral agents have been shown to be effec- tive in retarding or slowing disease progress, although amanta- dine (Symmetrel) has been reported occasionally to have had some success. Death usually ensues within 6 months to 2 years of onset.
10.4. The answer is B
The core features of dementia with Lewy bodies include (1) fluctuating cognition with pronounced variations in attention and alertness; (2) recurrent visual hallucinations, which are typically well formed and detailed; and (3) spontaneous motor features
Table 10.1
Pathological Features Associated with Dementia with Lewy Bodies
Essential for diagnosis Lewy bodies
Associated but not essential
Lewy-related neurites
Plaques (all morphological types)
Neurofibrillary tangles
Regional neuronal loss, especially brainstem (substantia nigra
and locus ceruleus) and nucleus basalis of Meynert Microvacuolation (spongiform change) and synapse loss Neurochemical abnormalities and neurotransmitter deficits
Reprinted with permission from McKeith IG, Galasko D, Kosaka K, et al. For the Consortium on Dementia with Lewy Bodies. Neurology. 1996;47: 1113.
of parkinsonism. At least one core feature must be present for clinical diagnosis, although the presence of two core features leads to a higher sensitivity. Dementia with Lewy bodies is a neurodegenerative dementia characterized by progressive cog- nitive decline of sufficient magnitude to interfere with normal social or occupational function. It is now considered to be the second most common cause of dementia after Alzheimer’s dis- ease. Figure 10.3 shows cortical Lewy bodies. Table 10.1 lists the pathological features of dementia with Lewy bodies, and Table 10.2 includes recently developed consensus guidelines for clinical diagnosis.
The core symptoms of delirium include a disturbance of con- sciousness that is accompanied by a change in cognition that de- velops rapidly, usually over hours or days, and tends to fluctuate during the course of the day.
Vascular dementia refers to cognitive decline caused by is- chemic, hemorrhagic, or oligemic injury to the brain as a conse- quence of cerebrovascular and cardiovascular disease. The di- agnosis is made if dementia is associated with focal neurological signs and symptoms and neuroimaging evidence of cerebrovas- cular disease is present. Frontotemporal dementia is a progres- sive change in personality and behavior, with variable degrees of language and other cognitive impairment. There are two core clinical patterns: (1) gradual changes in personality and behav- ior and (2) progressive language dysfunction. The hallmark of the behavioral presentation is typically a combination of disin- hibition, apathy, and limited insight.
10.5. The answer is E
Transient global amnesia is a syndrome characterized by the sudden onset of a profound anterograde amnesia and retrograde amnesia for the past weeks or months. There is an association with migraine in about 15 percent of cases, leading to etiologic speculation of vascular instability. It occurs in men more than
  
10. Delirium, Dementia, and Amnestic and Other Cognitive Disorders and Mental Disorders 89
Table 10.2
Consensus Criteria for the Clinical Diagnosis of Probable and Possible Dementia with Lewy Bodies
1. The central feature required for a diagnosis of dementia with Lewy bodies is progressive cognitive decline of sufficient magnitude to interfere with normal social or occupational function. Prominent or persistent memory impairment may not necessarily occur in the early stages but is usually evident with progression. Deficits on tests of attention and of frontal-subcortical skills and visuospatial ability may be especially prominent.
2. Two of the following core features are essential for a diagnosis of probable dementia with Lewy bodies, and one is essential for possible dementia with Lewy bodies:
a. Fluctuating cognition with profound variations in attention and alertness
b. Recurrent visual hallucinations that are typically well formed and detailed
c. Spontaneous motor features of parkinsonism
3. Features supportive of the diagnosis are:
a. Repeated falls
b. Syncope
c. Transient loss of consciousness d. Neuroleptic sensitivity
e. Systematized delusions
f. Hallucinations in other modalities
4. A diagnosis of dementia with Lewy bodies is less likely in
the presence of:
a. Stroke disease, evident as focal neurologic signs or on
brain imaging
b. Evidence on physical examination and investigation of
any physical illness or other brain disorder sufficient to account for the clinical picture
Reprinted with permission from McKeith IG, Galasko D, Kosaka K, et al. For the Consortium on Dementia with Lewy Bodies. Neurology. 1996; 47:1113.
women, and typically after age 50 years. The patient is gen- erally able to recall his or her own identity and that of rel- atives or close associates but is not able to retain his or her immediate context even when explained to the patient repeat- edly. There is no disturbance of consciousness and no detectable seizure activity on electroencephalogram obtained during the event.
10.6. The answer is A
Described by James Parkinson in 1817, Parkinson’s disease is
a prototype of a subcortical, not cortical, degenerative disease. It is idiopathic and must be distinguished from parkinsonian syndromes that arise from a variety of causes.
Parkinson’s disease is the result of the degeneration of subcor- tical structures, primarily the substantia nigra but also the globus pallidus, putamen, and caudate. Cells containing dopamine are predominantly affected, although serotonergic and other sys- tems are disrupted as well. Medication-induced parkinsonism presumably involves only a dysfunction of the basal ganglia structures without any obvious pathoanatomical abnormality. The typical age at the onset of Parkinson’s disease is between 50 and 60 years but may vary widely with the onset sometimes oc- curring 1 to 2 decades earlier. The clinical course is chronic and progressive with severe disability attained after approximately
10 years. A smaller proportion of patients have a more rapidly progressive disease, and a yet smaller group has a slowly progres- sive disorder in which deterioration plateaus or remains minimal for 2 to 3 decades.
In general, subcortical diseases are thought to impinge on the three Ms—movement, mentation, and mood. In Parkinson’s dis- ease, all three of these areas are affected, although not always uniformly. The movement abnormalities are characterized by the triad of tremor, rigidity, and bradykinesia. The tremor and rigid- ity may be unilateral or bilateral. Bradykinesia is manifested by slowness in the initiation and execution of movement. The typical presentation, with a masklike facies, minimal blink, and monotonic speech, is a concomitant of the rigidity and slowness of movement. Other prominent characteristics include postural changes such as chin-to-chest flexion and gait abnormalities. The gait is characteristically slow and shuffling, and the patient has difficulty turning (en bloc turning) and trouble initiating and stopping walking. Seborrhea, sialorrhea, excessive fatigue, and constipation are also common.
Mentation or cognition in Parkinson’s disease is an area of controversy. Most patients complain of slowed thinking, sometimes called bradyphrenia. In general, approximately 20 to 30 percent of patients with Parkinson’s disease are found to have dementia, with the likelihood greater in those with late-onset disease (after age 70 years). Neuropathologically, cases inter- mediate between Parkinson’s disease and Alzheimer’s disease exist, with Lewy bodies in the substantia nigra suggesting the former. There is no clear line of division as yet between a process resembling dementia of the Alzheimer’s type on which abnor- mal parkinsonian movements are superimposed and a clinical presentation of Parkinson’s disease in which the patient slowly develops a global progressive dementia.
10.7. The answer is E (none)
The clinical characteristics of a vascular dementia depend on the area of infarction. As such, there is a wide variabil- ity in the possible presenting features of a vascular dementia. Single infarctions may result in the discrete loss of one particu- lar function (e.g., language) without dementia per se. However, some strategically located infarctions can affect more than one domain of cognitive function and mimic the clinical picture of a global dementia. An example is the angular gyrus syndrome that can occur with large posterior lesions in the dominant hemi- sphere. It has been characterized as manifesting with alexia with agraphia, aphasia, constructional disturbances, and Gerstmann syndrome (acalculia, agraphia, right-left disorientation, and fin- ger agnosia). Although the findings are similar to those of de- mentia of the Alzheimer’s type, angular gyrus syndrome can be distinguished by its abrupt onset; the presence of focal neu- rological, electroencephalographic, and imaging abnormalities; and preservation of memory and ideomotor praxis.
Vascular dementia is more commonly associated with multi- ple infarctions. The infarctions may take the form of numerous large infarctions accompanied by widespread cognitive and mo- tor deficits. Tiny, deep infarctions—lacunae—result from dis- ease of the small arteries that usually involves subcortical struc- tures, such as the basal ganglia, thalamus, and internal capsule. Lacunar strokes are frequently associated with hypertension. The neurological and cognitive deficits may resolve quickly after each
  
90 10. Delirium, Dementia, and Amnestic and Other Cognitive Disorders and Mental Disorders
of the small strokes; however, the deficits may accumulate, lead- ing to a persisting functional and intellectual decline. In the past, a stepwise pattern of deterioration was described for that type of vascular dementia, but it was dropped from the text revision of the fourth edition of the Diagnostic and Statistical Manual of Mental Disorders (DSM-IV-TR) criteria because no specific pat- tern of deterioration has been reliably demonstrated for vascular dementias. Similarly, the description of patchy deficits has been deleted in light of the marked variability in presentation of vas- cular dementia, depending on the type of vasculature and the site and extent of infarction.
10.8. The answer is B
In Huntington’s disease, computed tomography and magnetic resonance imaging scans show caudate atrophy and character- istic “boxcar” ventricles. Functional imaging, such as positron emission tomography (PET), may show striatal hypometabolism. It is transmitted by a single autosomal dominant gene found on the short arm of chromosome 4. It is usually diagnosed in the late 30s or early 40s and affects men and women equally. Emotional symptoms often appear early and include irritability, depression, or psychosis.
10.9. The answer is E
The criteria proposed by the Mayo Clinic Alzheimer’s Disease Research Center (MCADRC) for mild cognitive impairment are (1) memory complaint, preferably qualified by an informant; (2) objective memory impairment for age and education; (3) pre- served general cognitive function; (4) intact activities of daily living; and (5) not demented.
10.10. The answer is E (all)
Prion disease is a group of related disorders caused by a transmis- sible infectious protein known as a prion. Included in this group are Creutzfeldt-Jakob disease (CJD), Gerstmann-Straussler syn- drome (GSS), fatal familial insomnia (FFI), and kuru. A variant of CJD (vCJD), also called “mad cow disease,” appeared in 1995 in the United Kingdom and is attributed to the transmission of bovine spongiform encephalopathy (BSE) from cattle to humans.
10.11. The answer is D
Frontotemporal dementia is a term that encompasses several variant forms of dementia, including Pick’s disease, progressive nonfluent aphasia, semantic dementia, and corticobasal degen- eration. The DSM-IV-TR lists dementia due to Pick’s disease within the category of dementia due to other general medical conditions; there are no separate diagnostic criteria for Pick’s disease or other frontotemporal dementias. Frontotemporal de- mentia is more likely to affect younger populations, and the age of onset ranges between 35 and 75 years of age. The etiology re- mains unclear, but in some cases, genetic linkage to chromosome 17, not 9, has been found. Frontotemporal dementia is typically characterized by asymmetrical focal atrophy of the frontotem- poral regions. There is underlying neuronal loss, gliosis, and subsequent spongiform change in the affected cortices. Pick’s bodies are pathognomonic in Pick’s disease, and they appear swollen and stain pink on hematoxylin and eosin stain.
10.12. The answer is D
No features of the physical examination or laboratory evalua- tion are pathognomonic for dementia of the Alzheimer’s type. Some studies have apparently identified patients with demen- tia of the Alzheimer’s type from patients with dementia of other etiologies by using techniques such as electroencephalog- raphy (EEG), magnetic resonance imaging (MRI), and single photon emission computed tomography (SPECT). These stud- ies have been difficult to replicate consistently, and at present, brain imaging studies are best used to exclude other identifi- able causes. Indeed, available technological diagnostic methods have not proved more sensitive and specific than astute clini- cal evaluation. Age at onset is earlier in patients with a family history of Alzheimer’s disease. Despite some data to suggest distinctive age-related clinical patterns, no phenomenological separation between early- and late-onset cases has been found; however, early-onset dementia of the Alzheimer’s type may have a more rapidly progressive course. A major component of the presenting symptoms is usually subjective complaints of mem- ory difficulty, language impairment (“I can’t find the word”), and dyspraxia (loss of learned skills). Diagnosis at this junc- ture is primarily based on exclusion of other possible causes of dementia.
10.13. The answer is D
Being older than 70, not 60, years of age is a predisposing factor for delirium. Numerous factors increase a person’s risk for delir- ium (Table 10.3). In one study of elderly hospitalized patients, the risk of delirium was higher if the patients had vision impair- ment, more severe medical illness, cognitive impairment, or an elevated ratio of blood urea nitrogen to creatinine. Other stud- ies of intensive care unit patients suggested that hypertension, abnormal total bilirubin, smoking history, epidural use, mor- phine use, and use of intravenous dopamine were associated with delirium.
Table 10.3
Factors That Predispose Patients to Delirium
 
Vision Impairment
Medical illnesses (severity and quantity)
Cognitive impairment
Older than 70 years of age Any iatrogenic
event
Use of physical restraints
Malnutrition
More than three medications added
Hypertension
Chronic obstructive pulmonary disease
Alcohol abuse
Smoking history
Abnormal sodium level
Abnormal glucose level
Abnormal bilirubin level
Ratio of blood urea nitrogen to creatinine >18
Use of Bladder Catheter
Preoperative cognitive impairment
Functional limitations
History of delirium
Abnormal potassium, sodium, or glucose test results
Preoperative use of benzodiazepines Preoperative use of
narcotic analgesics Epidural use
 
10. Delirium, Dementia, and Amnestic and Other Cognitive Disorders and Mental Disorders 91
10.14. The answer is C
Male gender is an independent risk factor for delirium, according to DSM-IV-TR. About 10 to 30 percent of medically ill patients who are hospitalized exhibit delirium. Advanced age is a major risk factor for its development, and approximately 30 to 40 per- cent of hospitalized patients older than 65 years of age have an episode of delirium. The highest rate of delirium, which is more than 90 percent, is found in postcardiotomy patients. Delirium is a poor prognostic sign. Rates of institutionalization are in- creased threefold for patients age 65 years and older who exhibit delirium while in the hospital, and the 1-year mortality rate for patients who have an episode of delirium may be as high as 50 percent.
10.15. The answer is A
The most common cause of delirium in this case is delirium tremens (called alcohol withdrawal delirium in DSM-IV-TR). It is a medical emergency that results in mortality in about 20 per- cent of patients if left untreated. It occurs within 1 week after the person stops drinking. It usually develops on the third hospi- tal day in a patient admitted for an unrelated condition (e.g., surgery) who has no access to alcohol and stops drinking sud- denly. Another less common cause of postoperative delirium is stress, especially in major procedures such as cardiac or trans- plantation surgery. Pain, pain medications, and infection must also be considered in the postoperative period.
10.16. The answer is E (all)
The core features of delirium include altered consciousness, al- tered attention, impaired cognition, rapid onset, brief duration, and unpredictable fluctuations in severity. Associated features that are often present include disorganized thought processes, perceptual disturbances such as illusions and hallucinations, psychomotor hyperactivity and hypoactivity, disruptions of the sleep-wake cycle, mood alterations, and other manifestations of altered neurological function (e.g., autonomic instability).
10.17. The answer is D
Of the drugs listed, the best choice is haloperidol (Haldol), a bu- tyrophenone. Depending on the patient’s age, weight, and phys- ical condition, the initial dose may range from 0.5 to 10 mg intramuscularly repeated in an hour if the patient remains agi- tated. As soon as the patient is calm, oral medication in liquid concentrate or tablet form should begin. Two daily oral doses should suffice, with two-thirds of the dose being given at bed- time. To achieve the same therapeutic effect, the clinician should give an oral dose about 1.5 times higher than a parenteral dose. The effective total daily dosage of haloperidol may range from 5 to 50 mg for the majority of delirious patients. The patient’s response should always be closely monitored for possible side effects.
Phenothiazines, such as chlorpromazine (Thorazine), should be avoided in delirious patients because these drugs are asso- ciated with significant anticholinergic activity. Benzodiazepines with long half-lives, such as diazepam (Valium), and barbiturates, such as amobarbital (Amytal), should be avoided unless they are being used as part of the treatment for the underlying disorder
(e.g., alcohol withdrawal). Sedatives can increase cognitive dis- organization in delirious patients. When the delirium is caused by anticholinergic toxicity, the use of physostigmine salicylate (Antilirium) may be indicated, but it is not the first drug to be used in an acute delirium in which the cause has not been deter- mined.
10.18. The answer is B
The reticular formation of the brainstem is the principal area regulating attention and arousal; the major pathway implicated is the dorsal tegmental pathway. The major neurotransmitter hy- pothesized to be involved is acetylcholine, not epinephrine. In delirium, there is no hyperactivity in the nucleus accumbens. The electroencephalogram characteristically shows a general- ized slowing of activity. A core feature of delirium is altered consciousness.
10.19. The answer is E (all)
An estimated 20 to 30 percent of patients with dementia have hallucinations, and 30 to 40 percent have delusions, primarily of a paranoid or persecutory nature. They may exhibit pathologi- cal laughter or crying with no apparent provocation. Primitive reflexes, such as the grasp, snout, suck, tonic-foot, and palmo- mental reflexes, may be present. Patients with dementia often have disregard for the conventional rules of social conduct and demonstrate coarse language, inappropriate jokes, and neglect of personal appearance and hygiene.
10.20. The answer is D
Although all of the diagnoses listed should be considered in the differential diagnosis of dementia, the cognitive impairment associated with depression is often the most difficult to distin- guish from the symptoms of dementia. The clinical picture is often referred to as pseudodementia or depression-related cog- nitive dysfunction (see Table 10.4). Although schizophrenia may be associated with some acquired intellectual impairment, it is much less severe than that seen in dementia. Aging is not necessarily associated with any significant cognitive decline. Minor memory problems can occur as a normal part of ag- ing, but these do not interfere significantly with a person’s social or occupational behavior. Delirium has a rapid onset, brief duration, fluctuating cognitive impairment, nocturnal ex- acerbations, and prominent disturbances in attention and per- ception. People who attempt to simulate memory loss, as in factitious disorder, do so in an erratic and inconsistent man- ner. In true dementia, memory for time and place is lost before memory for persons, and recent memory is lost before remote memory.
10.21. The answer is E (all)
The classic course of dementia is an onset in the patient’s 50s or 60s, with gradual deterioration over 5 to 10 years, leading eventually to death. The average survival expectation for patients with dementia of the Alzheimer’s type is 8 years. Progression of the disease involves increased agitation, frequent emotional outbursts, and poor sleep with night pacing and wandering. In the terminal phases of dementia, patients become empty shells
92 10. Delirium, Dementia, and Amnestic and Other Cognitive Disorders and Mental Disorders
Table 10.4
Major Clinical Features Differentiating Pseudodementia from Dementia
 
Pseudodementia
Clinical course and history
Family always aware of dysfunction and its severity Onset can be dated with some precision
Symptoms of short duration before medical help is sought Rapid progression of symptoms after onset
History of previous psychiatric dysfunction common
Complaints and clinical behavior
Patients usually complain much of cognitive loss
Patients’ complaints of cognitive dysfunction usually detailed Patients emphasize disability
Patients highlight failures
Patients make little effort to perform even simple tasks
Patients usually communicate a strong sense of distress Affective change often pervasive
Loss of social skills often early and prominent Behavior often incongruent with severity of cognitive
dysfunction
Nocturnal accentuation of dysfunction uncommon
Clinical features related to memory, cognitive, and intellectual dysfunctions
Attention and concentration often well preserved
“Don’t know” answers typical
On tests of orientation, patients often give “don’t know” answers Memory loss for recent and remote events usually severe
Memory gaps for specific periods or events common
Marked variability in performance on tasks of similar difficulty
Dementia
Family often unaware of dysfunction and its severity
Onset can be dated only within broad limits
Symptoms usually of long duration before medical help is sought Slow progression of symptoms throughout course
History of previous psychiatric dysfunction unusual
Patients usually complain little of cognitive loss
Patients’ complaints of cognitive dysfunction usually vague Patients conceal disability
Patients delight in accomplishments, however trivial
Patients struggle to perform tasks
Patients rely on notes, calendars, and so on to keep up
Patients often appear unconcerned
Affect labile and shallow
Social skills often retained
Behavior usually compatible with severity of cognitive dysfunction
Nocturnal accentuation of dysfunction common
Attention and concentration usually faulty
Near-miss answers frequent
On tests of orientation, patients often mistake unusual for usual Memory loss for recent events usually more severe than for remote
events
Memory gaps for specific periods unusuala
Consistently poor performance on tasks of similar difficulty
 
a Except when caused by delirium, trauma, seizures, and so on.
Reprinted with permission from Wells CE. Pseudodementia. Am J Psychiatry. 1979;36:898.
of their former selves, profoundly disoriented, amnestic, and incontinent of urine and feces.
10.22. The answer is C
Petit mal epilepsy or absence seizure is associated with a characteristic generalized, bilaterally synchronous, 3-Hz spike- and-wave pattern in the EEG and is often easily induced by hyperventilation. Petit mal epilepsy occurs predominantly in children. It usually consists of simple absence attacks lasting 5 to 10 seconds, during which the patient has an abrupt alter- ation in awareness and responsiveness and an interruption in motor activity. The child often has a blank stare associated with an upward deviation of the eyes and some mild twitching move- ments of the eyes, eyelids, face, or extremities. Petit mal epilepsy is usually a fairly benign seizure disorder, often resolving after adolescence.
A partial seizure is a type of epilepsy characterized by recur- rent episodes of focal motor seizures. It begins with localized tonic or clonic contraction, increases in severity, and may gen- eralize by spreading progressively through the entire body and terminating in a generalized convulsion with loss of conscious- ness. Grand mal epilepsy is the major form of epilepsy. Gross tonic-clonic convulsive seizures are accompanied by loss of con-
sciousness and often incontinence of stool or urine. Psychomotor epilepsy is a type of epilepsy characterized by recurrent behav- ior disturbances. Complex hallucinations or illusions, frequently gustatory or olfactory, often herald the onset of the seizure, which typically involves a state of impaired consciousness resembling a dream, during which paramnesic phenomena, such as de ́ja` vu and jamais vu, are experienced, and the patient exhibits repeti- tive, automatic, or semipurposeful behaviors. In rare instances, violent behavior may be prominent. The EEG reveals a localized seizure focus in the temporal lobe.
10.23. The answer is A
Dementia of the Alzheimer’s type becomes more common with increasing age; among persons older than 75 years, the risk is six times greater than the risk for vascular dementia. The preva- lence of dementia increases exponentially with age. The esti- mated prevalence of moderate to severe dementia in a population ages 65 years or older is consistently reported at approximately 5 percent. Dementia of the Alzheimer’s type is the most common dementing disorder in clinical and neuropathological prevalence studies reported from North America, Scandinavia, and Europe. Prevalence studies from Russia and Japan show vascular demen- tia to be more common in those countries. It remains unclear
10. Delirium, Dementia, and Amnestic and Other Cognitive Disorders and Mental Disorders 93
whether those apparent clinical differences reflect true etiologi- cal distinctions or inconsistent uses of diagnostic criteria. There is a suggestion of higher rates of dementia of the Alzheimer’s type in women and higher rates of vascular dementia in men. In geri- atric psychiatric patient samples, dementia of the Alzheimer’s type is a much more common etiology (50 to 70 percent) than vascular dementia (15 to 25 percent).
10.24. The answer is E
Delirium generally causes a diffuse slowing of brain activity on the EEG, which may be useful in differentiating delirium from depression and psychosis. The EEG of a delirious patient some- times shows focal areas of hyperactivity. In rare cases, differen- tiating delirium related to epilepsy from delirium related to other causes may be difficult. In general, delirium has a sudden, not in- sidious, onset. Patients with delirium commonly have associated neurological symptoms, including dysphasia, tremor, asterixis, incoordination, and urinary incontinence. Delirium does not gen- erally have an underlying cause residing in the central nervous system (CNS). Delirium has many causes, all of which result in a similar pattern of symptoms relating to the patient’s level of consciousness and cognitive impairment. Most of the causes of delirium lie outside the CNS (e.g., renal and hepatic failures). Delirium cannot be successfully treated with lithium (Eskalith). However, patients with variable lithium serum concentrations may be at risk for delirium.
10.25. The answer is A
When testing cognitive functions, the clinician should evaluate
memory, visuospatial and constructional abilities, reading and writing, and mathematical abilities. Abstraction ability is also valuable to assess, although a patient’s performance on tasks, such as proverb interpretation, may be difficult to evaluate when abnormal. Proverb interpretation may be a useful bedside pro- jective test in some patients, but the specific interpretation may result from a variety of factors, such as poor education, low intel- ligence, and failure to understand the concept of proverbs, as well as a broad array of primary and secondary psychopathological disturbances.
10.26. The answer is C
For some forms of amnestic disorder, events from the remote past may be better remembered than more recent events. However, such a gradient of recall is not present uniformly among individu- als with amnestic disorders. Typically, the ability to immediately repeat a sequential string of information (e.g., a digit span) is not impaired in amnestic disorder; when such impairment is evi- dent, it suggests the presence of attentional dysfunction that may be indicative of delirium. Amnestic disorders may be transient, lasting for several hours to a few days, as in transient global amnesia, or persistent, lasting at least 1 month.
Depending on the cause of the disorder, the onset of amnesia may be sudden or gradual. Head trauma, vascular events, and specific types of neurotoxic exposure (e.g., carbon monoxide poisoning) may lead to acute mental status changes. Prolonged substance abuse, chronic neurotoxic exposure, and sustained nutritional deficiency exemplify conditions that may lead to an
insidious memory decline, eventually causing a clinically defin- able cognitive impairment. Amnestic disorder may also develop as a result of alcohol dependence associated with dietary and vitamin deficiency.
10.27. The answer is A
Amnestic disorders are secondary syndromes caused by systemic medical or primary cerebral diseases, substance use disorders, or medication adverse effects. The essential feature of amnes- tic disorders is the acquired impaired ability to learn and recall new information coupled variably with the inability to recall previously learned knowledge or past events. The impairment must be sufficiently severe to compromise personal, social, or occupational functioning. The diagnosis is not made if the memory impairment exists in the context of reduced ability to maintain and shift attention, as encountered in delirium, or in association with significant functional problems caused by the compromise of multiple intellectual abilities, as seen in dementia.
10.28. The answer is E (all)
Cerebrovascular diseases together comprise the second most common cause of dementia. This category of dementia was re- ferred to in the past as arteriosclerotic dementia, reflecting the belief that vascular insufficiency was responsible for the cogni- tive degeneration. That has now been supplanted by the belief that tissue damage or infarction underlies the vascular dementias. Cerebral infarction can be the result of a number of processes, of which thromboembolism from a large vessel plaque or cardio- thrombus is the most common. Anoxia caused by cardiac arrest, hypotension, anemia, or sleep apnea can also produce ischemia and infarction. Cerebral hemorrhage related to hypertension or an arteriovenous malformation accounts for approximately 15 percent of cerebrovascular disease.
10.29. The answer is E (all)
In recent years, several authors have sought to distinguish de- mentias of the frontal lobe from other disorders. The uncertain status of dementias of the frontal lobe as distinct clinical and neuropathological entities has not yet warranted their formal in- clusion in DSM-IV-TR or ICD-10. They are described as cortical dementias that are found in as many as 10 to 20 percent of cases in some neuropathological series. The age at onset is appar- ently between 50 and 60 years for the majority, but the reported range is broad—20 to 80 years. The early clinical features of frontal lobe dementias are typified by damage to the frontal lobes and include prominent changes in personality and behavior. The personality changes include disinhibition, social misconduct, and lack of insight; these changes progress to apathy, mutism, and repetitive behaviors.
Neuropsychological testing in patients suspected of having dementia of frontal lobe origin may demonstrate disproportion- ate impairment in tasks related to frontal lobe function, such as deficiency in abstract thinking, attentional shifting, or set forma- tion. Structural neuroimaging, such as computed tomography or magnetic resonance imaging, may reveal prominent atrophy of the frontal lobe, especially early in the disease process.
94 10. Delirium, Dementia, and Amnestic and Other Cognitive Disorders and Mental Disorders
10.30. The answer is E (all)
There have been relatively few studies of the incidence and preva- lence of delirium. Little is known about the epidemiology of delirium in community or other nonpatient, noninstitutionalized populations. An estimated 10 to 15 percent of general medical inpatients are delirious at any given time, and studies indicate that as many as 30 to 50 percent of acutely ill geriatric patients become delirious at some point during their hospital stay. Rates of delirium in psychiatric and nursing home populations are not well established but are clearly substantial. Risk factors for the development of delirium include increased severity of physical illness, older age, baseline cognitive impairment (e.g., caused by dementia), and use of anticholinergics.
Delirium is frequently unrecognized by treating physicians. Because of its wide array of associated symptoms, it may be de- tected but misdiagnosed as depression, schizophrenia, or another psychiatric disorder. Delirium is a frequent cause for psychiatric consultation in the general hospital but often is not recognized as delirium by the referring physician.
10.31. The answer is C
Bovine spongiform encephalopathy is also known as subacute spongiform encephalopathy because of shared neuropatholog- ical changes that consist of (1) spongiform vacuolization, (2) neuronal loss, and (3) astrocyte proliferation in the cerebral cortex. Amyloid plaques may or may not be present. A char- acteristic bull’s eye rash is found at the site of a tick bite and is not associated with bovine spongiform encephalopathy.
10.32. The answer is A
Psychotic symptoms in dementia generally occur in the middle stages (not in the early stages) of the illness and often co-occur with behavioral disturbance. A common early psychotic symp- tom is paranoia and the belief that belongings have been stolen. This may develop as a result of poor short-term memory and misplacing objects. This paranoia can become more pervasive as the illness progresses. In later stages, patients may have hal- lucinations in any modality, but these most commonly are visual hallucinations. They may say they see deceased relatives. The hallucinations are often congruent with delusions of the same theme. Psychotic symptoms can sometimes lead to agitation, aggression, and behavioral disturbance as patients act out on their hallucinations, paranoia, or delusional thinking.
10.33. The answer is E
A variety of neuropsychiatric disorders may explain the patient’s symptoms; however, he is most likely exhibiting normal changes related to aging. It is normal to have episodes of forgetfulness as one ages. Furthermore, the patient’s memory deficits do not appear to alter his social functioning. His independence in ac- tivities of daily living is not impaired. The patient is still able to operate a motor vehicle and care for himself on his own. As peo- ple age, they also become more easily tired, exhibit diminished energy levels, and may find it difficult falling asleep. Depres- sion may manifest as a pseudodementia in elderly individuals. It may involve forgetfulness and personality changes; however, this patient does not exhibit depressed mood or social withdrawal. Normal-pressure hydrocephalus is characterized by the triad of
“wet, whacky, and wobbly,” that is incontinence, dementia, and ataxia. Frontotemporal dementia is characterized by prominent personality changes and loss of executive function. Vascular de- mentia will present with a stepwise downward shift in cognitive functioning.
Answers 10.34–10.39 10.34. The answer is B 10.35. The answer is C 10.36. The answer is C 10.37. The answer is A 10.38. The answer is D
10.39. The answer is C
The differentiation between delirium and dementia can be dif- ficult, but several clinical features help in the differentiation. In contrast to the sudden onset of delirium, dementia usually has an insidious onset. Although both conditions include cognitive impairment, the changes in dementia are relatively stable over time and do not fluctuate over the course of a day, for example. A patient with dementia is usually alert; a patient with delirium has episodes of clouding of consciousness. Delirium and in rare cases dementia are reversible, although delirium has a better chance of reversing if treatment is timely. Insight, defined as the awareness that one is mentally ill, is absent in both conditions. Hallucina- tions can occur in both conditions and must be differentiated from those that occur in schizophrenia. In general, the hallucinations of schizophrenic patients are more constant and better formed than are the hallucinations of delirious patients. Sundowning is observed in both demented and delirious patients. Sundown- ing is characterized by drowsiness, confusion, ataxia, and ac- cidental falls just around bedtime. Kurt Goldstein described a catastrophic reaction in demented patients; it is marked by agi- tation secondary to the subjective awareness of one’s intellectual deficits under stressful circumstances. The presence of delirium is a bad prognostic sign. Patients with delirium have a high mor- tality rate. The 3-month mortality rate of patients who have an episode of delirium is estimated to be 23 to 33 percent; the 1-year mortality rate may be as high as 50 percent. The major neuro- transmitter hypothesized to be involved in delirium and dementia is acetylcholine. Several types of studies of delirium and demen- tia have shown a correlation between decreased acetylcholine activity in the brain and both delirium and dementia.
Answers 10.40–10.45 10.40. The answer is B 10.41. The answer is A 10.42. The answer is A 10.43. The answer is B
10. Delirium, Dementia, and Amnestic and Other Cognitive Disorders and Mental Disorders 95
10.44. The answer is B
10.45. The answer is A
Degenerative CNS diseases can be distinguished clinically from one another by the relative impairment and sparing of vari- ous cognitive and behavioral functions. Two basic patterns of dementia have been characterized clinically: cortical and sub- cortical.
The cortical pattern of dementia is characterized by impair- ments in memory (primarily a storage and recall deficit) and gnostic-practice abilities (primarily involving language, visu- ospatial abilities, calculation, and motor praxis). Executive or managerial functions such as organization, judgment, abstrac- tion, emotional control or modulation, and insight and social judgment are similarly affected. Personality often remains intact or displays subtle variations, with patients becoming more pas- sive or less spontaneous or becoming coarse and crude in their interactions. With disease progression, the changes in personality become more common and pronounced. Affective expression is generally preserved, although again, a coarsening may be noted in the form of emotional lability. Early in the disease, patients frequently discern and express dismay about their intellectual decline.
The subcortical pattern is characterized by a generalized slowing of mental processing. Specific cognitive skills, such as calculation, naming, or copying, are less affected initially in con- trast to their early decline in the cortical degenerative processes. Verbal and visual memory impairment may be present early in the course, although such impairment more often takes the form of forgetfulness or a failure of retrieval that is initially amenable to prompting. This is in contrast to the more severe recall deficits of cortical dementia. Patients also show deficits in learning new mo- tor movements or complex psychomotor procedures. Planning and organizational skills are disrupted. Abnormal movements are common and manifest as a slowing and awkwardness in nor- mal movement or chorea and tremor. In contrast to the early impairment of language function in cortical disease, language is relatively spared, although the motor production of speech may be abnormal. The personality change is often marked, with striking patterns of apathy, inertia, and diminished spontaneity. Mood disorders, including major depression and mania, occur frequently. The presenting symptoms in subcortical degenera- tive processes may be those of a personality change or a mood disorder at a time when cognitive impairment or motor dysfunc- tion is not yet obvious. In the cortical processes, by contrast, the presenting symptoms more often reflect cognitive impairment, particularly memory and language dysfunction. As the dementia and the degenerative process progress, the clinical presentations of cortical and subcortical diseases become nearly indistinguish- able from one another.
The term subcortical dementia was first used to describe the cognitive and behavioral deficits seen in patients with Hunt- ington’s disease. A similar clinical pattern was soon described for other subcortical diseases, such as progressive supranuclear palsy and Parkinson’s disease. Although the term was initially used in reference to a clinical picture that could be localized to the subcortex, subcortical dementia is now considered a pseu- doanatomical designation. It is clear from imaging and neu-
ropathological studies that cortical dementia (e.g., dementia of the Alzheimer’s type) is not restricted pathologically to the cor- tex; major affected cholinergic fiber pathways are subcortical in origin.
Answers 10.46–10.50 10.46. The answer is D 10.47. The answer is B 10.48. The answer is A 10.49. The answer is C
10.50. The answer is E
The lettered choices are all categorized in DSM-IV-TR under dementia due to a general medical condition. Dementia due to head trauma is the third most common cause of dementia in in- dividuals younger than 50 years old, most frequently occurring in men ages 14 to 24 years. Closed head injury is the most com- mon cause. Memory loss and impaired executive function are common features.
Parkinson’s disease is a movement disorder characterized by tremor, muscle rigidity, bradykinesia, and postural insta- bility. About 30 to 50 percent of patients with Parkinson’s disease develop dementia. Parkinson’s disease dementia is a subcortical dementia with slowed processing speed and memory loss, mood disorders, and difficulties with abstract thought and reasoning.
Human immunodeficiency virus type 1 (HIV-1) is commonly associated with motor and cognitive impairment ranging from subtle changes to overt dementia. HIV-1-associated dementia (HIV-D) occurs in approximately 10 percent of patients with acquired immune deficiency syndrome (AIDS). HIV-D presents with psychomotor slowing, personality changes, impaired ab- stract reasoning, and problem-solving and memory deficits. Psy- chosis can occur in later stages. Highly active antiretroviral ther- apy has reduced the incidence of HIV-D.
Huntington’s disease is a rare motor disorder characterized by involuntary dance-like movements (chorea) and lack of co- ordination (ataxia). It is an autosomal dominant, inherited dis- ease transmitted by a mutation on chromosome 4. Cognitive changes include psychomotor slowing, impaired executive func- tioning, and memory deficits. Huntington’s disease is inevitably fatal.
Creutzfeldt-Jakob disease is a very rare disorder caused by a transmittable prion that accumulates in the brain, leading to neu- ron death. It is characterized by a rapidly progressive dementia, with memory loss, personality changes, and hallucinations, and inevitably resulting in death over the course of a few months. There are no treatments other than supportive care.
Answers 10.51–10.55 10.51. The answer is B 10.52. The answer is A
96 10. Delirium, Dementia, and Amnestic and Other Cognitive Disorders and Mental Disorders
10.53. The answer is D
10.54. The answer is E
10.55. The answer is C
Huntington’s disease, which is inherited in an autosomal domi- nant pattern, leads to major atrophy of the brain with extensive degeneration of the caudate nucleus. The onset is usually insid- ious and most commonly begins in late middle life. The course is one of gradual progression; death occurs 15 to 20 years after the onset of the disease, and suicide is common.
Creutzfeldt-Jakob disease is a rare degenerative brain disease caused by a prion protein, with death occurring within 2 years of the diagnosis. A computed tomography scan shows cerebellar and cortical atrophy.
Neurosyphilis is a chronic dementia and psychosis caused by the tertiary form of syphilis affecting the brain. The presenting
symptoms include a manic syndrome with neurological signs in up to 20 percent of cases.
Normal-pressure hydrocephalus is associated with enlarged ventricles and normal cerebrospinal fluid (CSF) pressure. The characteristic signs include dementia, a gait disturbance, and urinary incontinence. The treatment of choice is CSF shunt from the ventricular space to either the atrium or the peritoneal space. Reversal of the dementia and associated signs is sometimes dra- matic after treatment.
Multiple sclerosis is characterized by diffuse multifocal le- sions in the white matter of the central nervous system. Its clinical course is characterized by exacerbations and remissions. It has no known specific cause, although research has focused on slow viral infections and autoimmune disturbances. Multiple sclero- sis is much more prevalent in cold and temperate climates than in the tropics and subtropics. It is more common in women than in men and is predominantly a disease of young adults.

11
Neuropsychiatric Aspects of HIV Infection and AIDS

The human immunodeficiency virus (HIV) is an epidemic that continues to be a major public health problem throughout the world. Currently, it is estimated that up to 40 million people are infected with HIV worldwide, approximately 1 million within the United States. Psychiatric disorders play a role in this epi- demic by increasing risk behaviors for infection and decreasing access to treatment. Thus, HIV has become a psychiatric epi- demic as well.
An extensive array of disease processes can affect the brain of a patient infected with HIV. The most important diseases for mental health workers to be aware of are HIV mild neurocogni- tive disorder and HIV-associated dementia. HIV-associated de- mentia can affect 50 percent of HIV infected patients. Multi- ple psychiatric syndromes can also affect HIV-infected patients, from adjustment disorders to mood disorders to substance abuse
disorders and suicide, all of which the psychiatrist must be fa- miliar. Similarly, the pharmacotherapies used in the management and treatment of HIV disease and AIDS may directly affect the brain or interact with the medications used by psychiatrists to treat the associated psychiatric syndromes.
Psychiatrists must be familiar with counseling patients about their risk factors for the disease, and the importance of HIV test- ing. Confidentiality issues are key in this matter. Psychotherapy plays an important role in working with this patient population, and psychiatrists must be familiar with the range of approaches that may be appropriate for these patients, including support- ive, cognitive, behavioral, or psychodynamic approaches, both as individual treatments or in groups.
Students should study the questions and answers below for a useful review of this topic.
The following terms should be known by students.
AIDS dementia complex
AIDS mania
AZT
Candida albicans Central nervous system
infections confidentiality
cytomegalovirus (CMV)
ddI
ELISA Guillain-Barre ́
syndrome
HIV encephalopathy Kaposi’s sarcoma
Pneumocystis carinii
pneumonia pretest and posttest
counseling protease inhibitors safe sex guidelines seropositive
T4 lymphocytes
Toxoplasma gondii and Cryptococcus neoformans
transmission tuberculosis
wasting syndrome Western blot analysis worried well
HELPFUL HINTS
QUESTIONS
Directions
Each of the questions or incomplete statements below is followed by five suggested responses or completions. Select the one that is best in each case.
11.1. Which of the following statements about Guillain-Barre ́ syndrome is false?
A. It is associated with late human immunodeficiency virus (HIV) infection.
B. It causes symmetrical paralysis.
C. It may impair respiration.
D. It usually occurs in young men.
E. It becomes serious if thoracic musculature is in-
volved.
11.2. Truestatementsassociatedwiththetreatmentofdelirium
in HIV illness include
A. PatientswithunderlyingHIV-associateddementiado not appear to be at higher risk for medication-induced movement disorders.
B. Symptoms of delirium in HIV illness can be managed effectively with low-potency antipsychotics such as chlorpromazine (Thorazine).
97
▲ ▲▲▲▲▲
▲▲▲▲▲▲
▲▲▲▲▲▲
▲▲▲▲▲▲
98
11. Neuropsychiatric Aspects of HIV Infection and AIDS
C. There is no increased incidence of extrapyrami- dal symptoms associated with high-potency typical agents in advanced HIV illness.
D. The use of benzodiazepines alone appears to be effective in delirious states.
E. None of the above.
. 11.3. TruestatementsabouttheassociationofsuicideandHIV disease include
A. Studies suggest that patients with advanced HIV dis- ease have a 30-fold increased risk of committing sui- cide compared with matched seronegative persons.
B. Some reports indicate that high-risk seronegative per- sons have an elevated lifetime prevalence of suicidal ideation and attempt compared with community con- trol subjects.
C. Psychiatric disorders are strongly implicated in sui- cidal ideation and attempted suicide.
D. HIV-infected adolescents are at a particularly high risk for suicide.
E. All of the above
. 11.4. Neuropathic pain related to HIV
A. should not be treated with acetaminophen (Tylenol) because it may diminish the metabolism of zidovu- dine.
B. is not effectively managed with anticonvulsants such as phenytoin (Dilantin) or carbamazepine (Tegretol).
C. is generally more effectively treated with selective serotonin reuptake inhibitors than with tricyclic an-
tidepressants.
D. is rarely effectively treated with opioid analgesics.
E. all of the above
. 11.5. Protease inhibitors can increase plasma levels of all of the following except
A. alprazolam and zolpidem B. nefazodone
C. valproate
D. bupropion
E. fluoxetine
. 11.6. Acquired immunodeficiency syndrome (AIDS) mania is
A. benign in its course
B. mild in its presentation
C. described in late HIV infection
D. not associated with cognitive impairment E. more episodic than chronic
. 11.7. A 28-year-old AIDS patient presents to clinic with his mother. The patient’s mother complains that her son stays up all night and speaks so fast that she cannot keep up with what he is saying. The patient confirms a grandiose scheme to take over the earth as a novel prophet with in- finite powers. He acknowledges a $4,000 shopping spree during which he purchased accessories to embellish him-
self with. In treating AIDS patients with mania, potential complications include
A. Carbamazepine may increase serum concentrations of protease inhibitors.
B. Lithiumandantipsychoticmedicationsmaybepoorly tolerated by individuals with HIV-related neurocog- nitive disorders.
C. The gastrointestinal disturbances associated with AIDS (e.g., vomiting and diarrhea) rarely affect lithium absorption or excretion.
D. Protease inhibitors increase valproate concentrations.
E. Valproate is usually poorly tolerated by individuals
with evidence of brain atrophy on MRI.
. 11.8. Which of the following about the pharmacotherapy of
HIV disease is true?
A. Two protease inhibitors and one reverse transcriptase is indicated.
B. A two-agent therapy is indicated in health care workers who have been pricked by a needle from an HIV-infected patient.
C. WhitemattersignalabnormalitiesonMRIinpatients who are HIV positive are permanent and not respon- sive to anti-HIV pharmacotherapy.
D. Zidovudine penetrates the blood-brain barrier well.
E. Protease inhibitors are metabolized primarily in the
kidney.
. 11.9. In persons infected by HIV
A. seroconversion usually occurs 2 weeks after infec- tion.
B. the estimated length of time from infection to the development of AIDS is 5 years.
C. 10 percent have neuropsychiatric complications.
D. the T4-lymphocyte count usually decreases to abnor-
mal levels during the asymptomatic period.
E. the majority are infected by HIV type 2 (HIV-2).
. 11.10. In a test for HIV
A. assays usually detect the presence of viral proteins.
B. the enzyme-linked immunosorbent assay (ELISA) is used to confirm positive test results of the Western
blot analysis.
C. the results cannot be shared with other members of a
medical treatment team.
D. pretest counseling should not inquire why a person
desires HIV testing.
E. a person may have a true-negative result even if the
person is infected by HIV if given prior to serocon- version.
. 11.11. Diseases affecting the central nervous system (CNS) in patients with AIDS include
A. atypical aseptic meningitis B. Candida albicans abscess
C. cerebrovascular infarction D. primary CNS lymphoma E. all of the above
. 11.12. Mild neurocognitive deficits associated with HIV infection
A. include attentional problems, slowing of information processing, and deficiencies in learning.
B. do not suggest selective involvement of subcortical structures.
C. are often characterized by confabulatory responses on formal memory testing.
D. do not occur independently of depression or anxiety.
E. are rarely associated with difficulties in abstract rea-
soning.
. 11.13. Which of the following personality disorders is a risk factor for HIV infection?
bered word or statement, select the one lettered heading most closely associated with it. Each lettered heading may be selected once, more than once, or not at all.
Questions 11.17–11.21
A. Nucleoside reverse transcriptase inhibitor
B. Nonnucleoside reverse transcriptase inhibitor C. Protease inhibitor
D. All of the above
E. None of the above
. 11.17. AZT
. 11.18. Nevirapine
. 11.19. Ritonavir
. 11.20. Stavudine
. 11.21. 3TC
ANSWERS
11.1. The answer is A
A small percentage of patients, usually young men, present with Guillain-Barre ́ syndrome associated with early (not late) HIV infection. Guillain-Barre ́ syndrome is an inflammatory demyeli- nating polyneuropathy causing symmetrical paralysis and few, if any, sensory symptoms, usually beginning in the lower extremi- ties and progressing upward. The condition becomes especially serious if the thoracic musculature is involved because it may impair respiration. The disorder is thought to be autoimmune in etiology and generally self-limited. Intravenous immunoglobu- lin and plasmapheresis have been used to shorten the course, but neither treatment has been studied well in HIV-infected individuals.
11.2. The answer is B
Symptoms of delirium in HIV illness can be managed effectively with modest dosages of either low-potency antipsychotic agents, such as chlorpromazine at 10 to 25 mg once to three times daily; or with high-potency agents, such as haloperidol (Haldol) at 0.25 mg to 5 mg once to three times daily; or with atypical serotonin-dopamine agonists, including risperidone (Risperdal) at 0.5 mg to 2 mg daily; or olanzapine (Zyprexa) at 10 mg daily. There may well be an increased incidence of extrapyramidal symptoms associated with high-potency typical agents in ad- vanced HIV illness, and patients with underlying HIV-associated dementia appear to be at higher risk for medication-induced movement disorders. For patients who do not respond to low- dosage oral therapy, excellent results have been reported with intravenous (IV) haloperidol given in individual boluses rang- ing from 2 to 10 mg every hour. Some clinicians have also had good results with a combination of IV haloperidol and lorazepam (Ativan), with an average daily IV dose of less than 50 mg of haloperidol and 10 mg of lorazepam. In general, no serious ad- verse effects have been noted with more aggressive IV regimens, although nearly half of the patients treated may have extrapyra- midal symptoms, and extreme care must be used.
Benzodiazepines alone (e.g., lorazepam) do not appear to be effective in delirious states, and they may accentuate confusion.
A. Antisocial personality disorder
B. Obsessive-compulsive personality disorder C. Histrionic personality disorder
D. Paranoid personality disorder
E. Avoidant personality disorder
. 11.14. Clinical symptoms associated with HIV encephalopathy diagnosis include all of the following except
A. psychomotor slowing
B. problems with memory and concentration C. mood and personality changes
D. hyperreflexia and paraparesis
E. early-onset aphasia
. 11.15. Which of the following statements about lymphoma and AIDS is false?
A. Lymphoma is the most common neoplasm seen in AIDS.
B. Seizures are present in about 15 percent of patients.
C. A brain biopsy is not required to confirm diagnosis.
D. Central nervous system lymphoma is at times misdi-
agnosed as toxoplasmosis.
E. Theprognosisdependsonthehighlyactiveantiretro-
viral therapy (HAART) response.
. 11.16. Which of the following is an early symptom of HIV- associated dementia?
A. Psychosis
B. Sleep disturbances C. Apathy
D. Weight loss
E. Anxiety
Directions
The group of questions below consists of lettered headings fol- lowed by a list of numbered words or statements. For each num-
11.
Neuropsychiatric Aspects of HIV Infection and AIDS 99
100 11. Neuropsychiatric Aspects of HIV Infection and AIDS
11.3. The answer is E (all)
Studies based on coroners’ reports suggest that patients with ad- vanced HIV disease have a 30-fold increased risk of committing suicide compared with seronegative persons matched for age and social position. Some survey reports indicate that seronegative persons who are in a high-risk group for HIV infection, as well as seropositive persons at all stages of HIV infection, have an elevated lifetime prevalence of suicidal ideation and suicide at- tempt compared with community control subjects. It is important to note that both sources of data suggest that psychiatric disorder is strongly implicated in suicide, attempted suicide, and suici- dal ideation. Psychological autopsies from coroners’ cases have identified psychiatric histories in almost 50 percent of the cases. Suicide attempt and suicidal ideation are correlated with histo- ries of major depressive disorder or substance-related disorders, and in more than half of the cases, these suicidal behaviors com- menced before the likely date of seroconversion. Conflicts about sexual orientation may be associated with suicide attempts by adolescents. This, together with the increase in HIV infection in adolescents, may place HIV-infected youths at particularly high risk. Suicide rates in women are not noted to be elevated, but the epidemic is now just starting to affect large numbers of women, and their greater vulnerability to major depressive disorder may mean that women are at increased risk. Advances in therapy may heighten hope and reduce the risk of suicide. However, those whose hopes are first raised but who then do not respond to or cannot tolerate these agents may require psychotherapeutic intervention.
11.4. The answer is A
Among the rheumatologic disorders are arthralgias, myalgias, and arthritides involving large joints of the leg. Patients with HIV-related arthralgias may respond to nonsteroidal antiinflam- matory agents, although acetaminophen (Tylenol) should be avoided because it may diminish the metabolism of zidovu- dine. HIV may also be associated with a polymyositis, which involves pain, weakness, and elevated creatine phosphokinase, along with changes on electromyography indicating a myopathic process. Long-term administration of zidovudine may also pro- duce a myositis that persists when the medication is discontin- ued. Psychopharmacologic interventions are not of demonstrated efficacy in these states.
Neuropathic pain related to HIV usually presents as a persist- ing, painful sensorimotor neuropathy with dysesthesia, stocking- glove distribution of sensory loss, diminished distal reflexes, and distal weakness. Similarly, postherpetic neuralgia (herpes zoster radiculitis) may involve pain of the face or trunk. Treatment of neuropathic pain syndromes is usually with low-dosage tricyclic antidepressant agents, such as desipramine or nortriptyline at 10 to 25 mg a day. The typical steady-state dosage is 50 mg a day, although some patients require higher amounts (75 to 100 mg daily). A response often occurs within 1 to 2 weeks, but 4 to 6 weeks of treatment may be necessary before response oc- curs or another tricyclic agent is chosen. In general, tricyclic antidepressants are more effective than the selective serotonin reuptake inhibitors (SSRIs) for chronic neuropathic pain. Opi- oid analgesics are also useful. Anticonvulsants such as pheny- toin (Dilantin) or carbamazepine (Tegretol), at usual therapeutic
concentrations required for seizure management, may also be effective. Postherpetic neuralgia may likewise be treated with topical capsaicin (Dolorac) and may respond to clonazepam at 1to5mgdaily.
Finally, studies of acute postoperative pain and chronic cancer pain generally indicate that for conditions in which opiate anal- gesia is indicated, those medications are often underprescribed or irrationally prescribed in subtherapeutic doses at too extended an interval. The clinician should always be alert to that possibility in advanced HIV disease.
11.5. The answer is C
A growing list of agents that act at different points of viral repli- cation has raised for the first time the hope that HIV can be permanently suppressed or actually eradicated from the body.
Antiretroviral agents have many adverse effects, too numer- ous to describe. Of importance to psychiatrists is that protease in- hibitors are metabolized by the hepatic cytochrome P450 oxidase system and can therefore increase levels of certain psychotropic drugs that are similarly metabolized. These include bupropion (Wellbutrin), meperidine (Demerol), various benzodiazepines, and SSRIs. Therefore, prescribing psychotropic drugs to persons taking protease inhibitors must be done with caution. For exam- ple, plasma concentrations of alprazolam (Xanax), midazolam (Versed), triazolam (Ilalcion), and zolpidem (Ambien) may be increased, and dosage reduction and careful monitoring may be required to prevent oversedation or other toxic effects. Protease inhibitors have been reported to increase concentrations of bu- propion, nefazodone (Serzone), and fluoxetine (Prozac) to toxic levels and to increase desipramine plasma concentrations by 100 to 150 percent. Drug interactions with antipsychotic agents are less well studied, but ritonavir may particularly increase concentrations. Concentrations of methadone and meperidine are also reported to be elevated. Additionally, concentrations of some drugs of abuse such as methylenedioxymethamphetamine (MDMA) may be increased. In turn, protease inhibitors may induce the metabolism of valproate (Depakene) and of lorazepam (Ativan) and lead to lower plasma concentrations.
Some psychotropic medications may induce metabolism of protease inhibitors. Carbamazepine and phenobarbital may reduce serum concentrations of protease inhibitors. The clinical relevance of this potential interaction is not clear, but use of an alternate mood stabilizer may be indicated. Interactions with lithium (Eskalith) and gabapentin (Neurontin) have not been reported.
Finally, psychotropic drugs may reduce the metabolism of some protease inhibitors, with an increase of protease inhibitor adverse effects; this has been reported with nefazodone and fluoxetine.
11.6. The answer is C
Mania can occur any time in the course of HIV infection for individuals with preexisting bipolar disorder, but AIDS mania has been described in late HIV infection, thus appearing to be a consequence of HIV brain involvement. Mania is a mood state characterized by elation, agitation, hyperactivity, hypersexual- ity, and accelerated thinking and speaking (flight of ideas). AIDS mania is a specific type of mania that appears to be specifically
associated with late-stage HIV infection and is associated with cognitive impairment and a lack of previous episodes or family history. Clinical experience has suggested that AIDS mania is usually quite severe (not mild) in its presentation and malignant (not benign) in its course. AIDS mania seems to be more char- acteristically chronic than episodic, has infrequent spontaneous remissions, and usually relapses with cessation of treatment.
11.7. The answer is B
For immediate control of manic excitement, up to 10 mg of clonazepam (Klonopin) daily is effective in many instances, al- though the risk for disinhibition or delirium must always be monitored. If psychotic features are present, low doses of an- tipsychotic agents, such as risperidone at 0.5 to 2 mg daily, olan- zapine up to 10 mg daily, chlorpromazine at 25 to 150 mg daily, or haloperidol at 0.5 to 5 mg daily, may be used. For longer-term management, lithium is effective but may not be as well tolerated as carbamazepine and valproate. For example, in some studies, lithium and antipsychotic medications are poorly tolerated by individuals with HIV-associated neurocognitive disorders, espe- cially if brain magnetic resonance imaging (MRI) abnormalities are present (e.g., atrophy), but valproate (dosage range, 750 to 1,750 mg daily; plasma concentration >50 μg per mL) is more successful. Good control is usually possible within 7 days, and treatment gains have been maintained for up to 4-year follow- up. Lithium has been used to treat patients who develop manic syndromes as an adverse effect of zidovudine with good control of symptoms, which allows a patient to continue antiretroviral therapy. It may be that valproic acid and carbamazepine would be effective for patients with these iatrogenic manias.
In HIV-infected patients treated with lithium for the control of bipolar I disorder, care must be taken to monitor lithium con- centrations closely, especially if the patient has significant gas- trointestinal disturbances (e.g., vomiting and diarrhea) that may affect lithium absorption and excretion. Carbamazepine may reduce serum concentrations of protease inhibitors, and these agents themselves may lower valproate concentrations.
11.8. The answer is D
High-dosage zidovudine monotherapy can ameliorate HIV- associated neurocognitive impairment because it penetrates the blood–brain barrier well. Since the introduction of protease inhibitors and combination antiretroviral therapy, zidovudine monotherapy is no longer an option of treatment, but it remains an important component of combination regimes. Current rec- ommendations are that treatment should be initiated with triple therapy, that is, a combination of two reverse transcriptase in- hibitors and one protease inhibitor. Triple therapy may also be used for people who have had an unexpected sexual encounter with a potentially infected partner and in health care workers who have been pricked by a needle from an infected patient. Protease inhibitors are metabolized in the liver by the hepatic cytochrome P450 oxidase system and can therefore increase levels of cer- tain psychotropic drugs that are similarly metabolized. Clinical improvement, including enhanced performance on standardized neuropsychological testing and of the pattern and severity of white matter signal abnormalities on magnetic resonance imag- ing, can be seen within 2 to 3 months of beginning therapy.
11.9. The answer is D
The T4-lymphocyte count usually decreases to abnormal levels during the asymptomatic period of HIV infection. The normal values are greater than 1,000/mm3, and grossly abnormal values can be fewer than 200/mm3 .
Seroconversion is the change after infection with HIV from a negative HIV antibody test result to a positive HIV antibody test result. Seroconversion usually occurs 6 to 12 weeks after infection. In rare cases, seroconversion can take 6 to 12 months. The estimated length of time from infection to the development of AIDS is 8 to 11 years, although that time is gradually increasing because of the early implementation of treatment. At least 50 percent of HIV-infected patients have neuropsychiatric compli- cations, which may be the first signs of the disease in about 10 percent of patients. At least two types of HIV have been identi- fied, HIV type 1 (HIV-1) and HIV type 2 (HIV-2). The majority of HIV-positive patients are infected by HIV-1. However, HIV-2 infection seems to be increasing in Africa.
11.10. The answer is E
A person may have a true negative result, even if the person is infected by HIV, if the test takes place after infection but be- fore seroconversion. Assays do not usually detect the presence of viral proteins. The enzyme-linked immunosorbent assay (ELISA) is not used to confirm positive test results of the Western blot anal- ysis. Rather, the ELISA is used as an initial screening test because it is less expensive than the Western blot analysis and more easily used to screen a large number of samples. The ELISA is sensi- tive and reasonably specific; although it is unlikely to report a false-negative result, it may indicate a false-positive result. For that reason, positive results from an ELISA are confirmed by us- ing the more expensive and cumbersome Western blot analysis, which is sensitive and specific.
Confidentiality is a key issue in serum testing. No persons should be given HIV tests without their prior knowledge and consent, although various jurisdictions and organizations (e.g., the military) now require HIV testing for all their inhabitants or members. The results can be shared with other members of a medical treatment team but should be provided to no one else.
Pretest counseling should inquire why a person desires HIV testing to detect unspoken concerns and motivations that may merit psychotherapeutic intervention. Table 11.1 lists general guidelines for HIV testing and counseling.
11.11. The answer is E (all)
Most of the infections secondary to HIV involvement of the cen- tral nervous system (CNS) are viral or fungal. Atypical aseptic meningitis, Candida albicans abscess, primary CNS lymphoma, and cerebrovascular infarction can all affect a patient with AIDS.
11.12. The answer is A
A person experiencing mild neurocognitive disorder associated with HIV infection typically has some difficulty concentrating, may experience unusual fatigability when engaged in demanding mental tasks, may feel subjectively slowed down, and may notice difficulty in remembering. Such persons may say they are not as sharp or as quick as they once were.
11. Neuropsychiatric Aspects of HIV Infection and AIDS 101
102 11. Neuropsychiatric Aspects of HIV Infection and AIDS Table 11.1
Pretest HIV Counseling
1. Discuss the meaning of a positive result and clarify distortions (e.g., the test detects exposure to the AIDS virus; it is not a test for AIDS).
2. Discussthemeaningofanegativeresult(e.g.,seroconversionrequirestime;recenthigh-riskbehaviormayrequirefollow-uptesting). 3. Be available to discuss the patient’s fears and concerns (unrealistic fears may require appropriate psychological intervention).
4. Discuss why the test is necessary (not all patients will admit to high-risk behaviors).
5. Explore the patient’s potential reactions to a positive result (e.g. “I’ll kill myself if I’m positive”). Take appropriate necessary steps to
intervene in a potentially catastrophic reaction.
6. Explore past reactions to severe stresses.
7. Discuss the confidentiality issues relevant to the testing situation (e.g., is it an anonymous or nonanonymous setting?). Inform the
patient of other possible testing options where the counseling and testing can be done completely anonymously (e.g., where the
result is not made a permanent part of a hospital chart). Discuss who has access to the test results.
8. Discuss with the patient how being seropositive can potentially affect social status (e.g., health and life insurance coverage,
employment, housing).
9. Explore high-risk behaviors and recommend risk-reducing interventions.
10. Document discussions in chart.
11. Allow the patient time to ask questions.
Reprinted with permission from Rosse RB, Giese AA, Deutsch SI, Morihisa JM. Laboratory and Diagnostic Testing in Psychiatry. Washington, DC: American Psychiatric Press; 1989:55.
  
Such a set of presenting complaints, especially in younger in- dividuals who may be struggling to accept their seropositive sta- tus, may lead the clinician to conclude that anxiety, depression, or hypochondriasis is responsible. Although affective features are occasionally the best explanation for such complaints, that is not generally the case. Rather, comprehensive neuropsycho- logical testing may reveal that the individual does indeed have difficulties with speeded information processing, divided atten- tion, and sustained effortful processing as well as deficiencies in learning and recalling new information.
Some individuals with mild neurocognitive disorder also have difficulties with tasks involving problem solving and abstract reasoning, and there may also be slowing of simple motor per- formance (e.g., speed of finger tapping). Verbal skills are less affected, although there may be some decrement in fluency (e.g., quickly reciting as many animals as possible or as many words beginning with a particular letter as possible).
These neuropsychological findings, which emphasize atten- tional problems, slowing of information processing, and deficien- cies in learning, are reminiscent of neuropsychological patterns seen in patients with so-called subcortical dementias (e.g., Hunt- ington’s disease and Parkinson’s disease). Fine-grained analysis of memory breakdown in HIV-infected persons also confirms a subcortical pattern. For example, persons with HIV-associated cognitive disorders have difficulty recalling words from a list but do not make intrusion errors (i.e., confabulatory responses) the way patients with cortical dementias (e.g., dementia of the Alzheimer’s type) tend to do. Neuropsychological features that suggest selective involvement of subcortical structures are con- sistent with neuropathological findings. It is important to stress that these mild neurocognitive deficits occur independently of depression, anxiety, and other non-HIV sources of cognitive deficit.
11.13. The answer is A
Antisocial personality disorder is a risk factor for HIV infection. Personality disorders represent extremes of normal personality characteristics and are disabling conditions. Prevalence rates of
personality disorders among HIV-infected (19 to 36 percent) and HIV at-risk (15 to 20 percent) individuals are high and signifi- cantly exceed rates found in the general population (10 percent). Antisocial personality disorder is the most common. Individuals with personality disorder, particularly antisocial personality disorder, have high rates of substance abuse and are more likely to inject drugs and share needles compared with those without an Axis II diagnosis. Approximately half of drug abusers meet criteria for a diagnosis of antisocial personality disorder. Indi- viduals with antisocial personality disorder are also more likely to have higher numbers of lifetime sexual partners, engage in un- protected anal sex, and contract sexually transmitted infections compared to individuals without antisocial personality disorder.
11.14. The answer is E
HIV encephalopathy is a subacute encephalitis that results in a progressive subcortical dementia without focal neurological signs. The major differentiating feature between subcortical de- mentia and cortical dementia is the absence of classical cortical symptoms (e.g., aphasia) until late in the illness. Patients with HIV encephalitis or their friends usually notice subtle mood and personality changes, problems with memory and concentration, and some psychomotor slowing. The presence of motor symp- toms may also suggest a diagnosis of HIV encephalopathy. Motor symptoms associated with subcortical dementia include hyper- reflexia, spastic or ataxic gait, paraparesis, and increased muscle tone.
11.15. The answer is C
Brain biopsy is required for confirmation of the diagnosis of cen- tral nervous system (CNS) lymphoma. Lymphoma is the most common neoplasm seen in AIDS patients, affecting between 0.6 to 3 percent. AIDS is the most common condition associated with primary CNS lymphoma. The patient is generally afebrile and may develop a signal lesion with focal neurological signs or small, multifocal lesions most commonly presenting with mental status change. Seizures present in about 15 percent of patients. CNS lymphoma is at times misdiagnosed as
toxoplasmosis, HIV dementia, or other encephalopathy. CT scan of the brain may be normal or show multiple hypodense or patchy, nodular-enhancing lesions. MRI generally shows en- hanced lesions that may be difficult to differentiate from CNS toxoplasmosis, but thallium single photon emission computed tomography scanning may help to differentiate the two disor- ders and is 90 percent sensitive and specific to lymphoma. Lym- phoma may respond in part to radiation therapy and steroids. Chemotherapy is generally adjunctive for lymphoma. Although CNS lymphoma had a grim prognosis with an average survival of 3 to 5 months before the advent of highly active antiretroviral therapy (HAART), the prognosis now depends on the HAART response, with considerable improvement possible in patients who respond to HAART.
11.16. The answer is C
Apathy is a common early symptom of HIV-associated demen- tia, often causing a noticeable withdrawal by the patient from social activity. A frank depressive syndrome also commonly de- velops, typically with irritable mood and anhedonia instead of sadness and crying spells. Sleep disturbances are common, as is weight loss. Restlessness and anxiety may be complicating factors. Psychosis develops in a significant number of patients, typically with paranoid ideas and hallucinations. In about 5 to 8 percent of patients, a syndrome known as AIDS mania develops. Overall, HIV-associated dementia is rapidly progressive, usually ending in death within 2 years.
Answers 11.17–11.21 11.17. The answer is A 11.18. The answer is B 11.19. The answer is C 11.20. The answer is A
Antiretroviral Agents
Generic Name Trade Name
Nucleoside reverse transcriptase inhibitors
Usual Abbreviation
AZT or ZDV ddI
ddC
d4T
3TC
11.
Neuropsychiatric Aspects of HIV Infection and AIDS 103 Table 11.2
  
Zidovudine Didanosine Zalcitabine Stavudine Lamivudine Abacavir
Retrovir Videx Hivid Zerit Epivir Ziagen
Nonnucleoside reverse transcriptase inhibitors
Nevirapine Delavirdine Efavirenz
Protease inhibitors Saquinavir Ritonavir Indinavir Nelfinavir
11.21. The answer is A
Viramune Rescriptor Sustiva
Invirase Norvir Crixivan Viracept

A growing list of agents that act at different points in viral repli- cation has raised for the first time the hope that HIV might be permanently suppressed or actually eradicated from the body. The reverse transcriptase inhibitors are further subdivided into the nucleoside reverse transcriptase inhibitor group and the non- nucleoside reverse transcriptase inhibitors. In addition to the new nucleoside reverse transcriptase inhibitors, nonnucleoside reverse transcriptase inhibitors, and protease inhibitors, other classes of drugs are under investigation. These include agents that interfere with HIV cell binding and fusion (e.g., enfuvirtide [Fuzeon]) or the action of HIV integrase and certain HIV genes such as gag, among others. Table 11.2 lists some of the available agents in these three categories.

12
Substance-Related Disorders

Substance disorders are complex psychiatric conditions that gov- ern both internally perceived mental states, such as mood, and externally observable activities, such as behavior. As with most other psychiatric disorders, both biological factors and environ- mental circumstance govern a person’s involvement in the dis- order. Substances can cause neuropsychiatric symptoms indis- tinguishable from those of common psychiatric disorders with no known causes (e.g., schizophrenia and mood disorders), and thus primary psychiatric disorders and disorders involving the use of substances are possibly related. It is important to under- stand and appreciate the common features found in these disor- ders and how they differ from other psychiatric illnesses in order to provide adequate care and treatment for patients with these conditions.
Diagnosing a psychiatric disorder in the context of substance abuse can be complicated. A careful and detailed chronolog- ical history of symptom development and its relationship to
substance use is critical to clarifying diagnoses. Although a pri- mary diagnosis may be unclear at times, what does seem clear is that substance abuse worsens the course, prognosis, and pre- sentation of any preexisting psychiatric disorder. A patient with schizophrenia who abuses crack cocaine or a depressed patient abusing cocaine or benzodiazepines will undoubtedly be more impaired than a patient who is not. In fact, most experienced clin- icians will agree that effectively treating any psychiatric disorder in the context of ongoing substance abuse is not possible.
Clinicians need to be clear about the definitions of many terms relating to substance use, including addiction, dependence, abuse, tolerance, cross-tolerance, intoxication, and withdrawal. Each substance-related disorder also has its own definition, epi- demiology, and clinical features, and skilled clinicians must be knowledgeable about each one.
Students should study the questions and answers below for a useful review of these disorders.
Students should know each of the terms below.
AA
abuse
addiction Al-Anon
alcohol delirium alcohol psychotic
disorder
alcohol withdrawal amotivational syndrome anabolic steroids arylcyclohexylamines belladonna alkaloids binge drinking blackouts
caffeine
cocaine delirium cocaine-induced
psychotic disorder cocaine intoxication and
withdrawal
codependence cross-tolerance
DEA
dispositional tolerance disulfiram DSM-IV-TR course
modifiers DTS
dual diagnosis
fetal alcohol syndrome flashback
freebase
hallucinogen hallucinogen persisting
perception disorder idiosyncratic alcohol
intoxication illicit drug use inhalant intoxication ketamine
Korsakoff’s and Wernicke’s syndromes
LAMM
LSD
MDMA
methadone withdrawal misuse MPTP-induced
parkinsonism nicotine receptor NIDA
nitrous oxide opiate
opioid antagonists opioid intoxication and
withdrawal PCP
persisting amnestic disorder
persisting dementia physical dependence psychedelics psychoactive psychological
dependence RFLP
“roid” rage sedative-hypnotic
anxiolytic STP alcohol
intoxication; blood
levels
substance dependence
and abuse THC
tolerance
type I and II alcoholism volatile hydrocarbons withdrawal
HELPFUL HINTS
104
▲▲▲▲▲ ▲▲▲ ▲ ▲ ▲▲▲▲▲
▲ ▲▲▲▲▲▲ ▲▲▲▲▲▲ ▲▲
▲▲▲▲▲▲ ▲▲▲▲▲▲▲ ▲ ▲▲▲
▲▲▲▲▲▲ ▲▲▲▲▲▲▲▲▲▲ ▲
QUESTIONS
Directions
Each of the questions or incomplete statements below is followed by five suggested responses or completions. Select the one that is best in each case.
. 12.1. The Diagnostic and Statistical Manual of Mental Disor- ders (DSM-IV-TR) states specifically that the diagnosis of dependence can be applied to every class of substances except
A. anabolic steroids B. caffeine
C. nicotine
D. nitrous oxide
E. none of the above
. 12.2. FT is a 73-year old successful emeritus professor at a small liberal arts college in the Midwest. He was in rel- ative good health when he entered the hospital for an elective hernia repair. The brief history contained no de- tailed notes of his drinking pattern and made no mention of his γ -glutamyltransferase value of 55 U/L along with the mean corpuscular volume (MCV) of 93.5 μm3 . Eight hours postsurgery, the nursing staff noted an acute in- crease in the pulse rate to 110 beats/min, an increased in blood pressure to 150/100 mm Hg, prominent diaphore- sis, and a tremor to both hands, after which the patient demonstrated a brief but intense grand mal convulsion. He awoke agitated and disoriented to time, place, and person. A reevaluation of the history and an interview with the patient’s wife documented his alcohol depen- dence with a consumption of approximately six standard drinks per night. Over the following 4 days, the patient’s autonomic nervous system dysfunction decreased as his cognitive impairment disappeared. Which condition is the best classification for Mr. FT in the above case?
A. Alcohol withdrawal delirium
B. Wernicke’s encephalopathy
C. Korsakoff’s syndrome
D. Alcohol-induced anxiety disorder
E. Alcohol-induced persisting dementia
. 12.3. Which of the following statements regarding cancer and alcohol use is correct?
A. Cancer may be caused by the immunosuppressive effects of ethanol.
B. Canceristhesecondleadingcauseofprematuredeath in alcoholics.
C. Increases in breast cancer have been noted with just two drinks per day.
D. Alcohol can be directly linked to cancers of mucous membranes.
E. All of the above
. 12.4. Which of the following statements regarding alcohol’s effect on sleep is false?
A. Heavy drinkers often awaken at night and have diffi- culty going back to sleep.
B. Alcoholics tend to have more dreams later in the night.
C. Alcohol increases rapid eye movement (REM) sleep. D. Alcohol can significantly impair normal sleep pat-
terns.
E. Alcohol use tends to inhibit stage 4 sleep.
12.5. Acute PCP intoxication is not treated with
A. diazepam (Valium)
B. cranberry juice
C. phentolamine (Regitine) D. phenothiazines
E. all of the above
12.6. In distinguishing schizophrenia from amphetamine- induced toxic psychosis, the presence of which of the following is most helpful?
A. tactile or visual hallucinations B. paranoid delusions
C. intact orientation
D. clear consciousness
E. auditory hallucinations
12.7. The image in Figure 12.1 shows decreased dopamine type 2 (D2) receptor binding in the striatum in drug users compared with normal control subjects. Which of the following types of substance withdrawal is this pattern not typical of?
A. Alcohol
B. Cocaine
C. Methamphetamine D. Opioids
E. All of the above
12.8. Whichofthefollowingisnotatherapeuticindicationfor use of anabolic-androgenic steroids?
A. Anemia
B. Hereditary angioedema C. Hyperthyroidism
D. Male hypogonadism
E. Osteoporosis
12.9. Ms.Eisa32-year-oldsinglewhitewomanemployedfull time at a local factory. She is a smoker and occasionally has flares of asthma. She typically drinks four to five mugs of coffee each day and prefers to drink it without cream, milk, or sugar. Physicians had recommended she cut down or stop her coffee use because of complaints of mild indigestion, and she abruptly stopped her caffeine intake as a result of these recommendations.
Which of the following statements regarding caffeine is true?
A. Caffeine is slowly absorbed and metabolized by the kidneys.
12. Substance-Related Disorders 105
106 12.
Substance-Related Disorders

FIGURE 12.1
B. Caffeine’s effects on the body include bronchocon- striction.
C. Caffeinemetabolismismarkedlyincreasedattheend of pregnancy.
D. Caffeine increases the metabolism of the antipsy- chotic clozapine.
E. The rate of caffeine elimination is increased by smoking.
. 12.10. The above patient would most likely experience all of the following because of her abruptly stopping caffeine intake except
A. decreased concentration B. headache
C. insomnia
D. irritability
E. muscle aches
. 12.11. Which of the following is contraindicated for the treat- ment of acute disulfiram (Antabuse) overdose?
A. Activated charcoal B. Decontamination C. Gastric lavage
D. Hemodialysis
E. Syrup of ipecac
. 12.12. Minor signs and symptoms of the benzodiazepine dis- continuation syndrome commonly include
A. hyperpyrexia
B. grand mal seizures C. psychosis
D. nightmares
E. death
. 12.13. Which of the following is not a therapeutic effect for which cannabinoids are commonly used?
A. Weight loss
B. Reduced muscle spasticity
C. Relief of nausea and vomiting D. Decreased intraocular pressure E. Appetite stimulant
. 12.14. Amphetamines and cocaine are similar in
A. their metabolic pathways
B. the induction of paranoia and production of major
cardiovascular toxicities
C. their mechanisms of action at the cellular level
D. their duration of action
E. all of the above
. 12.15. Inhalant use most often correlates with which of the following comorbid conditions?
A. Borderline personality disorder B. Conduct disorder
C. Major depression
D. Manic episode
E. Schizophrenia
. 12.16. The single photon emission computed tomography (SPECT) image in Figure 12.2 shows multifocal areas of hypoperfusion in a patient with chronic substance abuse. The patient’s ischemic cerebrovascular disorder is most likely precipitated by which of the following substances?
A. Barbiturates B. Cannabis C. Cocaine
D. Heroin
E. Phencyclidine (PCP)
. 12.17. Which of the following drugs is an opioid antagonist?
A. Naloxone
B. Naltrexone
C. Nalorphine
D. Apomorphine E. All of the above
. 12.18. Whichofthefollowingatypicalsubstancesisalsoknown to produce symptoms of intoxication?
A. Benadryl B. Betel nut

FIGURE 12.2
C. Catnip
D. Kava
E. All of the above
. 12.19. Youarecalledforaconsultona42-year-oldwomanwith alcohol dependence who is complaining of persisting se- vere depressive symptoms despite 5 days of abstinence. In the initial stage of the interview, she noted that she had “always been depressed” and believed that she “drank to cope with the depression.” Her current complaint in- cludes a prominent sadness that has persisted for several weeks, difficulties concentrating, initial and terminal in- somnia, and feelings of hopelessness and guilt.
What is the most appropriate next step to distinguish between alcohol-induced depression and an independent major depressive episode?
A. Chronological history
B. Trial of electroconvulsive therapy (ECT) C. Proton emission tomography (PET) scan D. Antidepressant treatment
E. Intensive psychotherapy
. 12.20. Cocaine
A. competitively blocks dopamine reuptake by the dopamine transporter.
B. does not lead to physiological dependence.
C. does not induce psychotic disorders.
D. has been used by 40 percent of the United States pop-
ulation since 1991
E. is no longer used as a local anesthetic
12. Substance-Related Disorders 107
. 12.21. Whichofthefollowingisnotacomponentofacutenico-
tine intoxication?
A. Tachycardia
B. Visual hallucinations C. Bizarre dreams
D. Lability of mood
E. Cardiac arrhythmias
. 12.22. Laboratorytestsusefulinmakingthediagnosisofalcohol abuse or dependence include
A. triglycerides
B. γ -glutamyltransferase (GGT) C. MCV
D. all of the above
E. none of the above
. 12.23. Mouth ulceration is associated with which of the follow- ing types of withdrawal?
A. Alcohol
B. Benzodiazepines C. Cocaine
D. Nicotine
E. Opioids
. 12.24. A 35-year-old male dentist with no history of other sub- stance problems complains of problems with nitrous oxide abuse for 10 years. This had begun as experimenta- tion with what he had considered a harmless substance. However, his rate of use increased over several years, eventually becoming almost daily for months at a time. He felt a craving before sessions of use. Then, using the substance while alone in his office, he immediately felt numbness, a change in his temperature and heart rate, and an alleviation of depressed feelings. “Things would go through my mind. Time was erased,” he said. He some- times fell asleep. Sessions might last a few minutes or up to 8 hours; they ended when the craving and euphoria ended. He had often tried to stop or cut down, sometimes consulting a professional about the problem.
Which category best fits the condition of the above patient?
A. Hallucinogen-related disorder not otherwise speci- fied (NOS)
B. Opioid-related disorder NOS
C. Inhalant-related disorder NOS
D. Phencyclidine-related disorder NOS
E. Anabolic steroid–related disorder NOS
. 12.25. A 31-year-old woman is hospitalized after sustaining multiple rib fractures from a motor vehicle accident. Forty-eight hours later, she complains to the hospital’s patient relations department that the nurses are constantly hiding in the bathroom of her hospital room and making fun of her. After extensively speaking with the nurses, the hospital administration finds no evidence that any such behavior has occurred. Her vital signs are stable.
108
12. Substance-Related Disorders
The patient has no psychiatric history, denies smoking or recreational drugs, and has four standard drinks daily. On the sixth day of hospitalization, the patient is no longer agitated and does not have the same complaints. What is the most likely diagnosis?
A. Paranoid personality disorder B. Brief psychotic disorder
C. Adjustment disorder
D. Alcoholic hallucinosis
E. Posttraumatic stress disorder
Directions
Each set of lettered headings below is followed by a list of num- bered words or phrases. For each numbered word or phrase, select
A. if the item is associated with A only
B. if the item is associated with B only
C. if the item is associated with both A and B D. if the item is associated with neither A nor B
Questions 12.26–12.33
A. Barbiturates
B. Benzodiazepines
. 12.26. Pose a lower risk when taken in overdose.
. 12.27. Delirium may occur with withdrawal.
. 12.28. Withdrawal severity is greater with high scores on
the Minnesota Multiphase Personality Inventory 2
(MMPI-2).
. 12.29. Has a risk of pharmacodynamic tolerance.
. 12.30. Withdrawal in a hospital is necessary to prevent death.
. 12.31. Have symptoms of withdrawal that usually appear within
3 days.
. 12.32. Cause REM–sleep suppression.
. 12.33. Are clinically used as muscle relaxants.
Questions 12.34–12.38
A. Alcohol dehydrogenase B. Aldehyde dehydrogenase
. 12.34. Converts acetaldehyde into acetic acid
. 12.35. Involved in alcohol metabolism
. 12.36. Converts alcohol into acetaldehyde
. 12.37. Inhibited by disulfiram (Antabuse)
. 12.38. Decreased in Asian people
Directions
Each group of questions below consists of lettered headings fol- lowed by a list of numbered phrases or statements. For each numbered phrase or statement, select the one lettered heading that is most associated with it. Each lettered heading may be selected once, more than once, or not at all.
Questions 12.39–12.48
A. Intoxication
B. Abuse
C. Physical dependence D. Withdrawal
E. Codependence
F. Misuse G. Addiction H. Tolerance
I. Cross-tolerance J. Neuroadaptation
12.39. Increasingly larger doses of a drug are necessary to obtain the effects of the original dose.
12.40. Neurochemical changes in the body caused by repeated administration of a drug.
12.41. The behavior of family members affected by the sub- stance use of the patient.
12.42. The reduction or cessation of a substance after prolonged regular use or abuse.
12.43. The use of a drug or substance beyond approved social or medical patterns.
12.44. A reversible nondependent experience with a substance. 12.45. A drug prescribed by a physician is not used as it was
prescribed.
12.46. One drug is substituted for another drug that produces
the same psychological effect.
12.47. The repeated and increased use of a substance. 12.48. Can be a feature of problematic use of a substance.
Questions 12.49–12.53
A. Amphetamines B. Cocaine
C. LSD
D. Marijuana
E. PCP
12.49. Horizontal and vertical nystagmus 12.50. Injected conjunctiva
12.51. Atrophic nasal mucosa
12.52. Amotivational syndrome
12.53. Colorful hallucinations
ANSWERS
12.1. The answer is B
The DSM-IV-TR section dealing with substance dependence and substance abuse presents descriptions of the clinical phenomena associated with the use of 11 designated classes of pharmaco- logical agents: alcohol; amphetamines or similarly acting agents; caffeine; cannabis; cocaine; hallucinogens; inhalants; nicotine; opioids; PCP or similar agents; and sedatives, hypnotics, and anxiolytics. A residual twelfth category includes a variety of agents, such as anabolic steroids and nitrous oxide, that are not in the 11 designated classes.
In addition to requiring the clustering of three criteria in a 12-month period, the DSM-IV-TR includes a few other qualifi- cations. It states specifically that the diagnosis of dependence can be applied to every class of substances except caffeine. That point is admittedly controversial, and some researchers believe, on the basis of the same DSM-IV-TR generic criteria, that caffeine pro- duces a distinct form of dependence, although it is relatively benign for most persons.
12.2. The answer is A
FT experienced alcohol withdrawal delirium postsurgery. Al- cohol intoxication delirium and alcohol withdrawal delirium occur when intoxication or withdrawal is accompanied by se- vere cognitive symptoms and occur in fewer than 5 percent of intoxications or withdrawals. When this agitated confu- sion is associated with tactile or visual hallucinations, the di- agnosis of alcohol withdrawal delirium (also called delirium tremens) can be made. During withdrawal, some alcoholic peo- ple show one or several grand mal convulsions, sometimes called rum fits.
Wernicke’s encephalopathy and Korsakoff’s syndrome are both subdivisions of alcohol-induced persisting amnestic disorder, which is the result of a relatively severe deficiency of vi- tamin B1, thiamine. Wernicke’s encephalopathy, also called alcoholic encephalopathy, is an acute neurological disorder characterized by ataxia (affecting primarily the gait), vestibu- lar dysfunction, confusion, and nystagmus. Wernicke’s en- cephalopathy may clear spontaneously within a few days or weeks or may progress into Korsakoff ’s syndrome. Korsakoff ’s syndrome is characterized by impaired mental syndrome (es- pecially recent memory) and anterograde amnesia in an alert and responsive patient. The patient may or may not have the symptom of confabulation. The syndrome is permanent in at least partial form in about 50 to 70 percent of affected patients, and 30 percent of patients may recover fully. Patients appear to respond to 50 to 100 mg of oral thiamine a day, usually admin- istered for many months.
Alcohol-induced persisting dementia is characterized by het- erogeneous long-term cognitive problems that develop in the course of alcoholism. Global decreases in intellectual function- ing, cognitive abilities, and memory are observed. Brain func- tioning tends to improve with abstinence, but perhaps half of all affected patients have long-term and even permanent disabilities in memory and thinking.
Anxiety symptoms fulfilling the diagnostic criteria for alcohol-induced anxiety disorder are also common in the context of acute and protracted alcohol withdrawal. Almost 80 percent of alcoholic people report panic attacks during at least one acute withdrawal episode. Similarly, during the first 4 weeks or so of abstinence, people with severe alcohol problems have symptoms resembling social phobia.
12.3. The answer is E (all)
High rates of most cancers are seen in alcoholic people, es- pecially cancers of the head, neck, esophagus, stomach, liver, colon, lungs, and breast tissue. An enhanced risk for breast ma- lignancies might be seen with as few as two drinks per day, especially in women with family histories of this disease. The as- sociation with cancer probably reflects alcohol-related immune
system suppression and the direct effects of ethanol on mucous membranes. The heightened rates of malignant tumors in alco- holic people remain significant even when the possible effects of smoking and poor nutrition are considered, and this is the second leading cause of premature death in alcohol-dependent men and women.
12.4. The answer is C
Alcohol intoxication can help a person fall asleep more quickly, but if the intake in an evening is more than one or two drinks, the sleep pattern can be significantly impaired. Most heavy drinkers awaken after several hours and can have prob- lems falling back asleep. Alcohol also tends to depress rapid eye movements (REMs) and inhibit stage 4 sleep and thus is likely to be associated with frequent alternations between sleep stages (sleep fragmentation) and with more dreams late in the night as the blood alcohol level decreases. Exaggerated forms of similar problems are seen in alcoholics in whom sleep stages might not return to normal for 3 or more months after abstinence.
12.5. The answer is D
Phenothiazines are not used in the treatment of acute PCP intox- ication because they have anticholinergic effects that may po- tentiate the adverse effects of PCP, such as seizures. Diazepam (Valium) is useful in reducing agitation. If agitation is severe, however, the antipsychotic haloperidol (Haldol) may have to be used. Cranberry juice is used to acidify the urine and to promote the elimination of the drug; ammonium chloride or ascorbic acid serves the same purpose. Phentolamine (Regitine) is a hypoten- sive agent that may be needed to deal with severe hypertensive crises produced by PCP.
12.6. The answer is A
Amphetamine-induced toxic psychosis can be exceedingly dif- ficult to differentiate from schizophrenia and other psychotic disorders characterized by hallucinations or delusions. Paranoid delusions occur in about 80 percent of patients and hallucinations in 60 to 70 percent. Consciousness is clear, and disorientation is uncommon. The presence of vivid visual or tactile hallucina- tions should raise suspicion of a drug-induced disorder. In areas where and populations in whom amphetamine use is common, it may be necessary to provide only a provisional diagnosis until the patient can be observed and drug test results are obtained. Even then, there may be difficulties because in some urban ar- eas, a high percentage of persons with established diagnoses of schizophrenia also use amphetamines or cocaine. Typically, symptoms of amphetamine psychosis remit within 1 week, but in a small proportion of patients, psychosis may last for more than 1 month.
12.7. The answer is D
Opioid withdrawal has not been associated with decreased dopamine binding in the striatum. It has been found repeatedly that there are perfusion deficits in brains of cocaine-dependent subjects recently withdrawn from cocaine. This deficit is prob- ably not related to tolerance or withdrawal, but several other
12. Substance-Related Disorders 109
110 12. Substance-Related Disorders
findings probably are. Many (but not all) studies using positron emission tomography and single photon emission computed to- mography to examine the brains of cocaine-dependent subjects have found a decreased number of dopamine transporters in the striatum, a finding consistent with postmortem studies. At 1 to 4 weeks and at 3 to 4 months postwithdrawal, cocaine abusers have lower metabolic rates in the frontal cortex that correlate with symptoms of depression and decreased availability of D2 receptors. These findings were similar in studies of patients after alcohol and amphetamine withdrawal.
12.8. The answer is C
Anabolic-androgenic steroids (AAS) have not been used in the treatment of hyperthyroidism. The anabolic steroids are a fam- ily of drugs comprising the natural male hormone testosterone and a group of more than 50 synthetic analogs of testosterone, synthesized over the past 60 years. These drugs all exhibit various degrees of anabolic (muscle-building) and androgenic (masculinizing) effects. Thus, they should more correctly be called AAS. Note that it is important not to confuse the AAS (testosterone-like hormones) with corticosteroids (cortisol-like hormones such as hydrocortisone and prednisone). Corticos- teroids have no muscle-building properties and hence little abuse potential. AAS, by contrast, have only limited legitimate medical applications. However, AAS are widely used illicitly, especially by boys and young men seeking to gain increased muscle mass and strength either for athletic purposes or simply to improve personal appearance.
AAS are primarily indicated for testosterone deficiency (male hypogonadism), hereditary angioedema (a congenital skin disorder), and some uncommon forms of anemia caused by bone marrow or renal failure. In women, they are given, although not as first-choice agents, for metastatic breast can- cer, osteoporosis, endometriosis, and adjunctive treatment of menopausal symptoms. In men, they have also been used ex- perimentally as a male contraceptive and for treating major de- pressive disorder and sexual disorders in eugonadal men. Re- cently, they have been used to treat wasting syndromes associated with AIDS. Controlled studies have also suggested that testos- terone has antidepressant effects in some HIV-infected men with major depressive disorder, and it is also a supplementary (aug- mentation) treatment in some depressed men with low endoge- nous testosterone levels who are refractory to conventional anti- depressants.
12.9. The answer is E
Caffeine is a methylxanthine, as are theobromine (found in chocolate) and theophylline (typically used in the treatment of asthma). Caffeine is well absorbed from the gastrointestinal tract, with peak plasma concentrations typically occurring within 1 hour after ingestion. Caffeine is readily distributed throughout the body and is metabolized by the liver. The half-life of caf- feine is approximately 5 hours, with large individual differences. The rate of caffeine elimination is increased by smoking, oral contraceptives, steroids, cimetidine, and fluvoxamine (Luvox). Caffeine inhibits the metabolism of the antipsychotic clozapine (Clozaril) and the bronchodilator theophylline (Theo-Dur) to an
extent that might be clinically significant. Caffeine metabolism is markedly slowed at the end of pregnancy.
Caffeine exerts effects throughout the body, including bron- chodilation (hence the therapeutic application of caffeine and theophylline in the treatment of asthma); modest increases in blood pressure (which are reduced in caffeine-tolerant individ- uals); increased production of urine; increases in gastric acid secretion; and increases in plasma epinephrine, norepinephrine, renin, and free fatty acids. Centrally, caffeine affects turnover or levels of various neurotransmitters, and it functions as a central nervous system stimulant.
12.10. The answer is C
Insomnia is a symptom of caffeine intoxication, not withdrawal. Even though the DSM-IV-TR does not include a diagnosis of caf- feine withdrawal, several well-controlled studies indicate that caffeine withdrawal is a real phenomenon. The appearance of withdrawal symptoms reflects the tolerance and physiological dependence that develop with continued caffeine use. Several epidemiological studies have reported symptoms of caffeine withdrawal in 50 to 75 percent of all caffeine users studied. The most common symptoms are headache and fatigue; other symp- toms include anxiety, irritability, mild depressive symptoms, im- paired psychomotor performance, nausea, vomiting, craving for caffeine, and muscle aches and stiffness. The number and sever- ity of the withdrawal symptoms are correlated with the amount of caffeine ingested and the abruptness of the withdrawal. Caf- feine withdrawal symptoms have their onset at 12 to 24 hours after the last dose; the symptoms peak in 24 to 48 hours and resolve within 1 week.
12.11. The answer is E
Disulfiram (Antabuse) is used to ensure abstinence in the treat- ment of alcohol dependence. Its main effect is to produce a rapid and violently unpleasant reaction in a person who ingests even a small amount of alcohol while taking disulfiram. Disulfi- ram is an aldehyde dehydrogenase inhibitor that interferes with the metabolism of alcohol and produces a marked increase in blood acetaldehyde levels because it prevents the conversion of acetaldehyde to acetyl coenzyme A. The accumulation of ac- etaldehyde, which may be 10-fold more than normal, leads to many unpleasant effects, including nausea, throbbing headache, vomiting, hypertension, flushing, sweating, thirst, dyspnea, chest pain, vertigo, and blurred vision.
No specific antidote is available for the treatment of acute disulfiram overdose. Using ipecac syrup is contraindicated because this syrup contains ethanol and could precipitate a disulfiram-ethanol reaction. Emesis is also not recommended because this may delay the administration of activated charcoal, worsen nausea and vomiting, and increase the risk of pulmonary aspiration by an unprotected airway if seizures and coma sud- denly supersede. The use of activated charcoal in multiple doses may be beneficial because it increases the rate of elimination of disulfiram. Gastric lavage may be useful in cases of massive alcohol ingestion. Neither decontamination nor hemodialysis is likely to be beneficial after the reaction begins; however, they are not contraindicated.
Table 12.1
Signs and Symptoms of the Benzodiazepine Discontinuation Syndrome
The following signs and symptoms may be seen when benzodiazepine therapy is discontinued; they reflect the return of the original anxiety symptoms (recurrence), worsening of the original anxiety symptoms (rebound), or emergence of new symptoms (true withdrawal)
Disturbances of mood and cognition:
Anxiety, apprehension, dysphoria, pessimism, irritability,
obsessive rumination, paranoid ideation Disturbances of sleep:
Insomnia, altered sleep–wake cycle, daytime drowsiness Physical signs and symptoms:
Tachycardia, elevated blood pressure, hyperreflexia, muscle tension, agitation—motor restlessness, tremor, myoclonus, muscle and joint pain, nausea, coryza, diaphoresis, ataxia, tinnitus, grand mal seizures
Perceptual disturbances:
Hyperacusis, depersonalization, blurred vision, illusions,
hallucinations
12.12. The answer is D
Studies in the early 1960s by Leo Hollister established that abrupt discontinuation of high doses of chlordiazepoxide or diazepam could lead to a withdrawal syndrome.
The American Psychiatric Association’s Task Force Report on Benzodiazepine Dependence, Toxicity, and Abuse defined withdrawal as a true abstinence syndrome consisting of “new signs and symptoms and worsening of preexisting symptoms following drug discontinuance that were not part of the disorder for which the drugs were originally prescribed.”
The signs and symptoms of the benzodiazepine discontinu- ation syndrome (Table 12.1) have been classified as major or minor, similar to those of the alcohol withdrawal syndrome. According to that classification, minor symptoms include anx- iety, insomnia, and nightmares. Major symptoms (which are extremely rare) include grand mal seizures, psychosis, hyper- pyrexia, and death.
The discontinuance syndrome may also be divided into symp- toms of rebound, recurrence, and withdrawal. Rebound symp- toms are symptoms for which the benzodiazepine was originally prescribed that return in a more severe form than they had before treatment. They have a rapid onset after termination of therapy and a brief duration. Recurrence refers to return of the original symptoms at or below their original intensity.
The temporal sequence of symptom development is not well established, but upon the abrupt cessation of benzodiazepines with short elimination half-lives, symptoms may appear within 24 hours and peak at 48 hours. Symptoms arising from abrupt discontinuation of benzodiazepines with long half-lives may not peak until 2 weeks later.
12.13. The answer is A
Tetrahydrocannabinol (THC) has been shown to stimulate ap- petite and assist with weight gain (not weight loss) in AIDS pa- tients in short-term trials. When cannabinoids and cannabis are
advocated for medical uses, it is primarily to relieve symptoms rather than to cure underlying diseases. The conditions for which cannabis is most commonly advocated are for symptomatic relief of nausea and vomiting caused by cancer chemother- apy, appetite loss in AIDS, and muscle spasticity and chronic pain in neurological disorders. Cannabis and THC have also been documented to reduce intraocular pressure by 25 per- cent, a therapeutic effect that may be useful in the treatment of glaucoma.
12.14. The answer is B
The reinforcing and toxic effects of amphetamines and amphetamine-like drugs play an important role in the genesis of amphetamine dependence and other amphetamine-related dis- orders. Amphetamines produce subjective effects that are very similar, if not identical, to those produced by cocaine. Both categories of drugs can produce a sense of alertness, eupho- ria, and well-being. Performance impaired by fatigue is usually improved. There may be decreased hunger and decreased need for sleep. Patterns of toxicity are also similar but not identi- cal. Both the amphetamines and cocaine can induce paranoia, suspiciousness, and overt psychosis that can be difficult to dis- tinguish from paranoid-type schizophrenia; both can produce major cardiovascular toxicities. However, the amphetamines and cocaine differ distinctly in their mechanisms of action at the cellular level, their duration of action, and their metabolic pathways.
Although amphetamines inhibit reuptake of monoamines to a small degree, their major action is the release of monoamines from storage sites in axon terminals, which in turn increases monoamine concentrations in the synaptic cleft. The release of dopamine in the nucleus accumbens and related structures is thought to account for their reinforcing and mood-elevating ef- fects; the release of norepinephrine is probably responsible for the cardiovascular effects. In contrast to cocaine, which binds to neurotransporters and inhibits reuptake of the neurotransmitters released into the synapse, amphetamine-like drugs are taken into the neurons, where they are transported into the neurotransmitter storage vesicles.
12.15. The answer is B
People with adolescent conduct disorder or adult antisocial per- sonality disorder are prone to taking extreme risks, and many inhalant users have these disorders. Several studies suggest an association of inhalant use and conduct problems. Among youths in grades 7 through 12, inhalant users (compared with others who used no drugs or who used only cannabis or alcohol) had many characteristics suggesting conduct disorder. They accepted cheating more readily; admitted to more stealing; perceived less objection to drug use from their families; liked school less; and reported more sadness, tension, anger, and a feeling of being blamed by others. In addition, school surveys showed that sol- vent users were more likely to be involved with other drugs. Similarly, among youths referred to court-mandated education for minor alcohol offenses, those who also had used inhalants reported fewer school honors and more expulsions, truancy, aca- demic failures, criminal offenses, running away, and associations
12. Substance-Related Disorders 111
  
112 12. Substance-Related Disorders
with troubled peers as well as many more drug and alcohol prob-
lems.
12.16. The answer is C
The development of an ischemic cerebrovascular disorder is an adverse effect of cocaine abuse. The most common cerebrovas- cular diseases associated with cocaine use are nonhemorrhagic cerebral infarctions. When hemorrhagic infarctions do occur, they can include subarachnoid hemorrhages. Other adverse ef- fects of cocaine use include seizures, myocardial infarction, and arrhythmias.
12.17. The answer is E (all)
Opioid antagonists block or antagonize the effects of opiates and opioids. Unlike methadone, they do not themselves exert narcotic effects and do not cause dependence. The antagonists include the following drugs: naloxone, (Narcan) which is used in the treat- ment of opiate and opioid overdose because it reverses the ef- fects of narcotics; naltrexone, (ReVia) which is the longest-acting (72 hours) antagonist; and nalorphine, levallorphan (Lorfan), and apomorphine (Apokyn).
12.18. The answer is E (all)
Many atypical substances are capable of producing mild intox- ication. These include catnip, which can produce states simi- lar to those observed with marijuana and which in high doses is reported to result in lysergic acid diethylamide–type percep- tions; betel nut, which is chewed in many cultures to produce a mild euphoria; and kava (a substance derived from the South Pacific pepper plant), which produces sedation, incoordination, weight loss, mild forms of hepatitis, and lung abnormalities. In addition, individuals can develop dependence and impairment through repeated self-administration of over-the-counter and prescription drugs, including cortisol, antiparkinsonian agents that have anticholinergic properties, and antihistamines (e.g., Benadryl).
12.19. The answer is A
In an effort to distinguish between an alcohol-induced mood dis- order and an independent major depressive episode, a timeline- based chronological history should be obtained. This should focus on the age of onset of this patient’s alcohol dependence, periods of abstinence that extended for several months or more since the onset of dependence, and the ages of occurrence of clear major depressive episodes lasting several weeks or more at a time.
Heavy intake of alcohol over several days results in many of the symptoms observed in major depressive disorder, but the intense sadness markedly improves within days to 1 month of abstinence. Eighty percent of alcoholic patients report histories of intense depression, including 30 to 40 percent who were depressed for 2 or more weeks at a time. However, when information from patients and resource people was carefully evaluated, only 5 percent of alcoholic men and 10 percent of alcoholic women ever had depressions that met the criteria for major depressive disorder when not drinking heavily.
Clinical data reveal that when even severe depression devel- ops in alcoholic people, they are likely to improve fairly rapidly
without medications or intensive psychotherapy aimed at the de- pressive symptoms. A recent study of almost 200 alcoholic men found that although 40 percent had severe levels of depression after 1 week of abstinence, these symptoms markedly improved in all but 5 percent after 3 additional weeks of sobriety. At the end of several weeks to 1 month, most alcoholic patients are left with mood swings or intermittent symptoms of sadness that can resemble cyclothymic disorder or dysthymic disorder. Electro- convulsive therapy is indicated only in depression resistant to all other treatments.
12.20. The answer is A
Cocaine competitively blocks dopamine reuptake by the dopamine transporter. This primary pharmacodynamic effect is believed to be related to cocaine’s behavioral effects, includ- ing elation, euphoria, heightened self-esteem, and perceived im- provement on mental and physical tasks. Cocaine does lead to physiological dependence, although cocaine withdrawal is mild compared with the effects of withdrawal from opiates and opi- oids. A psychological dependence on cocaine can develop after a single use because of its potency as a positive reinforcer of behavior. Cocaine-induced psychotic disorders are most com- mon in intravenous users and crack users, not in those who snort cocaine. The National Institute of Drug Abuse reported that cocaine has been used by 12 percent, not 40 percent, of the United States population since 1991. The highest use was in the 18- to 25-year-old age group. Although cocaine use is highest among unemployed individuals, cocaine is also used by highly educated persons in high socioeconomic groups. Co- caine use among men is twice as frequent as cocaine use among women.
Despite its reputation as the most addictive commonly abused substance and one of the most dangerous, cocaine does have some important medical applications. Cocaine is still used as a local anesthetic, especially for eye, nose, and throat surgery, for which its vasoconstrictive effects are helpful.
12.21. The answer is B
Nicotine is a highly toxic alkaloid. Doses of 60 mg in an adult are fatal secondary to respiratory paralysis; doses of 0.5 mg are deliv- ered by smoking an average cigarette. Acute intoxication caused by nicotine is evidenced by dysfunctional behavior or perceptual abnormalities, as well as physical signs that are often attributed to sympathetic activation. The behavioral abnormalities include insomnia, bizarre dreams, labile mood, derealization, and inter- ference with personal functioning. Physically, nicotine intoxi- cation can lead to nausea, vomiting, sweating, tachycardia, and cardiac arrhythmias. Visual hallucinations have not been noted to be an effect of nicotine intoxication.
12.22. The answer is D (all)
Establishing the diagnosis for alcohol abuse or dependence cen- ters on obtaining from the patient and a resource person a history of the patient’s life problems and the possible role played by al- cohol. Up to one-third of all psychiatric patients are likely to have an alcohol problem that either caused or exacerbated the presenting clinical condition.
The process of identification can also be facilitated by a se- ries of blood tests, outlined in Table 12.2. Those state markers of heavy drinking reflect physiological alterations likely to be observed if the patient regularly ingests four or more drinks a day over many days or weeks. One of the most sensitive and specific of the markers is a level of 30 or more units per liter of γ-glutamyltransferase (GGT), an enzyme that aids in the trans- port of amino acids and that is found in most areas of the body. Because this enzyme is likely to return to normal levels after 2 to 4 weeks of abstinence, even 20 percent increases in enzyme levels above those observed after 4 weeks of abstinence can be useful in identifying patients who have returned to drinking af- ter treatment. Equally impressive results have been reported for the measure of a deglycosylated form of the protein transferrin, known as carbohydrate-deficient transferrin (CDT). With a bi- ological half-life of about 16 days, this test can also be useful in monitoring abstinence in alcoholics. It appears that patients not identified by higher GGT values might still have elevations in CDT so that both tests should be used for identification and abstinence-monitoring functions in alcoholics.
The MCV blood test, with perhaps 70 percent sensitivity and specificity, is a state marker when the size of the red blood cell (RBC) is 91 or more cubic micrometers. The 120-day life span of the RBC does not allow the test to be used as an indicator of a return to drinking after about 1 month of abstinence. Other tests that can be helpful in identifying patients who are regularly consuming heavy doses of alcohol include those for high-normal concentrations of uric acid (greater than 6.4 mg/dL, with a range that depends on the sex of the person); mild elevations in the usual liver function tests, including aspartate aminotransferase and alanine aminotransferase with a 2:1 ratio of AST to ALT; and elevated levels of triglycerides or low-density lipoprotein cholesterol.
12.23. The answer is D
The development of mouth ulcers and dry mouth has been noted as a sign of chronic tobacco use and may occur acutely dur- ing withdrawal. The DSM-IV-TR does not have a diagnostic
category for nicotine intoxication, but it does have a diagnostic category for nicotine withdrawal.
Withdrawal symptoms can develop within 2 hours of smok- ing the last cigarette, generally peak in the first 24 to 48 hours, and can last for weeks or months. The common symptoms include an intense craving for nicotine, tension, irritability, dif- ficulty concentrating, drowsiness and paradoxical trouble sleep- ing, decreased heart rate and blood pressure, increased appetite and weight gain, decreased motor performance, and increased muscle tension.
12.24. The answer is C
The patient in this case has a nitrous oxide–related disorder, which is included in the DSM-IV-TR as an inhalant-related disorder not otherwise specified (NOS). Nitrous oxide–related disorders are included in DSM-IV-TR under this category be- cause of differences between nitrous oxide and other inhalants in modes of action and associated problems. However, it is rarely studies separately from other inhalants. Nitrous oxide produces feelings described as “drunk,” “dreamy,” “coasting or spaced out,” and “pleasant bodily sensations.” Nitrous oxide produces higher rates of euphoria but no significant increases in subjec- tive sleepiness, distinguishing it from the central nervous system depressant effects of most inhalants. Coordination and reaction time seem to be intact, but logical reasoning appears to be im- paired. Chronic use may produce diffuse polyneuropathy and myelinopathy with extensive, although sometimes reversible, neurological symptoms mimicking those of vitamin B12–related pernicious anemia.
Hallucinogen-related disorder NOS is diagnosed when a pa- tient with a hallucinogen-related disorder does not meet the di- agnostic criteria for any of the standard hallucinogen-related disorders. The DSM-IV-TR does not have a diagnostic category of hallucinogen withdrawal, but some clinicians anecdotally re- port a syndrome with depression and anxiety after the cessation of frequent hallucinogen use.
Opioid-related disorder NOS is diagnosed when the clinical situation does not fit into the standard opioid-related disorder categories, namely opioid-related disorder with symptoms of delirium, abnormal mood, psychosis, abnormal sleep, and sexual dysfunction.
Phencyclidine-related disorder NOS is appropriate when a patient does not fit into any of the standard diagnoses for phencyclidine-related disorders.
Anabolic steroid-related disorder NOS includes symptoms with anxiety disorders and associated substance abuse or depen- dence.
12.25. The answer is D
Auditory, tactile, and visual hallucinations are typical compo- nents of alcohol hallucinosis. This is characterized by symp- toms that frequently resolve by 24 to 48 hours and does not typically include autonomic dysregulation. Because the patient’s auditory hallucinations and agitation appeared 48 hours after ad- mission, hallucinosis caused by alcohol consumption is a likely explanation. Paranoid personality disorder consists of a long- standing history of suspiciousness. Personality disorders are un- likely to disappear within days of hospitalization. Brief psychotic
Table 12.2
State Markers of Heavy Drinking Useful in Screening for Alcoholism
12. Substance-Related Disorders 113
 
Test
γ -Glutamyltransferase (GGT) Carbohydrate-deficient transferrin
(CDT)
Mean corpuscular volume (MCV) Uric acid
Serum glutamic-oxaloacetic transaminase (aspartate aminotransferase) (SGOT [AST])
Serum glutamic-pyruvic transaminase (alanine aminotransferase) (SGPT [ALT])
Triglycerides
Relevant Range of Results
>30 U/L >20 mg/L
>91 μm3
>6.4 mg/dL for men >5.0 mg/dL for women >45 IU/L
>45 IU/L >160 mg/dL
 
114 12. Substance-Related Disorders
disorder is a psychosis that lasts no more than 1 month but at least 1 day. It consists of disorganized speech and behavior, hal- lucinations, and delusions and is an unlikely diagnosis because the patient has a history of alcoholism. An adjustment disorder typically occurs within 3 months of a specific stressor and re- sults in emotional turmoil that does not include hallucinations. Posttraumatic stress disorder is frequently triggered by natural disasters, military combat, sexual abuse, and even motor vehi- cle accidents. It includes flashbacks, nightmares, and recurrent thoughts of the event but does not involve hallucinations.
Answers 12.26–12.33 12.26. The answer is B 12.27. The answer is A 12.28. The answer is B 12.29. The answer is A 12.30. The answer is A 12.31. The answer is C 12.32. The answer is A
12.33. The answer is B
Benzodiazepines have become the primary drugs used to treat anxiety and insomnia, largely replacing barbiturates and other sedative-hypnotic agents. Benzodiazepines have lower abuse potential than most barbiturates, pose a lower risk when taken in overdose, and have fewer interactions with other drugs. The abrupt discontinuation of these drugs, particularly chlor- diazepoxide (Librium) and diazepam (Valium), can lead to a withdrawal syndrome. Minor signs of discontinuation include anxiety, insomnia, and nightmares. Major signs (which are ex- tremely rare) include grand mal seizures, psychosis, hyper- pyrexia, and death. Personality traits may be a risk factor for development of the benzodiazepine withdrawal syndrome. With- drawal severity is greater in patients with higher scores on the dependence scale of the Minnesota Multiphase Personal- ity Inventory 2 (MMPI-2), high prewithdrawal levels of anxiety and depression, lower education level, and passive-dependent personality disorder. Even when grossly excessive amounts (more than 2 g) of benzodiazepines are taken in suicide attempts, the symptoms include only drowsiness, lethargy, ataxia, some confusion, and mild depression of the user’s vital signs. A much more serious condition prevails when benzodiazepines are taken in overdose in combination with other sedative-hypnotic sub- stances, such as alcohol. In such cases, small doses of benzodi- azepines can cause death.
Since the advent of benzodiazepines, barbiturate use has been limited in modern medicine. Pentobarbital (Nembutal) is still prescribed as an anticonvulsant and as a sedative, especially in children. A major disadvantage of the use of barbiturates is the development of pharmacokinetic and pharmacodynamic toler-
ance. Tolerance is defined as reduced drug response as a result of decreased drug concentration at the site of action, usually the result of increased drug metabolism (pharmacokinetic) or of cellular adaptive changes with unchanged or higher drug con- centrations at the site of action (pharmacodynamic). Pharma- codynamic tolerance begins after acute doses and continues to develop over weeks to months. Minor symptoms of withdrawal include apprehension and uneasiness, insomnia, muscular weak- ness, twitches, coarse tremors, myoclonic jerks, and electroen- cephalographic changes within 24 hours of the last dose. Minor symptoms may persist as long as 2 weeks. At high doses, major symptoms develop, which include grand mal seizures and delir- ium. The delirium can sometimes be accompanied by hypother- mia, which may be fatal. Overdose is lethal with barbiturates because they induce respiratory depression. As with benzodi- azepines, the lethal affects of barbiturates are additive to those of other sedative-hypnotic drugs, including alcohol and benzodi- azepines. Barbiturates overdose is characterized by induction of coma, respiratory arrest, cardiovascular failure, and death. Bar- biturates cause REM sleep suppression. An abrupt withdrawal of a barbiturate causes a marked increase or rebound in REM sleep. Symptoms of withdrawal from both benzodiazepines and barbiturates usually appear within 3 days.
Answers 12.34–12.38 12.34. The answer is B 12.35. The answer is C 12.36. The answer is A 12.37. The answer is B
12.38. The answer is C
Alcohol is metabolized by two enzymes, alcohol dehydroge- nase (ADH) and aldehyde dehydrogenase. ADH catalyzes the conversion of alcohol into acetaldehyde, which is a toxic com- pound, and aldehyde dehydrogenase catalyzes the conversion of acetaldehyde into acetic acid. Aldehyde dehydrogenase is in- hibited by disulfiram (Antabuse), often used in the treatment of alcohol-related disorders. Some studies have shown that women have a lower ADH blood content than do men; this fact may account for women’s tendency to become more intoxicated than men after drinking the same amount of alcohol. The decreased function of alcohol-metabolizing enzymes in some Asian people can also lead to easy intoxication and toxic symptoms.
Answers 12.39–12.48 12.39. The answer is H 12.40. The answer is J 12.41. The answer is E 12.42. The answer is D
12.43. The answer is B 12.44. The answer is A 12.45. The answer is F 12.46. The answer is I 12.47. The answer is G
12.48. The answer is C
The lettered terms are all terms associated with dependence or abuse. See Table 12.3 for definitions of terms used in de- pendence or abuse. The concept of substance dependence has had many officially recognized and commonly used meanings over the decades. In behavior dependence, substance-seeking activities and related evidence of pathological use patterns are emphasized; physical dependence refers to the physical (phys- iological) effects of multiple episodes of substance use. Phys- ical dependence can be a feature of problematic use, but it is neither necessary nor sufficient for diagnosis of (syndromic) de- pendence. A person may be physically dependent on a substance (e.g., a patient treated with opioid analgesics for cancer pain) but not have problematic use and vice versa.
Answers 12.49–12.53 12.49. The answer is E 12.50. The answer is D
Table 12.3
Terms Used in Dependence and Abuse
12. Substance-Related Disorders 115 12.51. The answer is B
12.52. The answer is D
12.53. The answer is C
A common adverse effect associated with cocaine use is nasal congestion; serious inflammation, swelling, bleeding, and ul- ceration of the nasal mucosa can also occur. Long-term use of cocaine can also lead to perforation of the nasal septa. Freebasing and smoking crack can damage the bronchial passages and the lungs. The intravenous use of cocaine can result in infection, em- bolisms, and the transmission of HIV. The major complications of cocaine use are cerebrovascular (infarctions), epileptic, and cardiac (MI and arrhythmias). About two-thirds of these acute toxic effects occur within 1 hour of intoxication.
The most common physical effects of cannabis are dila- tion of the conjunctival vessels (injected or red eyes) and mild tachycardia. At high doses, orthostatic hypotension may appear. Increased appetite and dry mouth are also common effects of cannabis intoxication. Traditionally, amotivational syndrome has been associated with long-term heavy use and has been charac- terized by a person’s unwillingness to persist in a task, be it at school, work, or in any setting that requires prolonged attention or tenacity. Persons are described as becoming apathetic and anergic, usually gaining weight and appearing slothful.
People who have just taken phencyclidine (PCP) are fre- quently uncommunicative, appear to be oblivious, and report active fantasy production. They experience speedy feelings, eu- phoria, bodily warmth, tingling, peaceful floating sensations,
 
Dependence The repeated use of a drug or chemical substance with or without physical dependence. Physical dependence indicates an altered physiologic state caused by repeated administration of a drug, the cessation of which results in a specific syndrome.
Abuse Use of any drug, usually by self-administration, in a manner that deviates from approved social or medical patterns.
Misuse Similar to abuse but usually applies to drugs prescribed by physicians that are not used properly.
Addiction The repeated and increased use of a substance, the deprivation of which gives rise to symptoms of distress and an irresistible
urge to use the agent again and which leads also to physical and mental deterioration. The term is no longer included in the official
nomenclature, having been replaced by the term dependence, but it is a useful term in common usage.
Intoxication A reversible syndrome caused by a specific substance (e.g., alcohol) that affects one or more of the following mental
functions: memory; orientation; mood; judgment; and behavioral, social, or occupational functioning.
Withdrawal A substance-specific syndrome that occurs after stopping or reducing the amount of the drug or substance that has been
used regularly over a prolonged period of time. The syndrome is characterized by physiologic signs and symptoms in addition to psychological changes, such as disturbances in thinking, feeling, and behavior. Also called abstinence syndrome or discontinuation syndrome.
Tolerance Phenomenon in which, after repeated administration, a given dose of drug produces a decreased effect or increasingly larger doses must be administered to obtain the effect observed with the original dose. Behavioral tolerance reflects the ability of the person to perform tasks despite the effects of the drug.
Cross-tolerance Refers to the ability of one drug to be substituted for another, each usually producing the same physiologic and psychological effect (e.g., diazepam and barbiturates). Also known as cross-dependence.
Neuroadaptation Neurochemical or neurophysiologic changes in the body that result from repeated administration of a drug. Neuroadaptation accounts for the phenomenon of tolerance. Pharmacokinetic adaptation refers to adaptation of the metabolizing system in the body. Cellular or pharmacodynamic adaptation refers to the ability of the nervous system to function despite high blood levels of the offending substance.
Codependence Term used to refer to family members affected by or influencing the behavior of the substance abuser. Related to the term enabler, which is a person who facilitates the abuser’s addictive behavior (e.g., providing drugs directly or money to buy drugs). Enabling also includes the unwillingness of a family member to accept addiction as a medical-psychiatric disorder or to deny that person is abusing a substance.

116 12. Substance-Related Disorders
and occasionally feelings of depersonalization. Hypertension, hyperthermia, and vertical or horizontal nystagmus are common effects of PCP. The short-term effects last 3 to 6 hours and some- times give way to a mild depression, irritability, paranoia, and occasionally belligerent and violent behavior.
With hallucinogen use, such as LSD, perceptions become un- usually brilliant and intense. Colors and textures seem richer than in the past, contours sharpened, music more emotionally profound, and smells and tastes heightened. Synesthesia is com- mon; colors may be heard or sounds seen.
 
Schizophrenia is one of the most debilitating clinical syndromes within psychiatry. It is characterized by disordered cognition, emotions, perceptions, and abnormal behavior. The effect of the illness is always severe despite variations across patients and it is usually long lasting. The consequences of the illness for the patient, his or her family, and society in general are devastating. Schizophrenia affects about 1 percent of the world’s population, and in the United States, it has a financial cost that is estimated to exceed that of all cancers combined.
Two major figures in psychiatry who studied the disorder are Emil Kraepelin (1856–1926) and Eugene Bleuler (1857–1939). The term de ́mence pre ́coce, coined by French psychiatrist Bene- dict Morel (1809–1873), was translated by Kraepelin to dementia precox. Patients with dementia precox were described as having symptoms of hallucinations and delusions and having a long deteriorating course. Bleuler introduced the term schizophrenia in 1911. He chose it to express the presence of schisms among thought, emotion, and behavior in patients with the disorder. He stressed that, unlike Kraepelin’s dementia precox, schizophrenia need not have a deteriorating course. He identified four primary symptoms commonly known as the “four As”: associations, affect, autism, and ambivalence. He also identified secondary symptoms: hallucinations and delusions.
The cause of schizophrenia is still unknown. There is con- siderable evidence that genetic factors make a considerable contribution to its etiology. The presence of a proband with schizophrenia significantly increases the prevalence of this dis- order among biological relatives. Eight genetic linkage sites
13
Schizophrenia
have been identified, and specific candidate genes have been implicated. A number of environmental factors have also been identified that may contribute to the development of schizo- phrenia.
In the past, it was believed that schizophrenia was a West- ern disease. This belief has been disproven by the fact that no community has been found to be free of schizophrenia. The clin- ical presentation of schizophrenia is very similar across cultures. Schizophrenia is equally prevalent in men and women. Where they differ is in the onset and course of the illness. Patients with schizophrenia are at increased risk for substance abuse, espe- cially nicotine dependence. They are also at an increased risk for suicide and assaultive behavior; approximately 10 percent of patients commit suicide.
Antipsychotic medications, which are the mainstay of phar- macological treatment of patients with schizophrenia, are effec- tive in reducing the impact of psychotic symptoms such as hallu- cinations, delusions, and suspiciousness. After these symptoms are minimized, or in some cases eliminated altogether, medi- cations can decrease the likeliness that they will return. Psy- chosocial and rehabilitative interventions are fundamental parts of the treatment of patients with schizophrenia. These include cognitive behavioral therapy, supportive educationally oriented psychotherapy, family therapy and educational programs, social and living skills training, supported employment programs, and supervised residential living arrangements.
Students should test their knowledge by addressing the fol- lowing questions and answers.
  
HELPFUL HINTS
The following names and terms, including the schizophrenic signs and symptoms listed, should be studied and the definitions learned.
akathisia antipsychotics autistic disorder Gregory Bateson Eugen Bleuler bouffe ́e de ́lirante brain imaging—CT,
PET, MRI catatonic type deinstitutionalization delusions
dementia precox disorganized type dopamine hypothesis downward-drift
hypothesis dystonia
ECT
ego boundaries electrophysiology—
EEG
first-rank symptoms
flat affect and blunted affect
forme fruste
the four As fundamental and
accessory symptoms genetic hypothesis hallucinations
Emil Kraepelin mesocortical and
mesolimbic tracts
Adolf Meyer Benedict Morel paranoid type paraphrenia
positive and negative
symptoms residual type
RFLPs
schizoaffective disorder Kurt Schneider seasonality of birth
117
▲▲▲▲▲ ▲▲▲▲▲
▲ ▲▲▲ ▲▲▲▲
▲▲▲▲ ▲▲▲▲ ▲
▲▲▲▲▲▲▲ ▲▲▲
118 13. Schizophrenia serotonin hypothesis
social causation hypothesis
QUESTIONS
Directions
soft signs stress–diathesis
model
Harry Stack thought disorders Sullivan undifferentiated type
tardive dyskinesia
C. having a schizophrenic family member
D. having a history of temporal lobe epilepsy
E. having low levels of monoamine oxidase, type B, in
blood platelets
13.5. True statements about violence and schizophrenia in-
clude all of the following except
A. Violence in a hospital setting can result from undiag- nosed neuroleptic-induced acute akathisia.
B. Patients with schizophrenia are more violent as a group than the general population.
C. It is more difficult to prevent most schizophrenic homicides compared with the general population. D. Patients with disorganized schizophrenia are at much
greater risk to commit violence than those with para-
noid schizophrenia.
E. Command hallucinations do not appear to play a par-
ticularly important role in violence.
13.6. Which of the following statements best describes a char- acteristic of the epidemiology of schizophrenia?
A. Female patients with schizophrenia are more likely to commit suicide than are male patients.
B. In the northern hemisphere, schizophrenia occurs more often among people born from July to Septem- ber than in those born in the other months.
C. Reproduction rates among people with schizophre- nia are typically higher than those among the general population.
D. Patients with schizophrenia occupy about 50 percent of all hospital beds.
E. Some regions of the world have an unusually high prevalence of schizophrenia.
13.7. True statements about eye movement dysfunction in schizophrenia include
A. Abnormal eye movements occur more often in pa- tients with schizophrenia compared with control sub- jects.
B. Eye movement dysfunction is associated with a frontal lobe pathology.
C. Eye movement dysfunction is independent of drug treatment.
D. Eye movement dysfunction is seen in first degree probands.
E. All of the above
13.8. In general, pooled studies show concordance rates for schizophrenia in monozygotic twins of
  
Each of the questions or incomplete statements below is followed by five suggested responses or completions. Select the one that is best in each case.
. 13.1. Which of the following statements about the dopamine hypothesis of schizophrenia is true?
A. Dysregulation of dopaminergic neurotransmission is caused by postsynaptic sensitivity.
B. Dopamine release caused by amphetamine challenge is higher during remission.
C. Higher amphetamine-provoked dopamine release predicts worsening of psychotic symptoms.
D. Overactivity of dopamine in the subcortical basal ganglia contributes to negative symptoms.
E. There is a lower occupancy of D2/3 receptors in relapsed patients.
. 13.2. True statements about hypothesized neurobiological models of schizophrenia include
A. Genes function in part by increasing vulnerability to environmental factors.
B. Environmental factors increase risk by producing subtle brain damage.
C. The apparent lack of gliosis in postmortem studies implicates in utero factors.
D. As the prefrontal cortex matures, behavioral and cog- nitive sequelae of subtle structural deficits become manifest.
E. All of the above
. 13.3. With regard to the ventricular size in schizophrenia,
which of the following statements is true?
A. Patients with schizophrenia invariably demonstrate significant enlargement of the fourth ventricle only.
B. Ventricular enlargement is a pathognomonic finding in schizophrenia.
C. Ventricular changes in schizophrenia are likely to be specific for the pathophysiological processes under- lying this disorder.
D. All of the above
E. None of the above
. 13.4. All of the following lead to an increased risk of schizophrenia except
A. havingadeviantcourseofpersonalitymaturationand development
B. having previously attempted suicide
A. 0.1 percent
▲▲
▲▲
▲▲
▲▲
B. 5percent C. 25 percent D. 40 percent E. 50 percent
. 13.9. A schizophrenic patient who states that he feels his brain burning is most likely experiencing a
A. cenesthetic hallucination B. delusional feeling
C. gustatory hallucination D. haptic hallucination
E. hypnopompic hallucination
. 13.10. Childhood schizophrenia
A. tends to have a chronic course
B. tends to have a better prognosis that adult schizo-
phrenia
C. isnotdiagnosedusingthesamesymptomsasareused
for adult schizophrenia
D. tends to have an abrupt onset
E. all of the above
. 13.11. Late-onset schizophrenia
A. is more common in men
B. isassociatedwithapreponderanceofparanoidsymp-
toms
C. is clinically distinguishable from early-onset schizo-
phrenia
D. results in poorer response to antipsychotic medica-
tions
E. has an onset after age 60 years
. 13.12. Whichofthefollowingistrueofbrainimagingtechnolo- gies in the study of schizophrenia?
A. Computed tomography (CT) is used more often than magnetic resonance imaging (MRI) in schizophrenia research because its resolution is superior to that of MRI.
B. The abnormalities reported in CT studies of patients with schizophrenia are specific for the pathophysio- logical processes underlying the disease.
C. In studies of monozygotic twins discordant for schizophrenia, MRI studies have shown that the cere- bral ventricles in the affected twins are larger than in the nonaffected twins.
D. Positron emission tomography (PET) studies have shown almost no impairment of brain areas after psy- chological test stimulation.
E. Function magnetic resonance imaging (fMRI) has shown no differences in the brains of patients with schizophrenia compared with control subjects.
. 13.13. ThemajorityofCTstudiesofpatientswithschizophrenia have reported
A. atrophy of the cerebellar vermis
B. cortical atrophy in 10 to 35 percent of patients
C. enlargedlateralandthirdventriclesin10to50percent of patients
D. findings that are not artifacts of treatment E. all of the above
13.14. Which of the following is not typically associated with catatonia?
A. Mannerisms
B. Mutism
C. Stereotypies
D. Verbigeration E. Waxy flexibility
13.15. Persons in the United States who develop schizophrenia are more likely to
A. have been born abroad
B. have been born in the months from January to
April
C. havebeenborninthemonthsfromJulytoSeptember
D. have been exposed to the parainfluenza virus
E. none of the above
13.16. Which of the following statements comparing the serotonin-dopamine antagonists (SDAs) with dopamine receptor antagonists (DRAs) is true?
A. The DRAs remain the first choice of treatment for schizophrenia.
B. The SDAs affect both serotonin and glutamate recep- tors.
C. The SDAs produce more extrapyramidal symptoms than the DRAs.
D. TheSDAsproducemoreneurologicaladverseeffects than the DRAs .
E. The SDAs are less effective than the DRAs for posi- tive symptoms of schizophrenia.
13.17. Which of the following statements about the cause of negative symptoms in schizophrenia is false?
A. Patients lose drive because circumstances eliminate them.
B. Positive symptoms commonly cause alogia.
C. Excessive doses of antipsychotic medications cause
blunting of affect.
D. Persecutory delusions can lead to social withdrawal.
E. None of the above
13.18. Clozapine (Clozaril)
A. has been associated with few, if any, extrapyramidal side effects
B. is believed to exert its therapeutic effect mainly by blocking dopamine receptors
C. causes significant increases in prolactin levels
D. is associated with a 10 to 20 percent incidence of
agranulocytosis
E. requires monthly monitoring of blood chemistry
13. Schizophrenia 119
120 13. Schizophrenia
. 13.19. Investigations into the cause of schizophrenia have
revealed that
A. a specific family pattern plays a causative role in the development of schizophrenia.
B. the efficacy and potency of most antipsychotics cor- relate with their ability to act primarily as antagonists of the dopamine type 1 (D1) receptor.
C. a particular defective chromosomal site has been found in all schizophrenic patients.
D. no significant abnormalities appear in the evoked potentials in schizophrenic patients.
E. a monozygotic twin reared by adoptive parents has schizophrenia at the same rate as his or her twin raised by biological parents.
. 13.20. Epidemiological studies of schizophrenia have found all of the following except
A. Hospital records suggest that the incidence of schizophrenia in the United States has remained un- changed for the past 100 years.
B. The peak age of onset for schizophrenia is the same for men and women.
C. Schizophrenia is equally prevalent among men and women.
D. Approximately 50 percent of schizophrenic patients attempt suicide at least once in their lifetimes.
E. The lifetime prevalence is usually between 1 and 1.5 percent of the population.
. 13.21. Features weighing toward a good prognosis in schizophrenia include all of the following except
A. depression
B. a family history of mood disorders
C. paranoid features
D. undifferentiated or disorganized features E. an undulating course
. 13.22. MRI studies of patients with schizophrenia have found evidence for
A. increased cortical gray matter
B. increased temporal cortex gray matter C. increased volume of the amygdala
D. increased volume of basal ganglia nuclei E. increased volume of the hippocampus
. 13.23. Prefrontal cortex and limbic system hypotheses are the predominant neuroanatomical theories of schizophrenia because of the demonstration of
A. decreased volume of prefrontal gray or white matter
B. disturbed prefrontal metabolism and blood flow
C. disarray or abnormal migration of hippocampal neu-
rons
D. prefrontal cortical interneuron abnormalities
E. all of the above
13.24. The rationale for the role of excess dopamine in schizophrenia is based on observations that
A. dopaminergic drugs can induce paranoid psycho- sis
B. drugs that block postsynaptic dopamine receptors reduce symptoms of schizophrenia
C. metabolic alterations in limbic anatomy are consistent with a disturbance in dopamine metabolism
D. increased concentrations of dopamine have been found in the amygdalas in postmortem brains of schizophrenic patients
E. all of the above
13.25. True statements about structural brain abnormalities in patients with schizophrenia include
A. abnormalities are present from birth
B. abnormalities are present in a minority of patients
C. abnormalitieshavenotbeencorrelatedwithcognitive
deficits
D. cortical involvement is multifocal rather than dif-
fuse
E. none of the above
13.26. In simple deteriorative disorder,
A. delusions are common
B. early diagnosis is common C. hallucinations are common D. homelessness is common E. all of the above
13.27. A 63-year-old man who was diagnosed with schizophre- nia in his mid-20s and living off and on with family, neighbors, and in shelters since he first became ill wanted to find some source of regular income so he might escape the noise and threats of violence in the homeless shelters where he had been residing. He was advised to seek a disability pension and was given detailed instructions on how and where to go and apply. At the next visit, his physician inquired about his application. “I didn’t go,” he replied. The physician asked, “Is there something that kept you from going to apply?” He replied, “I guess, but I don’t remember anything, really.”
“Were you afraid anything might happen to you if you went?”
“No, I’ve been feeling pretty safe.”
“Was there any problem taking the bus to the Social
Security office?”
“No, the bus goes by there on the way to your office,
doc.”
“Would you like to have more money so you could pay
rent for your own place?” “I guess.”
“Can you think of a reason not to go to apply?”
“No, it would be good to go so I could get out of the
shelters.”
“Could you try to go and get the forms on the way home today, and we could help you fill them out on your next visit?”
“Yeah, sure.”
At the next visit, and at subsequent visits, the patient made no progress toward a disability application, although he could acknowledge that he would feel safer getting out of the shelters. It was not until months later, when a social worker was available to accompany him to the office, that he was able to apply for benefits.
The patient above is showing which negative symp- tom of schizophrenia?
A. Alogia
B. Avolition
C. Anhedonia
D. Blunting
E. Social inattentiveness
. 13.28. A married man, age 38 years, with a history of depend- able, conscientious work as a bookkeeper, became sleep- less, anxious, and unable to concentrate three months prior. He developed the belief that his vision was failing because of poisons secretly placed in his food by former neighbors. He found a misprint in a newspaper that he believed was placed there by the editor to shame him pub- licly. Admitted to the psychiatric service of a general hos- pital, he said that cars passing up and down the street con- tained agents who were spying on him. He believed that the electric light bulbs in his room were emanating a pu- rifying radiation to counteract syphilitic germs, which he was supposedly breathing into the atmosphere, although a physical examination was negative for syphilis.
Which of the following psychiatric conditions is the most likely diagnosis?
A. Brief psychotic disorder
B. Schizophrenia, paranoid type
C. Delusional disorder
D. Schizophrenia, disorganized type E. Malingering
. 13.29. Mr. G, a 36-year-old man, is admitted to a psychiatric unit after being brought to the emergency department by police. As he was walking past a hotel in the central part of the city, he saw a man and woman standing on the sidewalk about to take a photograph of a building across the street. Thinking that they were going to take his picture, he grabbed the camera, smashed it on the ground, and pulled out all the film. He explained his actions by saying the photograph would be used to control him and that it is illegal to take another person’s photograph.
Mr. G has a history of multiple hospitalizations dat- ing back to age 14 years. During the hospitalizations, his symptoms have been well controlled with a variety of typical and atypical antipsychotic medications. After being discharged, he begins drinking four to five beers
a day, neglects getting prescriptions refilled, and stops medication when his supply runs out. He made two prior suicide attempts, both by hanging, in which he experi- enced no serious medical sequelae. He reports numerous blackouts from drinking, but he has never had seizures or delirum tremens (DTs). He does not use illicit drugs.
Mr. G dropped out of high school in the 11th grade. He worked a number of short-term, unskilled jobs be- fore going on public assistance at age 21 years. He lives alone, is estranged from his family, and has no friends. On examination, Mr. G is lying motionless. He makes good eye contact and says, “I’m trying not to move.”
He fears that if he moves he may die. He currently hears voices saying, “Be good,” “Get the dog,” and “He’s the one.” He also sees shapes, which he describes as col- ored letters dancing in front of his eyes. He talks about being monitored by hidden cameras and microphones ev- erywhere he goes in the city. He is alert and oriented. He can recall three out of three objects after 5 minutes. His concentration is impaired.
Which of the following is the most likely diagnosis for the case described above?
A. Delusional disorder
B. Schizophrenia, catatonic type
C. Schizophrenia, paranoid type
D. Schizophrenia, undifferentiated type E. Schizoaffective disorder
. 13.30. Which of the following interventions is most likely to prevent relapse in the case above?
A. Alcohol counseling
B. Increased socialization
C. Use of an atypical antipsychotic
D. Use of a long-term depot antipsychotic E. Vocational rehabilitation
. 13.31. A 32-year-old woman with a history of schizophrenia presents to clinic with her sister. The patient has been treated for the past 3 years with risperidone (Risperdal). The patient’s sister states she believes the patient’s symp- toms are well controlled; however, she is worried about unusual movements she noticed recently. She states that the patient has been protruding her tongue and making strange noises with her lips. Which of the following is the most likely diagnosis?
A. Akathisia
B. Tardive dyskinesia C. Parkinsonism
D. Dystonia
E. None of the above
Directions
Each group of questions below consists of lettered headings fol- lowed by a list of numbered phrases or statements. For each numbered phrase or statement, select the one lettered heading
13. Schizophrenia 121
122 13. Schizophrenia
that is most associated with it. Each lettered heading may be
selected once, more than once, or not at all.
Questions 13.32–13.36
A. Paranoid schizophrenia
B. Catatonic schizophrenia
C. Residual schizophrenia
D. Disorganized schizophrenia
E. Undifferentiated schizophrenia
. 13.32. Inappropriate laughter
. 13.33. Mutism
. 13.34. Auditory hallucinations
. 13.35. Self-inflicted injury
. 13.36. No active symptoms
Questions 13.37–13.41
A. Clang association
B. Echolalia
C. Loosening of associations D. Neologism
E. Verbigeration
. 13.37. Loss of logical relations between thoughts
. 13.38. Creation of a new expression or word
. 13.39. Repetition of interviewer’s words when answering a
question
. 13.40. Words associated by sound rather than meaning
. 13.41. Use of words in stereotypically repetitive fashion
Questions 13.42–13.46
A. Catatonic posturing B. Echopraxia
C. Catatonic negativism D. Catatonic stupor
E. Catalepsy
. 13.42. A patient remains with his hands behind his back hours after restraints are removed.
. 13.43. A patient stands on one foot for 5 hours.
. 13.44. A patient sits unmoving in a recliner all day until a nurse
takes him to bed at night.
. 13.45. A patient resists attempts to move him from the fetal
position.
. 13.46. A patient crosses his leg after the physician does.
Questions 13.47–13.51
A. Emil Kraepelin B. Eugen Bleuler C. Kurt Schneider
. 13.47. More concerned with course and prognosis
. 13.48. Divided symptoms into first- and second-rank symp-
toms
. 13.49. More concerned with understanding the underlying psy-
chological mechanisms
13.50. Created the original subtypes of schizophrenia 13.51. Saw symptoms in a continuum with normal behavior
Questions 13.52–13.56
A. Vocational rehabilitation
B. Cognitive behavioral therapy C. Token economy program
D. Social skills training
E. Assertive community treatment
13.52. Patient engages in role play with a trainer
13.53. A multidisciplinary team provides services 24 hours a
day
13.54. A variety of methods are used to help patients regain old
skills
13.55. Therapist gains a clear understanding of the patient’s
experience of hallucinations
13.56. Promotes learning behavior to enhance patient’s func-
tioning
ANSWERS
13.1. The answer is C
According to the dopamine hypothesis of schizophrenia, higher amphetamine-provoked dopamine release predicts worsening of psychotic symptoms. The dopamine hypothesis of schizophrenia posits that overactivity of dopamine neurotransmission in the subcortical basal ganglia contributes to positive (not negative) symptoms, and the hypoactivity of prefrontal cortical dopamine neurotransmission contributes to negative symptoms and cogni- tive abnormalities in patients with schizophrenia. This theory evolved from two observations: (1) the efficacy and the potency of many antipsychotic drugs (i.e., the dopamine receptor an- tagonists [DRAs]) are correlated with their ability to act as an- tagonists of the dopamine 2 (D2) receptor; and (2) drugs that in- crease dopaminergic activity, notably cocaine and amphetamine, are psychotomimetic. It has been suggested that the dysregu- lation of dopaminergic neurotransmission in schizophrenia is caused by presynaptic reactivity, not postsynaptic sensitivity. Dopamine release provoked by amphetamine challenge is higher in schizophrenic patients at the onset of illness (drug-na ̈ıve pa- tients) and during a relapse and is normal (not higher) during remission. Depletion of endogenous presynaptic dopamine vesi- cles has demonstrated that there is a higher (not lower) occu- pancy of D2/3 receptors by dopamine treatment-na ̈ıve patients and in relapsed schizophrenic patients.
13.2. The answer is E (all)
The essential neurobiological features of schizophrenia may place some constraints on plausible pathophysiological pro- cesses. First, there is a major genetic contribution. Many genes are likely to be involved, and these may function in part by in- creasing vulnerability to the deleterious effects of environmental factors. Several environmental factors have been hypothesized to increase the risk of schizophrenia, perhaps by producing subtle brain damage. Structural abnormalities have played an important role in placing theoretical constraints on mechanisms. Because
they are present from early in the illness and do not appear to progress, they may predate the onset of illness. Neuropatholog- ical data and studies of obstetric and perinatal complications support the idea that an early lesion may account for structural changes. The apparent lack of gliosis in postmortem studies is particularly critical and implicates in utero factors. Structural and functional neuroimaging, as well as neuropsychological data and animal studies, present converging evidence for the impor- tance of frontal and temporal regions. Finally, altered dopamine and glutamate neurotransmission is likely to play a part in the expression of psychotic symptoms.
The neurodevelopmental model can account for many of these findings. In short, some process (genetic or environmental) produces damage to selected brain areas early in life. Temporal lobe regions such as the hippocampus may be particularly vul- nerable. Secondary functional abnormalities develop later. As the prefrontal cortex matures in late adolescence, the behav- ioral and cognitive sequelae of subtle structural deficits become manifest. One result is hypofrontality and cognitive impairment. Alterations in limbic and prefrontal function then produce downstream, secondary alterations in subcortical dopamine, glu- tamate, and other neurotransmitter systems. Dopamine dysfunc- tion, in particular, may lead to positive psychotic symptoms. The feasibility of this model has received substantial validation from animal studies showing the delayed behavioral and neurobiolog- ical effects of minor damage to the hippocampus in neonatal rats. Observations that children at risk for schizophrenia have a num- ber of subtle neuropsychiatric abnormalities, such as deficits in attention, motor control, and social interactions, also support the neurodevelopmental model.
13.3. The answer is E (none)
Magnetic resonance imaging (MRI) studies have consistently shown that the brains of many schizophrenic patients have lat- eral and third ventricular enlargement and some degree of re- duction in cortical volume. These findings can be interpreted as consistent with the presence of less than usual brain tissue in af- fected patients; whether that decrease is attributable to abnormal development or to degeneration remains undetermined.
However, the abnormalities reported in MRI studies of schizophrenic patients have also been reported in other neu- ropsychiatric conditions, including mood disorders, alcohol- related disorders, and dementias. Thus, these changes are not likely to be pathognomonic for the pathological processes un- derlying schizophrenia. Although the enlarged ventricles in schizophrenic patients can be shown when groups of patients and control subjects are used, the difference between affected and unaffected persons is variable and usually small.
One of the most important MRI studies examined monozy- gotic twins who were discordant with schizophrenia. The study found that virtually all of the affected twins had larger cerebral ventricles than did the nonaffected twins, although most of the affected twins had cerebral ventricles within a normal range.
13.4. The answer is B
Having previously attempted suicide does not increase the risk for developing schizophrenia, although at least 50 percent of patients with schizophrenia attempt suicide once in their life-
times. Having a schizophrenic family member, especially having one or two schizophrenic parents or a monozygotic twin who is schizophrenic, increases the risk for schizophrenia. Other risk factors include (1) having lived through a difficult obstetrical delivery, presumably with trauma to the brain; (2) having, for unknown reasons, a deviant course of personality maturation and development that has produced an excessively shy, day- dreaming, withdrawn, friendless child; an excessively compli- ant, good, or dependent child; a child with idiosyncratic thought processes; a child who is particularly sensitive to separation; a child who is destructive, violent, incorrigible, and prone to tru- ancy; or an anhedonic child; (3) having a parent who has para- noid attitudes and formal disturbances of thinking; (4) having low levels of monoamine oxidase, type B, in the blood platelets; (5) having abnormal pursuit eye movements; (6) having taken a variety of drugs, particularly lysergic acid diethylamide (LSD), amphetamines, cannabis, cocaine, and phencyclidine; and (7) having a history of temporal lobe epilepsy, Huntington’s dis- ease, homocystinuria, folic acid deficiency, or the adult form of metachromatic leukodystrophy.
None of those risk factors invariably occurs in schizophrenic patients; they may occur in various combinations. The vast majority of people who ingest psychotomimetic drugs do not become schizophrenic. Not every schizophrenic patient has ab- normal pursuit eye movements, and some well relatives of schizophrenic patients may also have abnormal pursuit eye movements.
13.5. The answer is D
Patients with schizophrenia are more violent as a group than the general population. This is particularly a problem for pa- tients with the paranoid type, who may act quite suddenly and impulsively on a delusional idea. Patients with paranoia tend to be intelligent and capable of forming plans; therefore, they represent a much greater risk than individuals who are disorga- nized and cannot plan an effective attack. Despite earlier beliefs, command hallucinations do not appear to play a particularly important role in violence. Violence between patients in hos- pitals frequently results from the attacking patient’s mistaken belief that another patient is behaving in a threatening way or getting physically too close. Studies have revealed that violence in a hospital setting can result from undiagnosed neuroleptic- induced acute akathisia. Persistently violent inpatients often do well in special treatment units that provide a more structured program and a less crowded environment. Patients who fail to respond to this kind of care usually show neurological signs in addition to their diagnosis.
Unfortunately, it is exceedingly difficult to prevent most schizophrenic homicides because there is usually no clear warn- ing. Most of the homicides come as a horrifying surprise. Patients who are known to be paranoid with homicidal tendencies should not, as a rule, be allowed to move about freely as long as they retain their delusions and their aggressive tension.
13.6. The answer is E
An important epidemiological factor in schizophrenia is that some regions of the world have an unusually high prevalence of the disorder. Certain researchers have interpreted this geographic
13. Schizophrenia 123
124 13. Schizophrenia
inequity as supporting an infectious cause for schizophrenia; oth- ers emphasize genetic or social factors.
Schizophrenic patients occupy 50 percent of mental hospital beds, not of all hospital beds.
Female patients with schizophrenia are no more likely to commit suicide than are male patients; the risk factors are equal. There is a difference in prevalence of schizophrenia accord- ing to season, but in the northern hemisphere, schizophrenia occurs more often among people born from January to April, not from July to September. The latter time range refers to a seasonal preference for the disorder in the southern hemisphere. Reproduction rates among people with schizophrenia have been increasing in recent years because of newly introduced medi- cations and changes in laws and policies about hospitalization and community-based care. The fertility rate among people with schizophrenia, however, is only approaching the rate for the gen-
eral population and does not exceed it.
13.7. The answer is E (all)
The inability to follow a moving visual target accurately is the defining basis for the disorders of smooth visual pursuit and disinhibition of saccadic eye movements seen in patients with schizophrenia. Eye movement dysfunction may be a trait marker for schizophrenia; it is independent of drug treatment and clin- ical state and is also seen in first-degree relatives of probands with schizophrenia. Various studies have reported abnormal eye movements in 50 to 85 percent of patients with schizophrenia compared with about 25 percent in psychiatric patients without schizophrenia and fewer than 10 percent in nonpsychiatrically ill control subjects. Because eye movement is partly controlled by centers in the frontal lobes, a disorder in eye movement is con- sistent with theories that implicate a frontal lobe pathological process in schizophrenia.
13.8. The answer is E
In general, pooled studies show concordance rates of about 50 percent in monozygotic twins. This is the most robust finding pointing to a genetic etiologic component to the disorder.
13.9. The answer is A
A person with schizophrenia often experiences a cenesthetic hal- lucination, a sensation of an altered state in body organs without any special receptor apparatus to explain the sensation—for ex- ample, a burning sensation in the brain, a pushing sensation in the abdominal blood vessels, or a cutting sensation in the bone marrow.
A delusional feeling is a feeling of false belief based on an incorrect inference about external reality. A gustatory halluci- nation involves primarily taste. A tactile or haptic hallucination involves the sense of touch (e.g., formication—the feeling of bugs crawling under the skin). A hypnopompic hallucination is a hallucination that occurs as one awakes. Neither hallucina- tions nor delusions are pathognomonic of schizophrenia; they may occur in other disorders.
13.10. The answer is A
Recent studies have established that the diagnosis of childhood schizophrenia may be based on the same symptoms used for
adult schizophrenia. What characterizes childhood schizophre- nia is not the nature but the dramatic intensity of its symptoms. Its onset is usually insidious, its course tends to be chronic, and the prognosis is mostly unfavorable. Briefly, it resembles the typi- cal Kraepelinian case of dementia precox. What gives childhood schizophrenia unique importance for research is the observation that anatomical features of the brain that are often associated with adult-onset schizophrenia (e.g., enlarged ventricles) are also present in this early-onset form of the disease. Neurobiological studies of children with schizophrenia may therefore provide sig- nificant clues to the developmental pathogenesis of adult-onset schizophrenia.
13.11. The answer is B
Late-onset schizophrenia is clinically indistinguishable from early-onset schizophrenia but has an onset after age 45 years. This condition tends to appear more frequently in women and tends to be characterized by a predominance of paranoid symp- toms. The prognosis is favorable, and these patients usually do well on antipsychotic medication.
13.12. The answer is C
Magnetic resonance imaging (MRI) studies demonstrate that in monozygotic twins who are discordant for schizophrenia, virtu- ally all the affected twins have larger cerebral ventricles than their nonaffected twins. MRI is used in schizophrenia research because its resolution is superior to that with computed tomog- raphy (CT). The abnormalities reported in CT studies of patients with schizophrenia have also been reported in other neuropsychi- atric conditions and are unlikely to be specific for the pathophysi- ological processes underlying schizophrenia. Positron emission tomography studies have shown evidence of impaired activa- tion of certain brain areas after psychological test stimulation in schizophrenics. Functional MRI has shown differences in sen- sorimotor cortex activation and a decreased blood flow to the occipital lobes in patients with schizophrenia.
13.13. The answer is E (all)
The majority of CT studies of patients with schizophrenia have reported enlarged lateral and third ventricles in 10 to 50 percent of patients and cortical atrophy in 10 to 35 percent of patients. Controlled studies have also revealed atrophy of the cerebellar vermis, decreased radiodensity of brain parenchyma, and reversals of the normal brain asymmetries. Those findings are not artifacts of treatment and are not progressive or reversible. The enlargement of the ventricles seems to be present at the time of diagnosis before the use of medication. Some studies have correlated the presence of CT scan findings with the presence of negative or deficit symptoms (e.g., social isolation), neuropsychological impairment, frequent motor side effects from antipsychotics, and a poor premorbid adjustment.
13.14. The answer is D
Verbigeration is a specific disorder in the form of thought. It is the meaningless repetition of specific words or phrases and is not associated with catatonia.
The catatonic type of schizophrenia, which was common sev- eral decades ago, has become rare in Europe and North America.
The classic feature of the catatonic type is a marked disturbance in motor function; this disturbance may involve stupor, nega- tivism, rigidity, excitement, or posturing. Sometimes the patient shows rapid alteration between extremes of excitement and stu- por. Associated features include stereotypies, mannerisms, and waxy flexibility (or cerea flexibilitas). Stereotypies are repeti- tive fixed patterns of voluntary physical action or speech. Man- nerisms are ingrained, habitual involuntary movements. Finally, waxy flexibility or cerea flexibilitas is a condition in which a per- son can be molded into a position and then maintained or when an examiner moves the person’s limb and the limb feels as if it were made of wax; this is another term for catatonia.
13.15. The answer is B
Persons who develop schizophrenia are more likely to have been born in the winter and early spring. In the Northern Hemisphere, including the United States, persons with schizophrenia are more often born in the months from January to April. In the Southern Hemisphere, persons with schizophrenia are more often born in the months from July to September. There are no data to suggest that being born abroad is a risk factor for developing schizophre- nia. Some studies show that the frequency of schizophrenia is increased after exposure to influenza, not parainfluenza.
13.16. The answer is B
The serotonin-dopamine antagonists (SDAs) affect both sero- tonin and glutamate receptors. However, the SDAs produce min- imal or no extrapyramidal symptoms and interact with different subtypes of dopamine receptors than do the standard antipsy- chotics. They also produce fewer neurological adverse effects and are effective in treating negative symptoms of schizophre- nia. They are at least as effective as haloperidol for positive symptoms and are uniquely effective for the negative symptoms. These drugs have replaced the dopamine receptor antagonists as the drug of first choice for the treatment of schizophrenia.
13.17. The answer is E (none)
All of these statements about negative symptoms in schizophre- nia are true. Negative symptoms can arise from multiple causes, including iatrogenic causes. Long-term institutional care or life on the streets and in shelters can remove normal, expectable pleasures and reinforcing activities and narrow the number of re- lationships someone with schizophrenia will have. People with schizophrenia may lose the capacity to experience drives and social relationships and reward simply because circumstances eliminate them. A frequently unrecognized cause of negative symptoms is treatment with antipsychotic medications. Medica- tions for the treatment of schizophrenia can cause a phenocopy of primary negative symptoms that can only be distinguished with a full history of illness or empiric trials of medication adjustment. Reports of excessive doses of antipsychotic medications caus- ing blunting of affect, diminution of motor activity, impairments in grooming and hygiene, and anhedonia have existed almost since the introduction of neuroleptics, but for many patients, overtreatment with medications remains the rule rather than the exception.
Negative symptoms can also arise from the presence of other psychiatric symptoms. Positive symptoms are a common cause
of apparent apathy, social withdrawal, and alogia. Persecutory delusions can precipitate frantic attempts to protect safety but can also engender fear, leading to a very active social withdrawal, and prominent hallucinations can reinforce these fears. Frequent and intrusive hallucinations can lead directly to withdrawal and inaction strictly through their effects on attention.
13.18. The answer is A
Clozapine (Clozaril) has been associated with few, if any, ex- trapyramidal side effects or tardive dyskinesia. It is an antipsy- chotic medication that is appropriate in the treatment of patients with schizophrenia who have not responded to first-line DRAs or who have tardive dyskinesia. It is not an appropriate first- line drug for the treatment of schizophrenia. Clozapine has been associated with a 1 to 2 percent (not 10 to 20 percent) inci- dence of agranulocytosis and thus requires weekly, not monthly, monitoring of blood chemistries. Clozapine is believed to exert its therapeutic effect by blocking serotonin type 2 (5-HT2) and, secondarily, dopamine receptors.
13.19. The answer is E
The cause of schizophrenia is unknown. However, a wide range of genetic studies strongly suggests a genetic component to the inheritance of schizophrenia. Monozygotic twins have the high- est concordance rate for schizophrenia. The studies of adopted monozygotic twins show that twins who are reared by adoptive parents have schizophrenia at the same rate as their twin siblings raised by their biological parents. That finding suggests that the genetic influence outweighs the environmental influence. In fur- ther support of the genetic basis is the observation that the more severe the schizophrenia, the more likely the twins are to be concordant for the disorder.
Nevertheless, a particular genetic defect has not been found in all patients with schizophrenia. Many associations between particular chromosomal sites and schizophrenia have been re- ported in the literature since the widespread application of the techniques of molecular biology.
The research literature also reports that a large number of abnormalities appear in the evoked potentials in schizophrenic patients.
Except for the serotonin-dopamine antagonists, the efficacy and the potency of most antipsychotics correlate with their ability to act as antagonists of the dopamine type 2 (D2) (not type 1) receptor.
No well-controlled evidence indicates that any specific family pattern plays a causative role in the development of schizophrenia.
13.20. The answer is B
Men have an earlier onset of schizophrenia than do women.
The peak ages of onset for men are 25 to 35 years. However,
schizophrenia is equally prevalent in men and women.
Hospital records suggest that the incidence of schizophrenia in the United States has probably remained unchanged for the past 100 years and possibly throughout the entire history of the country despite tremendous socioeconomic and population
changes.
13. Schizophrenia 125
126 13. Schizophrenia
Suicide is a common cause of death among schizophrenic patients. About 50 percent of patients with schizophrenia attempt suicide at least once in their lifetimes, and 10 to 15 percent of schizophrenic patients die by suicide during a 20-year follow-up period.
The lifetime prevalence of schizophrenia is usually between 1 and 1.5 percent of the population. Consistent with that range, the National Institute of Mental Health–sponsored Epidemio- logic Catchment Area study reported a lifetime prevalence of 1.3 percent.
13.21. The answer is D
Poor prognostic features in schizophrenia include a family his- tory of schizophrenia; poor premorbid social, sexual, and work histories; and undifferentiated or disorganized features. Fea- tures weighting toward a good prognosis in schizophrenia in- clude mood symptoms (especially depression), a family history of mood disorders, paranoid features, and an undulating course. Table 13.1 presents a summary of the factors used to assess prog- nosis in schizophrenia.
13.22. The answer is D
Studies using magnetic resonance imaging have found evidence in patients with schizophrenia for decreased (not increased) cor- tical gray matter, especially in the temporal cortex; decreased volume of limbic system structures (e.g., the amygdala, hip- pocampus, and parahippocampus); and increased volume of basal ganglia nuclei. These findings are consistent with the find- ings of neuropathological examinations of postmortem tissue, including ultrastructural examination, which in some cases indi- cates cell loss, misalignment of cells, altered intracellular struc- ture, abnormal protein expression, or gliosis.
Table 13.1
Features Weighting toward Good to Poor Prognosis in Schizophrenia
13.23. The answer is E (all)
Prefrontal cortex and limbic system hypotheses are the predomi- nant neuroanatomical theories of schizophrenia. The demonstra- tion of decreased volumes of prefrontal gray or white matter, pre- frontal cortical interneuron abnormalities, disturbed prefrontal metabolism and blood flow, decreased volumes of hippocampal and entorhinal cortex, and disarray or abnormal migration of hippocampal and entorhinal neurons provide strong support for the involvement of these brain regions in the pathophysiology of schizophrenia. In the context of neural circuit hypotheses linking the prefrontal cortex and limbic system, studies demonstrating a relationship between hippocampal morphological abnormalities and disturbances in prefrontal cortex metabolism or function are particularly interesting.
13.24. The answer is E (all)
The hyperdopaminergic hypothesis of schizophrenia arose from two sets of observations of drug action relating to the dopamin- ergic system. Drugs that increase dopamine system activity, such as d-amphetamine, cocaine, levodopa (Larodopa), and methylphenidate (Ritalin), can induce a paranoid psychosis that is similar to some aspects of schizophrenia. Substantial evidence supports the role of postsynaptic dopamine blockade as an ini- tiating factor in a cascade of events responsible for the mode of therapeutic action of antipsychotic drugs.
Functional imaging studies provide indirect evidence of dopamine involvement through the examination of metabolic rates in brain regions where dopamine is an important neuro- transmitter. For example, data confirming metabolic alterations in limbic anatomy are consistent with a disturbance in dopamine metabolism, but it is not possible to determine the extent to which this reflects an alteration of dopamine biochemistry versus an al- teration of any one of a number of interacting neurotransmitter and neuromodulatory systems.
Finally, there is the potential for the relatively precise bio- chemical study of dopamine in postmortem tissue, but here, as with the use of body fluids, sources of artifact and imprecision have been difficult to manage.
Despite these methodological limitations, postmortem stud- ies have reported differences between schizophrenic and control brains. For example, increased concentration of dopamine has been found in the left amygdala (a limbic system structure) in the postmortem brains of patients with schizophrenia. This finding has been replicated, and because it is lateralized, is not likely to be an artifact.
13.25. The answer is E (none)
Structural abnormalities in schizophrenia, such as enlarged ven- tricles and reduced cortical volume, are a prominent feature. It is unclear whether cortical involvement is multifocal or diffuse. Temporal and frontal lobe regions are certainly involved. These abnormalities are present very early in the illness. It is too early to say, however, whether they are present from birth or develop at a later stage. Structural abnormalities may be present in a majority of patients, although the exact percentage is unknown. The prevalence is most apparent when compared with ideally matched genetic control subjects. Structural abnormalities are
 
Good Prognosis
Late onset
Obvious precipitating factors Acute onset
Good premorbid social,
sexual, and work histories Mood disorder symptoms
(especially depressive
disorders) Married
Family history of mood disorders
Good support systems Positive symptoms
Poor Prognosis
Young onset
No precipitating factors
Insidious onset
Poor premorbid social, sexual, and
work histories
Withdrawn, autistic behavior
Single, divorced, or widowed Family history of schizophrenia
Poor support systems
Negative symptoms
Neurological signs and symptoms History of perinatal trauma
No remissions in 3 years
Many relapses
History of assaultiveness
 
correlated to some degree with clinical aspects of the illness, such as cognitive deficits. A key issue remains unresolved: what neurobiological processes account for these enigmatic changes?
13.26. The answer is D
The DSM-IV-TR diagnosis of simple deteriorative disorder (sim- ple schizophrenia) is characterized by a gradual, insidious loss of drive, interest, ambition, and initiative. Hallucinations and delusions are uncommon, and if these symptoms do occur, they do not persist. Patients with simple deteriorative disorder with- draw from contact with other people, tend to stay in their rooms, avoid meeting or eating with other members of the family, stop working, and stop seeing friends. If they are still in school, their grades drop to a low level even if they were consistently high in the past.
These patients avoid going out into the street during the day but may go for long walks alone at 2:00 or 3:00 a.m. They tend to sleep until noon or later after staying up alone most of the night. During the early stages of the illness, they may have many somatic complaints, variously described as fatigue, nervousness, neurosis, psychosomatic disease, and laziness.
Patients are often treated for 1 year or more before the correct diagnosis is made. In many cases, patients with simple deteriora- tive disorder later become homeless. They become increasingly shallow in their emotional responses and are quite content to drift aimlessly through life as long as they are left alone.
Patients with simple deteriorative disorder may resemble per- sonalities of the schizoid type. The distinguishing feature is the disorder makes its appearance at some time during or after puberty and from then on goes on to definite deterioration; per- sonality deviations usually start earlier and remain the same over the years.
To meet the International Classification of Diseases (ICD- 10) diagnostic criteria for simple schizophrenia, the individual must show over a period of at least 1 year all of the following manifestations: (1) a significant and consistent change in the overall quality of some aspect of personal behavior such as loss of drive and interest; (2) gradual appearance and deepening of negative symptoms such as marked apathy; and (3) a marked decline in social, scholastic, or occupational performance.
13.27. The answer is B
The patient in this case is showing signs of avolition, which is one of the most common negative symptoms seen in schizophre- nia. Avolition is defined as a loss of will or drive. Avolition is similar to apathy, and these may be considered closely related, with avolition identifying a deficit in the ability to act and apathy a loss of concern for an idea or task. In the case of the afore- mentioned man, he knows that getting the application for the disability pension is the best for him to get out of the homeless shelter, but he just does not possess the drive to actively pursue this goal. Avolition seems associated with deficits in grooming and hygiene, and it seriously impairs educational and vocational progress. Although often overlooked, this loss of will can be severely disabling for patients.
Anhedonia is the loss of the ability to find or derive pleasure from activities or relationships and may be the most persistent of the
negative symptoms. Although present in depressive disorders, when anhedonia in schizophrenia is part of a negative syndrome, it should not be considered a manifestation of depression.
Affective blunting, consisting of both an inability to under- stand or recognize displays of emotion from others and an inabil- ity to express emotion, is an important predictor of functional im- pairment in schizophrenia. The blunting of expression includes deficits in production, facial expression, gestures, and prosody, and understanding these social signals is similarly impaired. It is more common in men, in people with an early-onset illness, and in people with poor premorbid function, and it predicts lower scores on quality of life measures.
Alogia is a decrease in verbal communication, and it is found in up to 25 percent of people with schizophrenia. Although alo- gia has been considered both the loss of production and a deficit of content with a normal volume of words, only the loss of pro- duction is a negative symptom. The lack of speech production is considered to result from a decreased rate of verbal cognition. In this way, alogia is also conceived of as a “negative thought dis- order.” This loss of production can include an increased latency to response, short verbal responses, and a paucity or complete lack of spontaneous production.
13.28. The answer is B
The presence of hallucinations or delusions is not necessary for a diagnosis of schizophrenia. To make the diagnosis, the patient must demonstrate the presence of two or more of the follow- ing: delusions, hallucinations, disorganized speech, grossly dis- organized or catatonic behavior, or negative symptoms. Symp- toms must persist for at least 6 months. The paranoid type of schizophrenia is characterized by preoccupation with one or more delusions or frequent hallucinations. Classically the para- noid type of schizophrenia is characterized mainly by the pres- ence of delusions of persecution or grandeur. The disorganized type of schizophrenia is characterized by a marked regression to primitive, disinhibited, and unorganized behavior and by the absence of symptoms that meet the criteria for the catatonic type. Disorganized patients are usually active but in an aimless, non- constructive manner.
Brief psychotic disorder is an acute and transient psychotic syndrome. The disorder lasts from 1 day to 1 month, and the symptoms may resemble those of schizophrenia. In addition, the disorder develops in response to a severe psychosocial stressor or group of stressors.
Nonbizarre delusions present for at least 1 month without other symptoms of schizophrenia or a mood disorder warrant the diagnosis of delusional disorder.
Malingering is characterized by the voluntary production and presentation of false or grossly exaggerated physical or psycho- logical symptoms. The presence of a clearly definable goal is the main factor that differentiates malingering from factitious disorders.
13.29. The answer is D
Mr. G has a long-standing illness characterized by periods of hal- lucinations and delusions. He is socially isolated and not work- ing. The most likely diagnosis is schizophrenia.
There is no description of current or past mood symptoms that would make a diagnosis of schizoaffective disorder reasonable,
13. Schizophrenia 127
128 13. Schizophrenia
and the presence of prominent hallucinations is inconsistent with a diagnosis of delusional disorder. The designation of the sub- type of schizophrenia is based on the current episode. Although Mr. G’s attempt to not move superficially resembles catatonia, his openly discussing with the examiner his reasons for remain- ing still is most uncharacteristic of the catatonic subtype. The continued presence of delusions and auditory and visual halluci- nations makes the diagnosis of an undifferentiated subtype more appropriate.
13.30. The answer is A
Mr. G is lucky in having a good response to many different an- tipsychotics. There is no reason to suppose that an atypical agent will help him more than conventional antipsychotics. His multi- ple relapses result from his failure to fill prescriptions because of his drinking, and a long-acting depot medication is not likely to disturb that pattern. It is probable that he would get an injection, start drinking, and not go for his next injection. Alcohol coun- seling is, therefore, of the greatest importance in giving him some stability and freedom from the ongoing cycle of relapse and rehospitalization. Vocational counseling and increased so- cialization may help, but only if his drinking is brought under control.
13.31. The answer is B
Extrapyramidal symptoms may occur as a result of both typ- ical and atypical antipsychotics. Among the atypical antipsy- chotics, risperidone is the most likely to result in extrapyramidal symptoms. This patient has developed tardive dyskinesia. Tar- dive dyskinesia is characterized by involuntary movements such as tongue protrusion, lip smacking, and other perioral move- ments. However, involuntary movements of the limbs, trunk, and head are also possible. Tardive dyskinesia typically appears between 4 months to 4 years after treatment is initiated. Akathisia is characterized by restlessness in which the patient is constantly moving about. Parkinsonism classically results in bradykinesis, pill-rolling tremors, and cogwheel rigidity. Dystonia is charac- terized by sustained muscle contractions and stiffness. Dystonia occurs 4 hours to 4 days after treatment.
Patients show less regression of their mental faculties, emotional responses, and behavior than do patients with other types of schizophrenia. Patients are typically tense, suspicious, guarded, reserved, and sometimes hostile or aggressive. They can occa- sionally conduct themselves adequately in social situations. This subtype has a relatively favorable prognosis.
Catatonic schizophrenia is characterized by a marked dis- turbance in motor function; this disturbance may involve stu- por, negativism, rigidity, excitement, or posturing. Associated features include stereotypies, mannerisms, and waxy flexibility. Mutism is particularly common. During catatonic excitement, patients need careful supervision to prevent from hurting them- selves or others. Medical care may be needed because of malnu- trition, exhaustion, hyperpyrexia, or self-inflicted injury.
Residual schizophrenia is characterized by continuing evi- dence of schizophrenic disturbance in the absence of a com- plete set of active symptoms or of sufficient symptoms to meet the diagnosis of another type of schizophrenia. Emotional blunt- ing, social withdrawal, eccentric behavior, illogical thinking, and mild loosening of associations are common. The condition can be chronic or may be a transition to a complete remission of the illness.
Disorganized schizophrenia is characterized by a marked re- gression to primitive, disinhibited, and unorganized behavior and by the absence of symptoms that meet the criteria for the cata- tonic type. The onset is generally early, occurring before age 25 years. Patients are usually active but in an aimless, noncon- structive manner. Their thought disorders are pronounced, and their contact with reality is poor. Their personal appearance is di- sheveled, and their social behavior and their emotional responses are inappropriate. They often burst into laughter without any ap- parent reason. Incongruous grinning and grimacing are common in these patients, whose behavior is best described as silly or fatu- ous. A continuous course and a poor prognosis are characteristic of this subtype.
Undifferentiated schizophrenia is defined to include patients who meet criteria for schizophrenia but cannot be clearly clas- sified into any of the other subtypes. Its diagnosis is through exclusion of the other subtypes. When patients with schizophre- nia are carefully diagnosed, a large number of patients are found to meet criteria for this subtype.
Answers 13.37–13.41 13.37. The answer is C 13.38. The answer is D 13.39. The answer is B 13.40. The answer is A
13.41. The answer is E
Occasionally, patients with schizophrenia create a completely new expression, a neologism, when they need to express a con- cept for which no ordinary word exists.
A woman with schizophrenia who had been hospitalized for several years kept repeating (in an otherwise quite rational
Answers 13.32–13.36 13.32. The answer is 13.33. The answer is 13.34. The answer is 13.35. The answer is
D B A B
13.36. The answer is C
The DSM-IV-TR classifies the subtypes of schizophrenia as paranoid, disorganized, catatonic, undifferentiated, and resid- ual based predominantly on clinical presentation. Paranoid schizophrenia is characterized by the presence of one or more delusions and frequent auditory hallucinations. Delusions do not need to be persecutory in nature. The contents of the auditory hallucinations are often related to the contents of the delusions.
conversation) the word “polamolalittersjitterstittersleelitla.” Her psychiatrist asked her to spell it out, and she proceeded to ex- plain the meaning of the various components, which she insisted were to be used as one word. “Polamolalitters” was intended to recall the disease poliomyelitis because the patient wanted to in- dicate that she believed she was suffering from a serious disease affecting her nervous system; the component “litters” stood for untidiness or messiness, the way she felt inside; “jitterstitters” reflected her inner nervousness and lack of ease; and “leelita” was a reference to the French le lit la` (that bed there), meaning that she both depended on and felt handicapped by her illness. That single neologistic production thus enabled the patient to express—in a condensed, autistic manner—information about her preoccupations and apprehensions that otherwise would have taken a whole paragraph to explain in common language.
It is assumed that the disorders of language reflect an underly- ing disorder of thinking. A variety of features have been reported by clinicians for the past 100 years as characteristic of this syn- drome. These include the loss of the logical relations between antecedent and subsequent associations that is termed loosening of associations. For example, during a sentence completion test, a patient is asked to complete the sentence “The man fell on the street . . . ” The patient responds “because of World War I.” Although the thought of falling might be associated with falling in combat, it was an inappropriate association for the stimulus. Words can be combined on the basis of sound rather than on meaning (clang association). Verbigeration involves the use of words in a stereotypically repetitive fashion. This rare symptom is found almost exclusively in chronic and very regressed pa- tients with schizophrenia. It consists of the senseless repetition of the same words or phrases, and it may go on for days. Sim- ilar to neologisms and echolalia, verbigeration is a rare symp- tom today and is almost restricted to long-term institutionalized schizophrenia patients. Many psychiatrists working with patients with schizophrenia in the community may never encounter these manifestations of deterioration.
Echolalia involves the repetition of the examiner’s words.
Examiner: How did you sleep last night?
Patient: I slept well last night.
Examiner: Can you tell me the name of your head nurse? Patient: The name of my head nurse is Miss Brown.
Echolalia seems to signal two facts: patients are aware of some shortcomings in their ideation, and they are striving to maintain active rapport with the interviewer. They act much like someone learning a new language who answers the teacher’s questions with as many of the teacher’s words in the strange language as they can possibly manage.
Thought blocking involves the sudden and inexplicable block- ing of thoughts manifested by the patient’s inability to speak.
Answers 13.42–13.46 13.42. The answer is E 13.43. The answer is A
13.44. The answer is D
13.45. The answer is C
13.46. The answer is B
Catatonia is a syndrome of psychomotor disturbance that is char- acterized by periods of physical rigidity, negativism, or stupor. Catatonic negativism refers to the automatic resistance to at- tempts to move limbs, postures, or direct ambulation. For exam- ple, a patient may continually lie in the fetal position and resist any attempts to move him or her from said position. Catatonic posturing involves patients holding odd or exaggerated postures for prolonged periods (Figure 13.1). For example, patients may stand on one foot or sit with their feet under them for hours. Catatonic rigidity is similar and refers to patients holding sim- ple, fixed, rigid postures. Some people with catatonic stupor can be obviously awake but immobile without rigidity and will sit or lie in these postures until moved. For example, patients may sit in a chair all day, just staring into space, and will not move until a nurse or a loved one moves them to put them to bed at night. In catatonia, catalepsy refers to waxy flexibility or cerea flexibilitas, the tendency of patients to hold postures that are manipulated by others. For example, patient can be restrained with their hands behind their back and will continue to keep their hands in that position even after the restraints are removed.
FIGURE 13.1
A patient with chronic schizophrenia stands in a catatonic position. He maintained this uncomfortable position for hours. (Courtesy of Emil Kraepelin, MD.)
13. Schizophrenia 129

130 13. Schizophrenia
Patients may sometimes mimic the motor behaviors of others, which is referred to as echopraxia (i.e., a patient crossing his or her legs after the physician does).
Answers 13.47–13.51 13.47. The answer is A 13.48. The answer is C 13.49. The answer is B 13.50. The answer is A
13.51. The answer is B
Emil Kraepelin described two major patterns of primary in- sanity: manic depressive psychosis and dementia praecox. The two syndromes were divided based on the long-term progno- sis and course of the illnesses. It was believed that affective psychosis (i.e., manic depressive psychosis) had a nondeterio- rating course, but dementia praecox was deemed as deteriorat- ing and irreversible. Kraepelin believed that dementia praecox was a loss of the inner unity of intellect, emotion, and voli- tion. He went further by grouping illnesses that were previously described as paranoia, catatonia, and hebephrenia within de- mentia praecox and included these as the original subtypes of schizophrenia.
Eugen Bleuler first coined the term schizophrenia in 1911. He thought the name denoted the “splitting” of psychotic func- tions, which he believed to be the basis of the illness. Bleuler was less interested in the course of the illness, as Kraepelin had been, and was much more interested in the understanding of the psychological mechanisms underlying the disease process. He divided the symptoms into primary and secondary symptoms. Primary symptoms included what is known as the four As: abnor- mal associations, autistic behavior and thinking, abnormal affect, and ambivalence. Secondary symptoms include hallucinations, delusions, social withdrawal, and diminished drive. Bleuler also saw symptoms of schizophrenia in a continuum with normal behavior.
Kurt Schneider (1887–1967) contributed a description of first- and second-rank symptoms of schizophrenia. First-rank symptoms include symptoms such as various forms of halluci- nations, thought withdrawal, and delusional thoughts. Second- rank symptoms include perplexity and mood changes along with others (Table 13.2). Schneider emphasized that in patients who showed no first-rank symptoms, the disorder could be diagnosed exclusively on the basis of second-rank symptoms and an other- wise typical clinical appearance. Clinicians frequently ignore his warnings and sometimes see the absence of first-rank symptoms during a single interview as evidence that a person does not have schizophrenia.
Answers 13.52–13.56 13.52. The answer is D
1.
2.
Table 13.2
Kurt Schneider Criteria for Schizophrenia
First-rank symptoms
a. Audible thoughts
b. Voices arguing or discussing or both
c. Voices commenting
d. Somatic passivity experiences
e. Thought withdrawal and other experiences of influenced
thought
f. Thought broadcasting
g. Delusional perceptions
h. All other experiences involving volition made affects and
made impulses
Second-rank symptoms
a. Other disorders of perception
b. Sudden delusional ideas
c. Perplexity
d. Depressive and euphoric mood changes e. Feelings of emotional impoverishment
f. “. . . and several others as well”
  
13.53. The answer is E 13.54. The answer is A 13.55. The answer is B
13.56. The answer is C
In vocational rehabilitation, a variety of methods and settings are used to help patients regain old skills or develop new ones. Impairment of vocational role function is a common complica- tion related to schizophrenia. The unemployment rates among this population are much higher than among the general popu- lation and among others with mental illnesses. However, many patients with schizophrenia frequently express a desire to work. Many approaches to help people with schizophrenia gain em- ployment have developed. Many involve train-then-place models in which extensive pre-employment training is used to evaluate and improve the person’s “readiness” to work.
Cognitive behavioral therapy has been used in schizophrenia patients to improve cognitive distortions, reduce distractibility, and correct errors in judgment. In cases of auditory hallucina- tions, the therapist’s first task is to gain a clear understanding of the person’s experience of the hallucinations. From there, the therapist can begin to formulate a plan for helping the patient reduce or cope better with the hallucinations. Patients who may benefit generally have some insight into their illness.
Token economies are behavioral reinforcement programs based on the principles of social learning. The overall goal is to promote learning of behaviors that will enhance each patient’s functioning and allow him or her to make a transition to less restrictive care environments. Token economies are based on the premise that all psychiatric patients can learn to build on their strengths when treatment is offered in a positive, creative, and systematic manner. These programs also emphasize the impor- tance of positively reinforcing desired behaviors, thereby im- proving cooperation in the treatment setting; increasing active
participation in treatment; and decreasing the frequency of prob- lem behaviors, especially dangerous behaviors that traditionally lead to seclusion and restraint.
Social skills training emphasizes the role of behavioral re- hearsal in skill development rather than discussion. The primary modality of social skills training is through role play of simulated conversations. The trainer first provides instructions on how to perform the skill and then models the behavior to demonstrate how it is performed. After identifying a relevant social situation in which the skill might be used, the patient engages in a role play with the trainer. The trainer next provides feedback and positive reinforcement followed by suggestions for how the response can
be improved. The sequence of role play followed by feedback and reinforcement is repeated until the patient can perform the response adequately.
The Assertive Community Treatment (ACT) program was de- veloped for delivery of services for persons with chronic mental illness. Patients are assigned to one multidisciplinary team (e.g., case manager, psychiatrist, nurse, general physicians). The team has a fixed caseload of patients and delivers all services when and where needed by the patient, 24 hours a day, 7 days a week. ACT programs can effectively decrease the risk of rehospitaliza- tion for persons with schizophrenia, but they are labor-intensive and expensive programs to administer.
13. Schizophrenia 131

14
Other Psychotic Disorders

There are six disorders within the category of other psychotic disorders: schizophreniform, schizoaffective, delusional, shared psychotic, brief psychotic, and psychotic disorder not otherwise specified. These disorders occur less frequently and are less un- derstood than schizophrenia and mood disorders with psychotic features. They can be difficult to distinguish from other forms of psychosis. However, these disorders can have profound short- and long-term psychosocial consequences, and it is important to be able to identify and treat patients who have them.
Schizophreniform disorder is conceptualized as a variant of schizophrenia. Patients with this condition are floridly psychotic with a prodromal, active and residual phase between 1 and 6 months. If the duration of illness extends beyond 6 months, the diagnosis might be changed to schizophrenia. Risk factors include unemployment; residence in a metropolitan area; low income; being separated, widowed, or divorced; young age; low education; living with nonrelatives; obstetric and early neonatal complications; childhood emotional problems; and cannabis use.
Schizoaffective disorder combines the symptoms of mood disorders and schizophrenia. It may be a neurodevelopmental disorder, and gender differences parallel those seen in mood dis- orders. Although almost 85 percent of women experience some type of mood disturbance during the postpartum period, postpar- tum psychosis is rare. Students should be familiar with it because infanticide may occur. Hormonal hypotheses have been posited to explain its etiology, which remains unknown, however.
Delusional disorders, once referred to as paranoid disorders, are diagnosed when the individual reports nonbizarre delusions for more than 1 month without prominent hallucinations and with a relative preservation of functioning. Nonbizarre delusions are plausible, understandable, and derive from ordinary life experi- ence. The course appears to be less chronic, with less associated deterioration in functioning than the course of schizophrenic pa- tients. Shared psychotic disorder, commonly referred to as a folie a deux, refers to the condition in which two individuals with a close and generally long-term relationship share the same delu- sional belief, although it may involve more than two individuals, including entire families.
Brief psychotic disorder is a psychotic condition involving the sudden onset of psychotic symptoms that lasts one day or more but less than 1 month. Remission is full, and the individual returns to the premorbid level of functioning.
Knowledge of the culture-bound syndromes is increasingly important. The growing wave of immigration from developing countries to the United States over the past few decades has meant that doctors in the United States need to acquire a basic understanding of the formulations of health and illness in the culture from which their patients come. The course of these syn- dromes is generally favorable, and most present as self-limiting episodes after stressful events.
Students should study the questions and answers below for a useful review of these disorders.
HELPFUL HINTS
Students should know the psychotic syndromes and other terms listed here.
amok
Arctic hysteria
atypical psychoses autoscopic psychosis bouffe ́e de ́lirante
brief psychotic disorder conjugal paranoia Cotard’s syndrome culture-bound
syndromes Cushing’s syndrome delusional disorder double insanity erotomania
Fregoli’s syndrome Ganser’s syndrome heutoscopy
koro
Gabriel Langfeldt lithium lycanthropy mental status
examination mood-congruent and
-incongruent
psychotic features nihilistic delusion paranoid states
paraphrenia
piblokto
postpartum blues postpartum psychosis postpsychotic depressive
disorder of
schizophrenia pseudocommunity psychodynamic
formulation psychosis of association psychotic disorder not
otherwise specified reduplicative paramnesia
schizoaffective disorder schizophreniform
disorder
Daniel Paul Schreber SES
shared psychotic
disorder significant stressor simple schizophrenia suicidal incidence suk-yeong
TRH stimulation test wihtigo psychosis
132
▲▲ ▲▲▲ ▲▲▲▲▲▲
▲▲▲▲▲ ▲▲ ▲▲ ▲
▲▲▲▲▲▲▲▲ ▲ ▲▲
▲▲▲▲▲▲▲▲▲ ▲▲▲▲
QUESTIONS
Directions
Each of the questions or incomplete statements below is followed by five suggested responses or completions. Select the one that is best in each case.
. 14.1. The delusion that a familiar person has been replaced by an imposter is referred to as
A. intermetamorphosis
B. Cotard syndrome
C. Capgras syndrome.
D. olfactory reference syndrome E. conjugal paranoia
. 14.2. Which of the following statements is true about brief psychotic disorder?
A. Approximately 10 percent of patients diagnosed re- tain the diagnosis.
B. Fifty percent of the cases evolve into either schizophrenia or major mood disorder.
C. There are clear distinguishing features between brief psychotic disorder and acute-onset schizophrenia on initial presentation.
D. Poor prognosis is associated with emotional turmoil.
E. None of the above
. 14.3. The differential diagnosis of brief psychotic disorder in- cludes
A. malingering
B. severe personality disorders
C. substance-induced psychotic disorder
D. psychoticdisorderduetoageneralmedicalcondition E. all of the above
. 14.4. Allofthefollowingareassociatedwithagoodprognosis in a brief psychotic disorder except
A. confusion during psychosis B. severe precipitating stressor C. sudden onset of symptoms D. few premorbid schizoid traits E. no affective symptoms
. 14.5. Mrs. P is a 47-year-old, divorced, unemployed woman who lived alone and who experienced chronic psychotic symptoms despite treatment with olanzapine (Zyprexa) 20 mg per day and citalopram (Celexa) 20 mg per day. She believed that she was getting messages from God and the police department to go on a mission to fight against drugs. She also believed that an organized crime group was trying to stop her in this pursuit. The onset of her ill- ness began at age 20 years, when she experienced the first of several depressive episodes. She also described peri- ods when she felt more energetic; was more talkative; had a decreased need for sleep; and was more active, some- times cleaning her house throughout the night. About 4 years after the onset of her symptoms, she began to hear
“voices” that became stronger when she was depressed but were still present and disturbed her even when her mood was euthymic.
The case of Mrs. P is a classic presentation of
A. schizophreniform disorder B. schizoaffective disorder
C. delusional disorder
D. brief psychotic disorder
E. acute and transient disorders
14.6. In schizoaffective disorder, all of the following variables indicate a poor prognosis except
A. early onset
B. depressive type
C. bipolar type
D. no precipitating factor
E. a predominance of psychotic symptoms
14.7. Folie a ́ deux is another name for
A. erotomania
B. brief psychotic disorder
C. shared psychotic disorder
D. delusional disorder, persecutory type E. schizoaffective disorder
14.8. True statements concerning the treatment of shared psy- chotic disorder include all of the following except
A. Recovery rates have been reported to be as low as 10 percent.
B. The submissive person commonly requires treatment with antipsychotic drugs.
C. Psychotherapy for nondelusional members of the pa- tient’s family should be undertaken.
D. Separation of the submissive person from the domi- nant person is the primary intervention.
E. The submissive person and the dominant person usu- ally move back together after treatment.
14.9. The best-documented risk factor for delusional disorder is
A. family history
B. advanced age
C. social isolation
D. sensory impairment E. recent immigration
14.10. Delusional disorder may include
A. auditory hallucinations B. olfactory hallucinations C. tactile hallucinations D. visual hallucinations
E. all of the above
14.11. Delusional disorder
A. usually begins by age 20 years
B. is more common in men than in women
14. Other Psychotic Disorders 133
134 14. Other Psychotic Disorders
C. is less common than schizophrenia D. is an early stage of schizophrenia E. is caused by frontal lobe lesions
. 14.12. Mr. T was 42 years old when he was brought to a psy- chiatrist for evaluation. He lived with his parents and four brothers. His chief complaint was that his business partner was trying to force him to quit by giving him too much work. Although he appeared outwardly calm, Mr. T complained of nervousness and restlessness. He also noted an inability to sleep at night, believing that his neighbor was making noise on purpose. Mr. T’s brother corroborated that the neighbor was being quite noisy at night and did not seem to respond to requests to be qui- eter. The psychiatrist prescribed hypnotic and anxiolytic medication.
Three days later, the psychiatrist received a phone call from the patient’s brother, who requested that Mr. T be seen emergently. Upon arriving at the psychiatrist’s office, Mr. T appeared quite anxious and perplexed. He stated that the noises had grown louder and were now present throughout the day. He was also convinced that other people besides his neighbor and his business partner were trying to harm him. Mr. T’s brother now reported that Mr. T’s beliefs had no basis in reality.
This case illustrates which of the following difficulties in diagnosing delusional disorder?
A. The initial report appears believable but is actually a delusion.
B. The initial report appears delusional but is actually true.
C. The patient believes the physician is in on the plot against him.
D. The patient is unwilling to provide information.
E. None of the above
. 14.13. Evidence that suggests delusional disorder is a separate entity from schizophrenia or mood disorders includes
A. epidemiological data
B. family or genetic studies
C. natural history of the disorder D. premorbid personality data
E. all of the above
. 14.14. True statements about patients with delusional disorder, erotomanic type, include:
A. They exhibit what has been called “paradoxical con- duct.”
B. The course of the disorder is invariably chronic.
C. Separation from the love object is usually not an
effective treatment.
D. Women predominate in forensic populations.
E. All of the above
. 14.15. The following statements about psychotherapeutic treat- ment of delusional disorders is true except
A. It is essential that the patient trust the therapist.
B. The therapist should not challenge the patient’s delu-
sions.
C. Empathy toward the patient’s struggle with the delu-
sion is useful.
D. Individual therapy seems to be less effective than
group therapy.
E. When available, family members should be involved
in therapy.
14.16. Of the following somatic treatments for delusional disorder, which is considered the least likely to be successful?
A. Dopamine receptor antagonists
B. Electroconvulsive treatment
C. Selective serotonin reuptake inhibitors
D. Serotonin-dopamine antagonists
E. All of the above are considered equally effective
14.17. The postpartum blues
A. occurs in up to 50 percent of women after childbirth B. is self-limited
C. begins shortly after childbirth and lessens in severity
over the course of 1 week D. is considered to be normal E. all of the above
14.18. Postpartum psychosis
A. occurs more commonly in multigravida women
B. is rarely correlated with perinatal complications
C. almost always begins within 8 weeks of delivery
D. usuallyoccursabruptly,withnoprodromalpsychotic
symptoms
E. is essentially an episode of a psychotic disorder
14.19. All of the following are true statements about postpartum psychosis except
A. Generally, it is not considered a psychiatric emer- gency.
B. Delusional material may involve the idea that the baby is dead.
C. The risk is increased if the patient had a recent mood disorder.
D. Hallucinations involve voices telling the patient to kill her baby.
E. It is found in one to two per 1,000 deliveries.
14.20. Moststudiesofnormalpregnantwomenindicatethatthe percentage who report the “blues” in the early postpartum period is about
A. 10 percent B. 25 percent C. 50 percent D. 75 percent E. 100 percent
. 14.21. Attaque de nervios
A. is usually associated with acute fear
B. usually results in a deteriorating course
C. is most common in Puerto Ricans
D. usually has no precipitating stressful event
E. usually features a sense of being out of control
. 14.22. Erotomania is also referred to as
A. Ganser’s syndrome
B. Fregoli’s syndrome
C. Cotard’s syndrome
D. Cle ́rambault’s syndrome E. Capgras syndrome
. 14.23. Puerperal psychosis
A. is most likely to occur in patients with a previous history of the disorder
B. usually does not occur until 2 to 3 months postpar- tum
C. has a prevalence of 10 to 15 percent
D. usually has insidious onset
E. all of the above
. 14.24. Acute and transient psychotic disorder
A. can be definitively diagnosed without an extensive past history
B. is easily recognized early in its course
C. is often only definitively diagnosed retrospectively
D. may have a better outcome with acute onset
E. has a higher age of onset in developing countries
. 14.25. A13-year-oldgirlpresentstoclinicforunusualbehavior. Her father states that for the past 3 months, his daughter claims she has been “communicating with unidentified flying objects from outer space.” During this period, it has become increasingly difficult to communicate with his daughter, and she has become an introvert. What is the most likely diagnosis?
A. Schizophrenia
B. Brief psychotic disorder C. Schizoaffective disorder D. Schizophreniform disorder E. None of the above
Directions
Each group of questions below consists of lettered headings fol- lowed by a list of numbered phrases or statements. For each numbered phrase or statement, select the one lettered heading that is most associated with it. Each lettered heading may be selected once, more than once, or not at all.
14. Other Psychotic Disorders 135 Questions 14.26–14.30
A. Brief psychotic disorder
B. Schizoaffective disorder
C. Schizophreniform disorder
D. Acute and transient psychotic disorders E. Delusional disorder
14.26. Mood symptoms are present
14.27. Return to baseline state within 6 months
14.28. May progress to schizophrenia
14.29. Prominent hallucinations are absent
14.30. The presence of acute stress within 2 weeks of onset
Questions 14.31–14.35
A. Delusional disorder
B. Major depressive episode
C. Manic episode
D. Paranoid personality disorder E. Schizophrenia
14.31. Psychomotor retardation
14.32. Thought broadcasting
14.33. Easy distractibility with an elevated, expansive, or irri-
table mood
14.34. Nonbizarre persecutory or grandiose delusions
14.35. Suspiciousness and mistrust of people without psychotic
symptoms
Questions 14.36–14.40
A. Delusions of guilt
B. Delusions secondary to perceptual disturbances C. Grandiose delusions
D. Bizarre delusions of being controlled
E. Delusions of jealousy
. 14.36. Cognitive disorders
. 14.37. Delusional disorder
. 14.38. Depressive disorders
. 14.39. Mania
. 14.40. Schizophrenia
ANSWERS
14.1. The answer is C
Capgras syndrome is the belief that a familiar person has been replaced by an imposter. Capgras syndrome is a member of the unspecified type of delusional disorders. There are variants of Capgras syndrome such as the delusion that familiar persons can assume the guise of strangers (Fre ́goli’s phenomenon) and the very rare delusion that familiar persons can change them- selves into other persons at will (intermetamorphosis). Patients with Cotard syndrome (also known as de ́lire de ne ́gation or ni- hilistic delusional disorder) complain of having lost not only possessions, status, and strength, but also their heart, blood, and intestines. Each delusional disorder of the unspecified type is not
136 14. Other Psychotic Disorders
only rare but may be associated with schizophrenia, dementia, epilepsy, and other organic disorders.
Olfactory reference syndrome is a subcategory of delusional disorder, somatic type, in which the patient has delusions of foul body odors or halitosis. It differs from other delusions of the somatic type in that it has an earlier age of onset (mean of 25 years), male predominance, unmarried, and absence of past psychiatric treatment.
Conjugal paranoia is a delusional disorder of infidelity in which the patient has delusions that a spouse is being unfaithful. The eponym Othello syndrome has been used to describe morbid jealousy that can arise from multiple concerns. The delusion usually affects men, often those with no prior psychiatric illness. It may appear suddenly and serve to explain a host of present and past events involving the spouse’s behavior. The condition is difficult to treat and may diminish only on separation, divorce, or death of the spouse.
14.2. The answer is B
The course of brief psychotic disorder is found in the diagnostic criteria of the Diagnostic and Statistical Manual of Mental Dis- orders (DSM-IV-TR). It is a psychotic episode that lasts more than 1 day but less than 1 month, with eventual return to pre- morbid level of functioning. Approximately 50 percent patients diagnosed with brief psychotic disorder retain this diagnosis; the other 50 percent evolve into either schizophrenia or a major affective disorder. There are no apparent distinguishing features among brief psychotic disorder, acute-onset schizophrenia, and mood disorders with psychotic features on initial presentation. Several prognostic features have been proposed to characterize the illness, but they are inconsistent across studies. The good prognostic features are similar to those found in schizophreni- form disorder, including acute onset of psychotic symptoms, confusion or emotional turmoil at the height of the psychotic episode, good premorbid functioning, the presence of affective symptoms, and a short duration of symptoms. There is a rela- tive dearth of information on the recurrence of brief psychotic episodes, however, so the course and prognosis of this disorder have not been well characterized.
14.3. The answer is E (all)
Sharing rapid onset of symptoms, brief psychotic disorder must be differentiated from substance-induced psychotic disorders and psychotic disorders due to a general medical condition. A thorough medical evaluation, including a physical examination, laboratory studies, and brain imaging, helps rule out many of those conditions. With only cross-sectional information, brief psychotic disorder is difficult to differentiate from other types of functional psychosis.
The relationship between brief psychotic disorder and both schizophrenia and affective disorders remains uncertain. The DSM-IV-TR has made the distinction between brief psychotic disorder and schizophreniform disorder clearer by now requiring a full month of psychotic symptoms for the latter. If psychotic symptoms are present longer than 1 month, the diagnoses of schizophreniform disorder, schizoaffective disorder, schizophre- nia, mood disorders with psychotic features, delusional disorder, and psychotic disorder not otherwise specified need to be enter-
tained. If psychotic symptoms of sudden onset are present for less than 1 month in response to an obvious stressor, the diag- nosis of brief psychotic disorder is strongly suggested. Other diagnoses to differentiate include factitious disorder, malinger- ing, and severe personality disorders, with consequent transient psychosis possible.
14.4. The answer is E
Good prognostic features for brief psychotic disorders include good premorbid adjustment, few premorbid schizoid traits, a se- vere precipitating stressor, the sudden onset of symptoms, affec- tive symptoms, confusion and perplexity during psychosis, little affective blunting, a short duration of symptoms, and the absence of schizophrenic relatives.
14.5. The answer is B
The case of Mrs. P demonstrates a “classic” presentation of schizoaffective disorder. Schizoaffective disorder has features of both schizophrenia and mood disorders. Patients are diagnosed with schizoaffective disorder if they fit into one of six categories: (1) patients with schizophrenia who have mood symptoms, (2) patients with mood disorder who have symptoms of schizophre- nia even when euthymic, (3) patients with both mood disorder and schizophrenia, (4) patients with a third psychosis unrelated to schizophrenia and mood disorder, (5) patients whose disorder is on a continuum between schizophrenia and mood disorder, and (6) patients with some combination of the above. In the case of Mrs. P, clear depressive and hypomanic episodes are present in combination with continuous psychotic illness.
Schizophreniform disorder is an acute psychotic disorder that has a rapid onset and lacks a long prodromal phase. Symptoms must last at least 1 month but less than 6 months. In the case of Mrs. P, symptoms presented for more than 6 months and in- clude depressive and manic symptoms that are not indicated in the diagnostic criteria for schizophreniform disorder. Delusional disorder is diagnosed when a person exhibits nonbizarre delu- sions for at least 1 month’s duration that cannot be attributed to other psychotic disorders. The mood of patients with delu- sional disorders is consistent with the content of the delusions (i.e., a person with persecutory delusions is suspicious). Mrs. P experiences depressive and manic episodes that do not appear to coincide with the content of her delusions. Brief psychotic disorder is a psychotic condition that involves the sudden onset of psychotic symptoms and lasts 1 day or more but less than 1 month. It can also include the presence of marked stressors (i.e., war, torture, serious medical illness). Mrs. P’s symptoms present too long to qualify for this diagnosis. Acute and transient psychotic disorders are defined as psychotic conditions with an onset within 2 weeks and full remission within 1 to 3 months. It is also a nonaffective psychotic disorder; therefore, Mrs. P’s mood symptoms and the duration of the illness rule out acute and transient psychotic disorders.
14.6. The answer is C
The course and the prognosis of schizoaffective disorder are vari- able. As a group, patients with this disorder have a prognosis in- termediate between patients with schizophrenia and patients with
mood disorders. Patients with schizoaffective disorder, bipolar type, typically have a better prognosis. A poor prognosis is as- sociated with the depressive type of schizoaffective disorder. A poor prognosis is also associated with the following variables: no precipitating factor, a predominance of psychotic symptoms, early or insidious onset, a poor premorbid history, and a positive family history of schizophrenia.
14.7. The answer is C
Folie a ́ deux is another name for shared psychotic disorder. Other references for the disorder are shared paranoid disorder, induced psychotic disorder, folie impose, and double insanity. The disor- der is characterized by the transfer of delusions from one person to another. The key features of the disorder are the unquestion- ing acceptance of the other individual’s delusional beliefs and the temporal sequence of development of the disorder, with one of the individuals having an earlier onset. Both persons are closely associated for a long time and typically live together in relative social isolation. Shared psychotic disorder usually involves two individuals but may involve more than two individuals, includ- ing entire family units. Persecutory delusional beliefs are most commonly seen in shared psychotic disorder, comprising about 70 percent of patients in one study; however, religious, grandiose, and somatic delusions may also be observed.
14.8. The answer is C
Psychotherapy for nondelusional members of the patient’s family is usually not necessary. Clinical reports vary, but the prognosis is guarded—recovery rates have been reported to be as low as 10 percent. The submissive person often requires treatment with antipsychotic drugs, as does the dominant person. Separation of the submissive person from the dominant person is the primary intervention. The submissive person and the dominant person usually move back together after treatment.
14.9. The answer is A
The cause of delusional disorder is unknown. The epidemiologi- cal and clinical literature suggests that certain risk factors may be relevant to the etiology and deserve further research elaboration. These risk factors are found in Table 14.1. Whether they are risk predictors or simply characteristics or markers of the disorder is unknown. Familial psychiatric disorder, including delusional disorder, is the best documented risk factor at present.
14.10. The answer is E (all)
Generally, in delusional disorders, the patient’s delusions are well systematized and have been developed logically. The per-
Table 14.1
Risk Factors Associated with Delusional Disorder
Advanced age
Sensory impairment or isolation
Family history
Social isolation
Personality features (e.g., unusual interpersonal sensitivity) Recent immigration
son may experience auditory or visual hallucinations, but these are not prominent features. Tactile or olfactory hallucinations may be present and prominent if they are related to the delu- sional content or theme. Examples are the sensation of being infested by bugs or parasites, associated with delusions of in- festation, and the belief that one’s body odor is foul, associated with somatic delusions. The person’s behavioral and emotional responses to the delusion appear to be appropriate. Impairment of functioning is not marked and personality deterioration is min- imal, if it occurs at all. General behavior is neither obviously odd nor bizarre.
14.11. The answer is C
Delusional disorder is less common than schizophrenia. Its prevalence in the United States is estimated to be 0.03 percent— in contrast with 1 percent for schizophrenia and 5 percent for mood disorders.
The neuropsychiatric approach to delusional disorder derives from the observation that delusions are a common symptom in many neurological conditions, particularly those involving the limbic system and the basal ganglia. No evidence indicates that the disorder is caused by frontal lobe lesions. Long-term follow- up of patients with delusional disorder has found that their di- agnoses are rarely revised as schizophrenia or mood disorders; hence, delusional disorder is not an early stage of schizophre- nia or mood disorders. Moreover, delusional disorder has a later onset than does schizophrenia or mood disorders. The mean age of onset is 40 years; the disorder does not usually begin by age 20 years. The disorder is slightly more common in women than in men.
14.12. The answer is A
The initial report in this case appears believable but is actually a delusion. Upon the first evaluation, Mr. T’s complaint about noise seemed believable and was verified by his brother, but by the second visit 3 days later, it was clear that Mr. T’s complaints were delusions.
Delusions are false fixed beliefs not in keeping with the cul- ture. The diagnosis of delusional disorder is made when a person exhibits nonbizarre delusions of at least 1 month’s duration that cannot be attributed to other psychiatric disorders. The first step in the clinical assessment is establishing whether a delusion ex- ists. Some statements that initially seem to be delusional may, in fact, be true. In contrast, as in the above case, reports of circum- stances that initially seem believable may be clearly identified as delusions as the symptoms worsen, the delusion becomes less encapsulated, or more information comes to light. Certain behav- ioral features such as hypersensitivity, guardedness, evasiveness, secretiveness, litigiousness, overly detailed descriptions, hostil- ity, and humorlessness are characteristic of delusional patients yet are not pathognomonic. The patient may also be secretive and vague and may not voluntarily provide the clinician with necessary details. At other times, the patient may see no need to convey information, believing that the interviewer is part of the “conspiracy” and already knowledgeable about its specifics. For example, the patient may view the interview as a staged game in which both parties know the hidden meanings behind the overt text. As a result, certain phrases are commonly heard
14. Other Psychotic Disorders 137
  
138 14. Other Psychotic Disorders
from individuals with persecutory ideas in the context of delu- sional disorder (e.g., “Don’t play games with me,” “You know what ‘they’ have been doing”).
14.13. The answer is E (all)
An issue that is central to attributing causation is whether delu- sional disorder represents a separate group of conditions or is an atypical form of schizophrenic and mood disorders. The relevant data come from a limited number of studies and are inconclu- sive. Epidemiology data suggest that delusional disorder is a separate condition; it is far less prevalent than schizophrenic or mood disorders; the age of onset is later than in schizophrenia, although men tend to experience the illness at earlier ages than women; and the sex ratio is different from that of mood disorder, which occurs disproportionately among women. Findings from family or genetic studies also support the theory that delusional disorder is a distinct entity. If delusional disorder is simply an unusual form of schizophrenic or mood disorders, the incidence of these latter conditions in family studies of delusional disor- der patient probands should be higher than that of the general population. However, this has not been a consistent finding. A recent study concluded that patients with delusional disorder are more likely to have family members who show suspiciousness, jealousy, secretiveness, even paranoid illness, than families of controls. Other investigative efforts have found paranoid per- sonality disorder and avoidant personality disorder to be more common in the relatives of patients with delusional disorder than in the relatives of control subjects or of schizophrenic patients. A recent study documented modest evidence for an increased risk of alcoholism among the relatives of patients with delusional disorder compared with probands with schizophrenia, probands with psychotic disorder not otherwise specified, and probands with schizophreniform disorder.
Investigations into the patient’s natural history also lend sup- port to the suggestion that delusional disorder is a distinct cat- egory: age of onset appears to be later than in schizophrenia, and outcome is generally better for delusional patients with dis- order than for those with schizophrenia. Although fraught with methodological shortcomings, premorbid personality data indi- cate that people with schizophrenia and those with delusional disorder differ early in life. The former are more likely to be introverted, schizoid, and submissive and the latter extroverted, dominant, and hypersensitive. Patients with delusional disorder may have below-average intelligence. Precipitating factors, es- pecially related to social isolation, conflicts of conscience, and immigration, are more closely associated to delusional disorder than schizophrenia. These characteristics support the view that environmental factors may play an important etiological role.
14.14. The answer is A
Patients with erotomania have delusions of secret lovers. Most frequently, the patient is a woman, but men are also susceptible to the delusion. The patient believes that a suitor, usually more socially prominent than herself, is in love with her. The onset can be sudden, and the delusion becomes the central focus of the patient’s existence.
Patients with erotomania frequently show certain character- istics: they are generally but not exclusively women; may be
considered unattractive in appearance; are in low-level jobs; and lead withdrawn, lonely lives, being single and having few sexual contacts. They select secret lovers who are substantially differ- ent from themselves. They exhibit what has been called “para- doxical conduct,” the delusional phenomenon of interpreting all denials of love as secret affirmations of love. The course may be chronic, recurrent, or brief. Separation from the love object may be the only satisfactory means of intervention. Although men are less commonly affected by this condition than women, they may be more aggressive and possibly violent in their pursuit of love. Hence, in forensic populations, men with this condition predom- inate. The object of aggression may not be the loved individual but companions or protectors of the love object, who are viewed as trying to come between the lovers. The tendency toward vi- olence among men with erotomania may lead initially to police rather than psychiatric contact. In certain cases, resentment and rage in response to an absence of reaction from all forms of love communication may escalate to a point that the love object is in danger.
14.15. The answer is D
The use of individual therapy in the treatment of patients with delusional disorders seems to be more (not less) effective than group therapy. The essential element in effective psychotherapy is to establish a relationship in which the patient begins to trust the therapist. Initially, the therapist should neither agree with nor challenge the patient’s delusions. Although the therapist must ask about a delusion to establish its extent, persistent questioning about it should probably be avoided. The physicians may stimu- late the motivation to receive help by emphasizing a willingness to help the patient with his or her anxiety or irritability without suggesting that the delusions be treated, but the therapist should not actively support the notion that the delusions are real. A use- ful approach in building a therapeutic alliance is to empathize with the patient’s internal experience of being overwhelmed by persecution. It may be helpful to make such comments as, “You must be exhausted, considering what you have been through.” Without agreeing with every delusional misperception, a thera- pist can acknowledge that from the patient’s perspective, such perceptions create much distress. The ultimate goal is to help the patient entertain the possibility of doubt about his or her percep- tions. When family members are available, clinicians may decide to involve them in the treatment plan.
14.16. The answer is B
Patients with delusional disorders respond less well gener- ally to electroconvulsive treatment than do patients with major mood disorders with psychotic features. Some patients may respond to selective serotonin reuptake inhibitor (SSRIs), es- pecially cases of body dysmorphic disorder with delusional concerns. Dopamine-receptor antagonists (DRAs) (particularly pimozide [Orap]), serotonin-dopamine antagonists (SDAs), and SSRIs are pharmacological agents with reports of successful use in delusional disorder.
Delusional disorder is a psychotic disorder by definition, and the natural presumption has been that patients with the condition would respond to antipsychotic medication. Because controlled
studies are limited and the disorder is uncommon, the results required to support this practice empirically have been only par- tially obtained.
The disparate findings in the recent literature on delusional disorder treatment have been summarized recently, with several qualifications. Of approximately 1,000 articles published since 1961, the majority since 1980, 257 cases of delusional disorder (consistent with DSM-IV-TR criteria), of which 209 provided sufficient treatment detail to make comparison, were assessed. Overall treatment results indicated that 80.8 percent of patients recovered either fully or partially. Pimozide (the most frequently reported treatment) produced full recovery in 68.5 percent and partial recovery in 22.4 percent of patients treated, and there was full recovery in 22.6 percent and partial recovery in 45.3 per- cent of patients treated with DRAs that are typical neuroleptic agents (e.g., thioridazine [Mellaril], haloperidol [Haldol], chlor- promazine [Thorazine], loxapine [Loxitane], perphenazine [Tri- lafon], and others). The remaining patients were noncompliant with any treatment. There were no specific conclusions drawn regarding treatment with SSRIs, although a number of such re- ports have been published.
The results of treatment with the SDAs (i.e., clozapine [Clozaril], risperidone [Risperdal], olanzapine [Zyprexa], and others) are preliminary. Unfortunately, systematic case series will develop slowly, but these early results suggest that the atyp- ical neuroleptic agents may add to the available treatment op- tions.
14.17. The answer is E (all)
The so-called postpartum blues is a normal condition that occurs in up to 50 percent of women after childbirth. Postpartum blues is self-limited; lasts only a few days; and is characterized by tearfulness, fatigue, anxiety, and irritability that begin shortly after childbirth and lessen in severity over the course of 1 week.
14.18. The answer is C
The symptoms of postpartum psychosis most often begin within 8 weeks of delivery. About 50 to 60 percent of affected women with postpartum psychosis have just had their first child, and in that group, half of the newborns have perinatal complica- tions. The onset of florid psychotic symptoms is usually pre- ceded by prodromal signs, such as insomnia, restlessness, agi- tation, lability of mood, and mild cognitive deficits. The most robust data indicate that postpartum psychosis is related to a mood disorder, usually a bipolar disorder. Relatives of those with postpartum psychosis have an incidence of mood disorders that is similar to the incidence in relatives of persons with mood disorders.
14.19. The answer is A
Postpartum psychosis is found in one to two per 1,000 deliveries. The risk is increased if the patient or the patient’s mother had a previous postpartum illness or mood disorder. The symptoms are usually experienced within days of delivery and almost always within the first 8 weeks after giving birth. The patient begins to complain of insomnia, restlessness, and fatigue, and she shows lability of mood with tearfulness. Later symptoms include sus- piciousness, confusion, incoherence, irrational statements, and obsessive concerns about the baby’s health. Delusional material may involve the idea that the baby is dead or defective. The birth
14. Other Psychotic Disorders 139 may be denied, or ideas of persecution, influence, or perversity
may be expressed. Hallucinations may involve voices telling the patient to kill her baby. Postpartum psychosis is a psychiatric emergency. In one study, 5 percent of patients killed themselves, and 4 percent killed the baby. Postpartum psychosis is not to be confused with postpartum “blues.”
14.20. The answer is C
Postpartum psychosis should not be confused with postpartum “blues,” a normal condition that occurs in about 50 percent of women after childbirth. The “blues” are self-limited; last only a few days; and are characterized by tearfulness, fatigue, anxiety, and irritability that begin shortly after childbirth and lessen in severity each day postpartum. Postpartum psychosis is character- ized by agitation, severe depression, and thoughts of infanticide.
14.21. The answer is E
A general feature of attaque de nervios is a sense of being out of control. Attaque de nervios is reported in Latinos from through- out the Caribbean and it is not most common among Puerto Ricans. It also occurs in South America and Mediterranean coun- tries. It frequently occurs as a direct result of a stressful event relating to the family. It most closely resembles a panic attack, but unlike panic attacks, it is associated with a precipitating event. Panic attacks occur spontaneously. Persons may experience am- nesia for what occurred during the attack, but they otherwise return rapidly to their usual level of functioning.
14.22. The answer is D
Erotomania, the delusional disorder in which the person makes repeated efforts to contact the object of the delusion through let- ters, phone calls, gifts, visits, surveillance, and even stalking, is also called Cle ́rambault’s syndrome. Most patients with eroto- mania are women. In forensic samples in which harm is done to another person, most patients are men. In Cotard’s syndrome, patients may believe that they have lost everything: possessions, strength, and even bodily organs. Fregoli’s syndrome is the delu- sion that a persecutor is taking on a variety of faces, like an actor. Ganser’s syndrome is the voluntary production of severe psychiatric symptoms, sometimes described as the giving of ap- proximate answers (e.g., 2 + 2 = 5). Capgras syndrome is the delusion that familiar people have been replaced by identical impostors.
14.23. The answer is A
Puerperal psychosis is the most severe form of postpartum psy- chiatric illness. In contrast to postpartum blues and depression, puerperal psychosis is a rare event that occurs in approximately one to two per 1,000 women after childbirth. Its presentation is often dramatic, with onset of psychosis as early as the first 48 to 72 hours postpartum. Most women with puerperal psychosis develop symptoms within the first 2 to 4 weeks after delivery.
In women with this disorder, psychotic symptoms and disor- ganized behavior are prominent and cause significant dysfunc- tion. Puerperal psychosis resembles a rapidly evolving affective psychosis with restlessness, irritability, and insomnia. Women with this disorder may exhibit a rapidly shifting depressed or
140
14. Other Psychotic Disorders
Table 14.2
History of Psychiatric Illness and Risk for Puerperal Relapse
antipsychotic medication treatment. The ICD-10 also suggests that a distinction be made between acute onset (i.e., onset within 2 weeks) and abrupt onset (i.e., onset within 48 hours), noting that the latter may be associated with even better outcome.
14.25. The answer is D
Because symptoms are present for more than 1 month but less than 6 months, the diagnosis of schizophreniform disorder is likely. The patient is presenting with hallucinations and neg- ative symptoms. In schizophreniform disorder, the diagnosis must include at least two of any of the following: disorganized speech or behavior, negative symptoms, delusions, and halluci- nations. Schizophrenia would present with symptoms for at least 6 months. The diagnosis of brief psychotic disorder requires symptoms be present for at least 1 day but less than 1 month. Diagnosis requires one or more of hallucinations, delusions, or disorganized behavior or speech. Schizoaffective disorder may be diagnosed when a mood disorder and schizophrenia are both present.
Answers 14.26–14.30 14.26. The answer is B 14.27. The answer is C 14.28. The answer is C 14.29. The answer is E
14.30. The answer is D
Schizoaffective disorder has features of both schizophrenia and mood disorders. Patients must fit into one of six categories: (1) patients with schizophrenia who have mood symptoms, (2) pa- tients with mood disorder who have symptoms of schizophrenia during periods of euthymia, (3) patients with both mood disor- der and schizophrenia, (4) patients with a psychosis unrelated to schizophrenia as well as a mood disorder, (5) patients whose disorder is on a continuum between schizophrenia and mood disorder, and (6) patients with some combination of the above. However, it can be difficult to diagnose. The clinician must ac- curately diagnose the mood disorder, making sure it meets the criteria of either a manic or a depressive episode but also deter- mining the exact length of each episode (not always easy or even possible).
By definition, patients with schizophreniform disorder re- turn to their baseline state within 6 months. However, in some cases, the illness is episodic with more than one episode occur- ring after long periods of full remission. Some studies show that 45 to 75 percent of schizophreniform patients may progress to schizophrenia with longitudinal assessment. Schizophreniform disorder is an acute psychotic disorder that has a rapid onset and lacks a long prodromal phase. The initial symptom profile is the same as schizophrenia in that two or more psychotic symptoms (hallucinations, delusions, disorganized speech and behavior, or negative symptoms) must be present.
Acute and transient psychotic disorders are defined by the ICD-10 as psychotic conditions with onset within 2 weeks and
 
Disorders
Postpartum psychosis Postpartum depression Bipolar I disorder
Major depressive disorder
Risk of Relapse at Future Pregnancy (%)
70
50 20–50 30
 
elated mood, disorientation or depersonalization, and disorga- nized behavior. Delusional beliefs often center on the infant and include delusions that the child may be defective or dying, that the infant has special powers, or that the child is either Satan or God. Auditory hallucinations that instruct the mother to harm or kill herself or her infant are sometimes reported. Although most believe that this illness is indistinguishable from an affective (or manic) psychosis, some have argued that puerperal psychosis may be clinically distinct in that it is more commonly associated with confusion and delirium than nonpuerperal psychotic mood disorder.
Although it has been difficult to identify specific demographic and psychosocial variables that consistently predict risk for post- partum illness, there is a well-defined association between all types of postpartum psychiatric illness and a personal history of mood disorder (Table 14.2). At highest risk are women with a history of postpartum psychosis; up to 70 percent of women who have had one episode of puerperal psychosis will experi- ence another episode after a subsequent pregnancy. Similarly, women with histories of postpartum depression are at significant risk, with rates of postpartum depression recurrence as high as 50 percent. Women with bipolar disorders also appear to be par- ticularly vulnerable during the postpartum period, with rates of bipolar relapse ranging from 20 to 50 percent.
14.24. The answer is C
A definitive diagnosis of acute and transient psychotic disorder is often only possible retrospectively and with (not without) ex- tensive information on the patient’s past history. Acute and tran- sient psychotic disorders are defined by the tenth edition of the International Statistical Classification of Diseases and Related Health Problems (ICD-10) as psychotic conditions with onset within 2 weeks and full remission within 1 to 3 months. These conditions do not have a designated place in the DSM-IV-TR. Many of the cases of ICD-10 acute and transient psychotic dis- orders would be categorized as schizophreniform disorder, brief psychotic disorder, or psychotic disorder not otherwise specified in the DSM-IV-TR. The age of onset in developing countries ap- pears to be lower (not higher) than in industrialized settings. Recognition of acute and transient psychotic disorders early in their course is difficult (not easy). In the first few weeks of the illness, there are no specific indicators that would distinguish acute and transient disorder from other nonaffective psychotic disorders with acute onset. Furthermore, even in retrospect, it is often difficult to assess the duration of a psychotic episode or distinguish spontaneous remission from remission caused by
full remission within 1 to 3 months. The ICD-10 provides a se- quence of three “key features” to be used for diagnosing acute and transient psychotic disorders. In order of priority, these fea- tures are (1) an acute onset (within 2 weeks); (2) the presence of typical syndromes, which are the basis for the subcategorization into specific disorders; and (3) the presence of associated acute stress (within about 2 weeks of onset). Acute onset is defined as “a change from a state without psychotic features to a clearly abnormal psychotic state, within a period of 2 weeks or less.”
Patients with delusional disorder, by definition, do not have prominent or sustained hallucinations. The diagnosis of delu- sional disorder is made when a person exhibits nonbizarre delu- sions of at least 1 month’s duration that cannot be attributed to other psychotic disorders. There are five subtypes of delusional disorders: (1) persecutory type (the belief by the patient that he or she is being persecuted or harmed), (2) jealous type (i.e., the belief that one’s spouse is unfaithful), (3) erotomanic type (the belief that another person, usually of higher status, is in love with the patient), (4) somatic type (delusions of a hypochondriacal or somatic nature), and (5) grandiose type (delusions of grandeur).
The diagnosis of brief psychotic disorder is based on the presence of one or more psychotic symptoms, including delu- sions, hallucinations, disorganized speech, and disorganized or catatonic behavior for less than 1 month.
Answers 14.31–14.35 14.31. The answer is B 14.32. The answer is E 14.33. The answer is C 14.34. The answer is A
14.35. The answer is D
Psychomotor retardation is a general slowing of mental and physical activity. It is often a sign of a major depressive episode,
which is characterized by feelings of sadness, loneliness, despair, low self-esteem, and self-reproach. Thought broadcasting is the feeling that one’s thoughts are being broadcast or projected into the environment. Such feelings are encountered in schizophre- nia.
A patient in a manic episode is easily distracted, with an elevated, expansive, or irritable mood with pressured speech and hyperactivity.
Delusional disorder is characterized by nonbizarre persecu- tory or grandiose delusions and related disturbances in mood, thought, and behavior.
The essential feature of paranoid personality disorder is a long-standing suspiciousness and mistrust of people without the presence of psychotic symptoms. Patients with this disorder are hypersensitive and continually alert for environmental clues that will validate their original prejudicial ideas.
Answers 14.36–14.40 14.36. The answer is B 14.37. The answer is E 14.38. The answer is A 14.39. The answer is C
14.40. The answer is D
In delusional disorder, delusions of jealousy are most commonly found. In schizophrenia, bizarre delusions may occur (e.g., of being controlled by outside persons or forces and delusions of persecution). Grandiose delusions are most often seen in mania but can be observed in other psychotic disorder as well. In depres- sive disorders, delusions of guilt are especially characteristic. In cognitive disorders, such as dementia, delusions secondary to perceptual disturbances are most often evident.
14. Other Psychotic Disorders 141
 
Mood is defined as a pervasive and sustained feeling that is experienced internally and that, in the extreme, can markedly influence all aspects of a person’s behavior and his or her per- ception of the world. Affect is the external expression of mood. Mood disorders, formally known as affective disorders, include a group of psychological disorders that are mainly characterized by pathological moods and related vegetative and psychomotor disturbances. The term mood disorder is preferred over affec- tive disorder because it refers not only to the external (affective) expression of the present emotional state but also to sustained emotional states. Mood disorders are syndromes (not diseases) that consist of signs and symptoms that present a deviation from a person’s normal functioning and are sustained over a period of weeks to months. These signs and symptoms tend to recur, often in periodic or cyclical fashion.
The most common mood disorder is major depressive disor- der (unipolar depression). Major depressive disorder is charac- terized by one or more episodes of major depression without a manic episode. Patients with both manic and depressive episodes or with manic episodes alone are diagnosed with bipolar disor- der. Dysthymia and cyclothymia are less severe forms of major depression and bipolar disorder, respectively. Hypomania is an episode of manic symptoms but does not meet the criteria for a manic episode.
Mood disorders can sometimes be difficult to diagnose, given the subjective nature of the symptoms. All people have nor- mal periods of feeling either blue or elated, and most of these obviously are not diagnosable as disorders. A mood disorder is characterized by the intensity, duration, and severity of the symptoms. Symptoms interfere with normal thought process and
15
Mood Disorders
content and cognitive, speech, and social functioning. Unfortu- nately, many people with depressive disorders go untreated be- cause their symptoms are minimized or misinterpreted. People with bipolar disorders are more often treated because their symp- toms more frequently are bizarre or disruptive enough to bring them to medical and psychiatric attention.
Mood disorders are caused by a complex interplay of bio- logical and psychological factors. Biologic theories involve the role of the biogenic amines, particularly dysfunction in the nore- pinephrine, serotonin, dopamine, and GABA (γ -aminobutyric acid) neurotransmitter systems. Most antidepressant medications involve complex manipulations of these systems. There appears to be dysregulation as well in the adrenal, thyroid, and growth hormone axes, all of which have been implicated in the etiol- ogy of mood disorders. Abnormalities in the sleep cycle and in regulation of circadian rhythms have also been studied.
Genetics always play an important role in the etiology of mental disorders, but genetic input is especially relevant in mood disorders. Bipolar I disorder is one of the most genetically de- termined disorders in psychiatry. However, as with any mental disorder, psychosocial factors play a crucial role in the devel- opment, presentation, course, and prognosis of mood disorders. Issues of real and symbolic loss, family relationships and dynam- ics, environmental stress, and unconscious conflicts all strongly contribute to and determine mood symptoms. Some clinicians believe that these factors are particularly important in the first episodes of mood disorders, but in one form or another, they play a role in all episodes.
Students should study the questions and answers below for a useful review of these disorders.
  
142
HELPFUL HINTS
Students should know the following terms that relate to mood disorders.
adrenal axis anxiety-blissfulness
psychosis atypical features biogenic amines bipolar I disorder bipolar II disorder carbamazepine cognitive, behavioral,
family, and psycho- analytic therapies
cognitive theories cyclothymic disorder depression rating scales depressive equivalent double depression dysthymia
ECT
euthymic
foliea` double forme folie circulaire GABA
GH
5-HT
hypomania
kindling
learned helplessness LH, FSH
lithium
major depressive
disorder mania
MAOIs
melancholic features melatonin
mild depressive disorder mixed episode mood-congruent and
-incongruent
psychotic fear neurological, medical,
and pharmacological causes of mood disorders
▲▲▲▲▲ ▲
▲▲▲▲▲▲▲▲ ▲▲
▲▲▲▲▲▲▲▲▲▲▲
▲▲ ▲▲▲▲▲▲
norepinephrine phototherapy postpartum onset premenstrual dysphoric
disorder
QUESTIONS
Directions
premorbid factors pseudodementia rapid cycling
REM latency, density RFLP
seasonal pattern SSRI
suicide
T3 thymoleptics
15.
Mood Disorders 143 TSH, TRH
vegetative functions
Zeitgeber
  
Each of the questions or incomplete statements below is followed by five suggested responses or completions. Select the one that is best in each case.
. 15.1. Mild,nonpsychoticdepressionwithpredominantanxiety is called
A. Endogenomorphic depression B. Bipolar disorder
C. Chronic depression
D. Dysthymia
E. Anxiety disorder
. 15.2. The most consistent computer tomography (CT) and magnetic resonance imaging (MRI) abnormality ob- served in depressive disorders is
A. cortical atrophy
B. sulcal widening
C. ventricular enlargement
D. increasedfrequencyofhyperintensitiesinsubcortical
regions
E. none of the above
. 15.3. The following situations call for a break in doctor–patient confidentiality except
A. A patient with a delusional disorder thinks his boss is out to get him and threatens to kill her.
B. A patient with major depressive disorder who is sex- ually promiscuous contracts syphilis.
C. A patient with bipolar I disorder admits he is homo- sexual.
D. A patient with conduct disorder thrives on the sexual abuse of young children.
E. A patient with schizoaffective disorder hallucinates that he can fly.
. 15.4. A 27-year-old patient has been diagnosed with bipo- lar disorder. Before starting this patient on lithium for mood stabilization, which of the following laboratory tests should be obtained?
A. Thyroid function tests, creatinine, pregnancy test
B. Thyroid function tests, creatinine, liver function tests
C. Thyroid function tests, creatinine, complete blood
count
D. Thyroid function tests, liver function tests, pregnancy
test
15.5.
E. Thyroid function tests, complete blood count, preg- nancy test
Which of the following statements regarding mood dis- orders is false?
A. One of four patients with an acute depressive episode will have recurrences throughout life.
B. Approximately15percentofdepressedpatientseven- tually commit suicide.
C. Incidence of depression in younger age groups is in- creasing.
D. Manic forms of mood disorders predominate in men.
E. Depressive disorders are more common in women.
All of the following are vegetative disturbances of de-
pression except
A. Hypersexuality B. Anorexia
C. Hypersomnia D. Insomnia
E. Circadian dysregulation
Serotonin
A. helps to regulate circadian rhythms
B. is an important regulator of sleep, appetite, and libido
C. stores are increased by transient stress and depleted
by chronic stress
D. permits or facilitates goal-directed motor and con-
summatory behavior in conjunction with nore-
pinephrine and dopamine
E. all of the above
Which graph in Figure 15.1 depicts the pattern with the best future prognosis?
A.A
B. B
C. C
D.D
E. None of the above
Which of the graphs in Figure 15.1 depicts the prototyp- ical course of double depression?
A.A
B. B
C. C
D.D
E. None of the above
15.6.
15.7.
15.8.
15.9.
▲▲▲
▲▲▲▲▲
▲▲▲▲▲
▲▲▲▲
144 15.
Mood Disorders

FIGURE 15.1
. 15.10. Double depression is characterized by
A. two family members with major depressive disorder concurrently
B. recurrent major depressive disorder with current symptoms twice as disabling as usual
C. two episodes of major depressive disorder per month consistently
D. superimposed bipolar II disorder and atypical depres- sion
E. recurrent major depressive disorder superimposed with dysthymic disorder
. 15.11. Depression and mania share which of the following symptoms?
A. Psychomotor acceleration B. Low-self esteem
C. Grandiosity
D. Anger
E. Pessimism
. 15.12. The person least likely to develop major depressive dis- order in his or her lifetime is
A. a 60-year-old man with pancreatic cancer
B. a 19-year-old woman who was raped 3 weeks ago
C. a 12-year-old girl mourning the death of her mother
D. a 10-year-old boy diagnosed with dysthymia
E. an identical twin of a patient with major depressive
disorder who committed suicide
. 15.13. A hypomanic episode differs from a manic episode in that a hypomanic episode
A. lasts at least 1 week
B. lacks psychotic features
C. is severe
D. causes greater social impairment E. all of the above
15.14. Thedefensemechanismmostcommonlyusedindepres- sion is
A. undoing
B. sublimation C. projection D. introjection E. altruism
15.15. Which of the following is not part of the text revision of the fourth edition of the Diagnostic and Statistical Manual of Mental Disorders (DSM-IV-TR) criteria for diagnosing atypical depression?
A. Hypersomnia
B. Leaden paralysis
C. Shortening of REM latency D. Mood reactivity
E. Significant weight gain
15.16. l-Tryptophan
A. has been used as an adjuvant to both antidepressants and lithium
B. has not been associated with any serious side effects
C. is the amino acid precursor to dopamine
D. has been used as a stimulant
E. all of the above
15.17. Which of the following is not an indicator of a good prognosis for major depressive disorder?
A. Stable family functioning
B. No more than one previous hospitalization
C. A history of more than one previous depressive
episode
D. Advanced age of onset
E. The absence of psychotic symptoms
15.18. Reactive depression can best be compared to
A. Adjustment disorder
B. Atypical depression
C. Conduct disorder
D. Oppositional defiant disorder E. Schizoaffective disorder
15.19. Which of the following statements regarding rapid cycling bipolar disorder is true?
A. Alcohol, stimulants, and caffeine use are risk factors. B. It is defined as at least four episodes per month.
C. Hospitalization of these patients is rare.
D. It is more common in men than women.
E. It often responds to tricyclic antidepressants.
15.20. All of the following are common causes of misdiagnosis
of mood disorder as schizophrenia except
A. reliance on the longitudinal rather than cross- sectional picture
B. flight of ideas perceived as loose associations
C. ascribing irritable mood to paranoid delusions
D. mistaking depressive depersonalization for schizo-
phrenic emotional blunting
E. incomplete interepisodic recovery equated with
schizophrenic defect
. 15.21. All of the following statements regarding cyclothymic
disorder are true except
A. Symptoms must be present for at least 2 years.
B. It occurs at the same rate in men and women.
C. Symptoms may satisfy criteria for major depression.
D. It consists of hypomania alternating with depressed
mood.
E. Its lifetime prevalence rate is about 0.4 to 1 percent.
. 15.22. Dysthymic disorder differs from major depressive disor- der because in dysthymic disorder,
A. depression is episodic
B. the symptoms outnumber the signs C. the onset is usually late in life
D. manic episodes are common
E. has a high-grade chronicity
. 15.23. Psychomotor retardation is characterized by all of the following except
A. indecisiveness
B. paucity of spontaneous movements C. poor concentration
D. reduced speech amplitude and flow E. restlessness
. 15.24. Features of anhedonia may include all of the following except
A. derealization
B. difficulty describing or being aware of emotions C. inability to experience normal emotions
D. loss of pleasure
E. withdrawal from interests
. 15.25. The highest suicide rates are in which of the following age groups?
A. Younger than age 15 years B. 15 to 24 year olds
C. 25 to 44 year olds
D. 45 to 64 year olds
E. Older than age 65 years
. 15.26. Which of the following is the best predictor of the likeli- hood of attempting suicide in the future?
A. Alcohol abuse
B. Gender
C. Prior suicide attempt D. Recent divorce
E. Unemployment
15. Mood Disorders 145 15.27. In the differential diagnosis, the diagnosis of schizoaf-
fective disorder should be restricted to
A. mixed episodes of bipolar disorder
B. affective psychosis with concurrent brain disease
C. full affective and schizophrenic symptoms simulta-
neously
D. affective psychosis superimposed on mental retarda-
tion
E. a contagious expansive and elated affect
15.28. Drugs that may precipitate mania include all of the following except
A. Bromocriptine B. Disulfiram
C. Isoniazid
D. Propranolol
E. All of the above
15.29. Which of the following antidepressants would not be the best choice for a patient with a history of suicidal ideation?
A. Bupropion (Wellbutrin)
B. A monoamine oxidase inhibitor
C. A selective serotonin reuptake inhibitor D. A tricyclic antidepressant
E. Venlafaxine (Effexor)
15.30. Which of the following statements regarding electrocon- vulsive therapy (ECT) is false?
A. ECT should be used in cases of psychotic depression only.
B. Bilateral ECT is somewhat more effective than uni- lateral ECT.
C. Retrograde memory impairment is a common side effect.
D. ECT is often used for refractory mood disorders.
E. Eight to 12 treatments are usually needed for symp-
tomatic remission.
15.31. Mr. M is an 87-year-old man who, 6 weeks after a coro- nary artery bypass graft that was complicated by pneumo- nia and renal insufficiency, was admitted to an inpatient rehabilitation service for management of physical de- conditioning. A psychiatrist was consulted 10 days after admission to rule out depression in the context of persis- tent low appetite and energy associated with suboptimal participation in rehabilitation. Mr. M reported no prior psychiatric history. He had worked as a chemist until retirement nearly 2 decades earlier. Laboratory exami- nation revealed a low hematocrit of 21 and moderately elevated blood urea nitrogen of 65. On interview, Mr. M demonstrated psychomotor slowing and bland affect. He denied depression, hopelessness, worthlessness, and suicidal ideation. He expressed a desire to recover from his debilitated state but acknowledged uncertainty that he
146 15. Mood Disorders
was capable of doing so. He also complained of extreme weakness. He stated, “I just don’t seem to have an ap- petite anymore.” Cognition largely was intact; there was mild short-term memory deficit.
The most likely diagnosis in this patient is:
A. Anxiety disorder with depressed mood
B. Delirium
C. Dementia
D. Major depressive disorder
E. Mood disorder secondary to a general medical con- dition
15.32. A 19-year-old single woman presented with the chief complaint that “all men are bastards.” Since her early teens, with the onset of her menses, she had complained of extreme variability in her moods on a nearly daily basis; irritability with hostile outbursts was her main af- fect, although more protracted hypersomnic depressions with multiple overdoses and wrist slashings had led to at least three hospitalizations. She also had migrainous headaches that, according to her mother, had motivated at least one of those overdoses. Despite her tempestu- ous and suicidal moods that led to these hospitalizations, she complained of “inner emptiness and a bottomless void.” She had used heroin, alcohol, and stimulants to overcome this troubling symptom. She said that she was mentally disturbed because of a series of stepfathers who had all forced “oral rape” on her when she was between 11 and 15 years of age. She subsequently became sex- ually involved with any man that she met in bars, no longer knowing whether she was a “prostitute” or a “nice little girl.” On two occasions, she had inflicted cigarette burns inside her vagina “to feel something.” She had also engaged in a “brief lesbian relationship” that ultimately left her “emptier” and guilt ridden; nonetheless, she now believed that she should burn in hell because she could not get rid of “obsessing” about the excitement of mutual cunnilingus with her much older female partner. The pa- tient was given phenelzine (Nardil), eventually increased to 75 mg per day, at which point the mother described her as “the sweet daughter she was before age 13.” At her next premenstrual phase, the patient developed in- somnia, ran away from home at night, started “dancing like a go-go girl, met an incredibly handsome man” of 45 years of age (a pornography shop owner), and had a clandestine marriage to him.
Other than a mood disorder, this patient also shows signs of
A. an anxiety disorder
B. schizophrenia
C. borderline personality disorder D. schizoaffective disorder
E. none of the above
15.33. A 35-year-old woman has just been diagnosed with major depressive disorder. For the past 8 months, she has had a depressed mood, decreased energy and concentration,
and loss of interest in previously enjoyed activities. Al- though she never attempted suicide, she acknowledges that she thought she would probably jump off a local bridge if she ever had the chance. She denies any history of excessively elevated moods. You decide to start her on antidepressant therapy. Two weeks later, this patient is at greatest risk for
A. extrapyramidal symptoms B. hypomanic episode
C. manic episode
D. medication noncompliance E. suicide completion
15.34. A 57-year-old woman presents to you after being diag- nosed with major depressive disorder. She has been de- pressed ever since the death of her husband 2 years earlier. She has been taking the same antidepressant since her di- agnosis 1 year ago, with no relief of her symptoms. She states that she would like your help in ending her life. The best option for your next step is:
A. respect the patient’s wishes because she is of sound mind
B. seek to more adequately treat her depression
C. seek family members to make a more informed deci-
sion
D. contact the hospital ethics committee
E. obtain information from the physician-assisted suicide laws
state
regarding
15.35. Ms. S, a 24-year-old woman, is brought for a psychiatric consultation by her mother who complains of bizarre be- havior. One month ago, Ms. S was fired from her job at a local bookstore because of frequently arriving late and not performing her duties adequately. She states that she fell in love with another employee and tried to get his at- tention and spend time with him even though he seemed uninterested. Over the past 3 months, she increased her use of alcohol and marijuana to three beers and two to three joints per day. Her mother reports a 2-week history of increased energy, eating little, talking a great deal, and interrupting others frequently. A week ago, Ms. S reported that her former work colleagues were plotting against her and attempting to control her by broadcasting thoughts into her brain. She did not sleep the previous 2 nights. Ms. S has no significant psychiatric or medical history. She takes no medications.
Physical examination reveals a blood pressure of 135/75 mm Hg, heart rate of 84 beats/min, and temper- ature of 37◦C. Her conjunctivae are pink, and her pupils are equal, 3 mm, and reactive to light. Deep tendon re- flexes are normal throughout. Urine toxicology reveals the presence of cannabinoids. On mental status testing, her mood is euphoric, her speech is pressured, and she is emotionally labile and irritable. Her thinking is illogical and disorganized. She denies hallucinations. She is alert and oriented to person, place, and time. Immediate recall and recent and remote memory are intact. Throughout the
interview, she is preoccupied by thoughts of the coworker with whom she has fallen in love.
Ms. S is admitted to a psychiatric unit, and treatment is initiated with haloperidol, 10 mg/day, which is increased to 20 mg/day on day 5 because of continued agitation. On day 6, she becomes withdrawn and uncommunica- tive. She is diffusely rigid with a temperature of 39◦C. Her white blood count is 14,300 and her creatinine phos- phokinase is 2,100. Several blood cultures are negative.
Which of the following is the most likely diagnosis at the time of admission?
A. Bipolar disorder
B. Delusional disorder, erotomanic type C. Marijuana-induced psychotic disorder D. Schizoaffective disorder, bipolar type E. Schizophrenia
. 15.36. Which of the following is the most likely explanation for her behavior on day 6?
A. Anticholinergic delirium
B. Neuroleptic malignant syndrome C. Marijuana-induced delirium
D. Occult infection
E. Worsening psychosis
. 15.37. Which of the following pharmacologic approaches is most appropriate on day 6?
A. Increase dose of haloperidol.
B. Stop haloperidol and add risperidone.
C. Stop haloperidol, add bromocriptine, and seek medi-
cal consultation.
D. Continue the same dose of haloperidol and add
risperidone.
E. Continue the same dose of haloperidol and add ben-
ztropine.
. 15.38. A suicidal patient with chronic depressive disorder presents to your office very frustrated and in tears. He tells you he cannot stop thinking about ending his life because he is so depressed. You ask him if he has a plan, and he details where he could buy a handgun and where he would go to shoot himself. You fear the patient will carry out this plan because he has not had adequate control of his symptoms since his last antidepressant change 1 month ago. You discuss inpatient hospitaliza- tion for medication stabilization, but the patient refuses. You’re next step in management of this patient would best be:
A. Admit the patient to the hospital anyway.
B. Give the latest antidepressant more time to take
affect.
C. Change to another class of antidepressant.
D. Try to persuade the patient to admit himself to the
hospital.
E. Initiate psychotherapy to discuss the reasons behind
the suicidal thoughts.
15.39. A 64-year-old woman with an extensive smoking history has recently been diagnosed with small cell lung cancer. She develops a depressed mood, decreased interests, and difficulty concentrating soon thereafter because she re- ports she cannot stop thinking about how worthless her life has been. She eats incessantly and has gained 10 lb in the last 5 weeks; she also reports increased sleep. You de- cide to prescribe phenelzine for her symptoms of atypical depression. Which of the following is contraindicated in those patients taking phenelzine?
A. Valproic acid B. Trazodone
C. Lithium
D. Fluoxetine
E. Clomipramine
15.40. A 28-year-old woman presents to a clinic with a chief complaint of fatigue. She states that this feeling of “being tired” has persisted for the past 4 years. She has lost 12 lb over the past 2 years and admits to overeating. The patient states that she sleeps at least 11 hours per night. She denies suicidal ideation but complains about not being able to concentrate. What is the likely diagnosis?
A. Generalized anxiety disorder B. Dysthymia
C. Major depressive disorder D. Substance abuse
E. None of the above
Directions
Each set of lettered headings below is followed by a list of phrases or statements. For each numbered phrase or statement, select:
A. if the item is associated with A only
B. if the item is associated with B only
C. if the item is associated with both A and B D. if the item is associated with neither A nor B
Questions 15.41–15.45
A. Bereavement B. Depression
15.41. Is perceived as normal
15.42. Active suicidal ideation is common 15.43. Persons often experience guilt 15.44. Persons react to the environment 15.45. Delusions of worthlessness
Questions 15.46–15.49
A. Clozapine B. Imipramine
15.46. Cardiotoxic
15.47. Causes weight gain
15. Mood Disorders 147
148 15. Mood Disorders 15.48. Acts as an NE partial agonist
15.49. Teratogenic Questions 15.50–15.53
A. Unipolar depression B. Bipolar II depression
. 15.50. Never any history of acute mania
. 15.51. Typically has psychotic features present
. 15.52. Symptoms of hypomania are present
. 15.53. Can present with atypical features
Directions
Each group of questions below consists of lettered headings fol- lowed by a list of numbered phrases or statements. For each numbered phrase or statement, select the one lettered heading that is most associated with it. Each lettered heading may be selected once, more than once, or not at all.
Questions 15.54–15.59
A. Dysthymic disorder B. Neurasthenia
C. Bipolar I disorder
D. Cyclothymic disorder E. Bipolar II disorder
F. Hypomania
. 15.54. Is diagnosed more in China than the rest of the world
. 15.55. Subsyndromal depression and hypomania
. 15.56. Rarely progresses to manic psychosis
. 15.57. Insidious onset of depression dating back to child-
hood
. 15.58. Manic-like symptoms do not meet full manic syndrome
criteria
. 15.59. Includes a full set of mania symptoms
ANSWERS
15.1. The answer is D
Dysthymia is defined as a reactive nonpsychotic depression of mild to moderate intensity with predominant anxiety. It is char- acterized by the presence of a depressed mood that lasts most of the day and is present almost continuously. Patients complain that they have always been depressed. It is not a sequela to a major depressive episode as with chronic depression. There are associated feelings of inadequacy, guilt, irritability, and anger; withdrawal from society; loss of interest; and inactivity and lack of productivity. Most cases are of early onset, beginning in child- hood or adolescence and certainly by the time patients reach their 20s. The overall prognosis for dysthymia is good with treatment; however, 25 percent of dysthymic patients never attain complete recovery.
Bipolar disorder is defined as a mood disorder in which the patient exhibits both manic and depressive episodes. The criteria
for a manic episode require the presence of a distinct period of abnormal mood lasting at least 1 week. Typically beginning in the teenage years, the 20s, or the 30s, the first episode of bipolar I disorder could be manic, depressive, or mixed. On average, manic episodes predominate in youth, and depressive episodes predominate in later years. Although the overall sex ratio is 1:1, men, on average, undergo more manic episodes, and women experience more mixed and depressive episodes.
Endogenomorphic depression is a term used to describe in- hibition of the pleasure or reward system to such an extent that the patient no longer has the capacity for enjoyment. There may or may not be an apparent environmental precipitant or stress. In most cases, patients show dramatic abnormalities in psychomo- tor activity and somatic rhythms (i.e., sleep, appetite, libido). Such patients may also be delusional. The symptom profile in chronic depression usually displays low-grade intensity. Full criteria for a major depressive episode must have been met con- tinuously for at least the past 2 years. Instead of the customary remission within 1 year, patients are ill for years.
Anxiety disorders are disorders in which anxiety is the most prominent disturbance or in which patients experience anxiety if they resist giving in to their symptoms. They are among the most prevalent mental disorders in the general population. Women are affected nearly twice as frequently as men. They are associated with significant morbidity and often are chronic and resistant to treatment. They include (1) panic disorder with or without agoraphobia, (2) agoraphobia with or without panic disorder, (3) specific phobia, (4) social phobia, (5) obsessive-compulsive disorder (OCD), (6) posttraumatic stress disorder (PTSD), (7) acute stress disorder, and (8) generalized anxiety disorder.
15.2. The answer is D
Computed tomography (CT) and magnetic resonance imaging (MRI) scans provide sensitive, noninvasive methods to assess the brain, including cortical and subcortical tracts, as well as white matter lesions. The most consistent abnormality observed in the depressive disorders is increased frequency of abnormal hyper- intensities in subcortical regions, especially the periventricular area, basal ganglia, and thalamus (Figure 15.2). More common in bipolar I disorder and among the elderly, these hyperinten- sities appear to reflect the deleterious neurodegenerative effects of recurrent mood episodes. Ventricular enlargement, cortical atrophy, and sulcal widening have also been reported in patients with mood disorders compared with normal control subjects. In addition to age and illness duration, structural abnormali- ties are associated with increased illness severity, bipolar status, and increased cortisol levels. Some depressed patients also may have reduced caudate nucleus volumes, suggesting a defect in the mesocorticolimbic system. Cerebrovascular factors often in- volve subcortical frontal and basal ganglia structures and appear particularly relevant to late-life depression.
15.3. The answer is C
Confidentiality refers to the therapist’s responsibility to not re- lease information learned in the course of treatment to third par- ties. Confidentiality is an essential ingredient of psychiatric care because it is a prerequisite for patients to be willing to speak freely to therapists. People would be less likely to go for help and would tend to withhold crucial information if confidentiality

FIGURE 15.2
Magnetic resonance imaging scan of a patient with late-onset ma- jor depressive disorder illustrating extensive periventricular hyper- intensities associated with diffuse cerebrovascular disease.
was not assured. There is no reason to break doctor–patient con- fidentiality in situations involving a patient’s sexual preference (e.g., homosexuality) unless there is some threat to the patient’s safety. Confidentiality must give way to the responsibility to protect others when a patient makes a credible threat to harm someone (e.g, a patient threatening to kill his boss) or acts in a way that could harm him- or herself (e.g., a patient who thinks he can fly). Reportable diseases (e.g., syphilis) must also be reported to the proper authorities despite the bond of patient confidential- ity. Any suspicion of child abuse must also be reported to the authorities.
15.4. The answer is A
Lithium (Eskalith), used for mood stabilization in patients with bipolar disorder, is known for its low margin of safety, mak- ing frequent monitoring necessary. Common side effects include gastrointestinal disturbances, nephrotoxicity, hypothyroidism, tremors, leukocytosis, acne, psoriasis flares, hair loss, and edema. Because of these effects, it is important to get thyroid function tests and renal function tests (creatinine) before starting any pa- tient on this medication. Lithium is also teratogenic and has been associated with cardiac defects, so a pregnancy test is necessary before lithium therapy.
15.5. The answer is A
Three (not one) out of four patients with acute depression will experience recurrences, with varying degrees of residual symp- toms between episodes. Of note, although depressive disorders
15. Mood Disorders 149 are more common in women, more men than women die of sui-
cide because of more lethal methods chosen.
15.6. The answer is A
Hypersexuality is cardinal sign of mania and usually indicates a mixed episode of bipolar disorder in depressed patients. The small subgroup of depressed persons with increased sexual drive usually exhibit other atypical features; hence, increased sex- ual drive is considered the fifth reverse vegetative sign (after even worsening mood, initial insomnia, hypersomnia, and weight gain).
The biological concomitants of melancholia include pro- found reductions in appetite (i.e., anorexia or weight loss), sleep (i.e., insomnia), and sexual functioning, as well as alterations in other circadian rhythms, especially worsening of mood and psychomotor performance. An equally prominent subgroup of depressed persons exhibits a reversal of the vegetative and circa- dian functions, with increases in appetite and sleep—and some- times in sexual functioning—and an evening worsening of mood; in this atypical pattern, patients characteristically exhibit mood reactivity and sensitivity to rejection.
15.7. The answer is E (all)
Serotoninergic neurons project from the brainstem dorsal raphe nuclei to the cerebral cortex, hypothalamus, thalamus, basal gan- glia, septum, and hippocampus. Serotonin (5-HT) pathways have both inhibitory and facilitatory functions in the brain. For exam- ple, much evidence suggests that 5-HT is an important regulator of sleep, appetite, and libido. Serotonergic neurons projecting to the suprachiasmatic nucleus of the hypothalamus help to regu- late circadian rhythms (e.g., sleep–wake cycles, body temper- ature, and hypothalamic-pituitary-adrenocortical axis function). Serotonin also permits or facilitates goal-directed motor and consummatory behaviors in conjunction with norepinephrine and dopamine. Moreover, serotonin inhibits aggressive behavior across mammalian and reptilian species.
There is some evidence that serotonin neurotransmission is partly under genetic control. Nevertheless, whereas acute stress increases serotonin release transiently, chronic stress eventually depletes serotonin stores. Chronic stress may also increase syn- thesis of 5-HT1A autoreceptors in the dorsal raphe nucleus, which further decrease serotonin transmission. Elevated glucocorticoid levels tend to enhance serotonergic functioning and thus may have significant compensatory effects on chronic stress.
15.8. The answer is A
The course of major depressive disorder, recurrent, with no an- tecedent dysthymic disorder and a period of full remission be- tween the episodes predicts the best future prognosis. This is depicted in graph A.
15.9. The answer is D
Graph D depicts double depression, which is character- ized by recurrent major depressive disorder and antecedent dysthymic disorder with no period of full remission in between the two most recent episodes. Graph A is the course of ma- jor depressive disorder, recurrent, with no antecedent dysthymic
150 15. Mood Disorders
disorder and full remission between episodes. Graph B is the course of major depressive disorder, recurrent, with no an- tecedent dysthymic disorder but with prominent symptoms per- sisting between episodes (partial remission is attained). Graph C is the rare pattern major depressive disorder, recurrent, with antecedent dysthymic disorder but with full interepisodic recovery.
15.10. The answer is E
Double depression is characterized by recurrent major depres- sive disorder with antecedent dysthymic disorder and no period of full remission between the two most recent episodes. This pattern is seen in approximately 20 to 25 percent of the persons with major depressive disorder.
15.11. The answer is D
Despite their contrasts, depression and mania share such symp- toms as irritability, anger, insomnia, and agitation. Mania is defined as a mood disorder characterized by elation, agitation, hyperactivity, and hyperexcitability. The clinical features of ma- nia are generally the opposite of those of depression. Depres- sion is defined as a mental state characterized by feelings of sadness, loneliness, despair, low self-esteem, and self-reproach. Depression is commonly associated with cases of lowered mood, thinking, self-esteem, and activity. Mania, on the other hand, is commonly associated with elevated mood, a rush of ideas, psychomotor acceleration, and grandiosity. An excess of the shared symptoms of escalating intensity suggests a mixed phase of mixed episode of mania and depression occurring simultane- ously, commonly called bipolar I disorder.
15.12. The answer is B
The 19-year-old rape victim is more likely to develop a variation of posttraumatic stress disorder, which is highest among vic- tims of rape, military combat veterans, and survivors of torture. Childhood onset of dysthymia similarly presages extremely high rates of depression and bipolar disorder in adulthood. Monozy- gotic twins have been shown to have a two- to fourfold in- crease in concordance rates for mood disorders over dizygotic twins, compelling data for the role of genetic factors in mood disorders.
Parental loss before adolescence is also a well-documented risk factor for adult-onset depression. Medical problems of many types, such as cancer of the pancreas, multiple sclerosis, and space-occupying lesions of the brain, can produce depression.
15.13. The answer is B
A hypomanic episode lacks psychotic features, which can some- times be associated with mania. A manic episode is a distinct period of an abnormally and persistent elevated, expansive, or irritable mood lasting for at least 1 week or less if a patient must be hospitalized. A hypomanic episode lasts at least 4 days and is similar to a manic episode except that it is not sufficiently se- vere to cause impairment in social or occupational functioning. Patients often believe that they benefit from the energy and con- fidence of hypomania. Diagnostically, history of hypomania is preferably obtained from significant others who have observed
the patient; the experience is often pleasant, and the subject may be unaware of it or may tend to deny it. Both mania and hypo- mania are associated with inflated self-esteem, decreased need for sleep, distractibility, great physical and mental activity, and overinvolvement in pleasurable behavior.
15.14. The answer is D
In Sigmund Freud’s structural theory, introjection of the lost object into the ego leads to the typical depressive symptoms of a lack of energy available to the ego. The superego, which is unable to retaliate against the lost object externally, flails out at the psychic representation of the lost object, now internalized in the ego as an introject. When the ego overcomes or merges with the superego, energy previously bound in the depressive symptoms is released, and mania supervenes with the typical symptoms of excess.
Projection is the unconscious defense mechanism in which a person attributes to another person those generally unconscious ideas, thoughts, feelings, and impulses that are personally unde- sirable or unacceptable. Sublimation is an unconscious defense mechanism in which the energy associated with unacceptable impulses or drives is diverted into personally and socially ac- ceptable channels.
Undoing is an unconscious defense mechanism by which a person symbolically acts out to reverse something unacceptable that has already been done or against which the ego must defend itself. Altruism is regard for and dedication to the welfare of others.
15.15. The answer is C
Nearly two-thirds of patients with depressive disorders, whether experiencing typical or atypical symptoms, exhibit marked short- ening of rapid eye movement (REM) latency, the period from sleep onset to the first REM period. This fact is not specific to atypical depression as are the other choices listed. Mood reactiv- ity is characterized by mood elevation in response to something good happening. Leaden paralysis (a heavy, leaden feelings in one’s arms and legs), hypersomnia, increased appetite, and sig- nificant weight gain are also among the features of atypical de- pression.
15.16. The answer is A
l-Tryptophan, the amino acid precursor to serotonin, has been used as an adjuvant to both antidepressants and lithium in the treatment of bipolar I disorder. Tyrosine is the amino acid precur- sor to dopamine. l-Tryptophan has also been used alone as a hyp- notic and an antidepressant. l-Tryptophan and l-tryptophan– containing products have been recalled in the United States because l-tryptophan has been associated with eosinophilia- myalgia syndrome. The symptoms include fatigue, myalgia, shortness of breath, rashes, and swelling of the extremities. Con- gestive heart failure and death can also occur.
Although several studies have shown that l-tryptophan is an efficacious adjuvant in the treatment of mood disorders, it should not be used for any purpose until the problem with eosinophilia- myalgia syndrome is resolved. Current evidence points to a con- taminant in the manufacturing process.
15.17. The answer is C
A history of more than one previous depressive episode is not an indicator of a good prognosis for major depressive disorder. Rather, it increases the possibility for a poor prognosis. Other indicators of a poor prognosis include coexisting dysthymic dis- order, abuse of alcohol and other substances, and anxiety disor- der symptoms. Men are more likely than women to experience a chronically impaired course.
Mild episodes, the absence of psychotic symptoms, and a short hospital stay are good prognostic indicators. Psychosocial indicators of a good course include a history of solid friend- ships during adolescence, stable family functioning, and gener- ally sound social functioning for the 5 years preceding the illness. Additional good prognostic signs are the absence of a comorbid psychiatric disorder and of a personality disorder, no more than one previous hospitalization for major depressive disorder, and an advanced age of onset.
15.18. The answer is A
Reactive depression is defined as depression that results from a specific life event. It continues as long as the event is present, and it terminates with the reversal of the event (e.g., return of a lover after a breakup). With interpersonal support, most people can face life’s reversals, which explains why reactive depression tends to be self-limiting. Hence, adjustment disorder is the more appropriate diagnosis for many cases of reactive depression. Op- positional defiant disorder is a recurring pattern of negative, hos- tile, disobedient, and defiant behavior in a child or adolescent, lasting for at least 6 months without serious violation of the basic rights of others. Conduct disorder is a childhood behavioral con- dition involving a pattern of repetitive and persistent conduct that infringes on the basic rights of others or does not conform to es- tablished societal norms or rules that are appropriate for a child of that age. Atypical depression is characterized by “reversed vege- tative symptoms,” which include oversleeping, overeating, rejec- tion sensitivity, and temporary brightening of mood in response to positive events. Schizoaffective disorder is a disorder with symptoms of both schizophrenia and manic-depressive disorder.
15.19. The answer is A
Among the factors favoring the occurrence of rapid cycling bipo- lar disorder are alcohol, stimulant, and caffeine use. Other fac- tors include female gender because this subtype is much more common in women than men. Most antidepressants readily in- duce excited episodes and thus aggravate the rapid cycling pat- tern. Rapid cycling is defined as the occurrence of at least four episodes of depression and hypomania or mania per year (not per month). Hospitalization of these patients is often frequent to stabilize medication and achieve compliance.
15.20. The answer is A
Reliance on the cross-sectional rather than longitudinal picture (not vice versa) may cause a misdiagnosis of mood disorder as schizophrenia. Cross-sectionally, young patients with bipo- lar disorder might seem psychotic and disorganized and thus schizophrenic. Their thought processes are so rapid that they may seem loose, but unlike patients with schizophrenia, they
Table 15.1
15. Mood Disorders 151 Common Causes of Misdiagnosis of Mood Disorder
as Schizophrenia
Reliance on cross-sectional rather than longitudinal picture Incomplete interepisodic recovery equated with schizophrenic
defect
Equation of bizarreness with schizophrenic thought disorder Ascribing irritable and cantankerous mood to paranoid delusions Mistaking depressive anhedonia and depersonalization for
schizophrenic emotional blunting
Flight of ideas perceived as loose associations
Lack of familiarity with the phenomenological approach in
assessing affective delusions and hallucinations
Heavy weight given to incidental schneiderian symptoms
Adapted from Akiskal HS, Puzantian VR. Psychotic forms of depression and mania. Psychiatr Clin North Am. 1979;2:419
display an expansive and elated affect, which is often conta- gious. By contrast, a severely retarded bipolar depressive person, whose affect may superficially seem flat, almost never exhibits major fragmentation of thought. The clinician should therefore place greater emphasis on the pattern of symptoms than on in- dividual symptoms in the differential diagnosis of mood and schizophrenic psychosis. Table 15.1 lists the most common pit- falls in diagnosis.
15.21. The answer is C
Cyclothymia is characterized by at least 2 years of numerous periods with hypomanic symptoms and numerous periods with depressive symptoms that do not meet criteria for a major de- pressive episode. This disorder apparently occurs at the same rate in both men and women, but women seek treatment more often than men. The lifetime prevalence rate of cyclothymic disorder is 0.4 to 1 percent.
15.22. The answer is B
The profile of dysthymic disorder overlaps with that of major depressive disorder but differs from it in that symptoms tend to outnumber signs (more subjective than objective depression). This means that disturbances in appetite and libido are unchar- acteristic, and psychomotor agitation or retardation is not ob- served. This all translates into a depression with attenuated symp- tomatology. Subtle endogenous features are observed, however, including inertia, lethargy, and anhedonia that are characteristi- cally worse in the morning.
Dysthymic disorder is distinguished from major depressive disorder by the fact that patients complain that they have always been depressed. Most cases are of early onset beginning in child- hood or adolescence and certainly occurring by the time patients reach their 20s. A late-onset subtype, which is much less preva- lent and not well characterized clinically, has been identified among middle-aged and geriatric populations, largely through epidemiological studies in the community. Diagnostic criteria stipulate the presence of depressed mood most of the time for at least 2 years (not episodic) or 1 year for children or adolescents. Patients should not have symptoms that are better accounted for as major depressive disorder and should never have had a manic or hypomaniac episode. The DSM-IV-TR allows clinicians to
  
152 15. Mood Disorders
specify whether the onset was early (before age 21 years) or late (age 21 years or older).
Dysthymic disorder refers to a subaffective or subclinical de- pressive disorder with (1) low-grade (not high-grade) chronicity for at least 2 years; (2) insidious onset, with the origin often in childhood or adolescence; and (3) a persistent or intermittent course.
15.23. The answer is E
Features such as restlessness, agitation, and pressured speech are characteristic of psychomotor agitation, not retardation. Psychomotor retardation, which is often present in depressed patients, manifests as paucity of spontaneous movements, re- duced speech amplitude and flow, increased latency of responses, indecisiveness, poor concentration and forgetfulness, and over- whelming fatigue. Brain imaging research that has revealed sub- cortical (extrapyramidal system) disturbances in mood disorders tends to support the centrality of psychomotor dysfunction in these disorders.
15.24. The answer is B
Anhedonia is one of the DSM IV-TR criteria for major depres- sion. Patients with severe depression and anhedonia may com- plain of being emotionally cut off from others and experience depersonalization in a world that seems strange to them (dereal- ization). Loss of pleasure and withdrawal from previously enjoy- able activities is typical of anhedonia. The impact of the loss of emotional resonance can be so pervasive that patients may de- nounce values and beliefs that had previously given meaning to their lives (e.g., members of the clergy complain they no longer believe in the Church and have lost God). The inability of a person with depressive disorder to experience normal emotions (commonly observed among young depressed patients) differs from a schizophrenic person’s flat affect in that the loss of emo- tions is itself experienced as painful; that is, the patient suffers immensely from the inability to experience emotions. Anhedo- nia does not typically include alexithymia, which is defined as a person’s inability to describe or be aware of one’s emotions or mood.
15.25. The answer is E
Suicide is almost twice as frequent in older adults as in the general population. The suicide rate for white men older than 65 years of age is five times higher than that of the general population. Aging reduces suicide attempts but increases their lethality. Depression is the most common psychiatric diagnosis in elderly suicide victims, unlike younger adults, in whom sub- stance abuse alone or with comorbid mood disorders is the most frequent diagnosis. Older patients with major medical illnesses or a recent loss should be evaluated for depressive symptomatol- ogy and suicidal ideation or plans, and aggressive treatment can help prevent their acting on suicidal thoughts. Suicide rates are rising most rapidly in young adults, but the greatest risk remains in those people older than the age of 65 years.
15.26. The answer is C
The strongest indicator of the likelihood of attempting suicide is a history of previous suicide attempts. It has been shown that
women make more suicidal gestures, but men are more likely to choose lethal methods and thus are more likely to successfully commit suicide. Therefore, male gender is considered more a risk factor for suicide completion. Alcohol or substance abuse, unemployment, and recent divorce are all additional risk fac- tors but are less significant than a history of previous suicide attempts.
15.27. The answer is C
The concept of schizoaffective (or cycloid) psychosis should be restricted to recurrent psychoses, with full affective and schizophrenic symptoms occurring nearly simultaneously during each episode. This diagnosis should not be considered for a mood psychosis in which mood-incongruent psychotic features (e.g., Schneiderian and Bleulerian symptoms) can be explained on the basis of one of the following: (1) affective psychosis superim- posed by mental retardation, giving rise to extremely hyperactive and bizarre manic behavior; (2) affective psychosis complicated by concurrent brain disease, substance abuse, or substantial with- drawal, known to give rise to numerous schneiderian symptoms; or (3) mixed episodes of bipolar disorder (which are notorious for signs and symptoms of psychotic disorganization). Official diagnostic systems use the category of schizoaffective disorder broadly. Thus, patients with clear-cut manic episodes receive a schizoaffective diagnosis if delusions or hallucinations occur in the interepisodic period in the absence of prominent affective symptoms. Many psychotic symptoms in mood disorders are of- ten explanatory, whereby the patient tries to make sense of the core experiences of the affective illness.
15.28. The answer is D
Propranolol (Inderal), a β-blocker, is an antihypertensive and may actually cause depressive symptoms. Many pharmacolog- ical agents, such as bromocriptine (Parlodel), isoniazid (Ny- drazid), cimetidine (Tagamet), and disulfiram (Antabuse), may precipitate mania, as can antidepressant treatment or withdrawal.
15.29. The answer is D
In considering antidepressant use in suicidal or impulsive pa- tients, caution should be taken in using medications that may be lethal in overdose (e.g., tricyclic antidepressants [TCA]) or that may exacerbate disinhibition or cognitive deficits (e.g., ben- zodiazepines). Therefore, it is not the best idea to prescribe a TCA to this patient. Selective serotonin reuptake inhibitors (SSRIs) and monoamine oxidase inhibitors (MAOIs) are better choices. Care should be taken to avoid precipitating a hyperten- sive crisis in patients taking MAOIs. Bupropion and venlafaxine are other antidepressants that can be used alone or in combination with another medication.
15.30. The answer is A
Electroconvulsive therapy (ECT) is effective in psychotic and nonpsychotic forms of depression. Usually, 8 to 12 ECT treat- ments are required to achieve symptomatic remission, and this form of therapy has been shown to be effective even in patients who are refractory to several different medications. Bilateral ECT is somewhat more effective than unilateral therapy, but
bilateral ECT also appears to have more cognitive side effects, such as retrograde memory loss.
15.31. The answer is E
After an extended hospital course, many patients, like this one, demonstrate withdrawn behavior, anorexia, and fatigue, occa- sionally associated with mild cognitive loss, that do not meet criteria for cognitive or mood disorder but are a direct prod- uct of resolving medical illness and associated debilitation. The likelihood that a mood disorder is due to a general medical con- dition is increased if a temporal relationship exists between the onset, exacerbation, or remission of the medical condition and the mood disorder. Atypical features (e.g., unusual age of on- set, lack of family history, and lack of prior episodes of mood disorder) also raise the likelihood of a medical basis for mood symptoms. Mild short-term memory deficits can be normal with aging and do not necessarily indicate that Mr. M has dementia. He also demonstrated largely intact cognition, making delirium an unlikely diagnosis. Patients with a diagnosis of anxiety disor- der with depressed mood primarily exhibit symptoms of anxiety states, such as marked tension, phobias, and panic attacks, all of which predate any depressive symptoms. In addition, anxiety disorders rarely appear for the first time after 40 years of age.
15.32. The answer is C
This case illustrates the intimate relationship among atypical de- pression, borderline personality disorder, and bipolar II disorder. These three conditions, listed as distinct nosological entities in the DSM-IV-TR, may nonetheless share an underlying psychobi- ological or genetic diathesis. The complaint is often heard that even when a mood disorder is diagnosed in a “borderline” pa- tient, response to antidepressants is disappointing. The problem is that affective disorders in these patients usually conform to bipolar II disorder—often complicated by ultrarapid cycling— and many clinicians trained in an earlier era, including some with a biological orientation, may lack sufficient experience in the art of pharmacologically managing patients who markedly deviate from classic bipolar I disorder. Recently, lamotrigine has shown promise for such patients.
Table 15.2 shows that the overlap between borderline per- sonality and mood disorders is extensive, so giving a borderline diagnosis to a person with mood disorder is redundant. Use of personality disorder diagnoses may lead to a neglect of the mood disorder or, perhaps, half-hearted treatment of the mood disor- der; failure to respond would then be blamed on the patient’s “self-defeating character” or “resistance to getting well,” thus exculpating the clinician.
15.33. The answer is E
When antidepressant medications first begin to work, patients tend to report an increase in energy levels before significant improvement in mood symptoms. For this reason, carrying out suicide plans is more of a risk during this period. Medication non- compliance with effective psychopharmacological treatments during both acute and maintenance therapy is a major cause of morbidity among patients with schizophrenia and disorders with poor insight, not among patients with depression. Although
Table 15.2
15. Mood Disorders 153 Overlap of Borderline Personality Disorder and
Mood Disorders
Familial
High rates of mood disorder
Phenomenology Dysthymic disorder Cyclothymic disorder Bipolar II disorder Mixed state
Pharmacological response
Worsening on most antidepressants Stabilization on anticonvulsants
Prospective course Major mood episodes Suicide
Adapted from Akiskal HS, Chen SE, Davis GC, Puzantian VR, Kashgarian M. Borderline: An adjective in search of a noun. Clin Psychiatry. 1985;46:41.
noncompliance is a possibility with any patient, it is not the best choice listed. This patient has nothing in her history to indicate bipolar disorder as a more appropriate diagnosis; therefore, there is no reason to believe manic or hypomanic episodes will be pre- cipitated by antidepressant use. Extrapyramidal symptoms are a consequence of typical antipsychotic use, not antidepressant use.
15.34. The answer is B
Much political and theological heat has been generated in recent years by the debate on physician-assisted suicide. Many of the patients requesting death are depressed and would be likely to re- spond to psychotherapy, pharmacotherapy, or both. Frequently, they have been inadequately diagnosed or treated. Active depres- sion can be considered a state of incompetence, making such pa- tients unable to make informed decisions for themselves. Family members or a living will should be consulted only in cases of medical emergencies in which decisions must be made imme- diately. In this case, treating the patient’s depression is the best option. It is not necessary to involve the hospital ethics commit- tee in this decision. Currently, Oregon is the only state that has passed enabling laws regarding physician-assisted suicide.
15.35. The answer is A
The diagnosis of bipolar disorder best fits the history and symp- toms, but it is by no means certain. She presents with a 1-month history of impaired judgment and erratic behavior followed by increased energy, pressured speech, mood lability, and decreased need for sleep, all of which indicate a manic episode. The emer- gence of paranoid delusions is also consistent with mania, and a single manic episode, with or without major depressive episodes, qualifies for a diagnosis of bipolar disorder. That diagnosis can only be made, however, if it is believed that the symptoms are not the result of a general medical condition or substance use. We know that she has been using increased amounts of marijuana and alcohol and that she is probably intoxicated with marijuana at the time of admission. Heavy marijuana use in some individuals can cause a psychotic state with paranoid delusions and halluci- nations. There are some features, however, that are inconsistent
  
154 15. Mood Disorders
with a purely marijuana-induced state, which more typically presents with decreased talkativeness and long response latency than with the pressured speech seen here. In addition, increased energy and activity and decreased need for sleep are much more likely in bipolar disorder than in a marijuana psychosis. The delusional belief of her thoughts being controlled by an outside force is strikingly similar to delusions of control that are so often seen in schizophrenia, but the prominent mood symptoms and time course (1 month) preclude that diagnosis. The diagnosis of schizoaffective disorder would require a 2-week period of psy- chotic symptoms without prominent mood symptoms, which is not the case here. An erotomanic delusional belief is that the patient is loved by another (often famous) person, not as is the case here, which the patient herself is preoccupied with being in love with someone else.
15.36. The answer is B
The events of days 5 and 6 almost certainly represent the emergence of a neuroleptic malignant syndrome, an idiosyn- cratic response to antipsychotics (especially high-potency, typ- ical agents) characterized by fever, rigidity, and obtundation. The clinical diagnosis is confirmed, with the typical findings of leukocytosis and greatly increased creatinine phosphokinase (CPK). Anticholinergic delirium includes fever but not the rigid- ity or laboratory findings. In addition, patients with an anticholin- ergic delirium are more likely to be agitated than withdrawn.
15.37. The answer is C
Neuroleptic malignant syndrome (NMS) is a life-threatening medical emergency. All medications must be stopped; switching to an atypical agent such as risperidone will not help her. Ap- propriate treatment includes life support, maintaining fluid and electrolyte balance, and decreasing her fever. Bromocriptine is a centrally acting dopamine agonist that presumably works by re- versing the effects of the antipsychotic-caused dopamine block- ade. The treatment of NMS commonly combines bromocrip- tine with dantrolene, a peripheral muscle relaxant. Consultation with the medical service is crucial because the treatment may be complex.
15.38. The answer is A
Suicidal patients with intent and specific plans should always be taken seriously. For patients believed to be too much at risk for outpatient therapy or partial hospital programs, inpatient treat- ment is required. Such hospitalization is preferably on a vol- untary basis, but if the patient refuses, involuntary admission is required. The options of changing antidepressants and giv- ing more time for the medication to take effect are not the best options in this circumstance, although they may be in patients when suicide is less of an acute risk. Psychotherapy is also not an option when immediate intervention is needed with actively suicidal patients. Contact with the family and friends of suicidal patients is essential, and maintaining patient confidentiality is not mandatory if divulged material is believed to be necessary to protect the patient’s life. Some patients appear so imminently and acutely suicidal that the clinician is afraid to let them out of the office. Patients can be admitted to the hospital against their will if they are a danger to themselves or to other people.
15.39. The answer is D
MAOIs have been shown to be particularly effective in the treat- ment of patients with atypical depression. Important side effects of MAOI therapy include hypertensive crisis and serotonin syn- drome. Hypertensive crisis can be precipitated when foods rich in tyramine (e.g., wine, cheese) are ingested by someone who is taking an MAOI. Serotonin syndrome is caused by the interaction of an MAOI with a SSRI, pseudoephedrine, or meperidine. Sero- tonin syndrome is characterized by hyperthermia, muscle rigid- ity, and altered mental status. Therefore, fluoxetine, an SSRI, is contraindicated in this patient.
15.40. The answer is B
Given the chronological course and degree of severity of the pa- tient’s symptoms, the most likely diagnosis is dysthymia. Dys- thymia is diagnosed when a patient experiences depressed mood for most of the day for a minimum of 2 years plus at least two of the following: insomnia or hypersomnia, poor concentration, low energy, change in appetite, or lack of self-worth. Unlike dysthymia, in major depressive disorder, patients must have five or more symptoms for most of the day over 2 weeks. General- ized anxiety disorder is characterized by sleep disturbance, ir- ritability, inability to concentrate, uncontrollable worrying, and fatigue. Substance abuse is a possible, although it is a less likely diagnosis given the duration of symptoms.
Answers 15.41–15.45 15.41. The answer is A 15.42. The answer is B 15.43. The answer is C 15.44. The answer is A
15.45. The answer is B
Bereaved persons exhibit many depressive symptoms during the first 1 to 2 years after their loss. However, there is a distinction between depression and bereavement. Whereas grieving persons and their relatives perceive bereavement as a normal reaction to the loss, those with depressive disorder often view themselves as sick and may actually believe they are losing their minds. Active suicidal ideation is rare in grief but common in major depressive disorder. Unlike a melancholic person, a grieving person reacts to the environment and tends to show a range of positive effects. Delusions of worthlessness or sin and psychotic experiences in general point toward a mood disorder. Guilt is experienced in both bereavement and depression. Bereaved persons often feel guilty about not having done certain things that they believe might have saved the life of the deceased loved one.
Answers 15.46–15.49 15.46. The answer is B 15.47. The answer is C
15.48. The answer is D
15.49. The answer is D
Tricyclic antidepressants such as imipramine (Tofranil) are known for their cardiotoxicity because they prolong cardiac con- duction time and can cause a variety of arrhythmias. Also among the substantial side effects of tricyclic antidepressants are weight gain, sedation, anticholinergic effects, and orthostatic hypoten- sion. Weight gain is also a common adverse effect of clozapine (Clozaril), as well as agranulocytosis; therefore, weekly blood monitoring is necessary. Neither clozapine nor imipramine acts as norepinephrine partial agonists; instead, clozapine acts by blockade of serotonin-2a and D2 receptors, and imipramine acts by reducing the reuptake of norepinephrine and serotonin. There are no known teratogenic effects of clozapine or imipramine.
Answers 15.50–15.53 15.50. The answer is C 15.51. The answer is D 15.52. The answer is B
15.53. The answer is C
Between the extremes of manic-depressive illness defined by at least one acute manic episode, which could be mood-congruent or -incongruent (bipolar I disorder), and strictly defined major depressive disorder without any personal or family history of ma- nia (pure unipolar disorder), there exists an overlapping group of intermediary forms characterized by recurrent major depressive episodes and hypomania (bipolar II disorder). There is never any history of acute mania in either unipolar depression or bipolar II disorder. Psychotic features are typically not present in either form of depression. There can be recurrent episodes of major de- pressive symptoms, as well as atypical features in both unipolar and bipolar II forms. Symptoms of hypomania are characteristic only of bipolar II disorder.
Answers 15.54–15.59 15.54. The answer is B 15.55. The answer is D 15.56. The answer is F 15.57. The answer is A
erated by overstimulation is so excessive that it is replaced by a chronic disposition to irritability, fatigue (especially mental fa- tigue), lethargy, and exhaustion. It seems as if the patient’s mind refuses to take on new stresses. Anxious manifestations include headache, backache, heavy limbs, vague neuralgias, yawning, dyspepsia, palpitations, sweating of the hands and feet, chills, flushing, sensitivity to weather changes, insomnia, nightmares, pantophobia, asthenopia, and tinnitus.
Cyclothymic disorder is characterized by frequent short cy- cles of subsyndromal mild depression and hypomania. The course of cyclothymia is continuous or intermittent, with infre- quent periods of euthymia. Shifts in mood often lack adequate precipitants. Persons with cyclothymic disorder are dilettantes; they show great promise in many areas but rarely bring any of their efforts to fruition.
Hypomania occurring as part of bipolar II disorder rarely pro- gresses to manic psychosis, and insight is relatively preserved. Hypomania refers to a distinct period of at least a few days of mild elevation of mood, sharpened and positive thinking, and in- creased energy and activity levels, typically without the impair- ment characteristic of manic episodes. It is not merely a milder form of mania. Hypomania is distinguished from mere happi- ness in that it tends to recur and can sometimes be mobilized by antidepressants.
The most typical developmental background of dysthymic disorder is an insidious onset of depression dating back to late childhood or the teens, preceding any superimposed major de- pressive episodes by years or even decades. The typical features of dysthymic disorder are the presence of a depressed mood that lasts most of the day and is present almost continuously. There are associated feeling of inadequacy, guilt, irritability, and anger; withdrawal from society; loss of interest; and inactivity and lack of productivity. It differs from major depression in that patients complain that they have always been depressed.
The designation bipolar I disorder is synonymous with what was formally known as bipolar disorder—a syndrome in which a complete set of mania symptoms occurs during the course of the disorder. Typically beginning in the teenage years, the 20s, or the 30s, the first episode could be manic, depressive, or mixed. Bipo- lar I disorder, single manic episode, describes patients having a first episode of mania (most such patients eventually develop depressive episodes). Bipolar I disorder, recurrent, is diagnosed on the basis of the symptoms of the most recent episode (i.e., bipolar I disorder, most recent episode manic; bipolar disorder I disorder, most recent episode depressed).
Bipolar II disorder is characterized by depressive episodes and hypomanic episodes, but the episodes of manic-like symp- toms do not quite meet the diagnostic criteria for a full manic syndrome. A few studies indicate that bipolar II disorder is asso- ciated with more marital disruption and with an onset at an earlier age than bipolar I disorder. Evidence also indicates that patients with bipolar II disorder are at greater risk of both attempting and completing suicide than patients with bipolar I disorder or major depressive disorder.
15.58. The answer is
15.59. The answer is
E
C
The diagnosis of neurasthenia is currently used more in China than the rest of the world. It is a condition characterized by vague functional physical and mental fatigue. The anxiety gen-
15. Mood Disorders 155
 
Anxiety disorders are among the most prevalent mental disorders in the general population. Nearly 30 million persons are affected in the United States. Anxiety disorders are associated with signif- icant morbidity and often are chronic and resistant to treatment. The text revision of the fourth edition of the Diagnostic and Statistical Manual of Mental Disorders (DSM-IV-TR) contains eight anxiety disorders: (1) panic disorder with or without ago- raphobia, (2) agoraphobia with or without panic disorder, (3) specific phobia, (4) social phobia, (5) obsessive-compulsive dis- order (OCD), (6) posttraumatic stress disorder (PTSD), (7) acute stress disorder, and (8) generalized anxiety disorder (GAD).
Anxiety disorders, similar to most psychiatric disorders, are usually the result of a complex interplay of biological, psycho- logical, and psychosocial elements. Treatment of patients with
16
Anxiety Disorders
these disorders can be correspondingly complex. Understanding the neuroanatomy and molecular biology of anxiety promises new insights into the etiology and more effective treatments in the future. An array of treatment approaches is currently avail- able, including psychoanalytic, cognitive, behavioral, and psy- chopharmacologic treatments. Many times, a combination of these treatments is used to best address the multiplicity of etio- logic forces.
Another aspect of anxiety disorders is the exquisite interplay of genetic and experiential factors. Students should also be aware of the role of specific neurotransmitters in the development of anxiety and the mechanisms of anxiolytic medications.
Students should study the questions and answers below for a useful review of these disorders.
HELPFUL HINTS
Students should know the following names, cases, terms, and acronyms related to anxiety disorders.
acute stress disorder anticipatory anxiety anxiety
Aplysia
aversive conditioning benzodiazepines clomipramine
(Anafranil) counterphobic attitude Jacob M. DaCosta Charles Darwin
ego dystonic
fear
Otto Fenichel flooding
Sigmund Freud GABA
generalized anxiety
disorder hypnosis
imipramine (Tofranil) implosion intrapsychic conflict isolation
lactate infusion
limbic system
Little Albert
Little Hans
locus ceruleus and raphe
nuclei
MHPG
mitral valve prolapse norepinephrine numbing obsessive-compulsive
disorder (OCD) panic attack
panic disorder panicogens phobias
◦ agoraphobia ◦ social
◦ specific
posttraumatic stress disorder (PTSD)
propranolol (Inderal) reaction formation repression
secondary gain serotonin
shell shock
sleep EEG studies soldier’s heart systematic
desensitization thought stopping undoing
John B. Watson Joseph Wolpe
QUESTIONS
Directions
Each of the questions or incomplete statements below is followed by five suggested responses or completions. Select the one that is best in each case.
16.1. Posttraumatic stress disorder (PTSD) differs from acute stress disorder in that
A. acute stress disorder occurs earlier than PTSD
B. PTSD is associated with at least three dissociative
symptoms
C. reexperiencing the trauma is not found in acute stress
disorder
D. avoidance of stimuli associated with the trauma is
only found in PTSD
E. PTSD lasts less than 1 month after a trauma
156
▲▲▲▲▲▲▲▲▲ ▲▲▲▲
▲▲▲▲▲ ▲▲▲▲ ▲
▲▲▲ ▲▲▲▲▲▲▲▲▲▲
▲▲▲▲▲▲▲ ▲▲▲▲▲▲▲
. 16.2. Theriskofdevelopinganxietydisordersisenhancedby
A. eating disorders B. depression
C. substance abuse D. allergies
E. all of the above
. 16.3. Which of the following is not a sign of poor prognosis in obsessive-compulsive disorder (OCD)?
A. Childhood onset
B. Coexisting major depression C. Good social adjustment
D. Bizarre compulsions
E. Delusional beliefs
. 16.4. Whichofthefollowingstatementsregardinganxietyand gender differences is true?
A. Women have greater rates of almost all anxiety dis- orders.
B. Gender ratios are nearly equal with OCD.
C. No significant difference exists in average age of
anxiety onset.
D. Women have a twofold greater lifetime rate of agora-
phobia than men.
E. All of the above
. 16.5. Which of the following epidemiological statements is true regarding anxiety disorders?
A. Panic disorder has the lowest heritability.
B. The mean age of onset is higher in girls.
C. The age of onset is earlier than that of mood dis-
orders.
D. Rates in males peak in the fourth and fifth decades of
life.
E. All of the above
. 16.6. Sigmund Freud postulated that the defense mechanisms necessary in phobias are
A. regression, condensation, and dissociation B. regression, condensation, and projection C. regression, repression, and isolation
D. repression, displacement, and avoidance E. repression, projection, and displacement
. 16.7. Anxiety disorders
A. are greater among people at lower socioeconomic levels
B. are highest among those with higher levels of educa- tion
C. are lowest among homemakers
D. have shown different prevalences with regard to social
class but not ethnicity
E. all of the above
16. Anxiety Disorders 157 16.8. Generalizedanxietydisorder
A. is least likely to coexist with another mental disorder
B. has a female-to-male ratio of 1:2
C. is a mild condition
D. has about a 50 percent chance of a recurrence after
recovery
E. has a low prevalence in primary care settings
16.9. Physiological activity associated with PTSD include all except
A. decreased parasympathetic tone B. elevated baseline heart rate
C. excessive sweating
D. increased circulating thyroxine E. increased blood pressure
16.10. Unexpected panic attacks are required for the diagnosis of
A. generalized anxiety disorder
B. panic disorder
C. social phobia
D. specific phobia
E. all of the above
16.11. Isolated panic attacks without functional disturbances
A. usually involves anticipatory anxiety or are phobic B. are part of the criteria for diagnostic panic disorder C. occur in less than 2 percent of the population
D. rarely involve avoidance
E. none of the above
16.12. Which of the following is not a component of the DSM- IV-TR diagnostic criteria for OCD?
A. Children need not recognize that their obsessions are unreasonable.
B. Obsessions are acknowledged as excessive or unrea- sonable.
C. Obsessions or compulsions are time consuming and take more than 1 hour a day.
D. The person recognized the obsessional thoughts as a product of outside him- or herself.
E. Thepersonattemptstoignoreorsuppresscompulsive thoughts or impulses.
16.13. All of the following are true for the course of panic dis- order except
A. patients become concerned after the first one or two panic attacks
B. excessive caffeine intake can exacerbate symptoms
C. comorbid depression increases risk for committing
suicide
D. the overall course is variable
E. patients without comorbid agoraphobia have a higher
recovery rate
158 16. Anxiety Disorders
. 16.14. Tourette’s disorder has been shown to possibly have a
familial and genetic relationship with
A. generalized anxiety disorder B. obsessive-compulsive disorder C. panic disorder
D. social phobia
E. none of the above
. 16.15. Allofthefollowinghavebeennotedthroughbrainimag- ing in patients with panic disorder except
A. magnetic resonance imaging (MRI) studies have shown pathological involvement of both temporal lobes
B. generalized cerebral vasoconstriction
C. right temporal cortical atrophy
D. increased blood flow to the basal ganglia
E. positron emission tomography scans have implicated
dysregulation of blood flow in panic disorder
. 16.16. A patient with OCD might exhibit all of the following
brain imaging findings except
A. longer mean T1 relaxation times in the frontal cortex than normal control subjects
B. significantly more gray matter and less white matter than normal control subjects
C. abnormalities in the frontal lobes, cingulum, and basal ganglia
D. decreasedcaudatevolumesbilaterallycomparedwith normal control subjects
E. lower metabolic rates in basal ganglia and white matter than in normal control subjects
. 16.17. Buspirone (Buspar) acts as a
A. dopamine partial agonist useful in the treatment of OCD
B. serotonin partial agonist useful in the treatment of OCD
C. dopamine partial agonist useful in the treatment of generalized anxiety disorder
D. serotonin partial agonist useful in treatment of gen- eralized anxiety disorder
E. none of the above
. 16.18. Whichofthefollowingchoicesmostaccuratelydescribes the role of serotonin in OCD?
A. Serotonergic drugs are an ineffective treatment.
B. Dysregulation of serotonin is involved in the symp-
tom formation.
C. Measures of platelet binding sites of titrated
imipramine are abnormally low.
D. Measures of serotonin metabolites in cerebrospinal
fluid are abnormally high.
E. None of the above
16.19. Which of the following medical disorders are not as- sociated with panic disorder due to a general medical condition?
A. Cardiomyopathy
B. Parkinson’s disease
C. Epilepsy
D. Sjo ̈gren’s syndrome
E. Chronic obstructive pulmonary disease (COPD)
16.20. Which of the following disorders is rarely confused with anxiety that stems primarily from medical dis- orders?
A. Panic disorder
B. Specific phobia
C. Obsessive-compulsive disorder D. Posttraumatic stress disorder E. Generalized anxiety disorder
16.21. Inductionofpanicattacksinpatientswithpanicdisorder can occur with
A. carbon dioxide B. cholecystokinin C. doxapram
D. yohimbine
E. all of the above
16.22. First-line medication treatments of anxiety disorders may
generally include all of the following except
A. diazepam (Valium) B. fluoxetine (Prozac) C. fluvoxamine (Luvox) D. nefazodone (Serzone) E. venlafaxine (Effexor)
16.23. Therapy for phobias may include all of the following except
A. counterphobic attitudes B. flooding
C. phenelzine (Nardil)
D. propranolol (Inderal)
E. systematic desensitization
16.24. Mr. A was a successful businessman who presented for treatment after a change in his business schedule. Al- though he had formerly worked largely from an office near his home, a promotion led to a schedule of frequent out-of-town meetings requiring weekly flights. Mr. A re- ported being “deathly afraid” of flying. Even the thought of getting on an airplane led to thoughts of impending doom in which he envisioned his airplane crashing to the ground. These thoughts were associated with intense fear, palpitations, sweating, clamminess, and stomach upset. Although the thought of flying was terrifying enough, Mr. A became nearly incapacitated when he went to the airport. Immediately before boarding, Mr. A would often
have to turn back from the plane, running to the bathroom to vomit. Which of the following is the most appropriate treatment for this patient who has another flight sched- uled tomorrow?
C. Obsessive-compulsive personality disorder D. Generalized anxiety disorder
E. None of the above
Directions
Each set of lettered headings below is followed by a list of num- bered words or phrases. For each numbered word or phrase, select
A. if the item is associated with A only
B. if the item is associated with B only
C. if the item is associated with both A and B D. if the item is associated with neither A nor B
Questions 16.27–16.31
A. Social phobia B. Agoraphobia
. 16.27. Symptoms include blushing and muscle twitching
. 16.28. Is associated with a sense of suffocation
. 16.29. Is chronic without a history of panic disorder
. 16.30. May be associated with panic attacks
. 16.31. Patients are comforted by the presence of another person
Questions 16.32–16.36
A. Generalized anxiety disorder B. Panic disorder
. 16.32. Response rates between 60 and 80 percent have been reported to buspirone
. 16.33. Patients with the disorder may still be responsive to bus- pirone after being exposed to benzodiazepine
. 16.34. Buspirone’s use is limited to potentiating the effects of other antidepressants and counteracting the adverse sex- ual effects of selective serotonin reuptake inhibitors
. 16.35. Relapse rates are generally high after discontinuation of medication
. 16.36. Tricyclic drugs have been reported to worsen anxiety symptoms in patients in whom the first symptoms were precipitated by cocaine
Questions 16.37–16.40
A. Cognitive behavioral therapy B. Psychodynamic therapy
16.37. Produces 80 to 90 percent panic-free status in panic dis- order within at least 6 months of treatment
16.38. Maybenearlytwiceaseffectiveinthetreatmentofsocial phobia as a more educational-supportive approach
16.39. Goals are more ambitious and require more time to achieve
16.40. Combining treatment with medication may be superior to either treatment alone
A. β-agonists
B. Exposure therapy C. Lorazepam
D. Paroxetine
E. None of the above
. 16.25. Ms.Kwasreferredforpsychiatricevaluationbyhergen- eral practitioner. On interview, Ms. K described a long history of checking rituals that had caused her to lose several jobs and had damaged numerous relationships. She reported, for example, that because she often had the thought that she had not locked the door to her car, it was difficult for her to leave the car until she had checked repeatedly that it was secure. She had broken several car door handles with the vigor of her checking and had been up to an hour late to work because she spent so much time checking her car door. Similarly, she had recurrent thoughts that she had left the door to her apartment un- locked, and she returned several times daily to check the door before she left for work. She reported that check- ing doors decreased her anxiety about security. Although Ms. K reported that she had occasionally tried to leave her car or apartment without checking the door (e.g., when she was already late for work), she found that she became so worried about her car being stolen or her apartment being broken into that she had difficulty going anywhere. Ms. K reported that her obsessions about security had be- come so extreme over the past 3 months that she had lost her job because of recurrent tardiness. She recognized the irrational nature of her obsessive concerns but could not bring herself to ignore them.
Which of the following symptom patterns of OCD does Mrs. K present?
A. Intrusive thoughts B. Symmetry
C. Pathological doubt D. Contamination
E. None of the above
. 16.26. A23-year-oldwomanpresentstoclinicwithachiefcom- plaint of “difficulty concentrating because I worry about my child.” She had recently gone back to teaching after having her third child. The patient states she is constantly wondering about other things as well. For example, she is going to help her sister-in-law throw a goodbye party and finds herself constantly going over what she needs to do to prepare for the party. At the end of the day, her husband claims she is irritable and tired. At night, she is unable to sleep and keeps thinking about her tasks for the next day. What is the most likely diagnosis?
A. Avoidant personality disorder B. Obsessive-compulsive disorder
16. Anxiety Disorders 159
160 16. Anxiety Disorders Directions
Each group of questions below consists of lettered headings fol- lowed by a list of numbered phrases or statements. For each numbered phrase or statement, select the one lettered heading that is most associated with it. Each lettered heading may be selected once, more than once, or not at all.
Questions 16.41–16.45
A. Panic disorder
B. Generalized social phobia C. Posttraumatic stress disorder D. Generalized anxiety disorder E. Acute stress disorder
. 16.41. Is associated with depersonalization
. 16.42. Must include at least two spontaneous panic attacks
. 16.43. Symptoms must persist at least 1 month after the trauma
. 16.44. Must include three somatic or cognitive symptoms asso-
ciated with worry
. 16.45. Difficult to distinguish from avoidant personality disor-
der
Questions 16.46–16.49
A. Imaginal exposure
B. Interoceptive exposure
C. In vivo exposure
D. Systematic desensitization
. 16.46. A patient is presented with photographs of snakes while practicing various relaxation techniques to overcome fear; gradually, he practices relaxation while in the pres- ence of live snakes.
. 16.47. ApatientwithOCDattemptstousepublictelephonesand doorknobs while intentionally refraining from washing her hands afterward.
. 16.48. A patient is asked to imagine his wartime experiences as vividly as possible to confront his memory of the trau- matic events.
. 16.49. A patient breathes through a thin straw to produce the sensation of not getting enough air; this activity produces a similar sensation to the distressing feeling of getting on an airplane.
Questions 16.50–16.54
A. Acrophobia B. Ailurophobia C. Cynophobia D. Mysophobia E. Xenophobia
. 16.50. Fear of dirt and germs
. 16.51. Fear of heights
. 16.52. Fear of strangers
. 16.53. Fear of dogs
. 16.54. Fear of cats
ANSWERS
16.1. The answer is A
Acute stress disorder is a disorder that is similar to posttraumatic stress disorder (PTSD), but acute stress disorder occurs earlier than PTSD (within 4 weeks of the traumatic event) and remits within 2 days to 1 month after a trauma (not PTSD).
PTSD shows three domains of symptoms: reexperiencing the trauma; avoiding stimuli associated with the trauma; and expe- riencing symptoms of increased autonomic arousal, such as en- hanced startle. Flashbacks, in which the individual may act and feel as if the trauma is recurring, represent a classic form of re- experiencing. Other forms of reexperiencing symptoms include distressing recollections or dreams and either physiological or psychological stress reactions on exposure to stimuli that are linked to the trauma. Symptoms of avoidance associated with PTSD include efforts to avoid thoughts or activities related to trauma, anhedonia, reduced capacity to remember events related to trauma, blunted effect, feelings of detachment or derealization, and a sense of a foreshortened future. Symptoms of increased arousal include insomnia, irritability, hypervigilance, and ex- aggerated startle. The diagnosis of PTSD is only made when symptoms persist for at least 1 month; the diagnosis of acute stress disorder is made in the interim.
Acute stress disorder is characterized by reexperiencing, avoidance, and increased arousal, similar to PTSD. Acute stress disorder (not PTSD) is also associated with at least three disso- ciative symptoms.
16.2. The answer is E (all)
Disorders that may enhance the risk for the development of anxi- ety disorders include eating disorders, depression, and substance use and abuse. In contrast, anxiety disorders have been shown to elevate the risk of subsequent substance use disorders and may comprise a mediator of the link between depression and the subsequent development of substance use disorders in a clinical sample.
Several studies have also suggested that there is an associ- ation between anxiety disorders and allergies, high fever, im- munological diseases and infections, epilepsy, and connective tissue diseases. Likewise, prospective studies have revealed that the anxiety disorders may comprise risk factors for the develop- ment of some cardiovascular and neurological diseases, such as ischemic heart disease and migraine.
16.3. The answer is C
A good prognosis for people with obsessive-compulsive disorder (OCD) is indicated by good social and occupational adjustment, the presence of a precipitating event, and an episodic nature of symptoms. About one-third of patients with OCD have major depressive disorder, and suicide is a risk for all patients with OCD. A poor prognosis is indicated by yielding to (rather than resisting) compulsions, childhood onset, bizarre compulsions, the need for hospitalization, a coexisting major depressive dis- order, delusional beliefs, the presence of overvalued ideas (i.e., some acceptance of obsessions and compulsions), and the pres- ence of a personality disorder (especially schizotypal personality
disorder). The obsessional content does not seem to be related to the prognosis.
16.4. The answer is E (all)
The results of community studies reveal that women have greater rates of almost all of the anxiety disorders. Despite differences in the magnitude of the rates of specific anxiety disorders across studies, the gender ratio is strikingly similar. Women have an approximately twofold elevation in lifetime rates of panic, gen- eralized anxiety disorder, agoraphobia, and simple phobia com- pared with men in nearly all of the studies. The only exception is the nearly equal gender ratio in the rates of OCD and social phobia.
Studies of youth report similar differences in the magnitude of anxiety disorders among girls and boys. Similar to the gender ratio for adults, girls tend to have more of all subtypes of anxi- ety disorders irrespective of the age composition of the sample. However, it has also been reported that despite the greater rates of anxiety in girls across all ages, there is no significant difference between boys and girls in the average age at onset of anxiety.
16.5. The answer is C
Anxiety disorders have been shown to have the earliest age of onset of all major classes of mental and behavioral disorders with a median onset by the age of 12 years. This is far earlier than the onset of mood disorders or substance use disorders and comparable to that of impulse control disorders. Women have greater rates of anxiety disorders than men. This difference in gender rates can be seen as early as 6 years of age. Despite the far more rapid increase in anxiety disorders with age in girls than in boys, there are no gender differences in the mean age at onset of anxiety disorders (not higher in girls) or in their duration. Female preponderance of anxiety disorders is present across all stages of life but is most pronounced throughout early and mid- adulthood. The rates of anxiety disorders in men are also rather constant throughout adult life, but the rates in women peak in the fourth and fifth decades of life and decrease thereafter.
Studies show a three- to fivefold increased risk of anxiety disorders among first-degree relatives of persons with anxiety disorders. Twin studies reveal that panic disorder has the highest heritability and has been shown to have the strongest degree of familial aggregation, with an almost sevenfold elevation in risk.
16.6. The answer is D
Sigmund Freud viewed phobias as resulting from conflicts cen- tered on an unresolved childhood oedipal situation. In adults, because the sexual drive continues to have a strong incestuous coloring, its arousal tends to create anxiety that is characteristi- cally a fear of castration. The anxiety then alerts the ego to exert repression to keep the drive away from conscious representation and discharge. Because repression is not entirely successful in its function, the ego must call on auxiliary defenses. In phobic patients, the defenses, arising genetically from an earlier pho- bic response during the initial childhood period of the oedipal conflict, involves primarily the use of displacement—that is, the sexual conflict is transposed or displaced from the person who evoked the conflict to a seemingly unimportant, irrelevant object
Table 16.1
16. Anxiety Disorders 161 Psychodynamic Themes in Phobias
Principal defense mechanisms include displacement, projection, and avoidance.
Environmental stressors, including humiliation and criticism from an older sibling, parental fights, or loss and separation from parents, interact with a genetic constitutional diathesis.
A characteristic pattern of internal object relations is externalized in social situations in the case of social phobia.
Anticipation of humiliation, criticism, and ridicule is projected onto individuals in the environment.
Shame and embarrassment are the principal affect states. Family members may encourage phobic behavior and serve as
obstacles to any treatment plan.
Self-exposure to the feared situation is a basic principle of all
treatment.
or situation, which has the power to elicit anxiety. The phobic ob- ject or situation selected has a direct associative connection with the primary source of the conflict and has thus come naturally to symbolize it. Furthermore, the situation or object is usually such that the patient is able to keep out of its way and by the additional defense mechanism of avoidance to escape suffering from serious anxiety.
Regression is an unconscious defense mechanism in which a person undergoes a partial or total return to early patterns of adaptation. Condensation is a mental process in which one sym- bol stands for a number of components. Projection is an uncon- scious defense mechanism in which persons attribute to another person generally unconscious ideas, thoughts, feelings, and im- pulses that are undesirable or unacceptable in themselves. In psychoanalysis, isolation is a defense mechanism involving the separation of an idea or memory from its attached feeling tone. Dissociation is an unconscious defense mechanism involving the segregation of any group of mental or behavioral processes from the rest of the person’s psychic activity. Table 16.1 describes a more current view of seven of the psychodynamic themes in phobias.
16.7. The answer is A
Community studies have consistently found that rates of anxi- ety disorders in general are greater among those at lower levels of socioeconomic status and education level. Anxiety disorders are negatively associated with income and education levels. For example, there is almost a twofold difference between rates of anxiety disorders in individuals in the highest income bracket and those in the lowest and between those who completed more than 16 years of school and those who completed less than 11 years of school. In addition, certain anxiety disorders seem to be elevated in specific occupations. Anxiety disorders are higher in home- makers and those who are unemployed or have a disability. Sev- eral community studies have also yielded greater rates of anxiety disorders, particularly phobic disorders, among African Ameri- cans. The reasons for ethnic and social class differences have not yet been evaluated systematically; however, both methodolog- ical factors and differences in exposure to stressors have been advanced as possible explanations.
  
162 16. Anxiety Disorders
16.8. The answer is D
Generalized anxiety disorder (GAD) is a chronic (not mild) con- dition, and nearly half of patients who eventually recover experi- ence a later recurrence. GAD is characterized by frequent, per- sistent worry and anxiety that is disproportionate to the impact of the events or circumstances on which the worry focuses. The dis- tinction between GAD and normal anxiety is emphasized by the use of the words “excessive” and “difficult to control” in the cri- teria and by the specification that the symptoms cause significant impairment or distress. The anxiety and worry are accompanied by a number of physiological symptoms, including motor tension (i.e., shakiness, restlessness, headache), autonomic hyperactiv- ity (i.e., shortness of breath, excessive sweating, palpitations), and cognitive vigilance (i.e., irritability). The ratio of women to men with the disorder is about 2:1 (not 1:2). The disorder usu- ally has its onset in late adolescence or early adulthood, although cases are commonly seen in older adults. Also, some evidence suggests that the prevalence is particularly high (not low) in pri- mary care settings. This is because patients with GAD usually seek out a general practitioner or internist for help with a somatic symptom. GAD is probably the disorder that most (not least) of- ten coexist with another mental disorder, usually social phobia, specific phobia, panic disorder, or a depressive disorder.
16.9. The answer is D
According to current conceptualizations, PTSD is associated with objective measures of physiological arousal. This includes elevated baselines heart rate, increased blood pressure, and ex- cessive sweating. Furthermore, evidence from studies of baseline cardiovascular activity revealed a positive association between heart rate and PTSD.
The finding of elevated baseline heart rate activity is con- sistent with the hypothesis of tonic sympathetic nervous system arousal in PTSD. Disturbance in autonomic nervous system ac- tivity in individuals with PTSD is characterized by increased sympathetic and decreased parasympathetic tone. Preliminary evidence suggests that this autonomic imbalance can be normal- ized with selective serotonin reuptake inhibitor treatment. There is no change in blood level of thyroxine in those with PTSD.
16.10. The answer is B
Unexpected panic attacks are required for the diagnosis of panic disorder, but panic attacks can occur in several anxiety disor- ders. The clinician must consider the context of the panic attack when making a diagnosis. Panic attacks can be divided into two types: (1) unexpected panic attacks, which are not associated with a situational trigger, and (2) situationally bound panic at- tacks, which occur immediately after exposure in a situational trigger or in anticipation of the situational trigger. Situationally bound panic attacks are most characteristic of social phobia and specific phobia. In generalized anxiety disorder, the anxiety can- not be about having a panic attack.
16.11. The answer is A
Some differences between the DSM-IV-TR and earlier versions in the diagnostic criteria of panic disorder are interesting. For ex- ample, no longer is a specific number of panic attacks necessary
in a specific period of time to meet criteria for panic disorder.
Rather, the attacks must be recurrent, and at least one attack must be followed by at least 1 month of anticipatory anxiety or phobic avoidance. This recognizes for the first time that although the panic attack is obviously the seminal event for diagnosing panic disorder, the syndrome involves a number of disturbances that go beyond the attack itself. Isolated panic attacks without functional disturbances are not diagnosed as panic disorder. Furthermore, isolated panic attacks without functional distur- bance are common, occurring in approximately 15 percent of the population.
16.12. The answer is D
Obsessions and compulsions are the essential features of OCD. An individual must exhibit either obsessions or compulsions to meet DSM-IV-TR criteria. The DSM-IV-TR recognizes obses- sions as “persistent ideas, thoughts, impulses, or images that are experienced as intrusive and inappropriate,” causing distress. Obsessions provoke anxiety, which accounts for the categoriza- tion of OCD as an anxiety disorder. However, they must be differ- entiated from excessive worries about real-life problems and as- sociated with efforts to either ignore or suppress the obsessions. The DSM-IV-TR diagnostic criteria for OCD indicate that the obsessions must be acknowledged as excessive or unreasonable (with the exception that children need not acknowledge this fact), there must be attempts to suppress these intrusive thoughts, and the obsessions or compulsions are time consuming to the point of requiring at least 1 hour a day, among other diagnostic criteria. As part of the criteria, however, is not that the thoughts are a product of outside the person, as in thought insertion, but that the person recognizes that the thoughts are a product of his or her own mind.
16.13. The answer is A
After the first one or two panic attacks, patients may be relatively unconcerned about their condition. With repeated attacks, how- ever, the symptoms may become a major concern. Patients may attempt to keep the panic attacks secret and thereby cause their families and friends concern about unexplained changes in be- havior. Panic disorder, in general, is a chronic disorder, although its course is variable, both among patients and within a single patient. The frequency and severity of the attacks can fluctuate. Panic attacks can occur several times a day or less than once a month. Excessive intake of caffeine or nicotine can exacerbate the symptoms. Depression can complicate the symptom picture in anywhere from 40 to 80 percent of all patients. Although the patients do not tend to talk about suicidal ideation, they are at increased risk for committing suicide. Recovery rates appear to be higher in patients without comorbid agoraphobia than in those who meet criteria for both conditions. Family interactions and performance in school and at work commonly suffer. Pa- tients with good premorbid functioning and symptoms of brief duration tend to have a good prognosis.
16.14. The answer is B
An interesting set of findings concerns the possible relationship between a subset of cases of OCD and certain types of motor tic
syndromes (i.e., Tourette’s disorder and chronic motor tics). In- creased rates of OCD, Tourette’s disorder, and chronic motor tics were found in the relatives of Tourette’s disorder patients com- pared with relatives of control subjects whether or not the patient had OCD. However, most family studies of probands with OCD have found elevated rates of Tourette’s disorder and chronic mo- tor tics only among the relatives of probands with OCD who also have some form of tic disorder. Taken together, these data suggest that there is a familial and perhaps genetic relationship between Tourette’s disorder and chronic motor tics and some cases of OCD. Cases of the latter in which the individual also manifests tics are the most likely to be related to Tourette’s disorder and chronic motor tics. Because there is considerable evidence of a genetic contribution to Tourette’s disorder, this finding also supports a genetic role in a subset of cases of OCDs.
16.15. The answer is D
Structural brain imaging studies, such as magnetic resonance imaging (MRI), in patients with panic disorder have implicated pathological involvement in the temporal lobes, particularly the hippocampus. One MRI study reported abnormalities, especially cortical atrophy, in the right temporal lobes of these patients. Functional brain imaging studies, such as positron emission tomography (PET), have implicated dysregulation of cerebral blood flow. Specifically, anxiety disorders and panic attacks are associated with cerebral vasoconstriction, which may result in central nervous system symptoms such as dizziness and in pe- ripheral nervous system symptoms that may be induced by hy- perventilation and hypocapnia. Increased blood flow to the basal ganglia has not been noted in patients with panic disorder.
16.16. The answer is E
Brain imaging studies of patients with OCD using PET scans have found abnormalities in frontal lobes, cingulum, and basal ganglia. PET scans have shown higher (not lower) levels of metabolism and blood flows to those areas in OCD patients than in control subjects. Volumetric computed tomography scans have shown decreased caudate volumes bilaterally in OCD patients compared with normal control subjects. Morphometric MRI has revealed that OCD patients have significantly more gray matter and less white matter than normal control subjects. MRI has also shown longer mean T1 relaxation times in the frontal cortex in OCD patients than is seen in normal control subjects.
16.17. The answer is D
Buspirone (Buspar) is a serotonin receptor partial agonist and is most likely effective in 60 to 80 percent of patients with gen- eralized anxiety disorder (GAD). Data indicate that buspirone is more effective in reducing the cognitive symptoms of GAD than in reducing the somatic symptoms. The major disadvantage of buspirone is that its effects take 2 to 3 weeks to become evi- dent in contrast to the almost immediate anxiolytic effects of the benzodiazepines.
16.18. The answer is B
Clinical trials of drugs have supported the hypothesis that dys- regulation of serotonin is involved in the symptom formation of
obsessions and compulsions. Data show that serotonergic drugs are an effective treatment, but it is unclear whether serotonin is involved in the cause of OCD.
Clinical studies have shown that measures of platelet binding sites of imipramine and of serotonin metabolites in cerebrospinal fluid are variable, neither consistently abnormally low nor ab- normally high.
16.19. The answer is D
A high prevalence of generalized anxiety disorder (not panic disorder) symptoms has been reported in patients with Sjo ̈gren’s syndrome. Sjo ̈gren’s syndrome is a chronic autoimmune dis- ease in which a person’s white blood cells attack their moisture- producing glands. The hallmark symptoms are dry eyes and dry mouth; however, it may also cause dysfunction of other organs.
The symptoms of anxiety disorder caused by a general med- ical condition can be identical to those of the primary anxiety disorders. A syndrome similar to panic disorder is the most com- mon clinical picture. Patients who have cardiomyopathy may have the highest incidence of panic disorder secondary to a gen- eral medical condition. Cardiomyopathy is a disease of the heart muscle (myocardium). One study reported that 83 percent of pa- tients with cardiomyopathy awaiting cardiac transplantation had panic disorder symptoms. Increased noradrenergic tone in these patients may be the provoking stimulus for the panic attacks. In some studies, about 25 percent of patients with Parkinson’s disease and chronic obstructive pulmonary disease have symp- toms of panic disorder. Other medical disorders associated with panic disorder include chronic pain; primary biliary cirrhosis (an autoimmune disease of the liver); and epilepsy (a chronic dis- order characterized by paroxysmal brain dysfunction caused by excessive neuronal discharge), particularly when focus is in the right parahippocampal gyrus.
16.20. The answer is B
Specific phobia is usually easily distinguished from anxiety stem- ming from primary medical problems by the focused nature of the anxiety. Such specificity is not typical of anxiety disorders related to medical problems.
Panic disorder with or without agoraphobia must be dif- ferentiated from a number of medical conditions that produce similar symptomatology. Panic attacks are associated with a va- riety of endocrinologic disorders, including hypo- and hyper- thyroid states, hyperparathyroidism, and pheochromocytomas. Episodic hypoglycemia associated with insulinomas can also produce panic-like states, as can primary neuropathologic pro- cesses. These include seizure disorders, vestibular dysfunction, neoplasms, and the effects of both prescribed and illicit sub- stances on the central nervous system. Finally, disorders of the cardiac and pulmonary systems, including arrhythmias, chronic obstructive disease, and asthma, can produce autonomic symp- toms and accompanying crescendo anxiety that can be difficult to distinguish from panic disorder.
A number of primary medical disorders can produce syn- dromes that bear a striking resemblance to obsessive-compulsive disorder (OCD). In fact, the current conceptualization of OCD
16. Anxiety Disorders 163
164 16. Anxiety Disorders
as a disorder of the basal ganglia derives from the phenomeno- logical similarity between idiopathic OCD and OCD-like dis- orders that are associated with basal ganglia diseases, such as Sydenham’s chorea and Huntington’s disease. It should be noted that OCD frequently develops before age 30 years, and new-onset OCD in an older individual should raise questions about potential neurological contributions to the disorder. Also, among children with pediatric autoimmune neuropsychiatric disorder associated with streptococcus (PANDAS), the syndrome appears to emerge relatively acutely, in contrast to the more insidious onset of child- hood OCD in the absence of infection. Hence, children with acute presentations, the role of such an infectious process should be considered.
It is particularly important to recognize potentially treatable contributors to posttraumatic symptomatology in the differential for posttraumatic stress disorder (PTSD). For example, neuro- logical injury after head trauma can contribute to the clinical picture, as can psychoactive substance use disorders or with- drawal syndromes, either in the period immediately surrounding the trauma or many weeks after the trauma. Medical contribu- tors can usually be detected through careful history and physical examination.
Generalized anxiety disorder (GAD) must be differentiated from both medical and other psychiatric disorders. Similar neurological, endocrinologic, metabolic, and medication-related disorders to those considered in the differential diagnosis of panic disorder are relevant to the differential diagnosis of GAD.
16.21. The answer is E (all)
Since the original finding that sodium lactate infusion can induce panic attacks in patients with panic disorder, many substances have shown similar panicogenic properties, including the no- radrenergic stimulant yohimbine (Yocon), carbon dioxide, the respiratory stimulant doxapram (Dopram), and cholecystokinin. Disordered serotonergic, noradrenergic, and respiratory systems are doubtless implicated in panic disorder, and the condition ap- pears to be caused both by a genetic predisposition and some type of traumatic distress. More recently, neuroimaging stud- ies revealed that patients with panic disorder have abnormally brisk cerebrovascular responses to stress, showing greater vaso- constriction during hypocapnic respiration than normal control subjects.
16.22. The answer is A
Antidepressant medication is increasingly seen as the medica- tion treatment of choice for the anxiety disorders. More specifi- cally, drugs with primary effects on the serotonin neurotrans- mission system have become first-line recommendations for panic disorder, social phobia, OCD, and PTSD. Evidence now exists that such medications are also effective for generalized anxiety disorder. Although they typically take longer to work than benzodiazepines, the selective serotonin reuptake inhibitors (SSRIs) such as fluoxetine (Prozac), sertraline (Zoloft), paroxe- tine (Paxil), fluvoxamine (Luvox), and citalopram (Celexa), as well as venlafaxine (Effexor) and nefazodone (Serzone), are probably more effective than benzodiazepines and easier to dis- continue. Increasingly, benzodiazepines such as diazepam (Val- ium) are used only for the temporary relief of extreme anxiety
as clinician and patient wait for the effects of antidepressants to take hold. Longer-term administration of benzodiazepines is reserved for patients who do not respond to or cannot tolerate antidepressants.
16.23. The answer is A
A counterphobic attitude is not a therapy for phobias, although it may lead to counterphobic behavior. Many activities may mask phobic anxiety, which can be hidden behind attitudes and be- havior patterns that represent a denial, either that the dreaded object or situation is dangerous or that one is afraid of it. Basic to this phenomenon is a reversal of the situation in which one is the passive victim of external circumstances to a position of attempting actively to confront and master what one fears. The counterphobic person seeks out situations of danger and rushes enthusiastically toward them. The devotee of dangerous sports, such as parachute jumping, rock climbing, bungee jumping, and parasailing, may be exhibiting counterphobic behavior.
Both behavioral and pharmacological techniques have been used in treating phobias. The most common behavioral technique is systematic desensitization, in which the patient is exposed se- rially to a predetermined list of anxiety-provoking stimuli graded in a hierarchy from least to most frightening. Patients are taught to self-induce a state of relaxation in the face of each anxiety- provoking stimulus. In flooding, patients are exposed to the pho- bic stimulus (actually [in vivo] or through imagery) for as long as they can tolerate the fear until they reach a point at which they can no longer feel it. The social phobia of stage fright in performers has been effectively treated with such β-adrenergic antagonists as propranolol (Inderal), which blocks the physio- logical signs of anxiety (e.g., tachycardia). Phenelzine (Nardil), a monoamine oxidase inhibitor, is also useful in treating social phobia.
16.24. The answer is C
Patients with specific phobias are often treated with as-needed benzodiazepines, such as lorazepam (Ativan). In the clinical case described, this is the most appropriate choice of treatment given their high safety margin (e.g., in overdose) and their overall ex- cellent efficacy and rapid onset of action. β-adrenergic receptor antagonists (not agonists) may be useful in the treatment of spe- cific phobia, especially when the phobia is associated with panic attacks. The most commonly used treatment for specific pho- bia is exposure therapy. In this method, therapists desensitize patients by using a series of gradual, self-paced exposures to the phobic stimulus; thus, this method would not be appropriate when immediate relief is required. Paroxetine, an SSRI, is not indicated for the immediate treatment of phobias.
16.25. The answer is C
The symptoms of an individual patient with OCD can overlap and change with time, but OCD has four major symptoms patterns. In this case, Mrs. K presents the symptom pattern of pathological doubt followed by a compulsion of checking. It is the second most common symptom pattern. The obsession often implies some danger of violence, in this case forgetting to lock the car door or the door to the apartment. The checking may involve multiple trips back into the house to check the stove, for example.
For Mrs. K, checking involves trips back to her car and her apartment to make sure both are secure, thereby making her constantly late for work. The patients have an obsessional self- doubt and always feel guilty about having forgotten or committed something.
The most common symptom pattern in OCD is an obsession of contamination followed by washing or accompanied by com- pulsive avoidance of the presumably contaminated object. The feared object is often hard to avoid (e.g., feces, urine, dust, or germs). Patients with contamination obsessions usually believe that the contamination is spread from object to object or person to person by the slightest contact.
In the third most common pattern, there are intrusive ob- sessional thoughts without a compulsion. Such obsessions are usually repetitious thoughts without a compulsion. Such obses- sions are usually repetitious thoughts of a sexual or aggressive act that are reprehensible to the patient. Patients obsessed with thoughts or aggressive or sexual acts may report themselves to the police or confess to a priest.
The fourth most common pattern is the need for symmetry or precision, which can lead to a compulsion of slowness. Patients can literally take hours to eat a meal or shave their faces.
16.26. The answer is D
Excessive and uncontrollable worry characterized by irritability, insomnia, and fatigue is the most likely attributable to gener- alized anxiety disorder. The patient’s worries typically include various aspects of the patient’s life and cause functional impair- ment. These symptoms must persist for at least 6 months. Patients with avoidant personality disorder have a long-standing pattern of avoiding activities because they fear judgment and feel in- adequate. These symptoms are part of a lifelong pattern rather than new onset. Obsessive-compulsive disorder involves intru- sive thoughts that result in compulsive activity to relieve anxiety. These patients’ symptoms are ego dystonic in that they are able to recognize their problematic compulsions and obsessions. Pa- tients with obsessive-compulsive personality disorder often seek perfection and organization to a degree that it causes functional impairment. Their symptoms are ego syntonic in that they do not recognize the unreasonable nature of their behaviors.
Answers 16.27–16.31 16.27. The answer is A 16.28. The answer is B 16.29. The answer is A 16.30. The answer is C
16.31. The answer is B
Social phobia is the excessive fear of humiliation or embarrass- ment in various social setting, such as speaking in public, urinat- ing in a public rest room (also called shy bladder), or speaking to a date. It can sometimes be difficult to differentiate from ago- raphobia, which is the fear of or anxiety regarding places from which escape may be difficult. Both disorders can be associ-
ated with panic attacks, agoraphobia more so than social phobia. Whereas patients with agoraphobia are often comforted by the presence of another person in an anxiety-provoking situation, pa- tients with social phobia are made more anxious than before by the presence of other persons. Breathlessness, dizziness, a sense of suffocation, and fear of dying are common with panic disorder and agoraphobia; however, the symptoms associated with social phobia usually involve blushing, muscle twitching, and anxi- ety about scrutiny. Most cases of agoraphobia are thought to be caused by panic disorder. When the panic disorder is treated, the agoraphobia often improves with time. Agoraphobia without a history of panic disorder is often incapacitating and chronic, and depressive disorders and alcohol dependence often complicate its course.
Answers 16.32–16.36 16.32. The answer is A 16.33. The answer is A 16.34. The answer is D 16.35. The answer is C
16.36. The answer is B
Buspirone was promoted as a less sedating alternative to ben- zodiazepines in the treatment of panic disorder. Buspirone has lower potential for abuse and dependence than benzodiazepines and produces relatively few adverse effects and no withdrawal syndrome. Buspirone does not alter cognitive or psychomotor function, does not interact with alcohol, and is not a muscle re- laxant or an anticonvulsant. However, the efficacy of buspirone in patients with panic disorder is disappointing, and with its further drawback of delayed onset of action and the need for multiple dosings, its use is limited to potentiating the efficacy of other antidepressants and counteracting the adverse sexual effects of SSRIs.
Although the short-term efficacy of antipanic medications has been established, the question of how long to treat a panic patient who responds to treatment remains open. The results of follow- up studies are mixed. Several reports indicate that most panic patients relapse within 2 months to 2 years after the medication is discontinued. Following medication discontinuation, only about 30 to 45 percent of the patients remain well, and even remitted patients rarely revert back to significant phobic avoidance or se- rious vocational or social disability. Improvement may continue for years after a single course of medication treatment. Given the uncertainty about the optimal duration of treatment, the cur- rent recommendation is to continue full-dosage medication for panic-free patients for at least 1 year. Medication taper should be slow, with careful monitoring of symptoms. Distinction should be made among return symptoms, withdrawal, and rebound anxiety.
Atypical responses to medications have been reported in panic patients whose first panic attacks were precipitated by cocaine use. These patients respond preferentially to benzodi- azepines and anticonvulsants, but tricyclic drugs seem to worsen
16. Anxiety Disorders 165
166 16. Anxiety Disorders
their anxiety symptoms. This pattern of medication response sug- gests that cocaine-induced panic attacks may be related to a kindling-like phenomenon.
Tolerance to the sedative effects of benzodiazepines devel- ops quickly, but the antianxiety effect of a given dosage is well maintained over time in those with generalized anxiety disor- der (GAD). However, the relapse rate upon discontinuation of benzodiazepines is high, as is the risk for dependency.
Buspirone is a potential alternative to benzodiazepine treat- ment in GAD. Response rates between 60 and 80 percent have been reported at levels ranging from 30 to 60 mg a day in three divided doses. Although response rates seem comparable, more patients drop out of buspirone trials than benzodiazepine tri- als. The relative merits of buspirone and benzodiazepines are further detailed under panic disorder. One notable exception is that patients with GAD exposed to benzodiazepines may still be responsive to buspirone, unlike panic patients.
Answers 16.37–16.40 16.37. The answer is A 16.38. The answer is A 16.39. The answer is B
16.40. The answer is C
Some studies have shown that cognitive-behavioral treatment of panic disorder, or panic control therapy, produces 80 to 90 percent panic-free status within at least 6 months of treat- ment. Two-year follow-up indicates that more than 50 percent of patients who originally responded to panic control therapy have occasional panic attacks, and more 25 percent seek additional treatment. Nonetheless, these treatment responders do tend to have a significant decline in panic-related symptoms and most maintain many of their treatment gains.
As with panic disorder, considerable progress in the psycholog- ical treatment of social anxiety or social phobia is linked to the application of cognitive-behavioral methods. Unlike more tradi- tional psychotherapies, cognitive-behavioral approaches do not focus on the origins of social anxiety but instead focus on the use of coping strategies that can be implemented in current fear- ful situations. The most thoroughly studied form of cognitive- behavioral therapy for social phobia is a group therapy consist- ing of several discrete entities, including (1) presentation of a three-system (cognitive-behavioral-physiological) model of so- cial anxiety, (2) training in identification and restructuring of irrational beliefs regarding social performance, (3) in-session exposure to feared social situations via group role-playing sce- narios, and (4) homework assignments directing patients to use cognitive and exposure techniques in vivo. Groups are partic- ularly amenable to the treatment of social phobia in that they provide natural opportunities for patients to practice feared be- haviors in a supportive and informative context.
Outcome research is somewhat limited, but one study showed that cognitive-behavioral group therapy was nearly twice as effective as standard educational-supportive group psychother- apy.
Psychodynamic psychotherapy is based on the concept that symptoms result from mental processes that may be outside of the patient’s conscious awareness and that elucidating these pro- cesses can lead to remission of symptoms. Moreover, to lessen the patient’s vulnerability to panic, the psychodynamic thera- pist considers it necessary to identify and alter core conflicts. The goals of psychodynamic psychotherapy are more ambitious and require more time to achieve than those of a more symptom- focused treatment approach. Thus, these therapies are inherently more difficult to study than more concrete, focused, manual- based therapies.
Investigators have examined use of the combination of med- ication and cognitive behavioral therapy for patients with panic disorder and agoraphobia. Several short-term treatment studies have shown that the combination of the tricyclic medication imipramine (Tofranil) with one component of cognitive behav- ioral therapy, behavioral exposure, may be superior to either treatment alone. Another study showed that selective serotonin reuptake inhibitors, such as paroxetine (Paxil), plus cognitive therapy worked significantly better for patients with panic disor- der than cognitive therapy plus placebo. There has been one study of the combination of psychodynamic psychotherapy with med- ication. This study suggested that psychodynamic psychother- apy may improve the long-term outcome of medication-treated patients.
Answers 16.41–16.45 16.41. The answer is E 16.42. The answer is A 16.43. The answer is C 16.44. The answer is D
16.45. The answer is B
To meet the criteria for panic disorder, an individual must have experienced at least two spontaneous panic attacks in the absence of any trigger or environmental cue. Furthermore, at least one of these attacks must be associated with concern about additional attacks, worry about attacks, or changes in behavior. A panic attack is defined as an episode of abrupt intense fear accompanied by at least four autonomic or cognitive symptoms (i.e., sweating, palpitations, fear of dying).
A generalized social phobia is a chronic and disabling con- dition characterized by a phobic avoidance of most social situa- tions. It can be difficult to distinguish from avoidant personality disorder. Both disorders include avoidance; however, in general- ized social phobia, the patient has a desire and capacity to inter- act, but with avoidant personality disorder, the patient appears to have given up.
Posttraumatic stress disorder (PTSD) is a condition marked by the development of symptoms after exposure to traumatic life events (e.g., war, natural disaster). The person reacts to this expe- rience with fear and helplessness, persistently relives the event, and tries to avoid being reminded of it. To make the diagnosis, the symptoms must last for more than 1 month after the event and
must significantly affect important areas of life, such as family and work. Acute stress disorder is similar to PTSD but occurs earlier than PTSD (within 4 weeks of the traumatic event) and remits within 2 days to 4 weeks. Acute stress disorder is also associated with at least three of the following dissociative symp- toms: (1) a subjective sense of numbing, detachment, or absence of emotional responsiveness; (2) a reduction in awareness of his or her surroundings; (3) derealization; (4) depersonalization; and (5) dissociative amnesia.
Generalized anxiety disorder (GAD) is characterized by a pattern of frequent, persistent worry and anxiety that is dispro- portionate to the impact of the events or circumstances on which the worry focuses. Patients with GAD do not necessarily ac- knowledge the excessive nature of their worries, but they must be bothered by their degree of worry. This pattern must occur “more days than not” for at least 6 months. Patients must find it difficult to control their worry and must report three or more of six somatic or cognitive symptoms, which include feeling restlessness, fatigue, muscle tension, or insomnia. Worry can be commonplace in many other anxiety disorders, but the worries in GAD must exceed in the breadth or scope the worries charac- terized in other anxiety disorders.
Answers 16.46–16.49 16.46. The answer is D 16.47. The answer is C 16.48. The answer is A
16.49. The answer is B
Exposure therapy involves intentionally confronting feared, but otherwise not dangerous, objects, situations, thoughts, memo- ries, and physical sensations for the purpose of reducing fear reactions associated with the same or similar stimuli. System- atic desensitization was the first exposure therapy technique to undergo scientific investigation. Although an effective treatment for some anxiety disorders, it has generally fallen out of use among researchers and cognitive-behavioral therapists. The con- temporary use of exposure therapy may be usefully divided into three classes of procedures: in vivo exposure, imaginal exposure, and interoceptive exposure.
In vivo exposure involves helping patients to directly confront feared objects, activities, and situations. It is usually conducted in a graduated fashion according to a mutually agreed-on (between patient and therapist) hierarchy. For example, a hierarchy for a specific animal phobia, such as a snakes or spiders, may begin with looking at pictures and other representations of the feared animal followed by looking at the actual animal kept in a cage, first at a distance and then gradually moving closer.
Imaginal exposure typically involves having the patient close his or her eyes and imagine feared stimuli as vividly as possible. The primary use of this type of exposure is to help patients confront feared thoughts, images, and memories. For example, individuals with OCD may experience obsessional thoughts and images about causing harm to people they love.
Interoceptive exposure is the most recent form of exposure therapy to be introduced. This procedure is designed to induce feared physiological sensations under controlled circumstances. A number of specific exercises have been developed to induce specific panic-like sensations. For example, the step-up exercise, in which the patient repeatedly steps up and down on a single step as rapidly as possible, produces rapid heart rate and shortness of breath.
Systematic desensitization requires initial training in progres- sive muscle relaxation and the development of one or more care- fully constructed hierarchies of feared stimuli. Treatment then involves the pairing of mental images of the lowest items on the hierarchy with relaxation until the image can be held in mind without it producing significant distress.
Answers 16.50–16.54 16.50. The answer is D 16.51. The answer is A 16.52. The answer is E 16.53. The answer is C
16.54. The answer is B
Specific phobia is divided into four subtypes (animal type, natu- ral environment type, blood injury type, and situational type) in addition to a residual category for phobias that do not clearly fall into any of these four categories. The key feature in each type of phobia is that the fear is circumscribed to a specific object, both temporally and with respect to other objects. Phobias have tradi- tionally been classified according to the specific fear by means of Greek or Latin prefixes, as indicted by the examples below.
Acrophobia: fear of heights Agoraphobia: fear of open places Ailurophobia: fear of cats Claustrophobia: fear of closed spaces Cynophobia: fear of dogs Hydrophobia: fear of water Mysophobia: fear of dirt and germs Pyrophobia: fear of fire
Xenophobia: fear of strangers Zoophobia: fear of animals
16. Anxiety Disorders 167

17
Somatoform Disorders

Six somatoform disorders are currently listed in the Diagnostic and Statistical Manual of Mental Disorders (DSM): somatiza- tion disorder, conversion disorder, hypochondriasis, body dys- morphic disorder, pain disorder, and undifferentiated somato- form disorder not otherwise specified.
The term somatoform is derived from the Greek word soma, which means body. Somatoform disorders are a broad group of illnesses that have bodily signs and symptoms as a major com- ponent. These disorders encompass mind-body interactions in which the brain, in ways still not well understood, sends vari- ous signals that impinge on the patient’s awareness, indicating a serious problem in the body. Additionally, minor or as yet undetectable changes in neurochemistry, neurophysiology, and neuroimmunology may result from unknown mental or brain mechanisms that cause illness.
Before a somatoform disorder is diagnosed, the clinician must initiate a thorough medical evaluation to rule out the presence of actual medical pathology. A certain percentage of these patients will turn out to have real underlying medical pathology, but it
does not usually account for the symptoms described by the patient. The disorders may be chronic or episodic, they may be associated with other mental disorders, and the symptoms described are always worsened by psychological stress.
Treatment is often very difficult because the symptoms tend to have deeply rooted and unconscious psychological meanings for most patients, and these are patients who do not or cannot express their feelings verbally. Unconscious conflicts are ex- pressed somatically and seem to have a particular tenaciousness and resistance to psychological treatment.
Treatment involves both biological and psychological strate- gies, including cognitive-behavioral treatments, psychodynamic therapies, and psychopharmacologic approaches. If other psy- chiatric disorders, such as depression or anxiety disorders, are also present, they must be treated concomitantly. Different med- ications are effective with the range of disorders, and students should be knowledgeable about this.
Students should study the questions and answers below for a useful review of these disorders.
Students should be able to define the terms listed below.
amobarbital (Amytal) interview
anorexia nervosa antisocial personality
disorder astasia-abasia autonomic arousal
disorder biofeedback
body dysmorphic disorder
Briquet’s syndrome conversion disorder dysmorphophobia endorphins hemianesthesia hypochondriasis hysteria identification instinctual impulse la belle indiffe ́rence malingering
pain disorder pimozide (Orap) primary gain and
secondary gain pseudocyesis
pseudoseizures secondary symptoms somatization disorder somatoform disorder not
otherwise specified somatosensory input
stocking-and-glove anesthesia
symbolization and projection
undifferentiated somatoform disorder
undoing
HELPFUL HINTS
QUESTIONS
Directions
Each of the questions or incomplete statements below is followed by five suggested responses or completions. Select the one that is best in each case.
168
17.1. Which of the following features are helpful in deciding whether idiopathic physical symptoms may have a psy- chiatric basis?
A. Symptoms have comorbid major psychiatric disor- ders such as depression.
B. Symptoms closely follow traumatic events.
▲▲▲▲
▲▲▲ ▲▲▲▲▲ ▲
▲▲▲▲▲▲▲▲▲▲▲
▲▲▲▲▲▲▲
C. Symptoms lead to psychological “gratification.” D. Symptoms represent predictable personality traits. E. All of the above
. 17.2. Conversion disorder
A. usually has a chronic course
B. is associated with antisocial personality disorder
C. is commonly comorbid with a schizoid personality
disorder
D. responds well to a confrontation of the “false nature”
of the symptoms
E. is associated with symptoms that conform to known
anatomical pathways
. 17.3. Which of the following statements regarding conversion
disorder and gender differences is true?
A. Women are often involved in occupational accidents.
B. Symptoms are more common on the right side than
the left side of the body in women.
C. The ratio of women to men among adult patients is
as high as 10:1.
D. In children, there is a higher predominance in boys.
E. There is an association with borderline personality
disorder in men.
. 17.4. Conversion reactions
A. seem to change the psychic energy of acute conflict into a personally meaningful metaphor of bodily dys- function
B. conform to usual dermatomal distribution of under- lying peripheral nerves
C. are invariably sensorimotor as opposed to autonomic
D. are always transient
E. all of the above
. 17.5. Characteristic signs of conversion disorder include all of the following except
A. astasia-abasia
B. cogwheel rigidity
C. hemianesthesiaofthebodybeginningpreciselyatthe
midline
D. normal reflexes
E. stocking-and-glove anesthesia
. 17.6. Whichofthefollowingdiseasesispartofthedifferential diagnosis for conversion disorder?
A. Multiple sclerosis
B. Guillain-Barre ́ syndrome
C. Acquired immunodeficiency syndrome (AIDS) D. Dementia
E. All of the above
. 17.7. Conversion disorder differs from somatization disorder in that
A. conversion disorder includes symptoms in many or- gan systems
B. somatization disorder begins early in life
C. complaints are limited to pain in conversion disorder
D. complaints are not limited to neurological symptoms
in conversion disorder
E. conversion disorder involves a particular disease
rather than a symptom
. 17.8. A patient with somatization disorder
A. has had physical symptoms for only 3 months
B. usually experiences minimal impairment in social or
occupational functioning
C. may have a false belief of being pregnant with objec-
tive signs of pregnancy, such as decreased menstrual
flow or amenorrhea
D. presents the initial physical complaints after age
30 years
E. has complained of symptoms not explained by a
known medical condition
. 17.9. All of the following mental disorders are frequently seen in patients with somatization disorder (relative to the general population) except
A. bipolar I disorder
B. generalized anxiety disorder
C. major depressive disorder
D. obsessive-compulsive personality disorder E. schizophrenia
. 17.10. The most frequently occurring of the somatoform disor- ders is
A. somatization disorder
B. pain disorder
C. hypochondriasis
D. conversion disorder
E. body dysmorphic disorder
. 17.11. Medical disorders to be considered in a differential diag- nosis of somatization disorder include
A. multiple sclerosis
B. systemic lupus erythematosus C. acute intermittent porphyria D. hyperparathyroidism
E. all of the above
. 17.12. Which of the following is not a recommended treatment strategy for a patient with somatization disorder?
A. Increasing the patient’s awareness that psychological factors may be involved
B. Having several different clinicians involved in caring for the patient
C. Avoiding additional laboratory and diagnostic proce- dures
D. Seeing patients during regularly scheduled visits at regular intervals
E. Listening to somatic complaints as emotional expres- sions rather than medical complaints
17. Somatoform Disorders 169
170 17. Somatoform Disorders
. 17.13. Chronic pain disorder is most frequently associated with
A. substance-related disorders B. anxiety disorder
C. dementia
D. depressive disorder
E. schizophrenia
. 17.14. All of the following depressive symptoms are most
prominent in patients with pain disorder except
A. decreased libido B. insomnia
C. weight loss
D. anhedonia
E. anergia
. 17.15. The most accurate statement regarding pain disorder is
A. It is diagnosed equally among men and women.
B. Peakagesofonsetareinthesecondandthirddecades.
C. It is least common in persons with blue-collar occu-
pations.
D. First-degree relatives of patients have an increased
likelihood of having the same disorder.
E. Depressivedisordersarenomorecommoninpatients
with pain disorder than in the general public.
. 17.16. People with hypochondriasis
A. are usually women
B. are often thanatophobic
C. do not respond to reassurance
D. seek treatment more than explanations
E. in postmortem examinations, have a greater degree
of upper gastrointestinal (GI) inflammation and con- gestion than normal control subjects
. 17.17. Which of the following is a theory for the cause of hypochondriasis?
A. Symptoms are viewed as a request for admission to the sick role made by a person facing challenges in his or her life.
B. Personswithhypochondriasishavelowthresholdsfor and low tolerance of physical discomfort.
C. Aggressiveandhostilewishestowardothersaretrans- ferred (through repression and displacement) into physical complaints.
D. Hypochondriasis is a variant form of other mental disorders, such as depressive or anxiety disorders.
E. All of the above
. 17.18. True statements about hypochondriasis include all of the
following except
A. Depression accounts for a major part of the total pic- ture in hypochondriasis.
B. Hypochondriasis symptoms can be part of dysthymic disorders, generalized anxiety disorder, or adjustment disorder.
C. Hypochondriasis is a chronic and somewhat disabling disorder.
D. Recent estimates are that 4 to 6 percent of the general medical population meets the specific criteria for the disorder.
E. Significant numbers of patients with hypochondri- asis report traumatic sexual contacts, physical vio- lence, and major parental upheaval before the age of 17 years.
. 17.19. Body dysmorphic disorder is associated with
A. a family history of substance abuse B. major depressive disorder
C. obsessive-compulsive disorder
D. social phobia
E. all of the above
. 17.20. In body dysmorphic disorder,
A. plastic surgery is usually beneficial
B. a comorbid diagnosis is unusual
C. anorexia nervosa may also be diagnosed
D. about 50 percent of patients may attempt suicide
E. serotonin-specific drugs are effective in reducing the
symptoms
. 17.21. A 34-year-old woman presented with chronic and inter- mittent dizziness, paresthesias, pain in multiple areas of her body, and intermittent nausea and diarrhea. She re- ported that these symptoms had been present most of the time, although they had been undulating since she was approximately 24 years old. In addition, she complained of mild depression; was disinterested in many things in life, including sexual activity; and had been to many doc- tors to try to find out what was wrong with her. Physical examination, including a neurological examination, was normal. There were no abnormalities on laboratory test- ing. Her doctor diagnoses somatization disorder. Which of the following about this disorder is true?
A. It occurs more commonly in men.
B. It is more common in urban populations.
C. These patients are no more likely to develop another
medical illness than people without the disorder.
D. The symptoms typically begin in middle age.
E. Thesepatientsusuallygiveaverythoroughandcom-
plete report of their symptoms.
. 17.22. Mr. J is a 28-year-old single man who is employed in a factory. He was brought to an emergency department by his father, complaining that he had lost his vision while sitting in the back seat on the way home from a family gathering. He had been playing volleyball at the gath- ering but had sustained no significant injury except for the volleyball hitting him in the head a few times. As was usual for this man, he had been reluctant to play volleyball because of his lack athletic skills, and he was placed on a team at the last moment. He recalls having
some problems with seeing during the game, but his vi- sion did not become ablated until he was in the car on the way home. By the time he got to the emergency department, his vision was improving, although he still complained of blurriness and mild diplopia. The double vision could be attenuated by having him focus on items at different distances.
On examination, Mr. J was fully cooperative, some- what uncertain about why this would have occurred, and rather nonchalant. Pupillary, oculomotor, and gen- eral sensorimotor examinations were normal. After be- ing cleared medically, the patient was sent to a mental health center for further evaluation.
At the mental health center, the patient recounted the same story as he did in the emergency department, and he was still accompanied by his father. He began to re- count how his vision started to return to normal when his father pulled over on the side of the road and be- gan to talk to him about the events of the day. He spoke with his father about how he had felt embarrassed and somewhat conflicted about playing volleyball and how he had felt that he really should play because of external pressures. Further history from the patient and his father revealed that this young man had been shy as an adoles- cent, particularly around athletic participation. He had never had another episode of visual loss. He did recount feeling anxious and sometimes not feeling well in his body during athletic activities. (Courtesy of Michael A. Hollifield, MD.)
Which of the following is the most likely diag- nosis?
A. Conversion disorder B. Somatization disorder C. Pain disorder
D. Hypochondriasis
E. Malingering
17.23. A 54-year-old man presents to clinic stating he “can no longer feel his legs.” He has no other accompanying symptoms, and his physical examination results are nor- mal, including motor strength bilaterally. However, after receiving an injection of sodium Amytal, his symptoms markedly improve. What is the diagnosis?
A. Factitious disorder
B. Conversion disorder
C. Malingering
D. Histrionic personality disorder E. None of the above
Directions
Each set of lettered headings below is followed by a list of num- bered phrases. For each numbered phrase, select:
A. if the item is associated with A only
B. if the item is associated with B only
C. if the item is associated with both A and B D. if the item is associated with neither A nor B
Questions 17.24–17.28
A. Body dysmorphic disorder B. Hypochondriasis
. 17.24. Beliefs and symptoms may reach delusional intensity
. 17.25. Has high rates of coexisting depressive and anxiety dis-
orders
. 17.26. Patients actively seek out attention for their symptoms
. 17.27. Causes persistent distress or interference with personal
functioning
. 17.28. May be related to the defense mechanisms of repression
and displacement
Questions 17.29–17.33
A. Pain disorder
B. Somatization disorder
17.29. Affects women more often than men
17.30. Most often begins during a person’s teens 17.31. Responds to antidepressants
17.32. May involve serotonin in its pathophysiology 17.33. Is commonly associated with anorexia nervosa
ANSWERS
17.1. The answer is E (all)
Because somatoform disorders are placed at the crossroads be- tween physical and mental disorder, their differential diagnosis tends to be quite inclusive and elaborate. However, several fea- tures of these disorders can help the differential diagnosis. Table 17.l lists features that can help in deciding whether idiopathic physical symptoms may have a psychiatric cause.
17.2. The answer is B
There is an association between conversion disorder and anti- social personality disorder. The onset of the disorder is usually acute, and symptoms or deficits are usually of short duration. The symptoms usually do not conform to known anatomical pathways and physiological mechanisms but instead follow the individual’s conceptualization of his or her illness. Confronting
Table 17.1
Features that May Help in the Differential Diagnosis of Somatoform Disorders
The symptoms coexist with major psychiatric disorders such as depression or panic.
The symptoms closely follow traumatic events.
The symptoms lead to psychological “gratification” or “secondary
gain.”
The symptoms represent predictable personality traits (coping
mechanisms).
The symptoms become persistent, join a conglomerate of other
symptoms, and convey such attitudes as overuse of medical services and dissatisfaction with medical care.
Adapted from Javier I. Escobar, MD.
17. Somatoform Disorders 171
  
172 17. Somatoform Disorders
the patient about the so-called “false nature” of his or her symp- toms is contraindicated. In acute cases, reassurance and sugges- tion of recovery coupled with early rehabilitation are the treat- ments of choice. Schizoid disorder is not comorbid in patients with conversion disorder.
17.3. The answer is C
The ratio of women to men among adult conversion disorder patients is at least 2:1 and as much as 10:1; among children, an even higher predominance is seen in girls (not boys). Symp- toms are more common on the left side than the right side of the body in women (not vice versa). Women who present with conversion symptoms are more likely to subsequently develop somatization than women who have not have conversion symp- toms. An association exists between conversion disorder and an- tisocial personality disorder in men (not borderline personality disorder). Men with conversion have more often been involved in occupational or military accidents (not women).
17.4. The answer is A
Many conversion disorders simulate acute neurological pathol- ogy (e.g., strokes and disturbances of speech, hearing, or vi- sion). However, conversion disorders are not associated with the usual pathological neurodiagnostic signs or the underlying somatic pathology. Conversion symptoms (e.g., anesthesias and paresthesias produced by a conversion disorder) do not conform to usual dermatomal distribution of the underlying peripheral nerves; rather, the signs and symptoms of a conversion disor- der typically conform to the patient’s concept of the medical condition.
Conversion disorders seem to change or convert the psychic energy of the turmoil of acute conflict into a personally mean- ingful metaphor of bodily dysfunction. Turbulence of the mind is transformed into a somatic statement, condensing and focusing concepts, role models, and communicative meanings into one or several physical signs or symptoms of dysfunction. These so- matic representations often simulate an acute medical calamity; initiate urgent, sometimes expensive medical investigation; and produce disability. In primitive settings, however, certain con- version symptoms have been taken as tokens of religious faith and even as expressions of witchcraft.
Although most conversion reactions are transient (hours to days), some can persist. Chronic conversion disorders can actu- ally produce permanent conversion complications, such as dis- use contractures of a “paralyzed” limb that remains long after the psychic strife that prompted the conversion has been re- solved. In many cases, a chronic conversion disorder serves to help stabilize an otherwise dysfunctional family. In addition to sensorimotor symptoms, marked autonomic disturbances such as protracted (psychogenic) vomiting, hyperemesis gravidarum, urinary retention, and pseudocyesis are also seen, although less commonly. Conversion disorders challenge the diagnostic com- petence of internists, neurologists, otolaryngologists, ophthal- mologists, and psychiatrists.
Similar to the other somatoform disorders, conversion dis- orders are not volitional. Rather, ego defense mechanisms of repression and dissociation act outside of the patient’s aware- ness. Many patients with conversion disorders experience la belle indiffe ́rence, an emotional unconcern or even flatness in
a setting of catastrophic illness, but some patients experience considerable anguish over their new symptoms.
17.5. The answer is B
Cogwheel rigidity is an organic sign secondary to disorders of the basal ganglia and not a sign of conversion disorder. In conversion disorder, anesthesia and paresthesia, especially of the extremi- ties, are common. All sensory modalities are involved, and the distribution of the disturbance is inconsistent with that of either central or peripheral neurological disease. Thus, one sees the characteristic stocking-and-glove anesthesia of the hands or feet or hemianesthesia of the body beginning precisely at the midline. Motor symptoms include abnormal movements and gait distur- bance, which is often a wildly ataxic, staggering gait accompa- nied by gross, irregular, jerky truncal movements and thrashing and waving arms (also known as astasia-abasia). Normal re- flexes are seen. The patient shows no fasciculations or muscle atrophy, and electromyography findings are normal.
17.6. The answer is E (all)
Neurological disorders (e.g., dementia and other degenerative diseases), brain tumors, and basal ganglia disease must be con- sidered in the differential diagnosis for conversion disorder. For example, weakness may be confused with myasthenia gravis, polymyositis, acquired myopathies, or multiple sclerosis. Optic neuritis may be misdiagnosed as conversion disorder blindness. Other diseases that can cause confusing symptoms are Guillain- Barre ́ syndrome, Creutzfeldt-Jakob disease, periodic paralysis, and early neurological manifestations of acquired immunodefi- ciency syndrome (AIDS). Conversion disorder symptoms occur in schizophrenia, depressive disorders and anxiety disorders, but these other disorders are associated with their own distinct symp- toms that eventually make differential diagnosis possible.
17.7. The answer is B
Conversion disorder is an illness of symptoms or deficits that af- fect voluntary motor or sensory functions, which suggest another medical condition, but that is judged to be caused by psycholog- ical factors because the illness is preceded by conflicts or other stressors. Somatization disorder is an illness of multiple somatic complaints in multiple organ systems that occurs over a period of several years and results in significant impairment, treatment seeking, or both. Sensorimotor symptoms also occur in somati- zation disorder. But somatization disorder is a chronic illness that begins early in life and includes symptoms in many other organ systems (not conversion disorder). In hypochondriasis (general and nondelusional preoccupation with fears of having, or the idea that one has a serious disease), as opposed to conversion disor- der, patients have no actual loss or distortion of function; the somatic complaints are chronic and are not limited to neurolog- ical symptoms, and the characteristic hypochondriacal attitudes and beliefs are present. If the patient’s symptoms are limited to pain, pain disorder (not conversion disorder) can be diagnosed. Conversion disorder is acute and generally transient and usually involves a symptom rather than a particular disease (not vice versa). This is in contrast to hypochondriasis, which involves a particular disease.
17.8. The answer is E
During the course of somatization disorder, the patient has com- plained of pain and gastrointestinal, sexual, and pseudoneuro- logical symptoms that are not explained by a known medical condition. In addition, the patient presents the initial physical complaints before, not after, age 30 years. The patient has had physical symptoms for at least several years, not just 3 months. The patient has had interpersonal problems and tremendous psy- chological distress and usually experiences significant, not mini- mal, impairment in social or occupational functioning. A patient who has a false belief of being pregnant and objective signs of pregnancy, such as decreased menstrual flow or amenorrhea, does not have somatization disorder. Instead, the patient has pseudocyesis, a somatoform disorder not otherwise specified.
17.9. The answer is A
Somatization disorder commonly coexists with other mental dis- orders. About two-thirds of all patients with somatization disor- der have identifiable psychiatric symptoms, and up to half have other mental disorders. Commonly associated personality traits or personality disorders are those characterized by avoidant, paranoid, self-defeating, and obsessive-compulsive features. Pa- tients with major depressive disorder, generalized anxiety dis- order, and schizophrenia may all have an initial complaint that focuses on somatic symptoms. In all of these disorders, however, the symptoms of depression, anxiety, and psychosis eventually predominate over the somatic complaints. Two disorders that are not seen more commonly in patients with somatization dis- order than in the general population are bipolar I disorder and substance abuse.
17.10. The answer is D
Conversion disorder may be the most frequently occurring of the somatoform disorders. The text revision of the fourth edi- tion of the Diagnostic and Statistical Manual of Mental Disor- ders (DSM-IV-TR) gives a range from a low of 11 to a high of 500 cases per 100,000. Affected persons can range in age from early childhood into old age. The annual incidence of conver- sion disorders seen by psychiatrists in a New York county has been estimated to be 22 cases per 100,000. In a general hospital setting, 5 to 16 percent of all psychiatric consultation patients manifest some conversion symptoms. In a study of a rural Veter- ans Administration general hospital, 25 to 30 percent of all male patients had a conversion symptom at some time during their admission. By contrast, in a psychiatric emergency department or psychiatric clinic, the incidence of conversion disorder is far lower (1 percent of all psychiatric admissions) because different selection factors supervene. Lifetime figures for ever having any conversion symptoms, even if only on a transient basis, are far higher, with some studies reporting a 33 percent prevalence rate. Conversion disorder occurs mainly in women, with a ratio of 2:1 up to 5:1 in some studies.
17.11. The answer is E (all)
The clinician must always rule out organic causes for the pa- tient’s symptoms. Medical disorders that present with nonspe- cific, transient abnormalities pose the greatest diagnostic diffi- culty in the differential diagnosis of somatization disorder. The
Table 17.2
17. Somatoform Disorders 173 Conditions Commonly Confused with Somatoform
Disorder
Multiple sclerosis
Central nervous system syphilis Brain tumor Hyperparathyroidism
Acute intermittent porphyria Lupus erythematosus Hyperthyroidism
Myasthenia gravis
disorders to be considered include multiple sclerosis, systemic lupus erythematosus, acute intermittent porphyria, and hyper- parathyroidism. In addition, the onset of many somatic symp- toms late in life must be presumed to be caused by a medical illness until testing rules it out. Table 17.2 lists a few of the disor- ders commonly confused with somatoform disorders, especially early in their courses.
17.12. The answer is B
Somatization disorder is best treated when the patient has a sin- gle identified physician as his or her primary caretaker. When more than one clinician is involved, patients have increased op- portunities to express somatic complaints. Primary physicians should see patients during regularly scheduled visits, usually at monthly intervals. The visits should be relatively brief, although a partial physical examination should be conducted to respond to each new somatic complaint. Additional laboratory and diagnos- tic procedures should generally be avoided. After somatization disorder has been diagnosed, the treating physician should listen to the somatic complaints as emotional expressions rather than as medical complaints. Nevertheless, patients with somatization disorder can also have bona fide physical illnesses; therefore, physicians must always use their judgment about what symp- toms to work up and to what extent. A reasonable long-range strategy for a primary care physician who is treating a patient with somatization disorder is to increase the patient’s aware- ness of the possibility that psychological factors are involved in the symptoms until the patient is willing to see a mental health clinician.
17.13. The answer is D
A pain disorder is characterized by the presence of and focus on pain in one or more body sites and is sufficiently severe to come to clinical attention. Pain disorder is associated with other psychiatric disorders, especially affective and anxiety disorders. Chronic pain appears to be most frequently associated with de- pressive disorders, and acute pain appears to be more commonly associated with anxiety disorders. The associated psychiatric dis- orders may precede the pain disorder, may co-occur with it, or may result from it. Depressive disorders, alcohol dependence, and chronic pain disorder may be more common in relatives of individuals with chronic pain disorder. Individuals whose pain is associated with severe depression and those whose pain is re- lated to a terminal illness, such as cancer, are at increased risk of suicide. Some investigators believe that chronic pain is almost
  
174 17. Somatoform Disorders
always a variant of a depressive disorder, a masked or somatized form of depression. The clinical picture of a patient with pain disorder can be complicated by substance-related disorders be- cause these patients attempt to reduce the pain through the use of alcohol and other substances.
17.14. The answer is C
The most prominent depressive symptoms in patients with pain disorder are anergia (lack of energy), anhedonia (absence of pleasure from acts that would ordinarily be pleasurable), decreased libido, insomnia, and irritability; diurnal variation, weight loss, and psychomotor retardation appear to be less com- mon. Major depressive disorder is present in about 25 to 50 per- cent of patients with pain disorder, and dysthymic disorder or depressive disorder symptoms are reported in 60 to 100 percent of patients.
17.15. The answer is D
The most accurate statement about pain disorder is that first- degree relatives of patients with pain disorder have an increased likelihood of having the same disorder, thus indicating the pos- sibility of genetic inheritance or behavioral mechanisms in the transmission of the disorder. Pain disorder is actually diagnosed twice as frequently in women as in men. The peak ages of onset are in the fourth and fifth decades of life, when the tolerance for pain declines. Pain disorder is most common in persons with blue- collar occupations, perhaps because of increased job-related in- juries. Depressive disorders, anxiety disorders, and substance abuse are also more common in families of patients with pain disorder than in the general population.
17.16. The answer is B
People with hypochondriasis are highly thanatophobic (fear of death), which is a central clinical feature of the disorder. They are persistent seekers of explanations rather than of treatment, are largely unsatisfied with their medical care, and often believe that physicians have not recognized their needs. The onset of the disorder is most commonly in the third and fourth decades of life, and it is equally common in men and women. Reassurance that is delivered confidently by a competent doctor using multiple modalities, including skillful examination, effective communi- cation, and helpful education, is the cornerstone of treatment of the patients with hypochondriasis. Without successful reassur- ance, more specific treatments are not likely to be accepted or adhered to by the patient. There is no known somatic pathology specific to hypochondriasis. Investigators who conducted post- mortem examinations of patients with hypochondriasis found no evidence of inflammation and congestion in the upper gastroin- testinal tract.
17.17. The answer is E (all)
A reasonable body of data indicates that persons with hypochon- driasis augment and amplify their somatic sensations; they have low thresholds for and low tolerance of physical discomfort. They may focus on bodily sensations, misinterpret them, and become alarmed by them because of a faulty cognitive scheme.
A second theory is that hypochondriasis is understandable in terms of a social learning model. The symptoms of hypochon- driasis are viewed as a request for admission to the sick role made by a person facing seemingly insurmountable and un- solvable problems. The sick role offers an escape that allows a patient to avoid noxious obligations, to postpone unwelcome challenges, and to be excused from usual duties and obligations. A third theory suggests that hypochondriasis is a variant form of other mental disorders, among which depressive disorders and anxiety disorders are most frequently included. An estimated 80 percent of patients with hypochondriasis may have coexisting depressive or anxiety disorders. Patients who meet the diagnos- tic criteria for hypochondriasis may be somatizing subtypes of these other disorders. The psychodynamic school of thought has produced a fourth theory of hypochondriasis. According to this theory, aggressive and hostile wishes toward others are trans- ferred (through repression and displacement) into physical com- plaints. The anger of patients with hypochondriasis originates in past disappointments, rejections, and losses. Hypochondriasis is also viewed as a defense against guilt, a sense of innate bad- ness, an expression of low self-esteem, and a sign of excessive self-concern.
17.18. The answer is A
Hypochondriacal symptoms can be a part of another disorder such as major depressive disorder, dysthymic disorders, general- ized anxiety disorder, or adjustment disorder. However, primary hypochondriasis or hypochondriacal disorder is a chronic and somewhat disabling disorder with hypochondriacal symptoms, not merely a part of another psychiatric condition.
Hypochondriasis is rather common in primary care settings. In various locales, the prevalence has varied from 3 to 14 percent. Recent work indicates that in a 6-month period of observation, 4 to 6 percent of the general medical population meets the spe- cific criteria for this disorder. The prevalence in either gender is comparable to that within the general medical population. There are no specific tendencies for overrepresentation based on social position, education, marital status, or other sociodemographic descriptors. There is a wide range of ages at onset. Although the disorder can begin at any age, onset is thought to be most common between 20 and 30 years of age.
Comorbidity with other psychiatric disorders is common with hypochondriasis and must be treated accordingly. Depression only accounts for a minor part of the total picture in hypochon- driasis, however, so it is a mistake to think that all hypochondri- asis is the result of some other Axis I disorder.
The developmental background of hypochondriacal patients is of interest in that significantly more of these patients than matched control subjects report traumatic sexual contacts, phys- ical violence, and major parental upheaval before the age of 17 years.
17.19. The answer is E (all)
Body dysmorphic disorder (BDD) is common as a comorbid condition in patients with major depressive disorder, obsessive- compulsive disorder (OCD), and social phobia. Indeed, in one study of 30 patients, all met the DSM-III-R criteria for at least one
other psychiatric diagnosis at some point in their lives, usually concurrently.
The cause of BDD is unknown. The high comorbidity with depressive disorders, a higher-than-expected family history of mood disorders and OCD, and the reported responsiveness to selective serotonin reuptake inhibitors indicates that, in at least some patients, the pathophysiology of the disorder may involve serotonin and may be related to other disorders. There may be significant cultural or social effects on patients with BDD because of the emphasis on stereotyped concepts of beauty that may be emphasized in certain families and within the culture at large.
Family histories of substance abuse and mood disorder are common in documented cases. Also predisposing to the disorder may be certain types of personality characteristics, especially a mixture of obsessional and avoidant traits, but no single per- sonality pattern predominates. Reportedly, the patients are shy, self-absorbed, and overly sensitive to their imagined defect as a focus of notice or criticism.
17.20. The answer is E
Serotonin-specific drugs, such as clomipramine (Anafranil) and fluoxetine (Prozac), are effective in reducing the symptoms in at least 50 percent of patients with body dysmorphic disorder (BDD). In any patient with a coexisting mental disorder or an anxiety disorder, the coexisting disorder should be treated with the appropriate pharmacotherapy and psychotherapy. How long treatment should be continued when the symptoms of BDD have remitted is unknown. Plastic surgery is not usually beneficial in the treatment of patients with BDD. In fact, surgical, derma- tological, dental, and other medical procedures to address the alleged defects rarely satisfy the patient.
A comorbid diagnosis is not unusual. BDD commonly co- exists with other mental disorders. One study found that more than 90 percent of patients with BDD had experienced a major depressive episode in their lifetimes, about 70 percent had an anxiety disorder, and about 30 percent had a psychotic disorder. However, anorexia nervosa should not be diagnosed along with BDD because distortions of body image occur in people with anorexia nervosa, gender identity disorders, and some specific types of brain damage (e.g., neglect syndromes).
The effects of BDD on a person’s life can be significant. Almost all affected patients avoid social and occupational expo- sure. As many as one-third of the patients may be housebound by their concern about being ridiculed for their alleged deformities, and as many as 20 percent, not 50 percent, of patients attempt suicide.
17.21. The answer is C
Patients with somatization disorder consider themselves to be medically ill. Despite this, there is good evidence that they are no more likely to develop another medical illness in the next 20 years than people without somatization disorder. The onset is before 25 years of age in 90 percent of people with the disorder, but initial symptoms generally develop during adolescence. Partly because of the undulating nature of the disorder, people with somatization disorder are usually poor historians, and they seem
to exaggerate various symptoms, each at different times. The female to male ratio ranges from five to one to 20 to one. It is relatively more common in rural areas and in people who are nonwhite, unmarried, and have less education.
17.22. The answer is A
The most likely diagnosis for Mr. J in this case is conversion disorder caused by the alternation in sensory functioning, the lack of a clear general medical condition, and the relation of the symptoms to a psychological issue. The patient complains of sensory disturbances (blurred vision, diplopia, loss of vision) that suggest a physical disorder; but thorough examinations have failed to detect a general medical condition that could account for the symptoms. The context in which these symptoms occurs suggests the role of psychological factor in their development: the patient lost his vision after play volleyball to which he had anxiety, and his vision started to improve after he began talking to his father about his anxiety. The diagnosis of a factitious disorder or malingering is ruled out because there is no evidence that the patient is conscious of intentionally producing the symptoms (e.g., taking a drug that would induce such symptoms, claiming to have the symptoms when they are not present). Somatization disorder is excluded because the patient’s complaints are not part of a long-standing polysymptomatic disturbance involving many organ systems. Pain disorder is ruled out because the patient does not complain of severe, persistent pain.
17.23. The answer is B
Subjective neurological symptoms are components of conver- sion disorder. These pseudoneurologic symptoms appear after a stressor and are not created intentionally. The definitive diag- nosis follows a thorough medical workup that does not explain the symptoms. Sodium Amytal may give relief to patients, but it is not unusual for patients to be indifferent (la belle indiffe ́rence). Factitious disorder consists of psychological or physical symp- toms that are intentionally created as the patient takes part in the “sick role.” Also intentionally produced, malingering con- sists of physical or psychological symptoms created to make secondary gains (e.g., avoiding work, obtaining drugs, avoiding criminal persecution). Histrionic personality disorder consists of a long-standing pattern of attention-seeking behavior and ex- cessive displays of emotion. It does not present acutely.
Answers 17.24–17.28 17.24. The answer is D 17.25. The answer is C 17.26. The answer is B 17.27. The answer is C
17.28. The answer is C
Hypochondriasis and body dysmorphic disorder (BDD) have several similarities in their proposed root causes and diag- nostic criteria, which is perhaps why the tenth edition of the
17. Somatoform Disorders 175
176 17. Somatoform Disorders
International Classification of Diseases (ICD-10) includes BDD as a subcategory of hypochondriacal disorder. In psychodynamic models of etiology, both hypochondriasis and BDD may be re- lated to the defense mechanisms of repression and displacement. In hypochondriasis, for example, aggressive and hostile wishes toward others are believed to be transferred through repression and displacement into physical complaints; BDD, meanwhile, is seen as reflecting the displacement of a sexual or emotional con- flict onto a nonrelated body part. Such association occurs through the defense mechanisms of repression, dissociation, distortion, symbolization, and projection. According to the DSM-IV-TR, the preoccupation with symptoms in both disorders cause clini- cally significant distress or impairment in social, occupational, or other important areas of functioning. In all of the somatoform disorders, patients firmly believe that their physical complaints are real, but these convictions do not reach delusional intensity; if they do, the appropriate diagnosis is delusional disorder. More- over, all of the somatoform disorders are related to high rates of comorbid mental disorders, such as depressive or anxiety dis- orders. Hypochondriasis, for example, is often accompanied by symptoms of depression and anxiety and commonly coexists with a depressive or anxiety disorder. Similarly, BDD commonly co- exists with other mental disorders: one study found that more than 90 percent of patients with BDD had experienced a major depressive episode in their lifetimes, and about 70 percent had experienced an anxiety disorder. Although these two categories of somatoform disorders have some similarities, the DSM-IV- TR maintains them as separate entities because of fundamental differences in their presentation. For example, whereas patients with BDD wish to appear normal but believe that others notice that they are not, those with hypochondriasis seek out attention for their presumed diseases.
Answers 17.29–17.33 17.29. The answer is C 17.30. The answer is B 17.31. The answer is C 17.32. The answer is C
17.33. The answer is D
Both somatization disorder and pain disorder affect women more than men. Somatization disorder has a female-to-male ratio of five to one. The lifetime prevalence of somatization disorder among women in the general population may be 1 to 2 percent. Pain disorder is diagnosed twice as commonly in women as in men. Somatization disorder is defined as beginning before age 30 years, and it most often begins during a person’s teens. As for pain disorder, the peak of onset is in the fourth and fifth decades of life, perhaps because the tolerance for pain decreases with age.
Antidepressants, such as fluoxetine (Prozac), sertraline (Zoloft), and clomipramine (Anafranil), are effective in the treatment of pain disorder and somatization disorder. Serotonin may be involved in the pathophysiology of both disorders. It is probably the main neurotransmitter in the descending inhibitory pathways. Endorphins also play a role in the central nervous system modulation of pain. Anorexia nervosa is not commonly associated with either pain disorder or somatization disorder. Anorexia nervosa is an eating disorder that presents a dramatic picture of self-starvation, distorted body image, peculiar attitudes toward food, weight loss (leading to the maintenance of the patient’s body weight at least 15 percent below that expected), and an intense fear of weight gain.
 
Patients with factitious disorder simulate, induce, or aggravate illness in order to receive medical attention whether or not they are actually ill. With factitious disorder patients, there is always the possibility of the patient’s inflicting painful, deforming, or life-threatening injury on themselves, their children, or other dependents. Unlike malingering, the motive of patients with factitious disorder is not for financial gain or avoidance of duties but to acquire medical care and to partake in the medical system.
According to the American Heritage Dictionary, the word factitious means “artificial; false,” derived from the Latin word facticius, which means “made by art.” The art and artifice of factitious patients often creates drama within the hospital and thus causes frustration and dismay for the clinicians and staff involved in their care. Clinicians may dismiss, avoid, or refuse to treat patients with factitious disorder. Strong countertransference of clinicians can be major obstacles toward the proper care of these patients, who arguably are among the most psychiatrically disturbed. Although the presenting complaints by these patients are false, it is still important to take the medical and mental needs of these patients seriously because factitious disorder can lead to significant morbidity and even mortality.
The best known factitious disorder is perhaps factitious dis- order with predominantly physical signs and symptoms, pop-
18
Factitious Disorders
ularly known as Munchausen syndrome. This presentation in- volves persons who travel from hospital to hospital, gaining ad- mission, receiving multiple diagnoses and treatments until they are found out by staff, and then they quickly move on to the next hospital to repeat the same rituals. Common complaints or presenting symptoms include hematomas, abdominal pain, fever, and seizures. Patients have been known to do such bizarre things as inject themselves with feces to induce infections or to willingly undergo repeated unnecessary surgeries.
Despite potentially high stakes, relatively little empirical knowledge is available about the etiology, epidemiology, course and prognosis, and effective treatment of factitious disorders. Most knowledge comes from case reports, information that is frequently suspect, given the false, unreliable nature of the in- formation these patients give. The text revision of the fourth edition of the Diagnostic and Statistical Manual of Mental Disorders (DSM-IV-TR) categories for the disorder include predominantly physical signs and symptoms, predominantly psychological signs and symptoms, both physical and psycho- logical signs and symptoms, and factitious disorder not otherwise specified.
Students should study the questions and answers below for a useful review of these disorders.
Students should be able to define each of these terms.
approximate answers as-if personality borderline personality
disorder
Briquet’s syndrome depressive-masochistic
personality dissociative disorder not
otherwise specified
factitious disorder ◦ by proxy
◦ not otherwise specified
◦ with predominantly physical signs and symptoms
◦
with predominantly psychological signs and symptoms
Munchausen syndrome pseudologia fantastica pseudomalingering regression
sick role
somatoform disorders symbolization unmasking ceremony
HELPFUL HINTS
Ganser’s syndrome gridiron abdomen impostorship malingering
QUESTIONS
Directions
Each of the questions or incomplete statements below is followed by five suggested responses or completions. Select the one that is best in each case.
18.1. Which of the following disorders when concurrent with factitious disorder with predominantly physical signs and symptoms bodes a poorer prognosis?
A. Alcohol abuse disorder B. Bipolar disorder
C. Anxiety disorder
177
▲▲▲▲▲▲▲▲
▲
▲▲▲▲▲▲
▲▲▲▲
178 18. Factitious Disorders
D. Antisocial personality disorder
E. None of the above
. 18.2. The perpetrators in factitious disorder by proxy
A. often suffer from psychotic or dissociative disorders
B. rarelyhavepersonalhistoriesoffactitiousorsomato-
form disorders
C. most often suffered direct abuse in childhood them-
selves
D. are commonly unresponsive to their infants when
their behavior is unwitnessed
E. all of the above
. 18.3. Whichofthefollowingisthegoldstandardfordiagnosis of factitious disorder by proxy?
A. Confession by the child
B. Finding inconsistencies in the medical records
C. Direct observation of the caretaker doing harm
D. Improvement when the child is removed from the
caretaker
E. Discovery of illness-inducing agents in the care-
giver’s possession
. 18.4. True statements about factitious disorder by proxy in-
clude all of the following except
A. The disorder currently accounts for fewer than 1,000 of the almost 3 million cases of child abuse reported each year in the United States.
B. The prevalence of the disorder in life-threatening episodes treated with cardiopulmonary resuscitation has been estimated to be as high as 9 percent.
C. The prevalence of the disorder has been estimated to be approximately 5 percent in children presenting with allergies.
D. The average length of time to establish a diagnosis after the initial presentation is about 2 months.
E. Oftenasiblinghasdiedofundiagnosedcausesbefore the disorder is recognized.
. 18.5. Which of the following occurs in factitious disorder by proxy?
A. The mother has had some medical education.
B. Thepatientfailstorespondtoappropriatetreatments. C. Maternal lying is observed.
D. Unexplained illnesses have occurred in the mother. E. All of the above
. 18.6. The following statements regarding the differentiation between factious disorders and somatoform disorders are true except
A. Somatization patients seem more willing to undergo numerous mutilating procedures.
B. Patients with conversion disorder symptoms have a symbolic reference to specific emotional conflicts.
C. The age of onset for hypochondriasis is typically later than that of factitious disorders.
D. The multiple hospitalizations of a patient with facti- tious disorder have an extreme course.
E. Patients with factitious disorder produce factitious symptoms voluntarily.
18.7. Factitious disorder
A. may result in death because of needless medical in- terventions
B. is associated with a history of childhood abuse
C. occurs more frequently in women than in men
D. is not associated with economic gain
E. all of the above
18.8. Factitious disorders
A. usually have a good prognosis
B. may occur by proxy
C. usually begin in childhood
D. are best treated with psychoactive drugs E. are synonymous with Ganser’s syndrome
18.9. The differential diagnosis of a factitious disorder includes
A. antisocial personality disorder B. hypochondriasis
C. malingering
D. somatization disorder
E. all of the above
18.10. Because of their dramatic flair, patients with factious disorder are often classified as having
A. histrionic personality disorder B. paranoid personality disorder C. schizotypal personality disorder D. narcissistic personality disorder E. none of the above
18.11. Patients with factitious disorders, either physical or psychological, most often demonstrate
A. poor sexual adjustment
B. generally adequate frustration tolerance C. a below-average IQ
D. a formal thought disorder
E. all of the above
18.12. The term Munchausen’s syndrome was coined by
A. Baron Karl Friedrich Hieronymus von Munchausen B. Richard Asher
C. Jean-Marie Charcot
D. Roy Meadow
E. Alan Gelenberg
18.13. Factitious disorder patients with Munchausen syndrome are typically
A. estranged from their families B. middle-aged men
C. unemployed
D. unmarried
E. all of the above
. 18.14. Psychosocial factors that contribute to factitious disor- ders include
A. childhood abuse
B. masochistic personality
C. parental rejection in childhood D. poor identity formation
E. all of the above
. 18.15. Which of the following symptoms would a patient with Munchausen syndrome most likely present with?
A. Amnesia
B. Depression C. Hemoptysis D. Pain disorder E. Psychosis
. 18.16. True statements about patients with factitious disorder with predominantly psychological signs and symptoms include
A. Virtually all patients with this type of factitious dis- order have a personality disorder.
B. The rate of suicide is generally reported to be low in this population.
C. The prognosis is slightly better than for most other Axis I disorders.
D. Factitious psychosis, in particular, almost never rep- resents the prodrome to an authentic psychosis.
E. None of the above
. 18.17. Factitious disorders are best treated by which of the fol- lowing?
A. Confrontation about the patient’s deceit
B. Immediate discharge from the hospital
C. Focusing on management rather than cure
D. Performing only minimally invasive procedures to
satisfy the patient
E. Using low-dose neuroleptics to decrease the patient’s
physical distress
. 18.18. You begin treatment of a new patient with a previously known history of factitious disorder. Within the first few sessions, you also become aware that this patient meets criteria for a diagnosis of antisocial personality disorder as well.
Which of the following statements regarding this pa- tient is true?
A. Persons with antisocial personality disorder do not usually volunteer for invasive procedures.
B. Persons with both of these disorders have repeated hospitalizations.
C. It is rare for persons with factitious disorder to also present with antisocial traits.
D. Factitious disorder symptoms almost always precede antisocial traits.
E. All of the above
18.19. You are asked by the court to evaluate a 21-year-old man arrested in a robbery because his lawyer raised the issue of his competence to stand trial. He has no known psy- chiatric history, and no psychotic symptoms have been previously reported. During the interview, the man ap- pears calm and in control, sits slouched in the chair, and has good eye contact. His affect shows a good range. His thought processes are logical, sequential, and spon- taneous even when he describes many difficulties with his thinking. He seems guarded in his answers, particularly to questions about his psychological symptoms.
He claims to have precognition on occasion, know- ing, for instance, what is going to be served for lunch in the jail, and that he does not like narcotics because Jean Dixon does not like narcotics either, and she is in control of his thoughts. He states that he has seen a vision of General Lee in his cell as well as “little green men from Mars” and that his current incarceration is a mission in which he is attempting to be an undercover agent for the police, although none of the local police realize this. De- spite the overtly psychotic nature of these thoughts as described, the patient does not seem to be really engaged in the ideas; he seems to be simply reciting a list of what appears to be crazy rather than recounting actual experi- ences and beliefs. When the interviewer expresses some skepticism about his described beliefs, he responds by saying that he has “many other crazy ideas” that he can share.
Which of the following is the most likely diag- nosis?
A. Capgras’ syndrome
B. Delusional disorder
C. Factitiousdisorderwithpredominantlypsychological
symptoms
D. Malingering
E. Schizophrenia, paranoid type
18.20. Ganser’s syndrome
A. is associated with a severe personality disorder B. has a chronic remitting and relapsing course C. is motivated by involuntary phenomena
D. is a factitious disorder
E. is more common in women than in men
18.21. Masquerade syndrome is when
A. achildisconfinedtoasickrolewhileavoidingphysi- cians and agencies
B. aparentexaggeratesthechild’ssymptomsinaneffort to increase a pediatrician’s attention to the child
C. illness fabrication results in the child’s increasing de- pendency on the parent
D. a hypochondriac parent is so preoccupied with the child’s health that he or she risks unnecessary proce- dures and iatrogenic illnesses for the child
E. none of the above
18. Factitious Disorders 179
180 18. Factitious Disorders ANSWERS
18.1. The answer is D
In factitious disorder with predominantly physical signs and symptoms, a concurrent mood (i.e., bipolar disorder), anxiety, or substance abuse disorder bodes a better prognosis; comor- bid personality disorder, especially antisocial personality disor- der, bodes a poorer prognosis. According to some experts, many patients with factitious disorder experience remission around 40 years of age, corresponding to the age of remission for many patients with borderline personality disorder.
Factitious disorder with predominately physical signs and symptoms is the best known type of Munchausen’s syndrome. The essential features of patients with the disorder is their ability to present physical symptoms so well that they can gain admis- sion to and stay in a hospital. The wide spectrum of this disorder should be kept in mind when considering the course and prog- nosis. At one end of the spectrum, factitious illness behavior can be considered within the range of normal, as when a child ex- aggerates distress from a knee scrape to gain attention or when a mother magnifies her child’s symptoms to seek reassurance from a physician. Further along the spectrum, factitious illness behavior can be a maladaptive way of coping with stress and does not necessarily imply an ongoing factitious disorder.
18.2. The answer is D
By definition, factitious disorder by proxy requires that any ex- ternal gains for the victim’s fabricated or induced illnesses, such as disability payments or respite from child-rearing responsibil- ities during hospitalization, are incidental to the pursuit of the vicarious sick role. In contrast to the devoted, even symbiotic, parenting style they reveal in public, these mothers are com- monly unresponsive to their infants when their behavior is un- witnessed. Despite the perversity of their behavior, they rarely have psychotic or dissociative disorders, although they often have personal histories of factitious or somatoform disorders. Although they may have been neglected or undervalued, most perpetrators did not suffer direct abuse in childhood. Table 18.1 lists the clinical indicators that may suggest factitious disorder by proxy.
18.3. The answer is C
Factitious disorder by proxy should not be considered a diagnosis of exclusion. Confirmatory evidence should be actively pursued, so as to lessen risk to the child. The safety of the child should be ensured at the same time. The gold standard for confirming facti- tious disorder by proxy is direct observation of a parent causing harm to a child. Covert video has also shown cases in which mothers, who appear concerned in the presence of staff, behave indifferently toward their children when they are not aware of being watched. Covert video should only be undertaken after consultation with legal counsel. A court order may need to be obtained, and a bioethics consultation may be helpful to weigh the potential benefits to the child versus compromises of privacy for the parent.
Other means of confirming factitious disorder by proxy in- clude searching the mother’s belongings for illness-inducing agents, reviewing collateral information and medical records
Table 18.1
Clinical Indicators That May Suggest Factitious Disorder by Proxy
The symptoms and pattern of illness are extremely unusual or inexplicable physiologically.
Repeated hospitalizations and workups by numerous caregivers fail to reveal a conclusive diagnosis or cause.
Physiological parameters are consistent with induced illness (e.g., apnea monitor tracings disclose a massive muscle artifact before respiratory arrest), suggesting that the child has been struggling against an obstruction to the airways.
The patient fails to respond to appropriate treatments.
The vitality of the patient is inconsistent with the laboratory
findings.
The signs and symptoms abate when the mother has not had
access to the child.
The mother is the only witness to the onset of signs and symptoms. Unexplained illnesses have occurred in the mother or her other
children.
The mother has had medical or nursing education or exposure to
models of the illnesses affecting the child (e.g., a parent with
sleep apnea).
The mother welcomes even invasive and painful tests.
The mother grows anxious if the child improves.
Maternal lying is proved.
Medical observations yield information that is inconsistent with
parental reports.
Adapted from Feldman MD, Eisendrath SJ. The Spectrum of Factitious
Disorders. Washington, DC: American Psychiatric Press; 1996.
for inconsistencies, gathering information on siblings, recording temporal associations between parental visits and the child’s signs and symptoms, observing the child’s well-being when re- moved from the parent’s care for extended periods, and analyz- ing specimens taken in the presence of the parent compared with those taken in the parent’s absence.
18.4. The answer is D
Factitious disorder by proxy currently accounts for fewer than 1,000 of the almost 3 million cases of child abuse reported each year in the United States, but this number may increase as mass media and professional attention increase recognition of these cases. Authors have attempted to elucidate the prevalence of factitious disorder by proxy within particular populations, such as children presenting with apnea (0.27 percent), allergy (5 per- cent), asthma (1 percent), apparent life-threatening episodes (1.5 percent), and life-threatening episodes treated with cardiopul- monary resuscitation (more than 9 percent among children in whom final diagnoses were established). The average length of time to establish a diagnosis of factitious disorder by proxy af- ter the initial presentation is 15 months, and often a sibling has died of undiagnosed causes before the disorder is recognized. Table 18.2 summarizes the most common presentations of the disorder.
18.5. The answer is E (all)
In factitious disorder by proxy, classified in DSM-IV-TR as a factitious disorder not otherwise specified, a person intentionally produces physical signs or symptoms in another person who is under the first person’s care. The most common cases involve
  
Table 18.2
Ranking of the Most Common Bibliographic References to Signs and Symptoms of Factitious Disorder by Proxy
Poisoning (includes Munchausen syndrome by proxy and intentional poisoning)
Seizures or vomiting Apnea
Diarrhea Unconsciousness Fevers
Lethargy
Dehydration or hematemesis Ataxia or hematuria
Adapted from Schreier HA, Libow JA. Hunting for Love: Munchausen by Proxy Syndrome. New York: Guilford Press; 1993; and Rosenberg DA. Web of deceit: a literature review of Munchausen syndrome by proxy. Child Abuse Negl. 1987;11:533.
mothers who deceive medical personnel into believing that their child is ill. In this disorder, it has been noted that the symptoms and pattern of illness are extremely unusual. The mothers have often had some medical or nursing education, are observed to lie, and are the only witnesses to the onset of signs and symp- toms. Unexplained illnesses have occurred in the mother or her other children, and she often welcomes even invasive and painful tests. The tenth edition of the International Classification of Dis- eases (ICD-10) classifies this condition under child abuse, not factitious disorders.
18.6. The answer is A
A factitious disorder is differentiated from somatization disorder (Briquet’s syndrome) by the voluntary production of factitious symptoms, the extreme course of multiple hospitalizations, and the seeming willingness of patients with a factitious disorder (not somatization disorder) to undergo an extraordinary number of mutilating procedures. Patients with conversion disorder are not usually conversant with medical terminology and hospital rou- tines, and their symptoms have a direct temporal relation or sym- bolic reference to specific emotional conflicts. Hypochondriasis differs from factitious disorder in that patients with hypochondri- asis do not voluntarily initiate the production of symptoms, and hypochondriasis typically has a later age of onset. As with som- atization disorder, patients with hypochondriasis do not usually submit to potentially mutilating procedures.
18.7. The answer is E (all)
The prevalence of factitious disorder in the general population is unknown, but it occurs more frequently in women than in men, and the severe syndromes are more frequent in women. Anecdo- tal case reports indicate that many of the patients suffered child- hood abuse or deprivation, resulting in frequent hospitalizations during early development. The motivation for the behavior is to assume the sick role, and external incentives, such as economic gain, avoiding legal responsibility, or improving physical well- being, as in malingering, are absent. The prognosis in most cases is poor, and although there are no adequate data about the ulti-
18. Factitious Disorders 181 mate outcome for the patients, a few of them probably die as a
result of needless medication, instrumentation, or surgery.
18.8. The answer is B
Factitious disorders may occur by proxy; such disorders are du- ally classified as factitious disorder by proxy and factitious dis- order not otherwise specified.
Factitious disorders usually begin in early adult life, although they may appear during childhood or adolescence. The onset of the disorder or of discrete episodes of treatment seeking may follow a real illness, loss, rejection, or abandonment. Usually, the patient or a close relative had a hospitalization in childhood or early adolescence for a genuine physical illness. Thereafter, a long pattern of successive hospitalizations unfolds, beginning insidiously.
Factitious disorders are not best treated with psychoactive drugs. Pharmacotherapy is of limited use. No specific psychiatric therapy has been effective in treating patients with factitious disorders. Although no adequate data are available about the ultimate outcome for patients, a number of them probably die as a result of needless medication, instrumentation, or surgery. They usually have a poor prognosis.
Factitious disorders are not synonymous with Ganser’s syn- drome, a controversial condition that is characterized by the use of approximate answers. Ganser’s syndrome may be a variant of malingering in that patients avoid punishment or responsibility for their actions. Ganser’s syndrome is classified as a dissociative disorder not otherwise specified.
18.9. The answer is E (all)
A factitious disorder is differentiated from somatization disorder (Briquet’s syndrome) by the voluntary production of factitious symptoms; the extreme course of multiple hospitalizations; and the patient’s seeming willingness to undergo an extraordinary number of painful, even mutilating, procedures.
Hypochondriasis differs from factitious disorder in that pa- tients with hypochondriasis do not voluntarily initiate the pro- duction of symptoms, and hypochondriasis typically has a later age of onset. As is the case with somatization disorder, patients with hypochondriasis do not usually submit to potentially muti- lating procedures.
Because of their pathological lying, lack of close relation- ships with others, hostile and manipulative manner, and asso- ciated substance and criminal history, patients with factitious disorder are often classified as having antisocial personality dis- order. However, persons with antisocial personality disorder do not usually volunteer for invasive procedures or resort to a way of life marked by repeated or long-term hospitalizations.
Factitious disorder must be distinguished from malingering. Malingerers have an obvious, recognizable environmental goal in producing signs and symptoms of illness. They may seek hos- pitalization to secure financial compensation, evade the police, avoid work, or merely obtain free bed and board for the night, yet they always have some apparent end for their behavior.
18.10. The answer is A
Patient with factitious disorder may be classified as having histri- onic personality disorder because of the attention seeking and
  
182 18. Factitious Disorders
dramatic flair of histrionic persons. But not all persons with histrionic have a dramatic flair; many are withdrawn and bland. Patients with factitious disorder may also be classified as hav- ing borderline personality disorder. This is because of these patients’ chaotic lifestyles, history of disturbed interpersonal re- lationships, identity crises, substance abuse, self-damaging acts, and manipulative tactics. Persons with factitious disorder usu- ally do not have the eccentricities or dress, thought, or commu- nication that characterize patients with schizotypal personality disorder.
Paranoid personality disorder is characterized by long- standing suspiciousness and mistrust of persons in general. They refuse responsibility for their own feelings and assign respon- sibility to others. They are often hostile, irritable, and angry. Bigots, injustice collectors, pathological jealous spouses, and litigious cranks often have paranoid personality disorder. Narcis- sistic personality disorder is characterized by a heightened sense of self-importance and grandiose feelings of uniqueness. They consider themselves special and expect special treatment. Their sense of entitlement is striking. They handle criticism poorly and may become enraged when someone dares to criticize them, or they may appear completely indifferent to criticism.
18.11. The answer is A
Patients with factitious disorders, whether physical or psycho- logical, most often demonstrate an average or above-average IQ, absence of a formal thought disorder, a poor sense of iden- tity, poor sexual adjustment, poor frustration tolerance, strong dependency needs, and narcissism.
18.12. The answer is B
Munchausen’s syndrome is a colorful term coined by Richard Asher in 1951 in his famous article in The Lancet. He wrote:
Here is described a common syndrome which most doctors have seen, but about which little has been written. Like the famous Baron von Munchausen, the persons affected have always trav- elled widely; their stories, like those attributed to him, are both dramatic and untruthful. Accordingly, the syndrome is respect- fully dedicated to the baron, and named after him.
Patients with Munchausen’s syndrome constantly seek medical care and hospitalization and often assume grandiose, false identi- ties, sometimes claiming to be royalty, relatives of celebrities, or figures in important historical events. It is also known as chronic factitious disorder with predominately physical signs and symp- toms. The two terms are used interchangeably.
The Baron Karl Friedrich Hieronymus von Munchausen (1720–1797) was a nobleman who served in the Russian army in the war against the Turks. After retirement, he entertained friends with embellished stories of his war adventures. The syndrome was named after him because of the dramatic nature of the false histories and symptoms and the travel from hospital to hospital that are characteristic of people with the syndrome.
Jean-Marie Charcot, around 1890, used the term mania op- erativa activa to describe a young girl who continually sought surgery for pain in a knee joint until her medical care seeking resulted in a surgeon amputating the leg. No pathology was ever found in the leg.
“Munchausen’s syndrome by proxy” was first described in 1977 by British pediatrician Roy Meadow. The essential fea- ture of this disorder is the intentional feigning or production of physical or psychological symptoms in another individual who is under the perpetrator’s care. The perpetrator’s motive is to assume the sick role by proxy.
Factitious disorder with psychological symptoms was first described by Alan Gelenberg in 1977, who mused that whereas other patients with factitious disorder avoided psychiatrists, his patient, a war veteran, gained admission to more than 30 psychiatric hospitals within a few years, usually feigning de- pression and suicidality under various pseudonyms.
18.13. The answer is E (all)
Overall, demographic analyses of factitious disorders in the liter- ature have distinguished two general patterns. Factitious disorder patients with Munchausen syndrome are typically middle-aged men who are unmarried, unemployed, and estranged from their families; the remaining patients are generally women ages 20 to 40 years. A number of reports suggest that those in the second group are commonly employed in or intimately familiar with health care occupations such as nursing and physical therapy. In a 10-year retrospective study of hospitalized patients, 28 of 41 patients with factitious disorder identified worked in medi- cally related fields, 15 as nurses.
18.14. The answer is E (all)
The psychodynamic underpinnings of factitious disorders are poorly understood because the patients are difficult to engage in an exploratory psychotherapy process. They may insist that their symptoms are physical and that psychologically oriented treat- ment is therefore useless. Anecdotal case reports indicate that many of the patients suffered childhood abuse or deprivation, resulting in frequent hospitalizations during early development. In such circumstances, an inpatient stay may have been regarded as an escape from a traumatic home situation, and the patient may have found a series of caretakers (e.g., doctors, nurses, and hospital workers) to be loving and caring. The patients’ fami- lies included a rejecting mother or an absent father. The usual history reveals that the patient perceives one or both parents as rejecting figures who are unable to form close relationships. The facsimile of genuine illness, therefore, is used to recreate the desired positive parent–child bond. The disorders are a form of repetitional compulsion, repeating the basic conflict of needing and seeking acceptance and love while expecting that they will not be forthcoming. Hence, the patient transforms the physicians and staff members into rejecting parents.
Patients who seek out painful procedures, such as surgical operations and invasive diagnostic tests, may have a masochistic personality makeup in which pain serves as punishment for past sins, imagined or real. Some patients may attempt to master the past and the early trauma of serious medical illness or hospital- ization by assuming the role of the patient and reliving the painful and frightening experience over and over again through multi- ple hospitalizations. Patients who feign psychiatric illness may have had a relative who was hospitalized with the illness they are simulating. Through identification, patients hope to reunite with the relative in a magical way.
Many patients have the poor identity formation and disturbed self-image that is characteristic of someone with borderline per- sonality disorder. Some patients are as-if personalities who have assumed the identities of those around them. If these patients are health professionals, they are often unable to differentiate themselves from the patients with whom they come in contact. The cooperation or encouragement of other persons in simulat- ing a factitious illness occurs in a rare variant of the disorder. Although most patients act alone, friends or relatives participate in fabricating the illness in some instances.
18.15. The answer is C
Munchausen syndrome is another name for factitious disorder with predominantly physical signs and symptoms. A primary feature of this disorder is a patient’s ability to present physical symptoms such as hemoptysis so well that he or she gains ad- mission to a hospital. A patient may feign symptoms of a severe disorder with which he or she is familiar and may give a history that is good enough to deceive a skilled clinician. The patient usually demands surgery or other treatment and can become abu- sive when negative test results threaten to reveal the factitious behavior. The other choices listed—depression, amnesia, pain disorder, and psychosis—are common presentations of patients with factitious disorder with predominantly psychological signs and symptoms.
18.16. The answer is A
The literature on factitious disorder with predominantly psycho- logical signs and symptoms is notable for the magnitude of the psychological dysfunction present in patients. Almost all have serious personality disorders, often associated with substance abuse. Several authors have reported that there is a high rate of suicide in this population and that patients with factitious psy- chological disorders have a worse prognosis than patients with most other Axis I disorders.
The patient’s simulation of a mental disorder may actually represent the prodrome to an authentic mental disorder with a serious outcome. In particular, clinicians should be cautious in diagnosing factitious psychosis because in two small stud- ies a majority of these patients eventually manifested clear-cut psychotic disorders such as schizophrenia. In other cases, an os- tensibly feigned condition such as depression has responded to psychotropic medications, validating at least some element of the presentation. Because virtually all patients with this type of factitious disorder have a personality disorder (usually border- line, histrionic, or antisocial), caregivers must also recognize that the simulated mental disorder coexists with an authentic one.
18.17. The answer is C
No specific psychiatric therapy has been effective in treating patients with factitious disorders. It is a clinical paradox that pa- tients with the disorders simulate serious illness and seek and submit to unnecessary treatment while they deny to themselves and others their true illness and thus avoid possible treatment for it. Treatment is thus best focused on management rather than cure. Perhaps the single most important factor in suc- cessful management is a physician’s early recognition of the disorder.
Physicians should try not to feel resentment when patients humiliate their diagnostic prowess, and they should avoid any unmasking ceremony that sets up the patients as adversaries and precipitates their flight from the hospital. The staff should not perform unnecessary procedures or discharge patients abruptly, both of which are manifestations of anger. Pharmacotherapy for factitious disorders is of limited use. Patients with comorbid Axis I disorders (e.g., schizophrenia) will respond to antipsychotic medication, but in all cases, medication should be administered carefully because of the potential for abuse.
18.18. The answer is A
Because of their pathological lying, lack of close relationships with others, hostile and manipulative behavior, and associated substance abuse and criminal history, patients with factitious disorder are often classified as having antisocial personality dis- order. Antisocial persons, however, do not usually volunteer for invasive procedures or resort to a way of life marked by repeated or long-term hospitalizations. There is no evidence that facti- tious disorder symptoms precede the development of antisocial personality traits or vice versa.
Because of attention seeking and an occasional flair for the dramatic, patients with factitious disorder may be classified as having histrionic personality disorder. Consideration of the pa- tient’s chaotic lifestyle, history of disturbed interpersonal rela- tionships, identity crisis, substance abuse, self-damaging acts, and manipulative tactics may lead to the diagnosis of borderline personality disorder.
18.19. The answer is D
Malingering is the most likely diagnosis based on the clinical presentation. Until his arrest, there was no psychiatric history or previously reported psychiatric symptoms. The man’s mental status examination results are apparently normal; there are no disorganized thoughts or loosening of associations. The patient claims a variety of unrelated bizarre beliefs, presenting responses in a manner that is inconsistent with the disorganization of psy- chological functioning that would be expected if the symptoms were genuine. In this case, the “psychotic” symptoms are un- der voluntary control, and because there is external incentive (avoiding prosecution) and no evidence of an intrapsychic need to maintain a sick role, the diagnosis of factitious disorder with predominantly psychotic features is ruled out. The patient ex- presses no paranoid feelings, as would be seen in schizophrenia, paranoid type. His delusions lack conviction and are therefore not indicative of the unshakable beliefs in a delusional disorder. Capgras’ syndrome, the delusion that familiar people have been replaced by identical impostors, is not seen here.
18.20. The answer is A
Ganser’s syndrome, the voluntary production of severe psychi- atric symptoms, sometimes described as giving approximate an- swers or talking past the point, is strongly associated with a severe personality disorder. It is more common in men than in women, and it most typically associated with prison inmates. It was previously classified as a factitious disorder but is com- monly associated with dissociative phenomena such as amnesia, fugue, perceptual disturbances, and conversion symptoms and
18. Factitious Disorders 183
184 18. Factitious Disorders
is thus classified as a dissociative disorder. Recovery from the
syndrome is sudden; patients claim amnesia for the events.
18.21. The answer is C
Masquerade syndrome is when illness fabrication results in the child’s increasing dependency on the parent. When considering the differential diagnosis for factitious disorder by proxy, other by-proxy conditions should also be considered because children can be made to manifest psychopathologies of their parents. For
example, in hypochondriasis by proxy, a hypochondriac parent preoccupied with her child’s health can repetitively seek pedi- atric care and thus risk unnecessary procedures and iatrogenic illness for the child. Other by-proxy syndromes include mother- ing to death in which the child is confined to the sick role as if the child were ill while avoiding physicians and agencies; extreme illness exaggeration in which a parent exaggerates the child’s symptoms in an effort to increase a pediatrician’s attention to the child; and achievement by proxy.

Dissociative disorders are a group of syndromes characterized by a sudden, temporary alteration in the normally integrated functions of consciousness, identity, or motor behavior in which some part of these functions is lost. Patients lose the sense of having one consciousness and feel that they either have more than one identity or no identity at all. The integrated thoughts, feelings, and actions that give every person his or her unique personality is abnormal in these patients. There are currently five dissociative disorders: (1) dissociative amnesia, (2) dissociative fugue, (3) dissociative identity disorder (for- mally known as multiple personality disorder), (4) deperson- alization disorder, and (5) dissociative disorder not otherwise specified.
Normal people can experience feelings of dissociation or de- personalization under a variety of circumstances, such as fatigue, isolation, or hypnosis. These feelings tend to be temporary, and although perhaps briefly uncomfortable, are not experienced as overly distressful. Dissociative disorders are much more severe and disabling. Pathological dissociative states are associated with histories of childhood physical, emotional, and sexual abuse, or may be seen in people who have undergone traumatic wartime or disaster experiences. A careful and thorough medical evalu- ation is necessary to rule out any possible organic cause for the dissociative symptoms.
Students should study the questions and answers below for a useful review of these disorders.
QUESTIONS
Directions
Each of the questions or incomplete statements below is followed by five suggested responses or completions. Select the one that is best in each case.
19.1. The systematized type of dissociative amnesia is
A. the failure to recall successive events as they occur B. the inability to recall events related to a circumscribed
C. the failure to recall one’s entire life
D. amnesia for certain categories of memory
E. theabilitytoremembersome,butnotall,oftheevents
occurring during a circumscribed period
19.2. Which of these statements regarding the prognosis of
dissociative identity disorder is incorrect?
A. Recovery is generally complete.
B. The earlier the onset of dissociative identity disorder,
the poorer the prognosis is.
C. The level of impairment is determined by the number
period of time
and types of various personalities.
19
Dissociative Disorders

HELPFUL HINTS
The terms below relate to dissociative disorders and should be defined.
anterograde amnesia approximate answers automatic writing brainwashing coercive persuasion continuous amnesia crystal gazing
denial depersonalization derealization disorder dissociation dissociative amnesia
Dissociative Experience Scale
dissociative fugue dissociative identity
disorder dissociative trance dominant personality double orientation doubling epidemiology of
dissociative disorders
false memory syndrome
Ganser’s syndrome hemidepersonalization highway hypnosis hypnotizability Korsakoff’s syndrome localized amnesia malingering
multiple personality disorder
paramnesia
possession state reduplicative
paramnesia repression
retrograde amnesia secondary gain selective amnesia sleepwalking disorder temporal lobe functions transient global amnesia unitary sense of self wandering
185
▲▲ ▲▲▲▲▲▲▲▲▲
▲ ▲▲▲▲▲▲▲▲ ▲
▲ ▲▲ ▲▲▲▲▲
▲▲▲▲▲▲▲▲▲▲▲▲
186 19. Dissociative Disorders
D. Individual personalities may have their own separate mental disorders.
E. One or more of the personalities may function relatively well.
. 19.3. Reduplicative paramnesia is a condition of which of the following disorders?
A.De ́ja`vu
B. Depersonalization disorder C. Dissociative amnesia
D. Dissociative fugue
E. Dissociative identity disorder
. 19.4. Culture-bound syndromes in which dissociative fugue is a prominent feature include
A. amok
B. grisi siknis
C. latah
D. piblokto
E. all of the above
. 19.5. Which of the following statements about the nonclassic presentation of dissociative amnesia is false?
A. Patients do not reveal the presence of dissociative symptoms unless asked directly.
B. Some patients may describe a history of fugue-like states.
C. Patients are quickly brought to medical attention for dissociative symptoms.
D. Amnesia presents as a circumscribed memory gap.
E. Primary complaints do not relate directly to amnesia.
. 19.6. Organic amnesias are distinguished from dissociative amnesias by which of the following?
A. They do not normally involve recurrent identity alter- ation.
B. The amnesia is not selectively limited to personal information.
C. The memories do not focus on an emotionally trau- matic event.
D. The amnesia is more often anterograde than retro- grade.
E. All of the above
. 19.7. Which of the following statements regarding transient
global amnesia is false?
A. Complete recovery does not occur.
B. It usually lasts 6 to 24 hours.
C. It is an acute retrograde amnesia.
D. It is most often caused by transient ischemic attacks.
E. It affects recent memories more than remote memo-
ries.
. 19.8. Patients with dissociative amnesia
A. do not retain the capacity to learn new information
B. commonly retain awareness of personal identity but have amnesia for general information
C. present very similarly to patients with dementia
D. typically behave in a confused and disorganized way E. none of the above
19.9. Dissociative amnesia is thought to be
A. decreased in times of war and natural disaster B. the least common of the dissociative disorders C. more common in older adults than younger D. more common in women than men
E. none of the above
19.10. The mental status examination of a patient with dissocia- tive identity disorder would most likely reveal which of the following?
A. Decreased concentration
B. Flat affect
C. Impaired recent memory D. Orientation difficulties
E. Normal examination results
19.11. Dissociative fugue
A. is caused by heavy alcohol use
B. occurs more often during wartime and natural disas-
ters
C. has new identities that are more complete than in dis-
sociative identity disorder
D. identities can alternate as in dissociative identity dis-
order
E. all of the above
19.12. The most common cause of organic fugue is probably
A. brain tumors B. epilepsy
C. head trauma D. hypoglycemia E. migraines
19.13. Allofthefollowingaretruestatementsaboutdissociative fugue except
A. It is usually a long-lasting state.
B. Recovery is spontaneous and rapid.
C. It is a rare type of dissociative disorder.
D. It is characterized by a lack of awareness of the loss
of memory.
E. It is not characterized by behavior that appears ex-
traordinary to others.
19.14. The mainstay of treatment of dissociative fugue is
A. antidepressant medication
B. hypnosis
C. psychodynamic psychotherapy D. sodium amobarbital interviewing E. none of the above
. 19.15. Patients predisposed to dissociative fugue include those with all of the following except
A. borderline personality disorders B. heavy alcohol abuse
C. histrionic personality disorders D. mood disorders
E. schizophrenia
. 19.16. Depersonalization disorder